Добавил:
Опубликованный материал нарушает ваши авторские права? Сообщите нам.
Вуз: Предмет: Файл:
Клянусь Вася решу! V.13(ФИНАЛ).pdf
Скачиваний:
240
Добавлен:
12.01.2023
Размер:
9.36 Mб
Скачать

Объективно: шея короткая, диапазон подвижности шейного отдела снижена. При осмотре ротовой полости визуализируется только твердое небо. Спрогнозируйте класс трудной интубации по Маллампати у данной пациентки?

4

32.В послеоперационной палате при дыхании воздухом пациент стал «серым». При быстром исследовании КОС отмечается: рН - 7,0; РаСО2 - 80 мм рт.ст.; РаО2 - 45 мм рт.ст.; BE – 0 ммоль/л. Выберите правильную интерпретацию ответов?

дыхательный ацидоз, артериальная гипоксемия, обусловлен­ные

гиповентиляцией

Применение

1.Пациент 25 л, поступил в сосудистую хирургию с ножевым раненением в шейный отдел. Сознание оглушенное. Кожа и видимые слизистые резко бледные; Дыхание учащенное.ЧД 28; АД 60/40ммHg; ЧСС 125 уд в м; индекс Алговера 2. Какой из компонентов крови первостепенно применить для оказания неотложной помощи?

А. Тромбоциты

B.Лейкоциты

C.Криопреципетат

D.Эритроциты

E.Альбумин

2.В отделение реанимации поступил пациент 25л, со слов сопровождающих, пациент обнаружен соседом в гараже внутри автомобиля с работающим двигателем. Объективно: Общее состояние тяжелое; сознание оглушенное; фотореакция вялая; кожные покровы бледно-синюшные; дыхание, учащенное ЧД 28; АД 80/40mmHg; SpO2 82%; НвCО>35%. Какой из нижеперечисленных методов оксигенации наиболее целесообразнее применить?

А. Подача О2 через дыхательные концентраторы

B.Подача О2 через дыхательной мешок

C. Подача О2 через носовые канюли

D. Подача О2 через кислородную камеру

E. Подача О2 через дыхательный аппарат

3.Пациент 34л., произведена пункция эпидурального пространства в положении на левом боку, введена 0,5% бупивакаина-20,0; после чего у больного отмечается неадекватное поведение с последующей потерей сознания с

присоединением судорог. Выбрать препараты для купирования судорог. А. Промедол

B.Диазепам

C.Дроперидол

D.Кетамин

E.Галоперидол

4.Пациент 67 л, предстоит ампутация правого бедра по поводу гангрены с интоксикационным синдромом; Состояние средней тяжести; Жалобы на жажду, сухость во рту и на выраженную общую слабость; Кожные покровы бледные, сухие. Дыхание жесткое проводится с обеих сторон, хрипов нет; АД 90/60mmHg; ЧСС 102 уд. Какой комплекс терапии необходимо использовать в предоперационном периоде?

А. Мочегонная терапия

B.Трансфузионная терапия

C.Энтеральнеое терапия D. Парентеральная терапия E. Гипотензивная терапия

5.Пациент 43л, в плановом порядке взята на оперативное вмешательство по поводу удалении миомы матки под спинномозговой анестезией. У больной на следующий день во второй половине дня отмечается сильнейшие головные боли в затылочной области, тошнота, рвота.

Выбрать оптимальный метод купирования болевого синдрома. А. Обезболивание кофеином

B.Обезболивание аспирином

C.Обезболивание калипсолом

D.Обезболивание морфином

E.Обезболивание кетамином

.

6.Пациент(ка) 58 лет, поступила в отд неврологии сопорозном сознании.Из анамнеза страдает ГБ 3 очень высоким риском. В неврологическом статусе: Анизокария. Правостороняя гемиплегия. По шкале Глазго 13 бл. Какое диагностическое вмешательство необходимо использовать для уточнения

диагноза?

А. Рентген головного мозга

B.КТ головного мозга

C.МРТ головного мозга

D.ЭЭГ головного мозга

E.УЗДГ головного мозга

7.Пациент(ка) 62л страдает бронхиальной астмой, после прогулки в лесу, участились приступы удушья; Жалуется на чувство нехватки воздуха, удушье, чувство страха смерти; кожа бледная, акроционоз губ, Дыхание частое, ЧД 28; жесткое дыхание проводится с обеих сторон, выслушиваются сухие, свистящие хрипы по всем легочным полям. Какой из нижеперечисленных

препаратов целесообразнее применить внтуривенно? А. преднизолон

B.фуросемида

C.пропофола

D.трепсина

E.промедола

8.Пациент 39 л, страдает полиаллергией, хроническим бронхитом с астмоидным формой. Аллергия на местные анестетики.Планируется удаление зуба. АД 90/70 мм рт.ст. Выбрать наиболее оптимальное сочетание препаратов для адекватной внутривенной анестезии.

А. Гексенал с промедолом

B.Кетамин с седуксеном

C.Тиопентал с азотом

D.Оксибутират с фентанилом

E.Диазепам с промедолом

9.Женщина 64 л., поступила в приемный блок с жалобами на тошноту, рвоту, боли в правом подреберье и поясничной области. Из анамнеза самостоятельно в течении 4 дней принимает парацетамол по 3 таб в день по поводу ОРВИ. В анализах АЛТ 102; АСТ 97; Какой антидот целесообразнее применить в данном отравлении?

А. Атропин

B.Купренил

C.Ацетилцистеин

D.Дипироксим

E.Протамин

10.Пациент 35 л., поступил в отд.токсикологии с жалобами на головные боли, слезотечение, затрудненное дыхание и общую слабость. Из анамнеза работал хлорофосом без средств защиты. Объективно: Сознание оглушенное.Зрачки узкие, фотореакции нет. Кожа влажная. Отмечается мышечные фибрилляции. Какой антидот целесообразнее применить в данном отравлении?

A. Унитиол

B. Флумазенил

C. Ацетилцистеин D. Атропин

E. Налоксон

11.Женщина 48 л., планируется операция по поводу удаления миомы матки. Со слов

пациента отягощённый аллергологический анамнез. Поп «В». Какой препарат целесообразнее применить для премедикации?

А. атропин B. диазепам

C. преднизолон D. адреналин E. эстроген

12.Пациент 67 л., поступил в сосудистую хирургию с декомпенсированной ишемией правой нижней конечности с болевым синдромом. Из сопутствующих заболеваний: ИБС. Постинфарктный кардиосклероз. ХОБЛ. Хронический обструктивный бронхит в ст.обострения. Из лаб-инструментальных данных: Нв 86: Эр 2.1: Нт-22%. Лейкоциты 18/мм3, п/24, СОЭ 36. Креатинин 570 ммол/л. Выбрать, какая тактика оперативного вмешательство планируется в данном случае?

A. Плановая тактика B. Срочная тактика

C. Экстренная тактика D. Отсроченная тактика

E. Выжидательная тактика

13.Женщина 65 л., 2-е сутки после экзартикуляции II-IV пальцев левой стопы, из анамнеза страдает сахарным диабетом в течение 15 л, принимает 20 ед инсулина утром и 14 ед вечером. Сознание оглушенное. Словесному контакту вяло доступна. Кожные покровы бледные, холодный липкий пот. Дыхание редкое с ЧД-14 р в мин. АД-90/60 мм.рт.ст. ЧСС-112 уд в мин. Какое лабораторное исследование первостепенно надо применить в данном случае?

A. Тропонин крови B. Гемоглобин крови C. Глюкоза крови

D. Электролиты крови E. Газы крови

14.Пациенту 25 л., планируется гемитиреиоидэктомия под тотальной внутривенной анестезией с ИВЛ, анестезиологический риск по ASA I, АД 120/70 мм.рт.ст. ЧСС 68 уд в мин. Вводный наркоз произведен 1% пропофолом с умеренной гипотонией АД 90/70 мм.рт.ст. Какой из нижеперечисленных препаратов целесообразнее применить для быстрой миорелаксации?

A. Галламин B. Метокурин

C. Тубокурарин D. Дитилин

E. Диплацин

15. Роженица 42 л., сроком 38 недель, планируется кесарево сечение. Из сопутствующих заболеваний варикозное расширение вен нижних конечностей. Сахарный диабет 2 типа. Кожные покровы бледно-розовые. АД 140/80 мм.рт.ст. ЧСС 87 уд в мин. ЧД 18 р в мин. Нв 110г/л. Эр 2.3. Глюкоза крови 6.8 ммол/л. Какое из нижеперечисленных видов анестезии целесообразнее применить?

А. Внутривенная анестезия B. Проводниковая анестезия C. Ингаляционная анестезия

D. Каудальная анестезия

E. Эпидуральная анестезия

16.Больной 60 лет с ИБС принимает оральные антикоагулянты, поступил в стационар в связи острым тромбозом мезентериальных сосудов. Выбрать правильный тактику приема препарата у данного пациента?

А. Пероральный прием антикоагулянта B. Подкожный прием антиагреганта

C. Внутримышечный прием антикоагулянта D. Внутривенный прием антикоагулянта

E. Внутриартериальный прием антиагреганта

.

17.У пациент 64 лет во время операции холецистэктомии произошла остановка сердечной деятельности. Выбрать в какой последовательности осуществляется СЛР у данного пациента?

A. Адреналин, непрямой массаж сердца B. Атропин, прямого массажа сердца. C. Гепарин, непрямой массаж сердца D. Гепарин, прямой массаж сердца

E. Aдреналин, прямой массаж сердца

18.Мужчина 25 л, доставлен после оказания первой мед.помощи после утопления. Сознание отсутствует. Кожа бледно-синюшной окраски. Дыхание поверхностное ЧД

32р SpO2 75%. Аус-но: масса проводных влажных хрипов по всем легочным полям, в нижних отделах не проводится. Сразу произведена оротрахеальная интубация с переводом на ИВЛ. Какое вмешательство необходимо использовать для улучшения прогноза лечение?

А. Прямая ларингоскопия

B.Лечебная бронхоскопия

C.Диагностическая бронхоскопия

D.Бронхоальвеолярный лаваж

E.Санационная бронхоскопия

19.В реанимацию переводится пациент 45 л с кровотечением из варикозно расширенных вен пищевода.Объективно: кожные покровы и видимые слизитые бледножелтушные.Дыхание учащенное с ЧД 25 р в мин. SpO2 92%АД 80/40. ЧСС97 уд в мин.В анализах кр.крови Hb-87 ; Эр 2.2; Ht 22%. АЛТ 102; АСТ 99. Какой зонд необходимо применить в данном случае?

А. Поляка

B.Кохера

C.Блэкмора

D.Боумена

E.Куликовского

20.Пациент 24 л, поступил в отделение реанимации спастическими судорогами. Из анамнеза страдает несахарным диабетом в течение 5 лет. Сознание оглушенное. Словесному контакту не доступен. Кожные покровы сухие. Тургор снижен. Дыхание учащенное, ЧД 25 р в мин. АД 90/60 мм.рт.ст. ЧСС112 уд в мин. В анализах : Эр 5.7; Нв -185; Нт -52%; Na 165 ммол/л: Какие растворы необходимо использовать в данном случае?

А. Изотонические B. Гипотонические C. Гипертонические D. Коллоидные

E. Кристаллоидные

21.Пациент 50л, доставлен в кардиологию с жалобами на чувство сжимающие боли за грудиной с одновременными неприятными ощущениями в левом плече. АД 90/60 мм.рт.ст. ЧСС 97 уд в мин. На ЭКГ патологический зубец Q, элевация RS, отриц.зубец T. Какой нижеперечисленных препаратов целесообразно применить для купирования болевого синдрома?

А. метамезол B. калипсол D. кетонал C. промедол

E. парацетамол

22.Пациентка 36л поступила с жалобами на одышку с затрудненным выдохом, кашель с трудноотделяемой мокротой. Страдает астмой. Положение ортопноэ, кожные покровы влажные, с цианотичным оттенком, дистанционные хрипы. При аускультации в легких жесткое дыхание, сухие хрипы по всем полям, ЧСС 66 в мин. ЧД 28 в мин. Какой из перечисленных препаратов целесообразно применить для оказания неотложной помощи?

А. эфедрин B. изадрин C. адреналин D. эуфиллин E. теофилин

23.Пациент 32 л, из анамнеза наркозависимость в течение 3 лет. После хирургического вмешательства по поводу гнойного аппендицита, осложненного перитонитом, испытывает сильную боль. Выбрать оптимальный метод купирования болевого синдрома.

А. Ненаркотическое обезболивание B. Методоновое обезболивание

C. Наркотическое обезболивание D. Эпидуральное обезболивание

E. Мультимодальное обезболивание

24.Пациентке 54 л планируется операция холецистэктомия. Из сопутствующих заболеваний ГБ 3ст, высокого риска; Нарушение жирового обмена, ИМТ +40. Объективно: шея короткая, диапазон подвижности шейного отдела снижена. Прогнозируется трудная интубация. Какие мероприятия необходимо использовать в данном случае?

А. Наложение трахеостомии B. Наложение коникотомии C. Слепая интубация

D. бронхоскопическая интубация E. назотрахеальная интубация

25.Мужчина 45 л, попал ДТП. На вопросы отвечает односложно, жалуется на боль в груди, чувство нехватки воздуха. Кожные покровы бледные с синюшным оттенком. Дыхание поверхностное, ЧДД 36 в мин. При аускультации: слева дыхательные шумы проводятся во все отделы, справа - резко ослаблены. АД 90/60 мм рт. ст., тахикардия с ЧСС 130 в мин. SaO2 - 82%. Какое диагностическое вмешательство необходимо применить в данном случае?

А. торокоскопия B. бронхоскопия C. рентгеноскопия

D. медиастиноскопия E. хромобронхоскопия

26.Больной 60 лет, 3 сутки после операции по поводу кишечной непроходимости. В послеоперационном периоде наблюдается выраженный болевой синдром с кратковременным купированием наркотическими анальгетиками. АД 160/90. ЧСС 116 уд. Живот вздут, перистальтика не выслушивается. Какую тактику анальгезии следует выбрать в данном случае?

А. Ненаркотические анальгетики B. Наркотические анальгетики C. Эпидуральная анальгезия

D. Внутривенный парацетамол E. Местные анестетики

27.Вы осматриваете пациента 25 лет перед аппендэктомией, со слов пациента последний прием пищи состоялся 2 часа назад, АД 120/80 мм.рт.ст. риск по ASA II E. Какое анестезиологическое обеспечение целесообразнее выбрать в данном случае?

А. Провести общею анестезию введением воздуховода

B.Провести общею анестезию ларенгиальной маской

C.Провести общею анестезию лицевой маской

D.Провести операцию под проводниковой анестезией

E.Провести операцию под эпидуральной анестезией

28.В приемный блок нейрохирургии поступил мужчина 36 л в тяжелом состоянии после ДТП. Сознание кома. Зрачки D <S. Отсутствует роговичный рефлекс. Дыхание поверхностное,ЧД 32 р в мин. АД 170/90. ЧСС117 уд. На КТ головного мозга гематома височно-теменной доле. Какое лечение целесообразнее применить для оказания неотложной помощи?

А. Гемостатическое B. Противоотечное C. Трансфузионное D. Гипотензивное E. Оперативное

29.У больного с ОПН в олиго-анурическую стадию имеет место выраженный ацидоз, умеренная рвота и понос. Какие растворы жидкостей предпочтительнее вводить для возмещения потерь электролитов?

А. Р-р бикарбоната натрия 5% до 400мл; изотоническими растворами

B. Р-р бикарбоната натрия 8,4% до 500мл; гипертоническими растворами C. Р-р бикарбоната натрия 4% до 100мл; гипотоническими растворами

D. Р-р бикарбоната натрия 4% до 200мл; сбалансированными растворами E. Р-р бикарбоната натрия 5% до 300мл; коллоидными растворами

30.У больного страдающего неукротимой рвотой и поносом, уровень мочевины крови

17ммоль/л, развилась олигурия и дегидратация; рН 7.15; Выбрать тактику лечения. А. Коррекция алкалоза, форсированный диурез

B. Коррекция ацидоза, восполнение объема C. Коррекция ацидоза, форсированный диурез D. Коррекция алкалоза, восполнение объема E. Коррекция алколоза, дезинтоксикация

31.Пациент 47 л., готовится к плановой операции по поводу тиреоидэктомии. Сопутствующие заболевания: Вирусный гепатит В. Из анамнеза перенес ЧМТ. Аллергологический анамнез поп В. Какой ингаляционный анестетик целесобразнее применить у данного пациента?

А. Фторотан B. Ксенон C. Эфир

D. Трихлорэтилен E. Хлорофором

32.У больного 25 л после заглатывания антифриза через 6 часов развилась выраженная слабость, головокружение с потерей сознания. Через 12 часов количество мочи снизилось до 200 мл, через 20 часов анурия, появились отеки. В анализе крови мочевина 22,5 ммоль/л, креатинин 847 мкмоль/л. Какую из нижеперечисленных процедур необходимо применить в данном случае:

А. Гемосорбция B. Плазмоферез C. Гемодиализ

D. Ультрафильтрация E. Гемодиафильтрация

33.У пациента с язвенной болезни желудка, возникла рвота, жидкий черный стул, общая слабость. При осмотре: кожные покровы бледные, ЧСС - 106 в минуту, АД в положении лежа - 90/50 мм рт. ст. Отмечается болезненность при пальпации живота в эпигастальной области. Какое диагностическое вмешательство необходимо использовать у данного пациента?

А. Эндоскопия B. Гастроскопия

C. Рентгеноскопия D. Колоноскопия E. Ирригоскопия

34.Пациент 52 л., с диагнозом: Острый коронарный синдром с подъёмом сегмента ST в нижней стенке левого желудочка. Какие исследования крови необходимо применить для уточнения диагноза?

А. Прокальцитонин B. Тропонин

C. Холестерин D. Гаптоглобин E. Трансферин

35.Больная С., 67 лет, поступила с жалобами на сухость во рту, жажду, зуд; около 6 лет страдает сахарным диабетом. Принимает манинил по 5 мг 2 раза в сутки за 30 минут до еды и сиофор по 500 мг 1 раз в сутки во время ужина. При осмотре повышенного питания, кожные покровы сухие, АД 160/90 mmHg, ЧСС 82. Глюкоза крови 21,5 ммоль/л, кетоацидоз. Выберите тактику лечения.

А. Гипотензивная терапия B. Мочегонная терапия

C. Гипергликемическая терапия D. Гипогликемическая терапия E. Дезинтоксикационная терапия

36.Пациент с тяжелой сердечной недостаточностью в течение длительного времени получал фуросемид. У него появилась выраженная слабость, участились желудочковые экстрасистолы, в крови выявлено снижение уровня калия и магния. Каким из перечисленных препаратов целесообразнее заменить фуросемид?

А. Гидрохлоротиазид B. Дихлотизид

C. Торасемид

D. Спиронолактон. E. Диакарб

37.Больная 68л поступила в стационар с жалобами на одышку при незначительной физической нагрузке, сердцебиение, отеки ног, общую слабость. В анамнезе инфаркт миокарда 6 лет назад. Вышеперечисленные жалобы беспокоят в течение месяца. К врачу не обращалась, не лечилась. Предложите препараты, которые необходимо назначить больной:

А. Сердечные гликозиды, ингибиторы ренина, диуретики

B.Ингибиторы-ренина, ангиопротекторы, тринитраты

C.Ингибиторы АПФ, бета-адреноблокаторы, диуретики.

D.Альфа-адреномиметики, диуретики, ингибиторы АПФ

E.Альфа-адреноблокаторы, гликозиды, тринитраты

38. Пациент 45 лет, находится в отделении с диагнозом «двухсторонняя пневмония». С антибактериальной целью введена цефтриаксон в/в, через 5 мин жалуется на ухудшение, холодный липкий пот. Сознание оглушенное, АД 40/0 мм рт.P/S нитевидный. Какое из нижеперечисленных препаратов целесообразнее применить для оказания неотложной помощи?

А. атропин

B. преднизолон C. адреналин

D.полиглюкин

E.фенилэфрин

39.Пациенту 45 л во время проведения сердечно-легочной реанимации на ЭКГ регистрируется фибрилляция желудочков (зубцы Р и комплексы QRST отсутствуют). Какое мероприятия необходимо использовать для оказания неотложной помощи?

А. Проведение электрической дефибрилляции

B.Проведение прекардиального удара

C.Внутрисердечное введение адреналина

D.Внутриартериальное введение антикоагулянтов

E.Внутривенное введение бетаблокаторов

40. У пациента с острым трансмуральным инфарктом миокарда возникли

частые желудочковые экстрасистолы. Какой из перечисленных препаратов целесообразнее применить данному пациенту?

А. Новокаинамид

B.Лидокаин

C.Верапамил

D.Дигоксин

E.Кордарон

41.Выбрать из перечисленных ниже инфузионных растворов обладающих

наиболее выраженным объем замещающим и противошоковым действием?

А. Раствор ксилата B. Раствор глюкозы C. Раствор лактата D. Раствор декстрана

E. Раствор электролитов

42.Пациент 34 л поступил в реанимацию в крайне-тяжлом состоянии, со слов внезапно потерял сознания, жалуется на одышку, нехватку воздуха и чувства страха смерти. Кожные покровы бледные, диффузный цианоз. Дыхание поверхностное, учащенное ЧД 28/мин; АД 80/40 мм.рт.ст. ЧСС 118 уд. Тоны сердца приглушены, ритм правильный. При обследовании установлена ТЭЛА. Какой из перечисленных препаратов целесобразнее применить для оказания неотложной помощи?

А. Варфарин B. Тромбокард C. Тромбопол D. Аспирин

E. Гепарин

43.Пациент 27 л обратился с жалобами на жажду, многократную рвоту, сухость во рту и общую слабость. Кожные покровы сухие, холодные. Дыхание учащенное ЧД 25/мин. АД 100/60 мм.рт.ст. ЧСС-110 уд/мин. Диурез снижен. Какой из нижеперечисленных растворов разумно применить для оказания неотложной помощи?

А. Реополиглюкин B. Полиглюкин

C. Ринлактин D. Рефортан E. Перфторан

44.Женщина 62 доставлена в тяжелом состоянии, со слов родственников страдает сахарным диабетом, принимает инсулин. Сознание оглушенное. Кожные покровы бледные, влажные. Дыхание поверхностное, учащенное. АД 100/60 мм.рт.ст. ЧСС 97 уд/мин. Нв 106г/л: Эр 2.3; Нт 24%; Сахар крови 3,2 ммол/л. Какой из

нижеперечисленных препаратов разумно применить для оказания неотложной помощи?

А. Декстроза 5%

B.Лактоза 15%

C.Гексоза 25%

D.Глюкоза 40%

E.Фруктоза 60%

45. Пациент 54 л с инфарктом миокарда. Выбрать из нижеперечисленных анальгетиков наиболее предпочтительнее для купирования болевого синдрома.

А. Анальгин

B.Кетонал

C.Трамадол

D.Промедол

E.Кофеин

46. Пациент 60 лет, после проведенной операции на сердце получал гепарин по 5 тыс ед в/в каждые 6 часов. На 4 сутки у пациента отмечается носовое кровотечение. Объективно: массивные кожные кровоподтёки в местах инъекции. Диурез по катетеру: гематурия. Какой из антидотов целесообразнее применить для оказания неотложной помощи?

А. Тетацин кальция

B.Тиосульфат натрия

C.Сульфат магния

D.Протамин сульфат

E.Атропин сульфат

47.Пациентка 36 л., с ожирением II, планируется холецистэктомия. Из анамнеза со слов пациента ранее была оперирована по поводу аппендэктомии отмечает трудную интубацию. При осмотре короткая шея, по Mallampati III. С целью профилактики регургитации, какие препараты необходимо применить?

А. Гастрацид

B. Фосфалюгель C. Пантопразол D. Алмагель

E. Маалокс

48.Пациент 30 л., поступил в приемный блок с признаками гиповолемии. Из анамнеза героиновая зависимость. Периферические вены отсутствуют. Катетеризация центральных вен затруднительна. Какое диагностическое вмешательство необходимо использовать для улучшения катетеризации?

А. Рентген

B. Флебография C. УЗДГ

D. Ультрасонография E. Платизмаграфия

49.Пациент 64 л., переведен из хирургического отделения с жалобами на одышку и затрудненное дыхание. Кожные покровы бледные, акроцианоз. Дыхание поверхностное, ЧД 25; аускультативно: проводные хрипы по всем легочным полям. АД 110/60; ЧСС 109; Нв 160г/л; Эр 4,5; Нт 40%; ЦВД 170 мм.вод.ст. Ухудшение состояние связывает после инфузии. Выбрать правильную тактику лечения?

А. Трансфузионная терапия B. Инфузионная терапия

C. Мочегонная терапия D. Гипотензивная терапия

E. Дезинтоксикационная терапия

50.Пациент 52 л., жалуется на стеснение в груди, боли в области эпигастрии, мышечную слабость вплоть до судорог и быструю утомляемость. Со слов пациента принимает длительное время верошпирон по 1 таб 2 раза. Дыхание учащенное, ЧД 26 р /мин. SpO2 86%; АД 140/80; ЧСС 48; На ЭКГ: высокий зуб Т, расширеный QRS: Na 132; Na 5,7: Какое из нижеперечисленных препаратов целесообразнее применить в данной ситуации?

А. Преднизолон B. Дексаметазон C. Адреналин D. Кальций

E. Эуфиллин

51.Пациент 60 л, находящийся в хирургическом отделении 5 сутки после холецистэктомии, при выходе в коридор внезапно упал. Дыхание поверхностное, учащенное. Кожа бледная, цианоз верхней половины туловища, набухания и пульсация шейных вен. АД 60/40; ЧСС 122; SpO2 68%. Неотложные мероприятия в данной ситуации.

А. Непрямой массаж

B. Оротрахеальная интубация C. Центральная катетеризация

D. Электрическая дефибрилляция E. Прекардиальный удар

52.Пациент 48 л., готовится к плановой операции эхинококкэктомии. При беседе пациент лабильный, беспокойный и раздраженный, такое поведение связывает со страхом перед предстоящей операцией. АД 160/90; ЧСС 92 уд в мин. Какой из нижеперечисленный препаратов необходимо назначить с целью премедикации? А. Атропин

B. Димедрол C. Диазепам D. Морфин E. Кетонал

53.Пациентка 28 л., планируется тотальная внутривенная анестезия. ASA II. Какой из нижеперечисленных препаратов необходимо применить для вводного наркоза?

А. Дроперидол B. Дитилин

C. Пропофол D. Ардуан E. Атропин

54.Пациент 30 л., поступил в отделении хирургии для планового оперативного лечения по поводу декомпенсированного язвенного стеноза. Показатели:рН 7,61; РаСО2 43 мм.рт.ст.; К 2,2 ммол/л Сl 90 ммол/л. Какой препарат необходим для предоперационной подготовки?

А. Фосфат сульфат B. Магний сульфат C. Кальций хлор D. Калий хлор

E. Натрий хлор

55.У больного с ОПН в олиго-анурическую стадию имеет место выраженный ацидоз, умеренная рвота и понос. Какие растворы жидкостей предпочтительнее вводить для возмещения потерь электролитов? р-р щелочей бикарбонат натрия проверено

56.Больной 60 лет, 3 сутки после операции по поводу кишечной непроходимости. В послеоперационном периоде наблюдается выраженный болевой синдром с кратковременным купированием наркотическими анальгетиками. АД 160/90 mmHg. ЧСС 116 уд. Живот вздут, перистальтика не выслушивается. Какую тактику анальгезии следует выбрать в данном случае?продленная эпидуральная или

спинальная анальгезия или инфузии лидокаиномПРОВЕРЕНО

57.Пациент 24 лет, поступил в отделение реанимации с спастическими судорогами. Из анамнеза страдает несахарным диабетом в течение 5 лет. Сознание оглушенное. Словесному контакту не доступен. Кожные покровы сухие. Тургор снижен. Дыхание учащенное, ЧД 25 р в мин. АД 90/60 mmHg, ЧСС112 уд в мин. В анализах : Эр 5.7х1012/л; Нв -185г/л; Нт -52%; Na 165 ммоль/л; Какие растворы необходимо использовать в данном случае? ●Гипотонические растворы (0,9 % NaCl до момента

получения нормального артериального давления, затем смесь 0,45 % раствора NaCl и 5 % раствор глюкозы в соотношении 1:1) ПРОВЕРЕНО

58.Мужчина 25 лет, доставлен после оказания первой мед.помощи после утопления. Сознание отсутствует. Кожа бледно-синюшной окраски. Дыхание поверхностное ЧД

32 р SpO2 75%. Аускультативно: масса проводных влажных хрипов по всем легочным полям, в нижних отделах не проводится. Сразу произведена оротрахеальная интубация с переводом на ИВЛ. Какое вмешательство необходимо использовать для улучшения прогноза лечение?Санация трахеобронхиального дерева (бронхоскопия) или

инфузионная терапия коррекция кос ПРОВЕРЕНО

59.Пациент 35 лет, поступил в отделение токсикологии с жалобами на головные боли, слезотечение, затрудненное дыхание и общую слабость. Из анамнеза работал хлорофосом без средств защиты. Объективно: Сознание оглушенное. Зрачки узкие, фотореакции нет. Кожа влажная. Отмечаются мышечные фибрилляции. Какой антидот целесообразнее применить при данном отравлении?Косвенным антидотом является атропин — вводить в тяжелых случаях внутривенно, капельно, до появления сухости во рту и расширения зрачков (30 — 50 — 100 мл в сутки).

Прямым антидотам является 15% раствор дипироксима — 1 мл в вену или под кожу 2 — 4 раза в сутки ПРОВЕРЕНО

60.У пациента 64 лет во время операции холецистэктомии произошла остановка сердечной деятельности. На мониторе – асистолия. Выберите в какой последовательности осуществляется СЛР у данного пациента? адреналин 1мл-1%р-

ра, ->СЛР 30:2 С повторной оценкой ритма каждые 2 мин, ->при возникновении ритма дефибрилляция( бифазный-150-200Дж, монофазный-360Дж) ПРОВЕРЕНО

61.Пациент 32 лет, из анамнеза наркозависимость в течение 3 лет. После хирургического вмешательства по поводу гнойного аппендицита, осложненного перитонитом, испытывает сильную боль. Выберите оптимальный метод купирования болевого синдрома?кетамин инфузии в/вПРОВЕРЕНО

62.Пациенту 45 лет, во время проведения сердечно-легочной реанимации на ЭКГ регистрируется фибрилляция желудочков (зубцы Р и комплексы QRST отсутствуют). Какое мероприятие необходимо использовать для оказания неотложной помощи? Дефибрилляция(с 360 Дж) ПРОВЕРЕНО

63.Пациент 52 лет, с диагнозом: Острый коронарный синдром с подъёмом сегмента ST в нижней стенке левого желудочка. На ЭхоКГ отмечается гипокинез левых отделов. Какие исследования крови необходимо применить для уточнения диагноза? кардиомаркеры (тропонин 1и 2), Ферменты(КФК-МВ,ЛДГ, АСТ) ПРОВЕРЕНО

64.Женщина 62 лет, доставлена в тяжелом состоянии, со слов родственников страдает сахарным диабетом, принимает инсулин. Сознание оглушенное. Кожные покровы бледные, влажные. Дыхание поверхностное, учащенное. АД 100/60 mmHg , ЧСС 97

уд/мин. Нв 106г/л: Эр 2.3; Нт 24%; Сахар крови 3,2 ммол/л. Выберите препарат для оказания неотложной помощи?●струйно вводят в/в 40% р-р глюкозы (20—100

мл);или глюкагон ПРОВЕРЕНО

65.Пациент 52 л., жалуется на стеснение в груди, боли в области эпигастрии, мышечную слабость вплоть до судорог и быструю утомляемость. Со слов пациента принимает длительное время верошпирон по 1 таб 2 раза. Дыхание учащенное, ЧД 26 р /мин. SpO2 86%; АД 140/80; ЧСС 48; На ЭКГ: высокий зуб Т, расширеный QRS: Na 132; Na 5,7: Какое из нижеперечисленных препаратов целесообразнее применить в данной ситуации? ● петлевые диуретики (фуросемид, торасемид) или глюконат кальция

ПРОВЕРЕНО

66.Пациент 48 л., готовится к плановой операции эхинококкэктомии. При беседе пациент лабильный, беспокойный и раздраженный, такое поведение связывает со страхом перед предстоящей операцией. АД 160/90; ЧСС 92 уд в мин. Какой из нижеперечисленный препаратов необходимо назначить с целью премедикации?

С целью премедикации сибазон(диазепам) назначают для приема внутрь на ночь за 20–30 минут до сна в дозе 5–10–20 мг. Утром, сразу после пробужде-ния, сибазон назначают внутрь в дозе 5–10 мг, запивая 100 мл кипяченой воды. За 10–15 минут до операции сибазон назначается повторно в/в в дозе 5–10 мг.

Нозепам (апо-оксазепам, тазепам и др.). Таблетки по 0,01 г (10 мг). По хи-

мическому строению и фармакологическим свойствам аналогичен сибазону, но обладает более мягким действием и менее токсичен. С целью премедикации но-зепам назначают на ночь в дозе 5–10 мг за 20–30 минут до сна.

Феназепам (Fenazepam). Таблетки по 0,0005 и 0,001 г (0,5 мг и 1 мг) и по

0,0025 г (2,5 мг). Феназепам по клиническому эффекту аналогичен действию сибазона. По силе седативного и, главным образом, противотревожного действия не уступает некоторым нейролептикам. С целью премедикации феназепам назначают на ночь в дозе от 0,25 до 2,5 мг за 20–30 минут до сна.

Диазепам(седуксен)

Мидазолам

ПРОВЕРЕНО

67.Пациентка 28 л., планируется тотальная внутривенная анестезия. ASA II. Какой из нижеперечисленных препаратов необходимо применить для вводного наркоза? пропофол,кетамин, тиапентал

ПРОВЕРЕНО

Гематология

Запоминание:

1.Отметьте заболевание, которое характеризуется избыточным накоплением железа в органах депо и в сыворотке крови? гемохроматоз/сидеробластная анемия

2.Определите причину развития синдрома повышенной кровоточивости при апластической анемии? Недостаточным образования тромбоцитов/недост миел.

ростка

3.Укажите, к какой группе заболеваний относится пароксизмальная ночная гемоглобинурия?Приобретенные гемолитические анемии

4.Покажите, при каком уровне тромбоцитов определяется клинико-гематологическая ремиссия первичной имммунной тромбоцитопении? Выше 50

5.Укажите, при каком варианте острых лейкозов образуется патологический белок – парапротеин?миеломная болезнь (острый плазмобластный лейкоз)

6.Укажите, при каком заболевании используются бисфосфанаты – ингибиторы резорбции костной ткани? множественная миелома

7.Выделите заболевание, которое характеризуется абсолютным истощением железа в организме человека? ЖДА

8.Укажите заболевание, которое характеризуется первичным поражением костного мозга? Лейкозы/В-клеточный хронический лимфолейкоз

Понимание:

1.Пациентка А. 27 лет, обратилась к врачу с жалобами на ломкость ногтей, выпадение волос, извращение вкуса и обоняния, затрудненное глотание пищи. Общий анализ крови: эритроциты 4,1х1012/л, Нв 134 г/л, MCV 60 fl, MCH 16 pg, тромбоциты 180,0х109/л, лейкоциты 3,9х109/л: пал.2, сегм.62, мон.2, баз.1, эозин.2, лимф.31, СОЭ 8 мм/час. Содержание ферритина в сыворотке крови 10,0 мкг/л. Объясните, причину развития гипохромии и микроцитоза эритроцитов? Латентный дефицит

железы(Опустошение тканевых резервов железа приводит к нарушению гемоглобинообразования, и раз мало железа в гемоглобине, соответсвенно, будет гипохромия и микроцитоз)

2.Женщина Б. 35 лет, находится в отделении гематологии НЦОГ с клиническими синдромами гипосидероза и анемии. В общем анализе крови: Нв 56 г/л, эритроциты

2,0х1012/л, MCV 45 fl, MCH 16 pg, ретикулоциты 0,2% тромбоциты 256,0х109/л,

лейкоциты 3,4х109/л: сегм.60, баз.3, эоз.1, лимф.36, СОЭ 12 мм/час. Чем обусловлено уменьшение эритроцитарных индексов MCV и MCH? ЖДА

3.Больная В. 42 года, жалуется на беспричинную общую слабость, быструю утомляемость, головокружение, шум в ушах, одышку и сердцебиение при незначительной физической нагрузке, ломкость ногтей, выпадение волос, нравится запах известки, частые запоры. Обследовалась у гинеколога, выявлена аденомиоз матки. В общем анализе крови – гипохромная, микроцитарная анемия тяжелой степени. Определите причину развития дефицита железа? Хроническая

постгеморрагическая анемия

4.Больной Г. 78 лет, обратился к врачу с жалобами на общую слабость, шум в ушах, бессонницу, ухудшение памяти, плаксивость, одышку и колющие боли в области сердца, без определенной иррадиации, чувство похолодания кончиков пальцев рук, шаткость походки. Объективно: общее состояние тяжелое, бледность кожи и видимых слизистых оболочек с желтушным оттенком, субиктеричность склер, язык гиперемирован, сосочки атрофированы. Со стороны общего анализа крови имеется

макроцитарная, гиперхромная анемия тяжелой степени. Дефицит какого вещества имеется у больного? Витамин В12

5.Девушка Д. 25 лет, обратилась к врачу с жалобами на слабость, раздражительность, плаксивость, быструю утомляемость, плохой сон.Из анамнеза выяснено, что недавно лечилась у врача инфекциониста, принимала антигельминтные препараты. Объективный статус удовлетворительный. Пониженного питания, бледность кожи и видимых слизистых оболочек с желтушным компонентом.В общем анализе крови Нв

84 г/л, эритроциты 2,4х1012/л, MCV 102 fl, MCH 34 pg, ретикулоциты 0,2%

тромбоциты 110,0х109/л, лейкоциты 3,7х109/л: сегм.64, баз.1, эоз.8, лимф.27, СОЭ 21 мм/час. Предположите, чем объясняется состояние пациентки? Фолиеводефицитная

анемия

6.Пациентка Ж. 48 лет, жалуется на беспричинную общую слабость, быструю утомляемость, головокружение, одышка и сердцебиение при незначительной физической нагрузке. Из анамнеза, находится на диспансерном учете у ревматолога с клиническим диагнозом «Ревматоидный артрит». Объективно: состояние тяжелое, бледность кожи и видимых слизистых оболочек, мелкие суставы конечностей деформированы. ОА крови: эритроциты 2,1х1012/л, Нв 62 г/л, МСV 65 fl, MCH 18 pg. Предположите, чем объясняется у больной микроцитарная, гипохромная анемия?Анемии хронических заболеваний

6-гепсидиновый механиз

7.Женщина З. 42 лет, жалуется на носовые, десневые и маточные кровотечения. Данные жалобы беспокоят в течение последних 2-х месяцев. Начало болезни ни с чем не связывает. Объективно: на коже конечностей и туловища имеются петехиальные высыпания и кровоподтеки. Внутренние органы без особенностей. В общем анализе крови (двухкратно) изолированная тромбоцитопения ниже 20,0х109/л. Выставлен диагноз «Первичная иммунная тромбоцитопения». Чем объясняется тромбоцитопения?Аутоимунная тромбоцитопения

8.Пациент Е. 20 лет, обратился к врачу с жалобами на боли и ограничение движение в левом коленном суставе. Объективно: Левый коленный сустав увеличен в объеме, резко болезнен, кожа гиперемирована, горячая на ощупь. Правый коленный и локтевые суставы деформированы, с ограничением движения в них. Лечащим врачом выставлен диагноз «Гемофилия В». Дефицит какого фактора свертывания крови привел к поражению суставов?Дефицит фактора IX

9.Женщина И. 26 лет, обратилась в клинику с жалобами на появление мелкоточечных высыпаний на коже нижних конечностей, боли в области правого коленного сустава, усиливающиеся при физической нагрузке. Объективно: Кожные покровы обычной окраски, на коже голеней имеются мелкоточечные петехиальные высыпания, не симметричные, не выступающие над уровнем кожи. Левый коленный сустав увеличен в объеме, болезнен при пальпации. Дефицит какого фактора свертывания крови привел к вышеперечисленным жалобам?Фактор Виллебранда и фактора VIII

10.Больной К. 17 лет, жалобы на ограничение движения и боли в коленных, локтевых суставах. Состоит на диспансерном учете с раннего детства с клиническим диагнозом «Гемофилия А, среднетяжелая форма». Объективно: общее состояние средней степени тяжести, кожные обычной окраски, на коже геморрагий нет. Коленные, локтевые суставы деформированы, с ограничением движения в них. Предположите, при каком уровне снижения активности фактора свертывания крови VIII, определяется среднетяжелая форма гемофилии А?1-5% или 0.01-0.05 МЕ

11.Мужчина Л. 35 лет, доставлен в клинику с жалобами на боли в животе, темный жидкий стул, повышение температуры тела до 38,50C, боли в голеностопных суставах. Объективно: состояние тяжелое, на коже голеней, бедер имеются мелкоточечные папулезно-пурпурные кровоизлияния. Язык обложен белым налетом, живот мягкий, болезненный в эпигастрии. В общем анализе крови: эритроциты 4,2х1012/л, Нв 154 г/л, тромбоциты 420,х109/л, лейкоциты 15,8х109/л: пал.5, сегм.72, баз.3, лимф.20, СОЭ 27 мм/час. Предположите, чем объясняется данная клиническая ситуация у пациента?

Геморрагический микротромбоваскулит

12.Женщина М. 36 лет, поступила в стационар с жалобами на десневые кровотечения при чистке зубов, продолжительные менструальные кровотечения, боли в коленных суставах. Объективно: на коже нижних конечностей имеются единичные петехиальные высыпания. Коленные суставы деформированы, с ограничением движения в них. Общий анализ крови: эритроциты 3,4х1012/л, Нв 95 г/л, MCV 68 fl, MCH 17 pg, тромбоциты 250,5х109/л, лейкоциты 7,9х109/л: сегм.65, эозин.1, баз.2, мон.4, лимф.28, СОЭ 10 мм/час. На гемостазиограмме агрегация тромбоцитов с ристомицином снижено. АЧТВ удлинена. Дефицит какого фактора свертывания крови приводит к нарушению агрегации тромбоцитов и удлинению АЧТВ? Фактор

Виллебранда

13.Больной Н. 36 лет, обратился к врачу с жалобами на повышение температуры тела до 38,50С, боли в горле, усиливающиеся при глотании, боли в животе, жидкий стул, носовые кровотечения, появление высыпаний на конечностях. В общем анализе крови: эритроциты 3,1х1012/л, Нв 82 г/л, тромбоциты 8,6х109/л, лейкоциты 0,7х109/л:

бластные клетки 2, лимф.18 (подсчет на 20 клеток), СОЭ 36 мм/час. Объясните, чем обусловлено тяжелое состояние больного? Миелотоксический агранулоцитоз

14.Пациент Р. 56 лет, поступил в стационар с жалобами на повышение температуры тела до 38,00С, общую слабость, потливость, снижение аппетита, головные боли, носовые кровотечение, появление высыпаний на конечностях. Общий анализ крови: эритроциты 2,4х1012/л, Нв 64 г/л, тромбоциты 12,0х109/л, лейкоциты 1,8х109/л: бластные клетки 34, сегм.12, мон.4, лимф.50, СОЭ 48 мм/час. Объясните, почему у пациента развилась панцитопения? Острый лейкоз (замещение нормальных

ростков на бластные)

15.Мужчина Т. 34 лет, наблюдается у врача с диагнозом «Хронический миелоидный лейкоз, развернутая стадия». В течение последних 2-3 месяцев стали увеличиваться периферические лимфатические узлы, селезенка, присоединились носовые, десневые кровотечения. Общий анализ крови: эритроциты 3,1х1012/л, Нв 83 г/л, тромбоциты 27,0х109/л, лейкоциты 18,6х109/л: бластные клетки 20, миел.5, метамиел.8, пал.5, сегм.34, баз.8, эозин.7, лимф.13, СОЭ 28 мм/час. Объясните, о чем свидетельствуют изменения со стороны клинических и гематологических данных? Бластный криз

(терминальная стадия)

16.Женщина С. 59 лет, поступила в клинику с жалобами на боли в ребрах, по ходу позвоночника, снижение аппетита, похудание на 4-5 кг за 2 месяца, общую слабость, головные боли. В анамнезе частые простудные заболевания. Объективно: кожные покровы и видимые слизистые оболочки бледные, геморрагий на коже нет. Внутренние органы без особенностей. В общем анализе крови резкое ускорение СОЭ. Гиперпротеинемия. Выявлены патологические иммуноглобулины класса G. Предположите, с какими клетками связаны патологические иммуноглобулины?

плазматические клетки, множественная миелома

17.Мужчина Ч. 70 лет, находится в стационаре с жалобами на увеличение подчелюстных, подмышечных лимфатических узлов, похудание, общую слабость. Объективно: кожные покровы бледные с желтушным оттенком, пальпируются практически все группы периферических лимфоузлов, диаметром от 0,5 до 3,0 см, плотноватой консистенции, малоподвижные, безболезненные. Общий анализ крови: эритроциты 3,4х1012/л, Нв 98 г/л, тромбоциты 105,0х109/л, лейкоциты 78,6х109/л: сегм.12, мон.2, лимф.86, СОЭ 14 мм/час. Объясните, с чем связан абсолютный лимфоцитоз у пациента? Хронический лимфолейкоз, обусловлены пролиферацией,

накоплением зрелых лимфоцитов, принадлежащих к малой субпопуляции СД5+ В-лимфоцитов

18.Больной Ф. 68 лет, находится в клинике с жалобами головные боли, шум в ушах, бессонницу, покраснение кончиков пальцев рук, боли в левом подреберье, снижение аппетита. При осмотре: пониженного питания, гиперемия лица и кончиков пальцев рук, спленомегалия. В общем анализе крови эритроцитоз, тромбоцитоз, лейкоцитоз. Интерпретируйте, какими изменениями в костном мозге объясняется панмиелоз у пациента? Гиперплазия всех ростков костного мозга

19.Мужчина 74 лет, доставлен по линии скорой помощи в палату интенсивной терапии. Состояние больного тяжелое, обусловленное клиническими синдромами тканевой гипоксии, фуникулярного миелоза. В общем анализе крови: гемоглобин 42 г/л, эритроциты 1,2х1012/л, средний объем эритроцитов 108fl, среднее содержание гемоглобина в эритроците 34 pg, ретикулоциты 0,1%, тромбоциты 70,0х109/л, лейкоциты 2,8х109/л: сегм.60%, баз.3%, эоз.1%, лимф.36%, СОЭ 8 мм/час. Дефицитом какого вещества обусловлены изменения среднего объема эритроцитов и среднего содержание гемоглобина в эритроците? Витамин В12

20.Пациентка 30 лет, обратилась к врачу с жалобами на общую слабость, шум в ушах, бессонницу, ухудшение памяти, одышку и сердцебиение при незначительной физической нагрузке. Объективно: общее состояние среднетяжелое, бледность кожи и видимых слизистых оболочек с желтушным оттенком, язык гиперемирован, сосочки атрофированы. Со стороны общего анализа крови: гемоглобин 85 г/л, эритроциты 2,0х1012/л; лейкоциты 3,3х109/л, средний объем эритроцитов 100 fl, среднее содержание гемоглобина в эритроцитах 36,0 pg, тромбоциты 100,0х109/л. Дефицит какого вещества в организме имеется у данной пациентки?

Дефицит фолиевой кислоты

21.Девушка 18 лет, обратилась к врачу с жалобами на появление мелкоточечных высыпаний, кровоподтеков на коже нижних конечностей, ограничение движения в правом коленном суставе. Объективно: Кожные покровы обычной окраски, на коже нижних конечностей имеются мелкоточечные петехиальные высыпания, не выступающие над уровнем кожи, безболезненные, кровоподтеки не выступающие над уровнем кожи. Левый коленный сустав увеличен в объеме, гиперемирован, резко болезнен. Дефицит какого фактора свертывания крови привел к геморрагическому синдрому?

фактора Виллебранда в сочетании со снижением фактора VIII:C

22.Больная 28 лет, находится в отделении гематологии с геморрагическим синдромом в виде частых носовых, десневых и маточных кровотечений, петехиальных кровоизлияний и подкожных кровоподтеков на коже конечностей. Объективно: на коже конечностей и туловища имеются мелкоточечные высыпания, не выступающие над уровнем кожи, не исчезающие при надавливании и кровоподтеки. Внутренние органы без особенностей. Спленомегалии нет. Общий анализ крови: эритроциты 4,0х1012/л, Нв 124 г/л, тромбоциты 30,0х109/л, лейкоциты 7,0х109/л: пал.2, сегм.62, мон.2, баз.1, эозин.2, лимф.31, СОЭ 18 мм/час. Предположите, возможную причину развития геморрагического синдрома? Идиопатическая тромбоцитопеническая

пурпура/Первичная иммунная тромбоцитопения

23.Больной 42 лет, поступил в клинику с жалобами на носовые, десневые кровотечения, появление высыпаний на конечностях, повышение температуры тела до 38,50С, снижение аппетита, потливость. Объективно: общее состояние тяжелое, пониженного питания, на коже конечностей и туловища имеются множественные петехиальные кровоизлияния. Гепатоспленомегалии нет. Общий анализ крови: эритроциты 2,1х1012/л, Нв 64 г/л, тромбоциты 18,0х109/л, лейкоциты 26,0х109/л: бластные клетки 28, пал.8, сегм.42, лимф.22, СОЭ 45 мм/час. Какой вариант острого лейкоза можно предположить? - Острый промиелоцитарный лейкоз-М3 вариант

23-миелобластный лейкоз

24.Женщина 54 лет, обратилась к врачу с жалобами на чувство тяжести и боли в левом подреберье, снижение аппетита, похудание, общую слабость, утомляемость, чувство общего дискомфорта.Объективно: кожные покровы и видимые слизистые оболочки бледные. Живот увеличен, ассиметричен за счет спленомегалии. Общий анализ крови: эритроциты 3,4х1012/л, Нв 92 г/л, тромбоциты 650,0х109/л, лейкоциты 140,0х109/л: промиел.5, миел.8, метамиел.14, пал.10, сегм.44, баз.4, эозин.5, лимф.10, СОЭ 20 мм/час. Определите изменение в костном мозге, характерное для уточнения клинического диагноза? -Ds: хронический миелоидный лейкоз. миелоидная

гиперплазия, инфильтрация к/м: преимущественно незрелые формы (промиелоциты, миелоциты, метамиелоциты). Жир вытесняется за счет гиперплазии.

Применение:

1.Девушка Ч. 20 лет, обратилась в женскую консультацию с жалобами на обильные, длительные менструации по 8-10 дней ежемесячно в течение последних 2-х лет, общую слабость, утомляемость, головокружение, шум в ушах, одышку и сердцебиение при физической нагрузке, сухость кожи, ломкость ногтей, выпадение волос. В общем анализе крови: Нв 64 г/л, эритроциты 3,0х1012/л, MCV 50 fl, MCH 21

pg, ретикулоциты 2,5% тромбоциты 420,0х109/л, лейкоциты 3,0х109/л: сегм.60, баз.3, эоз.1, лимф.36, СОЭ 8 мм/час. Что необходимо определить для уточнения диагноза?

ферритин + ОЖСС, железо сыворотки.

2.Мужчина Ц. 48 лет, находится в стационаре с жалобами на общую слабость, головные боли, одышку и сердцебиение при физической нагрузке, носовые кровотечение, появление кровоизлияний на коже конечностей. В общем анализе крови: Нв 60 г/л, эритроциты 1,4х1012/л, MCV 86 fl, MCH 29 pg, ретикулоциты 0,1% тромбоциты 12,0х109/л, лейкоциты 1,2х109/л: сегм.23, мон.2, лимф.75, СОЭ 54 мм/час. Ферритин

всыворотке крови 650 мкг/л. Какое исследование нужно проводить для уточнения клинического диагноза?

Апластическая анемия сверхтяжелая форма.Трэпанобиопсия костного мозга(гистология,миелограмма)

3.Пациент Ю. 29 лет, находится в клинике в тяжелом состоянии, тяжесть состояния обусловлена анемическим, геморрагическим синдромами. В общем анализе крови панцитопения. На трепанобиопсии аплазия костного мозга с замещением жировой тканью. Выберите комбинированную иммуносупрессивную терапию?

Апластическая анемия.Антитимоцитарный глобулин лошадиный(АТГ)/Антилимфоцитарный глобулин +Циклоспорин А(10 мг/кг)

4.Больной Я. 36 лет, обратился к врачу с жалобами на слабость, головокружение, шум

вушах, одышку и сердцебиение при незначительной физической активности, носовые и десневые кровотечения, появление мелкоточечных кровоизлияний и кровоподтеков на туловище и конечностях. В общем анализе крови: Нв 72 г/л, эритроциты 1,8х1012/л, MCV76 fl, MCH 27pg, ретикулоциты 0,1% тромбоциты 7,8х109/л, лейкоциты 1,9х109/л: сегм.32, эозин.4, баз.2, мон.8, лимф.54, СОЭ 46 мм/час. Под прикрытием чего можно проводить трепанобиопсию?

Тромбомассы(тромбоконцентрата)

5.Мужчина А. 69 лет, находится в палате интенсивной терапии с жалобами на общую слабость, одышку и сердцебиение при малейшей физической нагрузке, головокружение, головные боли, чувство пользания мурашек по всему телу, шаткость походки. В общем анализе крови: Нв 58 г/л, эритроциты 1,0х1012/л, MCV 118 fl, MCH 38 pg, ретикулоциты 0,1% тромбоциты 82,0х109/л, лейкоциты 3,2х109/л: сегм.64, эозин.2, мон.6, лимф.38, СО2446 мм/час. Что необходимо определить для уточнения диагноза?

Концентрацию вит. В12 в сыв. кр.

6.На основании клинических, лабораторных и эндоскопических исследований больному Б. 74 лет, выставлен клинический диагноз «Витамин В12 дефицитная анемия, тяжелая степень. Фуникулярный миелоз. Хронический атрофический гастрит». Выберите оптимальную суточную дозу витамина В12?

500 – 1000 мкг цианокобаламина 1 раз в сутки. Минимальный курс лечения – 6 недель, после поддерживающая терапия в/м 200 – 400 мкг цианокобаламина 2 – 4 раза в месяц.

7.Женщина Г. 32 лет, обратилась в женскую консультацию по поводу задержки беременности в течение 2-х последних месяцев. На УЗИ матки обнаружен плод, приблизительно 8-9 недель. Жалобы на общую слабость, утомляемость, шум в ушах, затемнение перед глазами при перемене положения тела. Общее состояние относительно удовлетворительное, бледность кожи и видимых слизистых оболочек с желтушным оттенком. Со стороны внутренних органов без особенностей. При исследовании периферической крови обнаружены: Нв 98 г/л, эритроциты 2,7х101 2/л, MCV 104 fl, MCH 36 pg, ретикулоциты 0,1% тромбоциты 90,0х109/л, лейкоциты 3,2х109/л: сегм.60, баз.3, эоз.1, лимф.36, СОЭ 18 мм/час. Что необходимо определить для уточнения диагноза?

уровень фолиевой кислоты сыв. кр.+стернальная пункция

8.Мужчина Д. 46 лет, доставлен в клинику в тяжелом состоянии. Со слов родственников, страдает хроническим алкоголизмом. Питается не регулярно, ведет бродяжнический образ жизни. Объективно: бледность кожи с желтушностью, иктеричность склер, язык гиперемирован, атрофия сосочков. В общем анализе крови макроцитоз, гиперхромия эритроцитов, панцитопения, гипербилирубинемия за счет непрямой фракции. Назначение какого препарата показано для оказания медицинской помощи?

фолиевой кислоты 5-10 мг/сут

9.Пациентка Ж. 37 лет, обратилась к врачу-гематологу с жалобами на желтушность кожи, изменение цвета мочи, боли в области левого подреберья, общую слабость, утомляемость, головокружение, одышку и сердцебиение при незначительной физической активности. Объективно: желтушность кожи и видимых слизистых оболочек, спленомегалия. Общий анализ крови: Нв 64 г/л, эритроциты 1,9х1012/л, MCV86fl, MCH 30pg, ретикулоциты 6,5% тромбоциты 350,0х109/л, лейкоциты 12,0х109/л: пал.6, сегм.52, баз.2, эоз.4, мон.3, лимф.33, СОЭ 28 мм/час. Общий

билирубин 78,2 ммоль/л: прямая фракция 12,8 ммоль/л, непрямая фракция 65,4 ммоль/л. Какое исследование нужно провести для уточнения диагноза?

проба Кумбса прямая

10.Женщина И. 43 лет, жалуется на общую слабость, утомляемость, одышку и сердцебиение при незначительной физической нагрузке, желтушность кожи, изменение цвета мочи в виде крепко заваренного чая. Лабораторно: нормоцитарная, нормохромная анемия тяжелой степени, ретикулоцитоз, гипербилирубинемия за счет непрямой фракции. Какую пробу необходимо провести для определения полных тепловых гемолизинов (аутоантител) против эритроцитов?

прямая сахарозная проба, часто выдает + результат при агрегатгемагглютинационном тесте или кислотная проба( проба Хема)

11.Женщине Е. 39 лет, в условиях стационара выставлен клинический диагноз «Аутоимммуная гемолитическая анемия с неполными тепловыми агглютининами тяжелой степени». Какую лекарственную группу препаратов необходимо использовать для проведения 1-й линии патогенетической терапии?

ГКС

12.Больной У. 18 лет, жалуется на желтушность кожи, темную мочу, боли в левом подреберье, общую слабость, одышку и сердцебиение при небольшой физической активности. Состоит на учете с диагнозом врожденная гемолитическая анемия. Объективно: общее состояние тяжелое, на левой кисти отсутствует мизинец. Желтушность кожи и слизистых оболочек. Живот увеличен за счет спленомегалии. В общем анализе крови: эритроциты 1,7х1012/л, Нв 54 г/л, МСV 74 fl, MCH 28 pg, ретикулоциты 16%, тромбоциты 310,х109/л, лейкоциты 5,8х109/л: пал.3, сегм.72, эозин.3, баз.2, мон.1, лимф.19, СОЭ 7 мм/час. Какую тактику лечения необходимо запланировать?

Спленэктомия

13.Мужчина И. 38 лет, доставлен в палату интенсивной терапии с жалобами на сильные боли в эпигастральной области, изменение цвета мочи в виде мясных помоев, преимущественно по утрам, желтушность кожи, общую слабость, утомляемость. В общем анализе крови панцитопения, ретикулоцитоз. Определение какого диагностического маркера необходимо запланировать пациенту для уточнения клинического диагноза?

DAF(CD55),MIRl(CD59)

14.Больной Ш. 18 лет, обратился в поликлинику с жалобами на общую слабость, утомляемость, головные боли, одышку и сердцебиение при незначительной физической нагрузке. Из анамнеза, находится учете у гематолога с наследственной анемией, систематически получает стационарное лечение в виде донорских эритроцитов. ОА крови: эритроциты 2,7х1012/л, Нв 62 г/л, МСV 68 fl, MCH 22 pg. Содержание ферритина в сыворотке крови 640,0 мкг/л. Глобиновые фракции гемоглобина: 2 пары α и 2 пары ß. Какое исследование позволяет уточнить клинический диагноз? Определение фермента протопорфирина

15.Пациент Х. 27 лет, находится на диспансерном учете у врача с клиническим диагнозом «ß-талассемия». Систематически получает гемотрансфузий отмытых эритроцитов. Объективно: состояние тяжелое, бледность кожи и видимых слизистых оболочек с коричневым оттенком. Со стороны внутренних органов систолический шум во всех точках выслушивания клапанов сердца, увеличение селезенки. ОАК: эритроциты 2,6х1012/л, Нв 61 г/л, МСV 71 fl, MCH 20 pg. Какое исследование необходимо провести для решения вопроса о хелаторной (железовыводящей)

терапии? - определение уровня ферритина

16.Пациент А. 42 года, находится на учете у врача гематолога. Его часто беспокоят, спонтанные носовые, десневые кровотечения, появление мелкоточечных геморрагий и кровоподтеков на коже туловища и конечностей. Общая слабость, утомляемость, головокружение практически у него постоянные. В общем анализе крови: панцитопения, относительный лимфоцитоз, ускорение СОЭ. На трепанобиопсии костного мозга аплазия костного мозга с замещением кроветворных клеток жировой тканью. Планируется комбинированная иммуносупрессивная терапия. При каком уровне тромбоцитопении должна назначается сопроводительная терапия донорскими тромбоцитами? - при тромбоцитопении менее 20х109/л

17.У женщины М. 28 лет, обнаружены частые носовые, десневые кровотечения, увеличения продолжительности менструальных кровотечений, мелкоточечные геморрагии на коже конечностей и туловища. Какое исследование нужно провести для уточнения диагноза? - подсчет тромбоцитов (в ОАК) и анализ на наличие

Антитромбоцитарных антител

18.Больная Н. 35 лет, обратилась в гематологический стационар с клиникой геморрагического синдрома. На основании клинико-гематологических показателей

выставлен клинический диагноз «Первичная иммунная тромбоцитопения, впервые диагностированная, градация геморрагического синдрома III». Какой лекарственный препарат нужно использовать для проведения 1-й линии патогенетической терапии?

- преднизолон/дексаметазон

19.Женщина К. 57 лет, обратилась к врачу с жалобами на эпизоды носовых, десневые кровотечений, появление спонтанных мелкоточечных высыпаний и кровоподтеков на конечностях и туловища. Эти симптомы беспокоят в течение последних 1,5-2лет. К врачам не обращалась. Внутренние органы без особенностей. В общем анализе крови изолированная тромбоцитопения 40,0х109/л. Консилиумом врачей принято решение проводить выжидательную тактику. Использование какого гемостатического препарата целесообразно при повторении кровотечений? эпсилон-

аминокапроновая кислота (ЭАКК), транексамовая кислота

20.У мужчины Т. 29 лет обнаружена клиника острого гемартроза левого голеностопного сустава. Определение какого фактора свертывания крови необходимо для диагностирования гемофилии А? -VIII фактор

21.Мужчина Л. 29 лет, жалуется на сильные боли в области правого бедра, ограничение движение в коленных, голеностопных суставах. Объективно: верхняя и средняя треть правого бедра увеличена в объеме, при пальпации болезненная, кожа гиперемирована, горячая на ощупь. Коленные, голеностопные суставы деформированы, движения в них ограничены. Общий анализ крови без патологии. АЧТВ удлинено. Какие факторы свертывания крови необходимо определить для решения вопроса о заместительной терапии? - VIII и IX фактор

22.Пациент З. 17 лет, обратился к врачу с жалобами на появление множественных папулезно-пурпурных геморрагических кровоизлияний на коже нижних конечностей, расположенных симметрично, сливающихся между собой, местами выступающие над уровнем кожи, временами сопровождающиеся кожным зудом. У него также отмечается летучая артралгия крупных нагрузочных суставов. Что необходимо определить для оценки состояния гемостаза? - определение агрегации

и адгезии тромбоцитов (агрегатограмма)

23.Больной Е. 26 лет, жалуется на появление множественных высыпаний на коже нижних конечностей, сливного характера, боли в голеностопных суставах. Объективно: общее состояние средней степени тяжести. На коже нижних конечностей имеются множественные папулезные геморрагические кровоизлияния. Голеностопные суставы увеличены в объеме, с ограничениями движения в них. В общем анализе крови лейкоцитоз, ускорение СОЭ. Выставлен диагноз «Геморрагический васкулит». Выберите тест, контролирующий дозу гепарина во время лечения? АЧТВ

24.Женщина О. 46 лет, жалуется на наличие множественных высыпаний на конечностях, сливного характера. Заболела остро, через 2 недели после переохлаждения. Объективно: на коже конечностей имеются множественные мелкоточечные яркокрасного цвета геморрагические кровоизлияния, расположенные симметрично, сливающие между собой, с участками атрофических язвочек. Внутренние органы без особенностей. В общем анализе крови: эритроциты 4,0х1012/л, Нв 124 г/л, тромбоциты 475 х109/л, лейкоциты 12,4х109/л: пал.3, сегм.69, эозин.5, баз.4, лимф.19, СОЭ 35 мм/час. Врачом выставлен диагноз «Геморрагический васкулит». Применение какого препарата показано для проведения базисной терапии? гепарин

300-400 Ед/кг п/к живота возле пупка, через каждые 4 часа или фраксипарин 0,6- 0,8 мл п/к возле пупка через каждые 12-24 часа.

25.Больной Р. 25 лет, находится в стационаре в течение 15 суток с клиническим диагнозом «Геморрагический васкулит, генерализованный кожный синдром, осложненный атрофическими язвочками, высокая степень активности». На фоне базисной терапии отмечается появления новых геморрагий на коже с увеличением площади атрофических язвочек. Консилиумом врачей решено активизировать базисную антикоагулянтную терапию. Использование какого препарата позволяет решить поставленную задачу? Свежезамороженная плазма

26.Мужчина К. 48 лет, жалуется на общую слабость, потливость, повышение температуры тела до 38,00С-39,00С, снижение аппетита, периодически носовые и десневые кровотечения. В общем анализе крови: эритроциты 3,4х1012/л, Нв 94 г/л, тромбоциты 17,0х109/л, лейкоциты 32,4х109/л: бластные клетки 20, сегм.31, лимф.49, СОЭ 50 мм/час. Какое диагностическое вмешательство нужно использовать пациенту для уточнения его диагноза? Стернальная пункция: миелограмма (исследование

клеток костного мозга): бластная инфильтрация костного мозга (20% и выше) и угнетение эритроидного и мегакариоцитарного ростков кроветворения.

27.Мужчина П. 40 лет, находится в отделении онкогематологии. На основании клиникогематологических данных выставлен клинический диагноз «Острый лейкоз, миелобластный вариант, I стадия». Планируется полиохимиотерапия – индукция ремиссии, переведен в палату интенсивной терапии. Какое исследование нужно провести для уточнения клональности развития болезни?- Цитогенетическое исследвание

28.У пациента Э. 39 лет, имеется клинические проявления геморрагического, анемического и интоксикационного синдромов. В общем анализе крови и в пунктате костного мозга появились недифференцируемые бластные клетки более 20%. Для уточнения морфологического варианта заболевания, какое исследование нужно проводить? Цитохимическое исследование

29.Женщина Ч. 57 лет, обратилась к врачу с жалобами на чувство тяжести и боли в левом подреберье, снижение аппетита, похудание на 3-4 кг за 2 месяца, общую слабость, периодически повышение температуры тела до 37,50С. В объективном статусе определяется спленомегалия. Общий анализ крови: эритроциты 3,6х1012/л, Нв 115 г/л, тромбоциты 567,0х109/л, лейкоциты 156,0х109/л: промиел.5, миел.12, метамиел.10, пал.17, сегм.36, баз.3, эозин.2, лимф.15, СОЭ 24. Какое исследование нужно провести для уточнения диагноза? -стернальная пункция: миелограмма, цитогенетическое

исследование к/м

30.Женщина Ю. 38 лет, поступила в клинику с жалобами на боли в области левого подреберья, усиливающиеся после физической нагрузки, снижение аппетита, чувство общего дискомфорта. Объективно: кожные покровы и видимые слизистые оболочки бледные, чистые. Периферические лимфатические узлы не пальпируются. Живот увеличен в объеме за счет увеличенной селезенки. В общем анализе крови: анемия легкой степени, гиперлейкоцитоз со сдвигом до промиелоцитов, миелоцитов. Какое исследование нужно проводить, чтобы определить цитогенетическое нарушение? Так наверное пойдет. Выберите форму цитогенетического исследования, направленное на уточнения диагноза? филадельфийская хромосома РН - T(9;22)

31.Мужчина Ы. 75 лет, жалуется на общую слабость, тяжесть в голове, головокружение, кожный зуд (особенно после приема водных процедур), чувство жжения и

покалывания кончиков пальцев рук. Объективно: пониженного питания, гиперемия кожи и видимых слизистых оболочек, спленомегалия. Общий анализ крови: эритроциты 7,4х1012/л, Нв 213 г/л, тромбоциты 657,0х109/л, лейкоциты 28,6х109/л: сегм.64, баз.3, мон.3, лимф.30, СОЭ 1 мм/час. Что необходимо провести в первую очередь для уточнения клинического диагноза? - трепанобиопсия костного мозга

32.У пациента Ш. 70 лет появились боли в позвоночнике. В общем анализе крови ускорение СОЭ. Общий белок 126 г/л, М-градиент 38%. Какое исследование прежде всего нужно провести для уточнения клинического диагноза? - аспирационная

биопсия или трепанобиопсия (миелограмма)

33.Мужчина А. 75 лет, состоит на учете у врача. Клинический диагноз «Множественная миелома». Несколько дней тому назад появились сильные боли в позвоночнике, усиливающиеся при малейшем движении. Объективно: состояние тяжелое, положение лежачее, при перемене положения тела боль в позвоночнике усиливается. На компьютерной томографии компрессионный перелом позвоночника. Какой из нижеперечисленных препаратов целесообразнее применить на фоне химиотерапии для ингибирования резорбции костной ткани? Бисфосфанаты

34.Пациент К. 58 лет, обратился в стационар с жалобами на увеличение подчелюстных, подмышечных, паховых лимфатических узлов, чувство тяжести и боли в подреберьях, снижение аппетита, похудание, повышение температуры тела до 37,8- 38,00С, общую слабость, утомляемость. Лечащим врачом выставлен диагноз «Хронический лимфоцитарный лейкоз». Для определения клеточной популяции моноклональных лимфоцитов какое исследование необходимо провести? -

иммунофенотипирование костного мозга

35.Женщина А. 64 лет. Наблюдается у врача с диагнозом «Хронический лимфоцитарный лейкоз». При поступлении состояние тяжелое: диффузное увеличение всех групп периферических лимфатических узлов в больших размерах, плотной консистенции; спленогепатомегалия. В ОАК: эритроциты 2,4х1012/л, Нв 62 г/л, тромбоциты

40,2х109/л (8/1000), лейкоциты 87,0х109/л: сегм.8, мон.5, лимф.87, СОЭ 20 мм/час.

Клетки Гумпрехта - Боткина 9/100. Какой из нижеперечисленных препаратов

целесообразно применить для проведения монохимиотерапии? - циклофосфан или

флударабин или лейкеран

36.Пациентка И. 29 лет, доставлена в реанимацию с множественными геморрагиями по

всему телу, уменьшением суточного диуреза. 5 дней тому назад было произведено искусственное прерывание беременности в сроке 24 недель. Объективно: на коже туловище и конечностей имеются множественные геморрагии от мелкоточечных до объемных подкожных кровоизлияний. Общий анализ крови: эритроциты 2,4х1012/л,

Нв 68 г/л, MCV 71 fl, MCH 24 pg, тромбоциты 34,0х109/л, лейкоциты 14,8х109/л:

метамиел.5, пал.8, сегм.74, мон.1, лимф.12, СОЭ 35 мм/час. Для уточнения ДВСсиндрома какое исследование необходимо проводить в первую очередь? -

определение продуктов паракоагуляции (сск, гемостазиограмма)

37.Пациентка 30 лет. Обратилась с жалобами на общую слабость, головокружение, одышку при физической нагрузке. В общем анализе крови гиперхромная, макроцитарная анемия тяжелой степени. На УЗИ определяется беременность 22-23 недели. Уровень какого вещества в крови наиболее целесообразно определить в данном случае? фолиевую кислоту

38.Больной 70 лет, обратился к врачу с жалобами на общую слабость, боли в позвоночнике, рёбрах, усиливающиеся при движениях. В общем анализе крови: нормохромная, нормоцитарная анемия, тромбоцитопения, ускоренное СОЭ. В биохимическом анализе крови высокий уровень общего белка. В миелограмме увеличенное количество плазматических клеток больше 10%. На рентгенограмме костей черепа, таза и ребер: множественные остеолитические очаги. Выберите, какой вид лабораторного анализа необходимо сделать для уточнения диагноза?

электрофорез белка сыворотки крови и мочи с последующей иммунофиксацией

39.Больная 28 лет, обратилась к врачу с жалобами на беспричинную общую слабость, утомляемость, раздражительность, головные боли, головокружение, шум в ушах, одышку и сердцебиение при физической нагрузке, желание есть кислые продукты, нравится запах лакокрасочных изделий. Объективно: кожные покровы и видимые слизистые оболочки бледные, сухость кожи, ногти плоские, сплюшенные. В общем анализе крови микроцитарная, гипохромная анемия тяжелой степени. Какое исследования крови необходимо провести в первую очередь для уточнения диагноза?

железо сыворотки крови

40.Больной 42 лет, поступил в клинику с жалобами на носовые и десневые кровотечения, появление мелкоточечных кровоизлияний и кровоподтеков на туловище, общую слабость, головокружение, шум в ушах, одышку и сердцебиение при незначительной физической активности. В общем анализе крови: гемоглобин82 г/л, эритроциты 2,4х1012/л, тромбоциты 7,8х109/л, лейкоциты 1,9х109/л: сегм.32, эозин.4, баз.2, мон.8,

лимф.54, СОЭ 46 мм/час. На трепанобиопсии: аплазия костного мозга с замещением кроветворных клеток жировой тканью. С целью предупреждения геморрагического синдрома, под прикрытием чего можно проводить комбинированную иммуносупрессивную терапию?:- гемотрансфузии –тромбомасса

41.Больной 20 лет, находится в отделении гематологии с острым гемартрозом правого коленного сустава, хронической артропатией обеих коленных и локтевых суставов. Наблюдается с раннего детства у гематологов с клиническим диагнозом «Гемофилия А». Для определения степени тяжести болезни, какое исследование необходимо провести? Количественное определение VIII фактора свертывания крови

42.Пациентка 55 лет, находится в отделении гематологии в тяжелом состоянии, тяжесть состояния обусловлена гемолитическим кризом и тканевой гипоксией. Со стороны лабораторных данных выявлены: нормоцитарная, нормохромная анемия тяжелой степени, ретикулоцитоз,лейкоцитоз, ускорение СОЭ; осмотическая резистентность эритроцитов снижена; гипербилирубинемия за счет непрямой фракции. Какое исследование необходимо провести пациенту для уточнения клинического диагноза и определения неполных тепловых агглютининов? Прямая проба Кумбса

43.Мужчина 52 лет. Находится на диспансерном учете у врача гематолога с клиническим диагнозом «Хронический лимфоцитарный лейкоз, прогрессирующая форма». В течение последних 3-4 месяцев отмечается опухолевая прогрессия в виде диффузного увеличения всех групп периферических и внутриорганных лимфатических узлов в больших размерах с нарастающей спленогепатомегалией. В общем анализе крови: гиперлейкоцитоз с абсолютным лимфоцитозом, клетки Гумпрехта - Боткина 14/100, анемия, тромбоцитопения. Консилиум врачей решил к химиотерапии подключить таргетную терапию. Какой из нижеперечисленных препаратов нужно использовать для выполнения данного режима терапии? - Мабтера(Ритуксимаб)

Гинекология (160)

ЗАПОМИНАНИЕ

1. Что является показанием для оперативного лечения миомы матки?

субмукозная форма опухоли

нарушение функции соседних органов

гиперполименорея, приводящая к анемии

+ величина миомы, превышающая размеры матки при беременности 12-14 недель;

рост миомы (за год 4-5 недельному сроку берем)

проверено

2.К злокачественным опухолям яичников относится:

Эпителиальные (Бреннера.Карцинома яичника)стромальные.липидоклеточные. серозная цистаденокарцинома, инвазивная слизеобразующая карцинома.

проверено

3. Гистеросальпингографию производят с целью установления. проходимости маточных труб,спайки,окклюзии,пороки развития,онкология.

проверено

4. Абсолютным ультразвуковым признаком эктопической беременности является: Эктопически расположенное плодное яйцо с живым эмбрионом,отсутствие плодного яйца в полости матки

проверено

5. Наиболее частая локализация плодного яйца при эктопической

беременности: маточная труба и трубный угол полости матки.

проверено

6. Какие основные изменения происходят в периоде полового созревания в организме девочек? активизация гормональной функции яичников-начало менструаций.развитие молочных желез,рост волос,акне,изменение фигуры

проверено

7. Назовите средства контрацепции, предохраняющие от инфекций, передаваемых половым путем: барьерные(презерватив и фемидом)

проверено

8. Что относится к барьерным методам контрацепции: Мужские: презерватив Женские:

• диафрагма • шеечный колпачок • женский презерватив

проверено

9. Причиной повышенной продукции пролактина может быть: Доброкачественные новообразования гипофиза (пролактиномы и аденомы).

Гипотиреоз (сниженная функция щитовидной железы).

Синдром поликистозных яичников.

Болезни почек (ХПН) и печени.

Дисфункции коры надпочечников.

Постоянный стресс.

Патологии головного мозга,кормление грудью,

проверено

10. Тяжесть климактерического синдрома определяют по:

по количеству приливов(Легкая форма— не более 10, Средняя степень тяжести— 10—20 «приливов» в сутки, Тяжелая форма—более 20), по наличию бессонницы, по степени трудоспособности (инд куппермана)

проверено

11. Бесплодие - отсутствие беременности при наличии регулярной половой жизни без применения контрацептивов в течение? от 6-12 мес и более

проверено

12. До скольки лет формирование вторичных половых признаков у девочек считается преждевременным? до 7-8 лет

проверено

13.Скопление гноя в маточной трубе — это? Пиосальпинкс

14.Восходящая гонорея — это поражение? Эндометрит/ сальпингит/ оофорит/ сальпингоофорит/ параметрит/ пельвиоперитонит

проверено

15. Главным эстрогенным гормоном в организме женщины в период постменопаузы является Эстрон

проверено

16.Укажите, что является частой причиной женского бесплодия: Трубное бесплодие, спкя, эндометриоз Спаечные процессы в маточных трубах(вследствии ИППП), Удаление маточных труб проверено

17.При анемической формы I-степени апоплексии яичника внутрибрюшная кровопотеря не превышает: 100-150 мл проверено

18.Назовите недостатки внутриматочной спирали: Необходимость медицинских манипуляций при введении и удалении ВМС и осложнения(воспаление, перфорация, кровотечения)

отсутствие защиты от ипп

изменения в мц

возможность воспаления в первые недели после установки

ПОНИМАНИЕ

1.На прием по бесплодию обратилась пациентка 30 лет с желобами на отсутствие беременности в течение 4 лет. ИЗ АНАМНЕЗА: менструальная функция не нарушена, гинекологические заболевания отрицает. Неоднократно проходила гормональное обследование, УЗИ органов малого таза. Патологии не выявлено. Посткоитальный тест положительный. Спермограмма мужа в норме. ОБЪЕКТИВНО: общее состояние удовлетворительное, вес 60 кг, рост 167 см. ГИНЕКОЛОГИЧЕСКИЙ СТАТУС: влагалище без особенностей, шейка матки без видимой патологии, наружный зев точечный, матка в ретрофлексии, не увеличена, безболезненная. Придатки с обеих сторон не определяются. Какой фактор бесплодия у данной пациентки наиболее вероятен? - лапароскопия, хромосальпингография иммунологический трубный

2.Пациентка 27 лет обратилась в женскую консультацию с жалобами на отсутствие беременности в течение 3 лет в браке. ИЗ АНАМНЕЗА: менструальная

функция не нарушена. В анамнезе эндоцервицит. Получала местное лечение. ИССЛЕДОВАНИЕ В ЗЕРКАЛАХ: влагалище свободное, слизистая розовая, шейка матки конической формы, в области наружного зева гиперемия, выделения слизистые. БИМАНУАЛЬНОЕ ИССЛЕДОВАНИЕ: матка в anteversio-flexio, не увеличена, безболезненная; придатки не определяются, своды глубокие. Наиболее вероятное носительство инфекции? - хламидийная инфекция

проверено

3.Пациентка 30 лет обратилась в женскую консультацию с жалобами на отсутствие беременности в течение 4 лет в браке. ИЗ АНАМНЕЗА: менструальная функция не нарушена. В анамнезе эндоцервицит. Получала местное лечение. ИССЛЕДОВАНИЕ В ЗЕРКАЛАХ: слизистая влагалища бледно-розовая, шейка матки цилиндрической формы наружный зев точечной формы, гиперемирован, выделения слизистые. БИМАНУАЛЬНОЕ ИССЛЕДОВАНИЕ: матка в anteversio-flexio, не увеличена, безболезненная; придатки не определяются, своды глубокие. Обследована на инфекции передающиеся половым путем: выявлена при ПЦР: Ch. trachomatis. Какой фактор бесплодия чаще всего встречается у женщин с данной инфекцией? - трубноперитонеальное бесплодие

проверено

4.Пациентке 42 лет назначена плановая операция по поводу миомы матки. Назовите наиболее благоприятные дни менструального цикла для проведения плановой гинекологической операции при ДЗ: Миома матка больших размеров: в любой день кроме периода менструации

1-я фаза 5-14 день (если будет вариант в первую неделю после мц то тоже верно)

5.Пациентка 37 лет, с целью подтверждения диагноза «Аденомиоз» назначена гистеросальпингография. В какой период менструального цикла следует провести данное исследование? - на 7-8 день менструального цикла (сразу после окончания менструации)

проверено

6.Пациентке 35 лет выставлен диагноз «Аденомиоз», продолжительность менструального цикла 28 дней. Назовите наиболее вероятный период возникновения болей у больной. - во время менструаций, т.е. с 1 по 7 день МЦ

проверено

7.Пациентка 30 лет обратилась в женскую консультацию с жалобами на отсутствие беременности в течение 4 лет в браке. ИЗ АНАМНЕЗА: менструальная функция не нарушена. В анамнезе эндоцервицит. Получала местное лечение. ИССЛЕДОВАНИЕ В ЗЕРКАЛАХ: слизистая влагалища бледно-розовая, шейка матки цилиндрической формы наружный зев точечной формы, гиперемирован, выделения слизистые. БИМАНУАЛЬНОЕ ИССЛЕДОВАНИЕ: матка в anteversio-flexio, не увеличена, безболезненная; придатки не определяются, своды глубокие. Обследована на инфекции, передающиеся половым путем: выявлена при ПЦР: Ch. trachomatis. Какой фактор бесплодия чаще всего встречается у женщин с данной инфекцией? - трубноперитонеальный

проверено

8.Больная 39 лет доставлена в отделение гинекологии бригадой скорой помощи в связи с жалобами на острые боли внизу живота, рвоту, учащённое мочеиспускание. Последние менструации были три недели назад. При осмотре: живот умеренно вздут,

положительный симптом Щёткина-Блюмберга. Пульс 88 в 1 мин, температура тела 37,70С. При влагалищном исследовании: тело матки плотное, не увеличено, подвижное, безболезненное, справа и спереди от матки пальпируется образование размером 8х8 см тугоэластической консистенции, резко болезненное при смещении; слева придатки не определяются; своды свободные; выделения слизистые. Какой диагноз наиболее вероятен? - перекрут ножки опухоли яичка

+апоплексия яичника

9.Пациентка 30 лет доставлена в отделение гинекологии с жалобами на острые боли внизу живота, больше справа, которые возникли внезапно при подъеме с постели, тошноту и однократную рвоту. Менструальный цикл не нарушен. ОБЪЕКТИВНО: живот умеренно вздут, резко болезненный в правой подвздошной области, симптом Щеткина-Блюмберга положительный. Пульс 90 уд/мин, температура тела 37,2 °С. ПРИ БИМАНУАЛЬНОМ ИССЛЕДОВАНИИ: шейка матки без видимой патологии, тело матки плотное, не увеличено, подвижное, безболезненное. Справа от матки в области придатков определяется округлое образование тугоэластической консистенции, резко болезненное, ограниченно подвижное, размерами до 6 см в диаметре. Слева придатки не определяются, своды свободные, выделения слизистые. Какой диагноз наиболее вероятен? перекрут ножки опухоли яичка проверено

10.Пациентка 27 лет доставлена в отделение гинекологии с жалобами на схваткообразные боли внизу живота, кровянистые выделения из половых путей. ОБЪЕКТИВНО: АД 100/60 мм рт. ст., пульс 90 в 1 минуту, температура тела 37 ºС. Последняя менструация 2 месяца назад. ПРИ БИМАНУАЛЬНОМ ИССЛЕДОВАНИИ: шейка матки без видимой патологии, цианотична, наружный зев пропускает палец. Матка увеличена до 6 недель беременности, размягчена. Придатки с обеих сторон не определяются. Своды глубокие, безболезненные, выделения кровянистые обильные. Какой диагноз наиболее вероятен? - беременность 8 недель, неполный аборт

проверено

11.Пациентка 19 лет поступила в отделение гинекологии с жалобами на резкие боли внизу живота, иррадиирующие в прямую кишку, озноб, тошноту, однократную рвоту, повышение температуры тела до 38,5 °С, Менструальная функция не нарушена. В анамнезе один медицинский аборт. Заболела остро на 5-й день менструального цикла. ОБЪЕКТИВНО: общее состояние средней тяжести, пульс 110–112 в 1 минуту, АД 125/70 мм рт. ст., язык сухой, обложен белым налетом, живот умеренно вздут, положительный симптом Щеткина—Блюмберга. ПРИ БИМАНУАЛЬНОМ ИССЛЕДОВАНИИ: шейка матки слизистая гиперемирована. Тело матки и придатки пальпировать не удается из-за резкого напряжения мышц передней брюшной стенки, задний свод влагалища нависает, резко болезненный. Какой диагноз наиболее вероятен? - острый пельвиоперитонит, абсцесс дугласова пространства

проверено

12.Больная 29 лет доставлена бригадой скорой помощи в отделение гинекологии с жалобами на сильные боли внизу живота, мажущие кровянистые выделения из половых путей, головокружение, слабость. АД=100/60 мм рт.ст., пульс 108 в 1 мин., ритмичный. Последняя менструация 1,5 месяца назад. При влагалищном исследовании: тело матки нормальных размеров, размягчена, шейка матки цианотична, движения за шейку матки резко болезненные, справа от матки пальпируется болезненное опухолевидное образование, размерами до 7 мм в диаметре. Задний свод нависает, болезненный. Какой диагноз наиболее вероятен? - нарушенная правосторонняя трубная беременность

разрыв кисты

13.Больная, 34 года поступила в отделение гинекологии с жалобами на боль внизу живота с иррадиацией в прямую кишку, кровянистые выделения из половых путей, головокружение. Жалобы появились внезапно. Последняя менструация 3 недели назад. Кожные покровы бледные, пульс -110уд/мин., t =36,6 , АД=90/60 мм. рт. ст. Живот напряженный, незначительно болезненный в нижних отделах, симптомы раздражения брюшины слабоположительные. Какой наиболее вероятный диагноз? эндометриоз яичников (боль накануне или во время менструации)

проверено

14.Пациентка 17 лет обратилась на прием к гинекологу с жалобами на отсутствие менструаций. Из анамнеза: росла и развивалась быстрее сверстников, успешно занимается спортом. Детские заболевания отрицает. Половой жизнью не живет. Объективно: рост 158 см, вес 55 кг, широкие плечи, узкий таз, молочные железы гипоплазированы, рост терминальных волос на бедрах, спине, грудине, по белой линии живота, в области подбородка, верхней губы. Осмотр наружных половых органов: развиты правильно,оволосение по мужскому типу. При ректо-абдоминальном исследовании: патологии со стороны гениталий не выявлено. Когда необходимо начинать лечение? Сразу после установки диагноза

проверено

15.Женщина 35 лет, обратилась на прием к гинекологу с целью установления я внутриматочной спирали, в каком случае введение внутриматочного контрацептива противопоказано? - беременность, воспалительные и опухолевые заболевания половых органов, нарушение МЦ, аллергия на ВМК, аномалии развития матки, необъяснимые вагинальные кровотечения

злокачественные новообразования тела или шейки матки

16. Пациентка 62 лет обратилась с жалобами к гинекологу на боли в костях и суставах, позвоночнике, дважды за последний год были переломы конечностей. Из анамнеза: менопауза 11 лет. Страдает артериальной гипертензией III, риск 3. Гинекологическое исследование: слизистая влагалища истончена, атрофична. При бимануальном исследовании тело матки уменьшено в размерах, подвижное, безболезненное.

Придатки с обеих сторон не пальпируются. Своды влагалища свободные. К какому состоянию относятся симптомы, наблюдаемые у данной пациентки? Постменопауза Поздние симптомы климактерического синдрома, постменопаузальный остеопороз

17.Женщина 36 лет, на приеме у гинеколога после медикаментозного прерывания беременности, через сколько дней целесообразно проведение контрольного УЗИ? 7- 10

проверено

18.Женщина 28 лет, на приеме у гинеколога, по поводу контрацепции. Половая жизнь нерегулярная, врач рекомендует метод естественного планирования семьи, к ним относится? Календарный, температурный, цервикальный, прерванный половой акт, половое воздержание, метод лактационной аменореи

19.Женщина 28 лет, на приеме у гинеколога с целью контрацепции, врач назначил прием комбинированных эстроген-гестагенных препаратов. С какого дня менструального цикла начинают прием контрацептивов? 1-го или 5 дня ПРОВЕРЕНО

20.Пациентка 32 лет госпитализирована в гинекологическое отделение на плановое оперативное лечение по поводу придаткового образования. Произведена лапаротомия, при ревизии брюшной полости обнаружена киста правого яичника. Произведено вылущивание кисты.Описание макропрепарата: тугоэластическое, правильной формы образование размером 6х5 см., на разрезе однокамерное, капсула средней толщины, содержимое образования – волосы, сало, хрящи, внутренняя поверхность капсулы – гладкая. Какая киста наиболее вероятна? Дермоидная киста (тератома) ПРОВЕРЕНО

21.Пациентка 32 лет обратилась в женскую консультацию с жалобами на выраженные головные боли, головокружение, сопровождающиеся тошнотой, рвотой, отеком век, лица, раздражительность, появляющиеся за неделю до менструации. ИЗ АНАМНЕЗА: в 10 лет была закрытая черепно-мозговая травма. Менархе в 12 лет, менструации по 5– 6 дней через 28 дней, умеренные, болезненные в первый день. Не замужем, беременностей не было. Головные боли появились около 2 лет назад. ГИНЕКОЛОГИЧЕСКИЙ СТАТУС: шейка матки без видимой патологии, матка не увеличена, плотная, подвижная, безболезненная, придатки с обеих сторон не увеличены, безболезненные, своды глубокие. Какой диагноз наиболее вероятен? ПМС менструальная мигрень ПРОВЕРЕНО

22.Пациентка 27 лет поступила в гинекологическое отделение на плановое оперативное лечение по поводу придаткового образования. Произведена лапаротомия, при ревизии

брюшной полости обнаружена киста правого яичника. Произведено вылущивание кисты. Описание макропрепарата: тугоэластическое, правильной формы образование размером 5х5 см., на разрезе однокамерное, капсула средней толщины, содержимое образования – волосы, сало, хрящи, внутренняя поверхность капсулы – гладкая. Какая киста наиболее вероятна? Дермоидная киста ПРОВЕРЕНО

23.На прием к гинекологу обратилась пациентка 50 лет с жалобами на приливы жара к лицу, шее по 15 раз в день, сердцебиение, раздражительность, плаксивость, нарушение сна, чувство «мурашек» по ночам в руках. Из анамнеза: последняя менструация 10 месяцев назад, до этого в течение года наблюдались задержки менструации до 3 месяцев. Из гинекологических заболеваний: миома матки. ОБЪЕКТИВНО: вес 70 кг, рост 168 см. АД 140/100 мм рт. ст., постоянно принимает гипотензивные препараты. Гинекологический статус: шейка матки без видимой патологии, слизистая влагалища атрофична, выделения слизистые скудные. Матка увеличена до 5 недель беременности, безболезненна, придатки не определяются. К какому состоянию относятся симптомы, наблюдаемые у данной пациентки? Климакс, перименопауза. Климактерический синдром средней степени тяжести ПРОВЕРЕНО

24.Пациентка 62 лет обратилась к гинекологу, с жалобами на боли в костях и суставах, позвоночнике, дважды за последний год были переломы конечностей. Из анамнеза: менопауза 11 лет. Страдает артериальной гипертензией III, риск 3, Н0. Специальное гинекологическое исследование: при исследовании в зеркалах слизистая влагалища атрофична, выделения слизистые скудные. При бимануальном исследовании тело матки уменьшено в размерах, подвижное, безболезненное. Придатки с обеих сторон не пальпируются, безболезненные. Своды влагалища свободные. К какому состоянию относятся симптомы, наблюдаемые у данной пациентки? Постменопауза постменопаузальный остеопороз. ПРОВЕРЕНО

25.Женщина 49 лет, на профилактическом приеме у гинеколога. Жалоб нет. Врач после осмотра заподозрила изменения. Укажите, что относится к предраковым изменениям влагалищной части шейки матки?эрозия ДИСПЛАЗИЯ ШЕЙКИ лейкоплакия с атипией клеток ПРОВЕРЕНО

26.Женщина Д.,29 лет обратилась в женскую консультацию с жалобами на повышение температуры тела до 38,7 С ,общую слабость ,сухость во рту, ЧСС 92 ударов в минуту ,боли внизу живота. В анамнезе, 2- родов и 1- медицинский аборт, 7 дней назад произведен медицинский аборт, выписана домой на следующий день. Живот мягкий,

слегка болезненный над лоном. Симптомов раздражения брюшины нет. Гинекологический осмотр: матка в anteversio-flexio, не увеличена, болезненная, придатки не определяются, своды глубокие, выделения обильные, гноевидные. Какой диагноз наиболее вероятен? Метроэндометрит, пиометра Острый эндометрит(МЭМ) после медицинского аборта, м.б.остатки плодного яйца после мед.аборта ПРОВЕРЕНО

27.На приеме у гинеколога супружеская пара с диагнозом: Бесплодие. Трубноперитонеальная форма. При проведении программы Экстракорпорального оплодотворения на первом этапе проводится? стимуляция суперовуляции под контролем эндокр. и эхогр мониторинга. ПРОВЕРЕНО

28.На приеме у гинеколога женщина 56 лет, с результатами гистологического исследования после проведенной биопсии. Морфологические изменения при краурозе вульвы больше выражены в? в базальной мембране СОЕДИНИТЕЛЬНОЙ ТКАНИ ПРОВЕРЕНО

29.На прием в женскую консультацию обратилась пациентка 40 лет с жалобами на постоянные слизисто-гнойные бели и контактные кровянистые выделения из половых путей. ИЗ АНАМНЕЗА: Менструальная функция не нарушена. В анамнезе 2 родов и 9 медицинских абортов, роды с наложением акушерских щипцов, осложнившиеся разрывом шейки матки. ГИНЕКОЛОГИЧЕСКОЕ ИССЛЕДОВАНИЕ: при осмотре в зеркалах шейка матки с эктопией цилиндрического эпителия, гипертрофирована, деформирована вследствие послеродовых разрывов, наружный зев зияет. При бимануальном исследовании матка и придатки без патологических изменений, параметрии свободны. Какой диагноз наиболее вероятен? эрозия ПСВЕДОЭРОЗИЯ (эндоцервицит шейки матки) ПРОВЕРЕНО

30.Пациентка 17 лет на приеме у детского гинеколога с жалобами на отсутствие развития молочных желез, отсутствие менструаций. Из анамнеза: родилась от вторых срочных родов. Росла и развивалась с опережением сверстников, половой жизнью не живет. Объективно: рост 160 см, вес 55 кг, широкие плечи, узкие бедра. Молочные железы гипопластичные, оволосение выраженное по мужскому типу. Соматической патологии не выявлено. Осмотр наружных половых органов: сформированы правильно по женскому типу, девственная плева кольцевидной формы не нарушена. Ректально: тело матки меньше нормальных размеров, безболезненное, подвижное. По УЗИ – гипоплазия матки 3 степени. Придатки не определяются. Ваш предварительный диагноз? синдром свайера?, дефицит 5-редуктазы,? синдром тестикулярной феминизации? ДИСГЕНЕЗИЯ ГОНАД ПРОВЕРЕНО (надпочечниковая гиперандрогения)

31.Женщина 23 лет, обратилась в ЦСМ к гинекологу с жалобами на редкие скудные менструации, увеличение веса. Из анамнеза: менархе в 12 лет. Менструации по 3–4 дня, через 45–60 дней скудные, безболезненные. Беременностей не было. Объективно: рост 160 см, вес 85 кг, АД 140/100 мм рт. ст., ИМТ28, кожа жирная, угревая сыпь на лице, рост пушковых волос в области подбородка, верхней губы. При бимануальном исследовании: тело матки несколько уменьшено в размерах, безболезненное. При пальпации яичники увеличены, плотные, безболезненные. Своды глубокие. Выделения бели, незначительно. Какова наиболее вероятная причина нарушения менструального цикла? СПКЯ ПРОВЕРЕНО

32.Пациентка 29 лет пришла на прием к гинекологу с жалобами на отсутствие менструации

втечение 2 лет после родов, отеки конечностей, лица, головные боли, общую слабость, Из анамнеза: менархе в 14 лет, менструации до беременности по 4–5 дней, через 28 дней. Роды 2 года назад через естественные родовые пути с ранним гипотоническим кровотечением с переливанием компонентов крови. Грудью не кормила из-за отсутствия лактации. Объективно: рост — 160 см, масса тела — 70 кг, за 2 года набрала15 кг. Пульс 76 уд/мин, АД 90/60 мм рт. ст. Молочные железы гипоплазированы. Гинекологический статус: слизистая влагалища истончена, легко ранима, выделения слизистые скудные. Тело матки нормальных размеров, подвижно, безболезненно. Какому синдрому соответствуют данные?

A.Ашермана

B.Шихана

C.Резистентных яичников

D.Истощения яичников

E.Поликистозных яичников

33.Пациентка, 32 года, на приеме у гинеколога с жалобами на обильные менструации, периодические межменструальные мажущие кровянистые выделения. Менструальный цикл регулярный, 26–28 дней. Объективно: состояние удовлетворительное, по органам и системам — без особенностей. Рост 168 см, масса тела 68 кг, ИМТ24.Узи органов малого таза на 6-й день менструального цикла: тело матки расположено в retroflexio, размеры 45×52×43 мм, структура миометрия однородная, М-эхо — 5,5 мм, неоднородное, по задней стенке визуализируется образование размером 8×9 мм. Структура шейки матки без особенностей, цервикальный канал не расширен, яичники расположены в типичном месте, объем правого 5,1 см3, левого 4,9 см3.Что является причиной кровотечения у данной пациентки? АМК: полип ПРОВЕРЕНО

34.Пациентка 15 лет на приеме у гинеколога, с жалобами на обильные и длительные до 10– 14 дней менструации. Из анамнеза: менструации регулярные через 28–29 дней. Отмечает 2–3 раза в месяц носовые кровотечения, частая кровоточивость десен. Половую жизнь отрицает. Объективно: состояние удовлетворительное. Рост 164 см, масса тела 58 кг, индекс массы тела — 21,5. При гинекологическом осмотре: девственная плева цела. Выделения из половых путей слизистые умеренные. При ректоабдоминальном исследовании: матка нормальных размеров, плотная, безболезненная, придатки с обеих сторон не увеличены. УЗИ органов малого таза без патологии. Наиболее вероятный диагноз? АМК: нарушение коагуляции ПРОВЕРЕНО

35.Пациентка 14 лет, поступила в гинекологическое отделение с жалобами на слабость, головокружение, обильные кровянистые выделения из половых путей в течение 8 дней. Менструации с 13 лет, нерегулярные (через 6-8 недель), обильные, со сгустками, по 8-10 дней, безболезненные. При осмотре: кожные покровы бледные. Пульс 94 удара в минуту, ритмичный, удовлетворительного наполнения и напряжения. АД 100/70мм рт. ст. Гинекологический статус: наружные половые органы развиты правильно, оволосение по женскому типу, девственная плева цела. Выделения из половых путей кровянистые, обильные. При ректоабдоминальном исследовании патологии не обнаружено. В анализе крови гемоглобин 90 г/л. Наиболее вероятный диагноз? Маточное кровотечение пубертатного периода. Анемия средней степени тяжести АМК в ювенильном периоде. Или ювенильное кровотечение. АНЕМИЯ СРЕДНЕЙ СТ ПРОВЕРЕНО

36.Девушка 25 лет, обратилась в гинекологу, с жалобами на недомогание, повышение температуры тела до 39° С, однократный озноб . Заболела остро на второй день после медицинского аборта. При обследовании ЧСС -99 ударов в минуту, бледность кожных покровов, повышение количества лейкоцитов до 10,0 х 10 9 /л, ускорение СОЭ до 35 мм в час. Живот обычной формы, мягкий, болезненный при пальпации над лоном. ПРИ БИМАНУАЛЬНОМ ИССЛЕДОВАНИИ: тело матки в anteversio-flexio, несколько больше нормы, мягковатой консистенции, подвижное, умеренно болезненное. Придатки с обеих сторон не определяются, область их безболезненна. Выделения скудные, кровянистые, с запахом. Какой диагноз наиболее вероятен? Острый эндометрит ПОСЛЕ МЕД АБОРТА ПРОВЕРЕНО

37.Пациентке 28 лет, на приеме у гинеколога, выставлен диагноз Гонорейный кольпит. Наиболее достоверным методом диагностики является? Мазок на флору Бактериологическая и бактериоскопическая диагностика — мазки и посевы из уретры, цервикального канала, влагалища (до применения антибиотиков!). или же пцр

38.Пациентка, Ж., 27 лет. Замужем. Обратилась в поликлинику с жалобами кровянистые выделения, нагрубание молочных желез, тошнота. АД 100/70 мм.рт.ст., пульс-66 уд в мин, ЧД-17. Из анамнеза: 2 беременности, 2 родов, менструальный цикл регулярный. Задержка менструации на 10 день, тест на беременность-слабо положительный. При пальпации: матка размягчено, наблюдается деформация матки, симптомы Гегара слабо выражены. Бимануально и в зеркалах: цианоз слизистых оболочек влагалища и шейки матки. Шейка уплощенная, а в области перешейка отмечается частичное размягчение. Поставьте предположительный диагноз? Эктопическая беременность ПРОВЕРЕНО

39.У пациентки Т., 36 лет, 28-дневный менструальный цикл. При обследовании по тестам функциональной диагностики на 18 день менструального цикла выявлено: симптом «зрачка» +++; КПИ - 65%. Дайте характеристику менструального цикла. Овуляторный ПРОВЕРЕНО

гиперэстрогения (на 18 день симптом зрачка +) ановуляторный МЦ

40.Больная 25 лет обратилась в женскую консультацию с жалобами на высыпания в лобковой зоне, на половых губах, зуд и жжение во влагалище. Заболела 2недели назад. ИЗ АНАМНЕЗА: половая жизнь с 18 лет. Не замужем, часто меняет половых партнеров. ГИНЕКОЛОГИЧЕСКИЙ СТАТУС: на слизистаой оболочке вульвы, влагалища и шейке матки имеются мелкопузырчатые высыпания. Матка не увеличена, безболезненная при пальпации, придатки с обеих сторон не определяются. На ПЦР выявлен ВПГ-2. Какой диагноз наиболее вероятен для данной пациентки? Генитальный Герпес

ПРОВЕРЕНО

41.Родильница 32 лет, вторые сутки после родов, при обходе жалобы на повышение температуры тела до 39°С, озноб, боли внизу живота над лоном, кровянисто-мутные выделения из половых путей с неприятным гнилостным запахом. Объективно: общее состояние средней тяжести, Ps-110 уд в мин. В ОАК - повышение количества лейкоцитов до 15,0х109, СОЭ 55 мм/час. Живот мягкий, болезненный при пальпации. Влагалищное исследование: шейка матки пропускает купол пальца, бледной окраски, матка увеличена до 16 нед беременности, без четких контуров, выделения гноевидные. Какой диагноз наиболее вероятен? Послеродовой эндометрит

ПРОВЕРЕНО

42.Пациентка 30-ти лет обратилась с жалобами на выделение молока из молочных желез, отсутствие менструаций в течение 5-ти месяцев. Роды одни, физиологические, четыре года назад. Объективно: развитие молочных желез нормальное. При бимануальном исследовании обнаружено уменьшение матки и нормальные размеры яичников. На МРТ: патологии головного мозга не обнаружено. Тиреотропный гормон - в пределах нормы. Уровень пролактина в сыворотке крови увеличен. Какой наиболее вероятный диагноз? Гиперпролактемия ПРОВЕРЕНО

43.Пациентка Р., 20 лет, предъявляет жалобы на отсутствие менструаций в течение 7 месяцев. Менструации с 13 лет, нерегулярные, с задержками на 2-3 мес., скудные, безболезненные. С 13 лет отмечает рост волос на задней поверхности бедер, голенях, над губой. При осмотре - гипертрихоз кожи; пигментация и гиперкератоз в подмышечных, паховых областях, под молочными железами; ожирение с равномерным распределением жировой ткани. При бимануальном исследовании: тело матки в anteversio-flexio, нормальных размеров, плотное, подвижное, безболезненное. С обеих сторон пальпируются несколько увеличенные, плотноватые, безболезненные яичники. Каков ваш диагноз? Синдром Штейна-Левенталя (СПКЯ гирсутизм олигоановуляция) ПРОВЕРЕНО

44.Пациентке 35 лет, с целью дифференциальной диагностики, проведена рентгенологическая гистеросальпигногография. На рентгенограмме определяется увеличенная площадь полости матки, деформация и зазубренные края контура полости матки. При контрастировании отмечено заполнение ходов эндометриоидных гетеротопий. Какой диагноз наиболее вероятен? Внутренний эндометриоз(аденомиоз) ПРОВЕРЕНО

45.Девочка 5 лет, на приеме у детского гинеколога в сопровождении материи с жалобами на увеличение молочных желез, кровянистые выделения из половых путей, боли в коленных суставах, периодическую хромоту. Анамнез жизни без особенностей. На осмотре рост 130см, масса 25 кг, имеются пигментные пятна на лице и шее цвета кофе с молоком, не выступающие над поверхностью кожи. Какой синдром у девочки? Преждевременное половое развитие. Гонадотропин-высвобождающий гормон зависимый(ГнВГ) => Синдром МакКьюна-Олбрайта-Брайцева ПРОВЕРЕНО

46.Девочка 7 лет с на приеме у детского гинеколога. Со слов мамы в течение 2-х дней у девочки кровянистые выделения из половых путей. Из анамнеза: родилась в срок, перенесла корь. Развитие вторичных половых признаков началось около 1 года назад. Объективно: рост 139 см, вес 38 кг. Соматическое развитие соответствует 11 летнему возрасту. Молочная железа значительно выдается, вместе с соском и околососковым кружком образует конус, в подмышечной впадине единичные волосы. Осмотр наружных половых органов: развиты правильно, единичные волосы на больших половых губах, девственная плева не нарушена. Выделения из влагалища умеренные, кровянистые. Ректально: тело матки отклонено кпереди, больше возрастной нормы, плотное, безболезненное. Придатки не определяются. Какое состояние у данной пациентки? Преждевременное половое развитие (ППР) изосексуального типа, полная форма ПРОВЕРЕНО

47.На прием к гинекологу в ЦСМ обратилась женщина 34 лет с жалобами на контактные кровянистые выделения. В анамнезе 2 родов, 1 медикаментозный аборт. Замужем. При

осмотре в зеркалах вокруг наружного зева имеется яркокрасное пятно с неправильными очертаниями. Цитологическое исследование показало клетки плоского и цилиндрического эпителия, лейкоциты, эритроциты, койлоциты. Кольпоскопически отмечается смещение зоны трансформации на экзоцервикс. Ваш диагноз? Эктопия шейки матки ПРОВЕРЕНО

48.Пациентке 35 лет, с целью дифференциальной диагностики, проведена рентгенологическая гистеросальпигногография. После процедуры был выставлен диагноз «Эндометриоз тела матки». Какой из перечисленных признаков помог при постановке диагноза? Расширение полости матки и «законтурные» тени . Характерным рентгенологическим признаком эндометриоза на фоне введенного контраста является неправильной треугольной формы полость матки, зубчатость ее контуров или наличие контрастированных канальцев, которые расположены перпендикулярно контурам полости матки. Увеличение продольных размеров,

расширение истмического отдела матки, расширение канала шейки матки, неровные контуры цервикального канала, дефекты наполнения, закругленные углы полости матки являются признаками аденомиоза.!!! Кроме этого, вне контуров полости матки могут наблюдаться различных форм и размеров затеки рентгенконтрастного вещества. ПРОВЕРЕНО

49.Пациентка 16 лет обратилась с жалобами на обильные кровянистые выделения из половых путей после 4 месяцев отсутствия месячных. Из анамнеза: менархе в 13 лет. Менструации не регулярные, последние месячные были 4 месяца назад. Половой жизнью не живет. Гинекологическое исследование не проведено. УЗИ: тело матки обычных размеров, без особенностей. Справа определяется киста яичника, придатки слева без особенностей. Какова наиболее вероятная причина нарушения менструального цикла у пациентки? СПКЯ с метаболическим синдромом ДМК ювенильного периода ПРОВЕРЕНО

50.Пациентка 26 лет обратилась в женскую консультацию с жалобами на отсутствие беременности в течение 5 лет. Из анамнеза: менархе в 14 лет. Менструации по 4–5 дней через 45–60 дней, нерегулярные, безболезненные. Об-но: рост 164 см, вес 90 кг. Гирсутное число по шкале Ферримана—Голлвея равно 9. Молочные железы развиты, мягкие б/б. PV: наружные половые органы развиты правильно, оволосенение по женскому типу. При бимануальном исследовании матка несколько уменьшена в размерах, плотная, подвижная, безболезненная. Придатки четко не пальпируются, область их безболезненна, своды свободны. Базальная температура монофазная. Какова наиболее вероятная причина бесплодия? Хроническая ановуляция (спкя) ПРОВЕРЕНО

ПРИМЕНЕНИЕ

1.На прием по бесплодию обратилась пациентка 30 лет с желобами на отсутствие беременности в течение 4 лет. ИЗ АНАМНЕЗА: менструальная функция не нарушена, гинекологические заболевания отрицает неоднократно проходила гормональное обследование, УЗИ органов малого таза. Патологии не выявлено. Посткоитальный тест положительный. Спермограмма мужа в норме. ОБЪЕКТИВНО: общее состояние удовлетворительное, вес 60 кг, рост 167 см. ГИНЕКОЛОГИЧЕСКИЙ СТАТУС: влагалище без особенностей, шейка матки без видимой патологии, наружный зев точечный, матка в ретрофлексии, не увеличена, безболезненная. Придатки с обеих сторон не определяются. Какой метод исследования является наиболее информативным для оценки проходимости маточных труб? - лапароскопия, хромосальпингография ГИСТЕРОСАЛЬПИНГОГРАФИЯ, ЭХОСАЛЬПИНГОГРАФИЯ проверено

2.Женщина 39 лет, на приеме у врача-гинеколога. Выставлен диагноз: Меланома вульвы. Какое лечение наиболее предпочтительно? - хирургическое иссечение (в пределах здоровой ткани)

ПРОВЕРЕНО

3.Женщина 56 лет, на профилактическом приеме у гинеколога. Жалоб нет. Менопауза 2 года. Врач после осмотра обнаружила патологические изменения на шейке матки. Что необходимо провести в первую очередь? -ПАП- тест Мазок на онкоцитологию ( цитология шейки матки, Пап-тест, мазок по Паппаниколау) и биопсия

проверено

4. Женщина 53 года, на приеме у гинеколога. С жалобами на контактные кровотечения в течение 1 месяца. В анамнезе 2 родов, осложнившиеся разрывом шейки матки. Менопауза 2 года. Гинекологическое исследование: при осмотре в зеркалах шейка матки с эктопией цилиндрического эпителия, гипертрофирована, деформирована вследствие послеродовых разрывов, наружный зев зияет. При бимануальном исследовании матка и придатки без патологических изменений, параметрии свободны. Выберите наиболее эффективный скрининг-тест для ранней диагностики рака шейки матки? - ко-тестирование (ПЦРдиагностика ВПЧ с генотипированием 16 и 18 типов отдельно и цитология)

Мазок на онкоцитологию (мазок по Паппаниколау и Пап-тест)

проверено

5.На приемеу гинеколога пациентка 29 лет, по поводу бесплодия. Какой из методов обследования наиболее эффективен в диагностике трубно-перитонеального бесплодия? -гистеросальпингография, лапароскопия ПРОВЕРЕНО

6.Пациентка 28 лет, на приеме у гинеколога, после длительного обследования по поводу бесплодия. Какой метод назначил врач для исключения цервикального фактора бесплодия? - пробу Шуварского-Хунера, пробу Курцрока-Миллера ПРОВЕРЕНО

7.На приеме у гинеколога, женщина 32 года, после проведенного обследования, установлен трубно-перитонеальный фактор бесплодия. Современным методом лечения трубного бесплодия является? - ЭКО ПРОВЕРЕНО

сальпинголизис, фимбриопластика, сальпингостомия,

8.Пациентке 49 лет, на приеме у гинеколога, выставлен диагноз: Крауроз и лейкоплакия вульвы. Какое лечение наиболее предпочтительно? Вульвэктомия ПРОВЕРЕНО

9.Пациентка 65 лет обратилась к гинекологу, с жалобами на боли в костях и суставах, позвоночнике, дважды за последний год были переломы конечностей. Из анамнеза: менопауза 11 лет. Страдает артериальной гипертензией III, риск 3. Гинекологическое исследование: слизистая влагалища истончена, атрофична. При бимануальном исследовании тело матки уменьшено в размерах, безболезненное. Придатки с обеих сторон не пальпируются. Своды влагалища свободные. Какая терапия наиболее оптимальна для данной пациентки? -заместительная гормональная терапия И консультация у ревматолога

препараты кальция и витамина Д, бифосфонаты

10.Пациентка 40 лет, на приеме у гинеколога, с жалобами на постоянные слизистогнойные бели и контактные кровянистые выделения из половых путей. ИЗ АНАМНЕЗА: Менструальная функция не нарушена. В анамнезе 2 родов, осложнившиеся разрывом шейки матки. ГИНЕКОЛОГИЧЕСКОЕ ИССЛЕДОВАНИЕ: при осмотре в зеркалах шейка матки с эктопией цилиндрического эпителия, гипертрофирована, деформирована вследствие послеродовых разрывов, наружный зев зияет. При бимануальном исследовании матка и придатки без патологических изменений, параметрии свободны. Необходимое обследование в первую очередь для уточнения диагноза? - микроскопия отделяемого влагалища, бакисследование, цитолог. исследование соскоба, биопсия шейки матки, УЗИ органов малого таза ПРОВЕРЕНО

11.Пациентка 40 лет, на приеме у гинеколога, с жалобами на постоянные слизистогнойные бели и контактные кровянистые выделения из половых путей. ИЗ

АНАМНЕЗА: Менструальная функция не нарушена. В анамнезе 2 родов, осложнившиеся разрывом шейки матки. ГИНЕКОЛОГИЧЕСКОЕ ИССЛЕДОВАНИЕ: при осмотре в зеркалах шейка матки с эктопией цилиндрического эпителия, гипертрофирована, деформирована вследствие послеродовых разрывов, наружный зев зияет. При бимануальном исследовании матка и придатки без патологических изменений, параметрии свободны. Какое лечение назначить данной пациентке? - экстирпация матки без придатков Радиоволновая конизация

+АБТ проверено

12.Пациентка 27 лет обратилась в отделении гинекологии по поводу миомы матки, выявленной на УЗИ. Жалоб не предъявляет. Из анамнеза: менархе в 13 лет. Менструации по 5–6 дней, через 27–28 дней умеренные, безболезненные. Беременностей не было. Гинекологическое исследование: у дна матки определяется субсерозный миоматозный узел в диаметре до 7-8см, придатки с обеих сторон не изменены; выделения слизистые. Выставлен диагноз: «Миома матки с субсерозным ростом узла». Выберите метод оперативного лечения для данной пациентки. Консервативная миомэктамия

проверено

13.Пациентка 59 лет обратилась к гинекологу, с жалобами на кровянистые выделения из половых путей. Из анамнеза: менопауза 5 лет. Гинекологическое исследование: наружные половые органы и влагалище с явлениями возрастной инволюции; слизистая оболочка влагалища легко ранима; шейка матки без видимой патологии. Из канала шейки матки скудные кровянистые выделения; матка увеличена до 15 недель беременности, ограничена в подвижности, плотная. Придатки не определяются; параметрии свободны. Выставлен предварительный диагноз: Миома матки больших размеров. С целью исключения злокачественного процесса какой метод исследования имеет первоочередное значение? - раздельное диагностическое выскабливание Гистология аспирата

узи

проверено

14.На прием к гинекологу обратилась пациентка 42 года, с жалобами на обильные продолжительные и болезненные менструации, периодические тянущие, ноющие боли внизу живота, запоры. Гинекологическое исследование: шейка матки сглжена, плотная, диаметром 4-5 см. Тело матки незначительно увеличено, чувствительно при пальтации. Придатки с обеих сторон не изменены. Выставлен диагноз: «Шеечная миома матки». Выберите метод оперативного лечения для данной пациентки. Миомэктомия Экстирпация матки без придадков

проверено

15.Пациетнка 38 лет, в гинекологическом отделении, выставлен диагноз: Двухсторонний пиосальпинкс, определите объем оперативного лечения? Сальпингостомия / Сальпингэктомия Сальпингэктомия обеих маточных труб

проверено

16.Больная 57 лет, поступила в отделение гинекологии с жалобами на умеренные кровянистые выделения из половых путейв течение 3 месяцев. Менопауза 4 года. УЗИ: тело матки 48×37×46 мм, полость матки не деформирована. Эндометрий неоднородный, толщиной 15 мм. Придатки без особенностей. Выставлен диагноз «Гиперплазия эндометрия». С целью исключения злокачественного процесса какой метод исследования имеет первоочередное значение? Гистологическое исследование аспирата ПРОВЕРЕНО

17.Пациентка 35 лет обратилась с жалобами на обильные менструации со сгустками крови до 8-10 дней, беспокоящие в течении 6 последних месяцев. Гинекологическое исследование: шейка матки без видимой патологии, выделения слизистые. Тело матки не увеличено, безболезненно при пальпации; придатки с обеих сторон не определяются, своды глубокие. УЗИ: тело матки 48×37×46 мм, полость матки не деформирована. Эндометрий неоднородный, толщиной 15 мм. Придатки без особенностей. Выставлен диагноз «Гиперплазия эндометрия». Выберите препараты для лечения гиперпластических процессов эндометрия после проведения гистологии? монофазные КОК (Предпочтение отдают препаратам с низким содержанием эстрогенов (логест, мерсилон, новинет, линдинет$20) или с повышенным содержанием прогестагенов (микрогинон, ригевидон)) ПРОВЕРЕНО

18.Пациентка 42 лет обратилась с жалобами на обильные менструации со сгустками крови до 8-10 дней, беспокоящие в течении 6 последних месяцев. УЗИ: тело матки 48×37×46 мм, полость матки не деформирована. Эндометрий неоднородный, толщина эндометрия – 15 мм. Придатки без особенностей. Выставлен предварительный диагноз «Гиперплазия эндометрия». Какое исследование наиболее приемлемо для определения типа гиперплазии? Обзорная гистероскопия ПРОВЕРЕНО

19.Пациентка 55 лет обратилась в ЦСМ с жалобами на сухость во влагалище, диспареунию. Из анамнеза: менопауза в течение 10 лет. Специальное гинекологическое исследование: при исследовании в зеркалах слизистая влагалища истончена, имеются микротрещины, при введении зеркал кровоточит. При бактериоскопии мазка специфической флоры не выявлено.Выберите вагинальные свечи для местного лечения атрофического кольпита в составе которых? препараты, содержащие 17βэстрадиол

свечи с эстриолом

проверено

20. Пациентка 25 лет доставлена в отделение гинекологии с жалобами на острые боли внизу живота, тошноту, рвоту, которые возникли внезапно при подъеме с постели. Менструальная функция не нарушена. Объективно: живот умеренно вздут, резко

болезненный в правой подвздошной области, симптом Щеткина—Блюмберга положительный. Пульс 90 уд. в 1 минуту, температура 37,2 °С. При бимануальном исследовании: шейка матки без особенностей, тело матки плотное, не увеличено, безболезненное. Справа в области придатков определяется округлое образование, тугоэластической консистенции, резко болезненное, размерами до 6 см в диаметре. Слева придатки не определяются, своды свободны, выделения слизистые. Какова тактика лечения при экстренном оперативном лечении? Дренирующие паллиативные операции(пункция и кольпотомия), в дальнейшем удаление придатков

Экстренное оперативное лечение. Объем оперативного вмешательства определяется после визаульного осмотра

проверено

21. Беременная 28 лет встала на учет в ЦСМ в сроке беременности 8 недель. Данная беременность первая. Проведено УЗИ: в полости матки один плод, соответствует 7-8 неделям беременности. Матка в нормальном тонусе. Справа, в области придатков определяется образование с четкими контурами, однокамерное, размером 5х6 см. Выставлен предварительный диагноз – киста яичника. В сроке беременности 12 сделано повторное УЗИ: в полости матки один плод, соответствует 12 неделям беременности. Беременность развивается нормально. Придатки с обеих сторон не увеличены. Какая тактика ведения пациентки? Динамическое наблюдение(Функциональные кисты обычно рассасываются самостоятельно в течение первого триместра). Если есть необходимость хирургического вмешательства, его осуществляют на сроке 14-16 недель(удаление кисты)

проверено

22.На прием к гинекологу, в ЦСМ обратилась пациентка 42 лет с жалобами на периодические тянущие боли внизу живота, поясничной области. Из анамнеза: менархе в 13 лет. Менструации по 5–6 дней, через 27–28 дней умеренные, безболезненные. Гинекологическое исследование: шейка матки цилиндрической формы; тело матки не увеличено, в области правых придатков, ближе к маточному углу, определяется образование с четкими контурами, размером 5х7 см. Придатки слева не определяются. Выставлен диагноз: «Параовариальная киста яичника». Рекомендовано оперативное лечение. Выберите метод оперативного лечения для данной пациентки. лапароскопическое Удаление кисты (энуклеация) с рассечением листка широкой связки матки

проверено

23.Пациентка 27 лет доставлена в отделение гинекологии бригадой скорой помощи с жалобами на схваткообразные боли внизу живота, обильные, со сгустками кровянистые выделения из половых путей, слабость. ОБЪЕКТИВНО: АД 110/60 мм рт. ст., пульс 90 в 1 минуту, температура тела 37 ºС. Последняя менструация 2 месяца назад. ПРИБИМАНУАЛЬНОМ ИССЛЕДОВАНИИ: шейка матки без видимой патологии, цианотична, наружный зев пропускает палец. Матка увеличена до 6 недель

беременности, в тонусе. Придатки с обеих сторон не определяются. Своды глубокие, безболезненные. Выделения кровянистые, обильные со сгустками. Объем неотложной медицинской помощи? Инструментальное удаление остатков плодного яйца на фоне комплексной гемостатической терапии, лечение геморрагического шока и постгеморрагической анемии.

проверено

24.Пациентка 23 лет обратилась в отделение гинекологии, с жалобами на задержку менструации на 10 дней. Нарушение менструальной функции отмечает впервые. Половая жизнь регулярная, без контрацепции. ОБЪЕКТИВНО: состояние удовлетворительное, живот мягкий, безболезненный, АД 120/80 мм рт. ст., пульс 72 в 1 минуту. ПРИ ТРАНСВАГИНАЛЬНОЙ ЭХОГРАФИИ: заподозрена прогрессирующая трубная беременность. Определите тактику врача? Экстренная госпитализация

проверено

25.Пациентка 15 лет, поступила в отделение гинекологии с жалобами на кровянистые выделения из половых путей, которые появились после задержки менструации на 4 месяца. ОБЪЕКТИВНО: кожные покровы бледные, пульс 86 в 1 мин, ритмичный, АД 110/70 мм рт. ст. Живот мягкий, безболезненный. ОАК: уровень гемоглобина 100 г/л, гематокрит 30 %. ПРИ ГИНЕКОЛОГИЧЕСКОМ ОСМОТРЕ: наружные половые органы развиты правильно, оволосение по женскому типу, девственная плева цела. При ректоабдоминалъном исследовании: матка нормальных размеров, плотная, безболезненная придатки с обеих сторон не увеличены, выделения

из половых путей кровянистые, обильные.

Какой гемостаз назначить данной

пациентке?

Комплексная гемостатическая терапия (антифибринолитики и

гормональный гемостаз)

 

проверено (гормональный гемостаз монофазными низкодозированными оральными контрацептивами начиная с 4-5 таблеток в сутки с постепенным снижением дозы до 1 таблетки в сутки курсом21 день; Аминокапроновая кислота)

26. Больная 31 года поступила в отделение гинекологии с жалобами на внезапно возникшие боли внизу живота схваткообразного характера с иррадиацией в крестец, слабость, головокружение, скудные кровянистые выделения из половых путей. Объективно определяется бледность кожных покровов, снижение АД до 90/50 мм рт.ст., пульс 110 ударов/мин. Живот слегка вздут, болезненный над лоном и в левой подвздошной области. Симптом Щеткина-Блюмберга слабо положительный. Мочеиспускание, стул не нарушены. При влагалищном исследовании выявлены укороченные, болезненные своды, матка несколько увеличена, левые придатки увеличены, правые не определяются. При пункции через брюшной полости через

задний свод влагалища получено 8 мл темной несворачивающейся крови. Какой должна быть тактика врача? Произвести срочную лапаротомию.

проверено

27. Пациентка 40 лет доставлена в приемный блок гинекологии с жалобами на схваткообразные боли внизу живота и обильные кровянистые выделения из половых путей. Последние 2 года менструации длятся до 16 дней, обильные, со сгустками, болезненные. При осмотре в зеркалах видны обильные кровянистые выделения из шейки. Нижний полюс рождающегося фиброматозного узла. Выберите тактику лечения? Удаление фиброматозного узла через влагалище

проверено

28.Пациентка 26 лет, обратилась в отделение гинекологии, с жалобами на тянущие боли внизу живота, альгоменорею, отсутствие беременности в течение 3 лет половой жизни без контрацепции. Из анамнеза: менструации по 5–6 дней через 28–30 дней. Гинекологическое исследование: Матка отклонена кзади, ограничена в подвижности, нормальных размеров, безболезненная; справа от матки пальпируется образование размером 10×8 см, тугоэластической консистенции, малоподвижное, умеренно болезненное; левые придатки не увеличены. Выставлен диагноз «Эндометриоидная киста правого яичника». Какое первоочередное лечение показано данной пациентке?

Кистэктомия

проверено

29.Больная 34 лет, три дня назад была удалена ВМС, которая была установлена 8 лет назад. Со вчерашнего дня отмечает повышение температуры тела до 390С, боли внизу живота, общую слабость. При поступлении отмечаются симптомы раздражения брюшины в нижних отделах живота. После гинекологического исследования выставили диагноз острый эндометрит, правосторонний пиосальпинкс, пельвиоперитонит. В динамике состояние больной ухудшается (усиливаются боли, нарастает лейкоцитоз). Какая тактика врача?Оперативное лечение – лапаротомия

проверено

30.Во время аборта произведена перфорация матки кюреткой. Ваша тактика?Лапароскопия ,ушивание перфарационного отверстия

проверено

31.Пациентка 58 лет обратилась к гинекологу, с жалобами на кровянистые выделения из половых путей. Из анамнеза: менопауза 5 лет. Гинекологическое исследование: наружные половые органы и влагалище с явлениями возрастной инволюции; слизистая оболочка влагалища легко ранима; шейка матки без видимой патологии. Из канала шейки матки скудные кровянистые выделения; матка увеличена до 15 недель беременности, ограничена в подвижности, плотная. Придатки не определяются; параметрии свободны. Выставлен предварительный диагноз: Миома

матки больших размеров. С целью исключения злокачественного процесса какой метод исследования имеет первоочередное значение? Биопсия опухоли+узи

проверено

32.На прием пришла женщина, 35лет для выбора метода контрацепции. Из анамнеза: замужем, родов-2, мед.абортов-1. Об-но: повышенного питания, варикозное расширение вен нижних конечностей. АД130/80 мм.рт.ст. Влагалищное исследование: шейка матки без видимой патологии. Тело матки нормальных размеров, придатки без особенностей. Выделения – светлые, без запаха. Какой из методов контрацепции Вы рекомендуете?ВМС

проверено

33.При выполнении медицинского аборта пациентке, 35 лет, хирургическим путем на сроке беременности 8 недель кюретка ушла за пределы полости матки в области передней стенки матки. Какая наиболее вероятная тактика лечения?Кульдоцентез\наблюдательная тактика

проверено

34.При выполнении медицинского аборта пациентке, 28лет, хирургическим путем на сроке беременности 9 недель кюретка ушла за пределы полости матки в области ее дна. ОБЪЕКТИВНО: кожные покровы розовые, пульс 86 в минуту, удовлетворительных свойств, ритмичный, АД 110/70 мм рт. ст. При пальпации живот мягкий во всех отделах. Пациентка находится под общим наркозом. Какие дополнительные методы исследования целесообразно использовать для уточнения диагноза? Лапароскопия

Зондирование полости матки трансвагинальное узи

проверено

35.Выберите метод контрацепции замужней женщине 28 лет, имеющей двоих детей, страдающей хроническим тромбофлебитом вен нижних конечностей:

Внутриматочная контрацепция (ВМС)

проверено

36.При проведении медицинского аборта пациентке, 32лет, в самом начале операции возникло обильное маточное кровотечение. Диагностирована перфорация матки.

Тактика врача? Чревосечение, экстирпация матки

проверено

37.Женщина 38 лет, на приеме у гинеколога по поводу контрацепции. Жалоб нет. Состоит на учете у терапевта по поводу Гипертонической болезни, получает лечение.

Оптимальным методом контрацепции у женщин репродуктивного

возраста с

гипертонической болезнью I, II стадии является? Мини-пили

 

вмс

 

проверено

 

38.При выполнении медицинского аборта хирургическим путем на сроке беременности 9 недель пациентке 35лет, кюретка ушла за пределы полости матки в области ее дна. ОБЪЕКТИВНО: кожные покровы розовые, пульс 86 в минуту, удовлетворительных свойств, ритмичный, АД 110/70 мм рт. ст. При пальпации живот мягкий во всех отделах. Пациентка находится под общим наркозом. Какова наиболее рациональная тактика лечения больной? Опорожнение матки через перфорационное

отверстие во время лапароскопии

проверенопроверено

39.Пациентка 39 лет, обратилась к гинекологу с жалобами на обильные менструации в течение 6 месяцев, нуждается в надежной контрацепции. Из анамнеза: родов 3, без особенностей, 5 медицинских абортов. Три месяца назад при обследовании по поводу обильных менструаций диагностирована гиперплазия эндометрия, проведена гистероскопия, гистологическое исследование соскоба эндометрия. Заключение: простая гиперплазия эндометрия без атипии. Принимала по рекомендации гинеколога

гормональные контрацептивы в течение 2 месяцев. Отмечала головные боли, выраженную болезненность молочных желез, в связи с чем, отказалась от дальнейшего их приема. Какой метод контрацепции рекомендовать данной пациентке?

Внутриматочную систему с левоноргестеролом «Мирена»

проверено

40.Пациентка 20 лет, обратилась к гинекологу, по поводу контрацепции. Из анамнеза: Роды 9 месяцев назад, кормит грудью. Была первая менструация после родов. Соматически здорова. ГИНЕКОЛОГИЧЕСКОЕ ИССЛЕДОВАНИЕ: при осмотре в зеркалах шейка матки деформирована ПОСЛЕРОДОВЫМ разрывом. Выделения умеренные молочного цвета. При бимануальном исследовании тело матки не увеличено, подвижное безболезненное, придатки с обеих сторон без особенностей. Своды свободные. Какой метод контрацепции рекомендовать данной пациентке?Гестагенные контрацептивы (Лактинет, мини-пили).

проверено

41.В кабинет планирования семьи обратилась девушка 25 лет, для выбора метода контрацепции. Из анамнеза: менархе 13 лет, менструальная функция не нарушена. Не замужем. Постоянного полового партнера нет. Половая жизнь не регулярная. Соматический и гинекологический анамнез не отягощен. ГИНЕКОЛОГИЧЕСКОЕ ОБСЛЕДОВАНИЕ: шейка матки без видимой патологии, тело матки не увеличено, подвижное, безболезненное, придатки с обеих сторон без особенностей. Своды глубокие. Какой метод контрацепции рекомендуете для данной пациентки?

Контрацептивное кольцо Новаринг Микродозированные 41 КОК-мерсилон, новинет, логест, линдинет

барьерный метод - презерватив

проверено

42. Пациентка 25 лет обратилась на прием к гинекологу для получения рекомендаций по контрацепции. Из анамнеза: менструации с 13 лет, нерегулярные через 25–40 дней, по 6–7 дней, обильные, болезненные, принимает спазмалгон н для купирования боли. Замужем. Были одни роды 3 года назад. Ближайшие 2–3 года беременность не планирует. Соматически здорова. При гинекологическом обследовании и УЗИ патологии не выявлено. Какой метод контрацепции рекомендовать данной пациентке?

Комбинорованные эстроген-гестагенные низкодозированные монофазные контрацептивны (Ярина, Регулон)

проверено

43. На прием к гинекологу обратилась пациентка 50 лет с жалобами на приливы жара к лицу, шее по 15 раз в день, сердцебиение, раздражительность, плаксивость, нарушение сна, чувство «мурашек» по ночам в руках. Из анамнеза: последняя менструация 10 месяцев назад, до этого в течение года наблюдались задержки менструации до 3 месяцев. Объективно: вес 70 кг, рост 168 см. АД 140/100 мм рт. ст., постоянно принимает гипотензивные препараты. гинекологический статус: при исследовании в зеркалах шейки матки без видимой патологии, слизистая влагалища атрофична, выделений нет. При бимануальном исследовании: матка не увеличена, безболезненна, придатки не определяются. Какое лечение необходимо данной пациентке?МГТ

эстроген-гестагенными препаратами.

проверено

44. Пациентка 65 лет на приеме у гинеколога в ЦСМ. Предъявляет жалобы на постоянные тянущие боли внизу живота, в поясничной области, ощущение инородого тела в половой щели, дизурические расстройства. В анамнезе четверо родов, последние роды крупным плодом, осложнились разрывом промежности II степени. Женщина страдает инсулинозависимым сахарным диабетом тяжелой степени. Гинекологический статус: половая щель зияет, отмечается расхождение леваторов, передняя и задняя стенки влагалища опущены, имеются цисто- и ректоцеле, при натуживании шейка матки выступает за пределы вульварного кольца. Какова тактика в коррекция пролапса гениталий у этой пациентки?Консервативное лечение: использование пессария реконструктивно-пластическая операция с использованием сетчатого протеза

проверено

45. Пациентка 46 лет, поступила в гинекологическое отделение с жалобами на обильные кровянистые выделения из половых путей. Последние 2 года интервал между менструациями составляет 2-3 месяца. 2 недели назад у пациентки после 2мес.

отсутствия менструации началось кровотечение, продолжающееся до настоящего времени. Гинекологические заболевания отрицает. В день поступления произведено раздельное диагностическое выскабливание матки и цервикального канала: длина матки по зонду 8 см, стенки гладкие. Получен обильный соскоб, Результат гистологического исследования: простая без атипии гиперплазия эндометрия. Какими препаратами будет проводиться лечение в данном случае? -КОКПрогестагены в

непрерывном режиме

проверено

46.Пациентка 38 лет, на приеме у гинеколога, с жалобами на общую слабость, быструю утомляемость, приливы жара к лицу, шее, интенсивное потоотделение. Из анамнеза:2 месяца назад произведена лапаротомия, двусторонняя аднексэктомия. Выписана на 12 сутки после операции в удовлетворительном состоянии. В анамнезе 2 родов, 3 медицинских аборта. Объективно: пульс 80 уд/мин, АД 140/100 мм рт. ст. Гинекологический осмотр: шейка матки без видимой патологии, выделения из половых путей слизистые, скудные. При бимануальном обследовании тело матки нормальных размеров, плотное, безболезненное, подвижность ограничена. Придатки не определяются. Какое лечение назначить данной пациентке? Заместительная

гормональная терапия (Климодиен)

проверено

47.Пациентка 30 лет обратилась к гинекологу, с жалобами на выраженные головные боли, головокружение, сопровождающиеся тошнотой, рвотой, отеком век, лица, раздражительность, появляющиеся за неделю до менструации. Из анамнеза: в 10 лет была закрытая черепно-мозговая травма. Менархе в 12 лет, менструации по 5–6 дней через 28 дней, умеренные, болезненные в первый день. Беременностей не было. Гинекологический статус: шейка матки без видимой патологии, матка не увеличена, плотная, безболезненная, придатки с обеих сторон не увеличены, безболезненные, своды глубокие. Какая терапия наиболее оптимальна для данной пациентки? Диета,

физиотерапия, психотерапия, седативные и транквилизаторы, НПВС, ноотропы, средства улучшающие обмен нейромедиатонров в ЦНС, бромкриптин при повышении пролактина.

проверено

48.Пациентка 38 лет обратилась к гинекологу с жалобами на вздутие живота, нагрубание и болезненность молочных желез, отечность лица и голеней, повышенную утомляемость, сердцебиение, подъемы артериального давления. Из анамнеза: менструации с 13 лет, через 28–29 дней, по 4–5 дней, умеренные. Заболела около 3 лет

назад, когда появилось нагрубание молочных желез накануне менструации. С годами количество и выраженность симптомов нарастают, появляются за 10–12 дней до менструации и прекращаются с ее окончанием. В анамнезе 2 родов. Беременности не планирует. При гинекологическом исследовании: патологических изменений не выявлено. Какое лечение назначить данной пациентке? КОК Прогестерон местно на область молочных желез.

Пролонгированная прогестероновая контрацепция

проверено

49. На прием к гинекологу, обратилась пациентка 57 лет с жалобами на сухость во влагалище, периодически жжение, зуд, недержание мочи. Из анамнеза: менопауза 6 лет, родов — 2 через естественные родовые пути. Объективно: состояние удовлетворительное, вес 70 кг, рост 168 см. АД 150/100 мм рт. ст., постоянно принимает гипотензивные препараты. Гинекологический статус: при исследовании в зеркалах шейка матки конической формы, слизистая влагалища истончена, гиперемирована, легко ранима, выделения слизистые скудные. При бимануальном исследовании: матка уменьшена в размерах, подвижна, безболезненна, придатки не определяются. Какое лечение наиболее оптимально для данной пациентки? - Заместительная гормональная терапия

местная гормональная терапия свечи с эстриолом

проверено

50.На прием к детскому гинекологу обратились девочка 8 лет с матерью. Жалобы на кровянистые выделения из половых путей в течение 2-х дней. Из анамнеза: родилась недоношенной, перенесла корь, краснуху. Развитие вторичных половых признаков началось около 1 года назад. Объективно: правильного телосложения, рост 140 см, вес

40кг. Соматическое развитие соответствует 12 летнему возрасту. Осмотр наружных половых органов: развиты правильно, оволосение по женскому типу, девственная плева не нарушена. Выделения из влагалища умеренные, кровянистые. Ректально: тело матки отклонено кпереди, больше возрастной нормы, плотное, безболезненное. Придатки не определяются. Необходимые обследования для уточнения диагноза?УЗИ органов малого таза, исследование уровней ФСГ, ЛГ, эстрадиола, КТ гипофиза

проверено

51.Девочка 8 лет, на приеме у детского гинеколога. Со слов матери жалобы на кровянистые выделения из половых путей в течение 2-х дней. Из анамнеза: родилась недоношенной, перенесла корь, краснуху. Развитие вторичных половых признаков началось около 1 года назад. Объективно: рост 140 см, вес 40 кг. Молочная железа значительно выдается, вместе с соском и околососковым кружком образует конус, в подмышечной впадине единичные волосы. Осмотр наружных половых органов:

развиты правильно, единичные волосы на больших половых губах, девственная плева не нарушена. Выделения из влагалища умеренные, кровянистые. Ректально: тело матки отклонено кпереди, больше возрастной нормы, плотное, безболезненное. Какой из инструментальных методов исследования необходим в первую очередь? Рентгенография

кистей рукИили

УЗИ органов малого таза

проверено

52. Женщина 23 лет, обратилась к гинекологу с жалобами на редкие скудные менструации, увеличение веса. Из анамнеза: менархе в 12 лет. Менструации по 3–4 дня, через 45–60 дней скудные, безболезненные. Беременностей не было. Объективно: рост 160 см, вес 68 кг, АД 140/100 мм рт. ст., ИМТ 28, кожа жирная, угревая сыпь на лице, рост пушковых волос в области подбородка, верхней губы. При бимануальном исследовании: тело матки несколько уменьшено в размерах, безболезненное. При пальпации яичники увеличены, плотные, безболезненные. Своды глубокие. Выделения бели. Какую группу препаратов назначить для патогенетического лечения? КОК с

антиандрогенным действием,

проверено

53.Пациентка 29 лет на приеме у гинеколога с жалобами на отсутствие менструации в течение 2 лет после родов, отеки конечностей, лица, головные боли, общую слабость. Менархе в 14 лет, менструации до беременности по 4–5 дней, через 28 дней. Одни роды

2года назад через естественные родовые пути с ранним гипотоническим кровотечением с переливанием компонентов крови. Грудью не кормила из-за отсутствия лактации. Объективно: рост 160 см, масса тела 70 кг, за 2 года набрала15 кг. Пульс 76 уд/мин, АД 90/60 мм рт. ст. Молочные железы гипоплазированы. Гинекологический статус: слизистая влагалища истончена, легко ранима, выделения слизистые скудные. Тело матки не увеличено, подвижно, безболезненно. Какое лечение необходимо назначить данной пациентке? ЗГТ эстроген-гестагенными препаратами, препаратами ТТГ,

глюкокортикоидами, препараты железа

проверено

54.Женщина 28 лет, обратилась к гинекологу с жалобами на редкие скудные менструации, увеличение веса. Из анамнеза: менархе в 13 лет. Менструации по 3–4 дня, через 47–60 дней скудные, безболезненные. Беременностей не было. Объективно: рост

158см, вес 83 кг, АД 130/100 мм рт. ст., ИМТ28, имеется угревая сыпь на лице, рост пушковых волос в области подбородка.. При бимануальном исследовании: тело матки несколько уменьшено в размерах, безболезненное. При пальпации яичники увеличены,

плотные, безболезненные. Своды глубокие. Выделения бели, незначительно. Назначена низкокалорийная диета Что назначить на первом этапе лечения?КОК с антиандрогенным действием/медроксипрогестерона ацетат перорально ежедневно однократно по 5–10 мг в течение 10–14 дней каждые 1–2 месяцев) или оральные контрацептивы

проверено

55.Пациентка, 32 года, на приеме у гинеколога с жалобами на обильные менструации, периодические межменструальные мажущие кровянистые выделения. Менструальный цикл регулярный, 26–28 дней. Объективно: состояние удовлетворительное. Рост 168 см, масса тела 68 кг, ИМТ 24.Узи органов малого таза на 6-й день менструального цикла: тело матки расположено в retroflexio, размеры 45×52×43 мм, структура миометрия однородная, М-эхо — 5,5 мм, неоднородное, по задней стенке визуализируется образование размером 8×9 мм. Структура шейки матки без особенностей, цервикальный канал не расширен, яичники расположены в типичном месте, объем правого 5,1 см3, левого — 4,9 см3. Выберите наиболее предпочтительное лечение для данной пациентки? полипэктомия + прием оральных контрацептивов

проверено

56.Пациентка 27 лет доставлена в гинекологическое отделение с жалобами на обильные кровянистые выделения из половых путей, задержку менструации на 3 недели. Со слов пациентки, накануне вечером почувствовала тянущие боли внизу живота; утром обильные кровянистые выделения из половых путей. Пациентка живет половой жизнью, использует барьерный метод контрацепции Общее состояние удовлетворительное, пульс 72 минуту, АД 120/80 мм рт. ст. Гинекологическое исследование: цервикальный канал пропускает 1 палец, выделения кровянистые, обильные, со сгустками. Тело матки несколько увеличено, безболезненное. Придатки с обеих сторон не пальпируются. Выберите метод лечения? Кюретаж полости матки, лечение постгеморрагической анемии+свечи спазмолитические

проверено

57.Пациентка 16 на приеме у гинеколога с жалобами на отсутствие менструаций. Половую жизнь отрицает. В 10 лет начался рост молочных желез, в 11 лет появились признаки оволосение на лобке, позже в подмышечных впадинах. При осмотре: рост 167 см, вес 58 кг. Вторичные половые признаки хорошо развиты. При осмотре: наружные половые органы развиты правильно. Большие половые губы прикрывают малые. Девственная плева цела. При ректоабдоминальном исследовании в месте расположения влагалища и матки пальпируется тяж. УЗИ органов малого таза: тело матки, влагалище отсутствует, в области придатков матки визуализируются образования, по структуре соответствующие яичникам. Кариотип 46 ХХ. Какое лечение необходимо данной пациентке? оперативно-хирургическое формирование влагалища + психотерапия, витаминотерапия.

проверено

58. Пациентка 17 лет обратилась на прием к гинекологу с жалобами на отсутствие менструаций. Из анамнеза: росла и развивалась быстрее сверстников, успешно занимается спортом. Детские заболевания отрицает. Половой жизнью не живет. Объективно: рост 158 см, вес 55 кг, широкие плечи, узкий таз, молочные железы гипоплазированы, рост терминальных волос на бедрах, спине, грудине, по белой линии живота, в области подбородка, верхней губы. Пульс 72 удара в минуту, АД 110/70 мм рт. ст. Осмотр наружных половых органов: развиты правильно, оволосение по мужскому типу. При ректо-абдоминальном исследовании: патологии со стороны гениталий не выявлено. Какое лечение необходимо данной пациентке? терапия ГКСДексаметазон 0,25-0,75 мг на ночь

Гидрокортизон10-30 мг на 2-3 приема в день

Преднизолон2,5-7,5 мг на ночь

Метипред2-6 мг на ночь

проверено

59. Женщина 28 лет, обратилась к гинекологу с жалобами на редкие скудные менструации, увеличение веса. Из анамнеза: менархе в 13 лет. Менструации по 3–4 дня, через 47–60 дней скудные, безболезненные. Беременностей не было. Объективно: рост 158 см, вес 83 кг, АД 130/100 мм рт. ст., ИМТ28, имеется угревая сыпь на лице, рост пушковых волос в области подбородка. При бимануальном исследовании: тело матки несколько уменьшено в размерах, безболезненное. При пальпации яичники увеличены, плотные, безболезненные. Своды глубокие. Выделения бели, незначительно. Назначена низкокалорийная диета. Что назначить на первом этапе лечения? бигуаниды

КОК с антиандрогенным действием/медроксипрогестерона ацетат перорально ежедневно однократно по 5–10 мг в течение 10–14 дней каждые 1–2 месяцев) или оральные контрацептивы

проверено

60. Пациентка 27 лет на приеме у гинеколога с жалобами на раздражительность, слабость, плаксивость, нагрубание молочных желез, отечность лица, голеней,увеличение массы тела. Выраженность жалоб растет с приближением менструации, после ее наступления указанные жалобы исчезают. Из перенесенных заболеваний отмечает хронический двусторонний аднексит. При осмотре в зеркалах и бимануальном исследовании патологических изменений не выявлено. Женщина отмечает, что прием диуретиков несколько облегчает состояние. Какое лечение назначить в данном случае? КОКи

проверено

61.Пациентка 14 лет, поступила в гинекологическое отделение с жалобами на слабость, головокружение, обильные кровянистые выделения из половых путей в течение 6 дней. Менструации с 13 лет, нерегулярные, обильные, со сгустками, по 8-10 дней, болезненные. При осмотре: кожные покровы бледные. Пульс 94 удара в минуту, ритмичный, АД 100/70мм рт. ст. Гинекологический статус: наружные половые органы развиты правильно, оволосение по женскому типу, девственная плева цела. Выделения из половых путей кровянистые, обильные. При ректоабдоминальном исследовании патологии не обнаружено. В анализе крови гемоглобин 90 г/л. Какое лечение необходимо пациентке в первую очередь? комплексная гемостатическая терапия - применение гормонального гемостаза с большим количеством эстрогенов

проверено

62.Пациентка 40 лет, на приеме у гинеколога, с жалобами на постоянные слизистогнойные бели и контактные кровянистые выделения из половых путей. ИЗ АНАМНЕЗА: Менструальная функция не нарушена. В анамнезе 2 родов, осложнившиеся разрывом шейки матки. ГИНЕКОЛОГИЧЕСКОЕ ИССЛЕДОВАНИЕ: при осмотре в зеркалах шейка матки с эктопией цилиндрического эпителия, гипертрофирована, деформирована вследствие послеродовых разрывов, наружный зев зияет. При бимануальном исследовании матка и придатки без патологических изменений, параметрии свободны. Какое лечение назначить данной пациентке? радиохирургическое лечение

Криодеструкция шейки матки+АБТ

проверено

63.Пациентка, 42 года, на приеме у гинеколога с жалобами на кровянистые выделения из половых путей. беспокоят обильные желтоватые выделения с неприятным запахом, иногда с примесью крови ИЗ АНАМНЕЗА: Менструации регулярные. ОСМОТР в зеркалах: шейка матки гипертрофирована, слизистая темнобагрового цвета, из цервикального канала сукровично-кровянистые мутные выделения с неприятным запахом. Бимануально: влагалищная часть шейки матки бочкообразно расширена, очень плотная, неподвижная. Тело матки несколько больше нормы. В параметриях с двух сторон имеются плотные инфильтраты, доходящие до стенок таза. Наиболее предпочтительный метод обследования в первую очередь для уточнения диагноза? жидкостная цитология, узи

проверено

64.Пациентка 25 лет обратилась в женскую консультацию с жалобами на обильные пенистые выделения из половых путей, зуд и жжение во влагалище, частые позывы на мочеиспускание. Заболела 1 неделю назад. ИЗ АНАМНЕЗА: половая жизнь с 17 лет, вне брака. Соматически здорова. ГИНЕКОЛОГИЧЕСКИЙ СТАТУС: слизистая оболочка вульвы, влагалища и шейка матки гиперемированы, отечны, с точечными

кровоизлияниями. Выделения из половых путей обильные, пенистые, серо-желтого цвета. Матка не увеличена, безболезненная при пальпации, придатки с обеих сторон не определяются. Был выставлен DS: Острый трихомониаз. Какой из нижеперечисленных антибиотиков наиболее предпочтителен для лечения этого заболевания? метртонидазол

проверено

65. Пациентка А.,32 лет поступила в гинекологическое отделение с жалобами на боли в области наружных половых органов, температура тела до 39,0 °С. Объективно: Общее состояние удовлетворительное, пульс 90 в минуту, АД 110/70 мм. рт. ст. Гинекологический осмотр: При осмотре в области правой большой половой губы определяется опухолевидное образование размером 4×4 см, закрывающее вход во влагалище, кожа над ним гиперемирована, определяется флуктуация, болезненна. Влагалище и шейка матки без видимой патологии, матка в антефлексии, не увеличена, безболезненная. Придатки с обеих сторон не определяются. Определите тактику лечения врача гинекологического стационара? Хирургическое лечение. Вскрытие абсцесса с формированием искусственного протока.

проверено

66.Пациентка В., 22 лет, обратилась к врачу женской консультации с жалобами на неприятные ощущения во влагалище, чувство жжения, зуд, бели. Считает себя больной в течение 5 дней. ОСМОТР В ЗЕРКАЛАХ: слизистая влагалища и шейки матки резко гиперемирована, отечна. На этом фоне имеются белесоватые налеты, которые легко снимаются марлевым шариком, выделения творожистого вида. Влагалищное исследование без особенностей. Какой вид терапии наиболее предпочтителен для этого диагноза? противогрибковая терапия: Клотримозол, флуконазол,кетоконазол

проверено

67.Пациентка А.,28 лет, обратилась к гинекологу, с жалобами на зуд и жжение наружных половых органов, болезненное мочеиспускание, повышение температуры до 37,4°С, отмечает усиление болей внизу живота перед менструацией. В браке не состоит, имеет несколько половых партнеров. ПРИ ОСМОТРЕ В ЗЕРКАЛАХ: слизистая наружного отверстия уретры гиперемирована, шейка матки цилиндрическая, выделения из цервикального канала слизисто-гнойные с желтоватым оттенком. ПРИ БИМАНУАЛЬНОМ ИССЛЕДОВАНИИ: ТЕЛО матки в anteversio-flexio, мягковатой консистенции, подвижное, безболезненная. Придатки с обеих сторон не определяются, область их безболезненна. При ПЦР выявлена: Chlamidia Trachomatis. Какой из

нижеперечисленных антибиотиков наиболее эффективен при лечении этого

заболевания? Препараты тетрациклинового ряда-доксициклин проверено

68. Пациентка Л., 37 лет, в гинекологическом отделении с жалобами на боли внизу живота и в пояснице, повышение температура тела до 39°С, жажду, сухость во рту в

течение 2 недель. В течение 9 лет с целью контрацепции использует ВМС. ОБЪЕКТИВНО: кожные покровы бледные, ЧС 106 уд/мин. АД 120/80 мм рт. ст. Живот симметричный, участвует в акте дыхания, при пальпации мягкий, незначительно болезненный в нижних отделах. Симптом раздражения брюшины нет. ПРИ ОСМОТРЕ В ЗЕРКАЛАХ: выделения гноевидные, из цервикального канала видны нити ВМС. ПРИ БИМАНУАЛЬНОМ ИССЛЕДОВАНИИ: тело матки в anteversio-flexio, отклонено вправо, болезненное при пальпации. Придатки без особенностей. В ОАК: лейкоциты – 15х109 /л, гемоглобин - 98 г/л. Какой вид терапии наиболее предпочтителен после удаления внутриматочной спирали? Антибиотикотерапия

проверено

69.Пациентка 27лет поступила в гинекологическое отделение с жалобами на боли в области наружных половых органов, повышение температуры тела до 38°С, в течение

3дней. ИЗ АНАМНЕЗА: менструальная функция не нарушена. В анамнезе 2 родов, 1 медицинский аборт. Гинекологические заболевания отрицает. ОБЪЕКТИВНО: общее состояние удовлетворительное, пульс 86 в минуту, АД 110/70 мм. рт. ст. При осмотре в области левой большой половой губы определяется опухолевидное образование размером 3,0×3,5 см, кожа над ним гиперемирована. ГИНЕКОЛОГИЧЕСКИЙ СТАТУС: шейка матки без видимой патологии, наружный зев щелевидный, матка в антефлексии, не увеличена, безболезненная. Придатки с обеих сторон не определяются. Определите тактику врача гинекологического отделения? Антибактериальная терапия с учетом чувствительности возбудите-ля, симптоматическое лечение,вскрытие +дренирование

проверено

70.Больная, 31 года, поступила в гинекологическое отделение с жалобами; на резкие боли внизу живота, чувство жара, озноб, слабость, повышение температуры до 38,8 С. В анамнезе воспаление придатков матки. ОБЪЕКТИВНО: Кожные покровы бледноватые, язык сухой, обложен белым налетом. Живот умеренно вздут, при пальпации болезнен во всех отделах, выражены симптомы раздражения брюшины. В ОАК: Нв - 126 г/л, лейкоцитов 18х109 в 1 л. ПРИ ВЛАГАЛИЩНОМ ИССЛЕДОВАНИИ: матка не контурируется, справа и слева в области придатков определяются

опухолевидные образования без четких границ, тугоэластической консистенции, интимно спаянные с маткой, неподвижные, резко болезненные при пальпации. Определите тактику врача? Оперативное вмешательство-лапаротомия

проверено

71. Пациентке 35 лет при профилактическом осмотре проведена кольпоскопия. На слизистой оболочке шейки матки выявлены йод-негативные участки. В анамнезе двое срочных родов и 2 медицинских аборта. Для установления окончательного диагноза необходимо выполнить? биопсия шейки матки(впч)

проверено

72.Девушка 20 лет на профосмотре у гинеколога.Жалоб нет. Менструации с 13 лет, регулярные по 3–4 дня через 28-30 дней. Замужем 3 месяца, половой партнер один. Гинекологических заболеваний не было. ГИНЕКОЛОГИЧЕСКОЕ ИССЛЕДОВАНИЕ: при осмотре в зеркалах выделения слизистые, вокруг наружного зева цервикального канала ободок гиперемии размером около 0,5 см. Матка и придатки пальпаторно не изменены. Определите тактику врача женской консультации? назначить повторный осмотр через 6 месяцев. ПЦР на УГИ

73.Пациентка 42 лет, у гинеколога для профилактического осмотра. Жалоб нет. Менструации по 4–5 дней, через 30 дней, темно-коричневые выделения из половых путей за 5–6 дней до менструаций, 6 лет назад произведена радиоволновая эксцизия шейки матки по поводу хронического цервицита. Родов-2. ПРИ ОСМОТРЕ В ЗЕРКАЛАХ: влагалищная часть шейки матки до 1 см, по передней губе синюшные «глазки» 0,7 и 0,9 см, при бимануальном исследовании патологии не выявлено. Какие исследования необходимо провести для уточнения диагноза? Кольпоскопия

пцр на уги

проверено

74.Пациента 27 лет поступила в отделение гинекологии на плановое оперативное лечение по поводу дермоидной кисты яичника. Какой объем оперативного лечения приемлем при дермоидной кисте яичника? Кистэктомия

проверено

75.У пациентки 16 лет, при обследовании в отделение гинекологии, обнаружена тератома яичника, какая тактика лечения является оптимальной? Резекция яичника

76.Выберите метод контрацепции для молодой девушки 19лет, с не регулярным менструальным циклом и болезненными менструациями: Гормональные

проверено

77.Женщина 29 лет, на приеме у гинеколога назначено лечение функциональной гиперпролактинемии. Какое лечение назначено врачом?

Бромокриптин с1/4 таб в день вечером во время еды, затем 2 раза в день по ¼ и ½, увеличивают постепенно до 1-3 таб 1 раз в день; Лечение основного заболевания

проверено

78. Женщина 52 года, обратилась на профилактический осмотр к гинекологу. Жалоб нет. Менопауза 1 год. Врач после осмотра заподозрила изменения. Какой метод диагностики дисплазии шейки матки наиболее предпочтителен? Гистологическое исследование,мазок по ПапаНиколау

проверено

79.Больная 52 лет. Жалобы на общую слабость, снижение работоспособности, нарушение сна, депрессивное состояние, чувство страха, тревоги, раздражительность, плаксивость. Вышеперечисленные жалобы появились год назад, в последние месяцы усилились. Последняя менструация 10 месяцев назад. Гинекологические заболевания: предменструальный синдром в течение 10 лет. Выберите наиболее оптимальный метод лечения? ЗГТ (эстрадиол 1 мг внутрь 1 раз в сутки затем 17в эстрадиол 2 мг и дидрогестерон 10 мг внутрь 1 раз 14 сут

проверено

80.Женщина 29 лет, обратилась с жалобами на отсутсвие менструаций в течение 8 месяцев. Из анамнеза менархе с 13 лет, относительно регулярные. С 26 лет интервалы между менструациями увеличились, к 29 годам развилась аменорея. Рост 172. Вес 60кг. Телосложение, оволосение женского типа. Молочные железы развиты правильно. Отделяемого из сосков нет. При гинекологическом обследовании: умеренная гипоплазия матки. ЛГ -4,5, ФСГ – 5,2, пролактин -982. МРТ головного мозга – без особенностей. Назначено лечение функциональной гиперпролактинемии. Какое лечение назначено врачом?

Агонисты дофаминовых рецепторов, алкалоиды спорыньи ( бромокриптин, бромэргон, парлодел, квиноголид, каберголин

81. Пациентка 29 лет обратилась в ЦСМ на женскую консультацию с жалобами на обильные кровянистые выделения из половых путей, задержку менструации на 4 недели. Со слов пациентки, накануне вечером почувствовала тянущие боли внизу живота; утром обильные кровянистые выделения из половых путей. Пациентка живет половой жизнью, не предохраняется. Общее состояние удовлетворительное, пульс 72 минуту, АД 120/80 мм рт. ст. Гинекологическое исследование: цервикальный канал пропускает 1 палец, выделения кровянистые, обильные, со сгустками. Тело матки несколько увеличено, безболезненное. Придатки с обеих сторон не пальпируются. Выберите метод лечения?

Кюретаж полости матки, лечение постгеморрагической анемии, свечи спазмолитические

82. У пациентки Ф., 44 лет, после 3-месячной задержки менструации наблюдаются обильные кровянистые выделения из половых путей, продолжающиеся 10 дней. Последние 2 года менструальный цикл нарушен: интервал между менструациями составляет 2-3 месяца. Гинекологические заболевания отрицает. При осмотре в зеркалах: слизистая влагалища и шейки матки чистая. Выделения кровянистые, обильные. При бимануальном исследовании патологии не обнаружено. Диагноз:

Аномальное маточное кровотечение позднего репродуктивного возраста. Какой метод остановки кровотечение показан для данной пациентки?

A.Хирургический гемостаз

B.Утеротоники

C.Антифибринолитическая терапия

D.Гормональный гемостаз

E.Гистерэктомия

83.К врачу в ЦСМ обратилась пациентка О., 17 лет, с жалобами на редкие, скудные менструации. Менструации с 15 лет, через 38-49 дней, скудные, безболезненные. При осмотре отмечается небольшой рост пациентки, широкие плечи, узкий таз, короткие конечности, недоразвитость молочных желез, оволосение по мужскому типу. При гинекологическом осмотре: наружные половые органы развиты по женскому типу, отмечаются увеличение клитора, гипоплазия малых и больших половых губ. При ректоабдоминальном исследовании: матка меньше нормы, плотная, подвижная, безболезненная, придатки с обеих сторон не определяются, область их безболезненна. Предварительный диагноз: Адреногенитальный синдром (пубертатная форма). Какие пробы надо провести для уточнения диагноза?

Пробы с дексаметазоном, АКТГ,

84.Пациентка 45 лет обратилась с жалобами на обильные, со сгустками крови, месячные. Из анамнеза: в последние 6 месяцев отмечаются менструации по 8–9 дней через 26–27 дней, обильные, умеренно болезненные. Гинекологическое исследование: шейка матки без видимой патологии; тело матки не увеличено, контуры четкие, безболезненно при пальпации; придатки с обеих сторон не определяются; своды глубокие; выделения кровянистые, обильные. УЗИ: тело матки обычных размеров, полость матки не деформирована, эндометрий неоднородный, толщиной 15 мм. Придатки без особенностей. Выставлен диагноз «Гиперплазия эндометрия». Какое первоначальное лечение следует провести пациентке для улучшения ее прогноза?

Местное внутриматочное введение левонгестрел-рилизинг-внутриматочной системы или Непрерывный пероральный прием прогестагенов норэтистерон 10-15мг/сут

85. Больная 42 года, поступила в отделение гинекологии после аборта, с жалобами на тянущие боли внизу живота и пояснице, которые периодический усиливается и исчезают. В момент приступа болей беспокоит тошнота, озноб, повышение температуры тела, тахикардия. При влагалищном исследовании обнаружено образование в области придатков матки тугоэластической консистенции, резко болезненное при смещении.

Что является самым информативном методом диагностики?

Трансвагинальное УЗИ

86. Больная 68 лет половой жизнью не живет. Обратилась в отделение гинекологии, жалуются на тянущие боли внизу живота, пояснице, ощущение тяжести, давление, наличие инородного тела во влагалище. Осмотр шейки матки-патология не выявлена. Диагноз: Полное выпадение матки. Какой вид операции нужно провести в данном случае?

Влагалищная экстирпация матки с пластикой мышц тазового дна

87. У больной 24 лет на профилактическом гинекологическом осмотре вокруг наружного зева шейки матки выявлен участок ярко-красного цвета 0.5-0.7 мм с четкими контурами, который не кровоточит при контакте. При двуручном исследовании патологии не обнаружено. Ваша тактика?

Диагноз Эрозия шейки матки. Обследование на инфекцию, кольпоскопия. Местное лечение эрозии

88. Пациентка 23 лет обратилась на прием к гинекологу для получения рекомендаций по контрацепции. ИЗ АНАМНЕЗА: менструации с 14 лет, нерегулярные через 25–45 дней, по 6–7 дней, обильные, болезненные, принимает спазмалгон для купирования боли. Замужем. Были одни роды 3 года назад. Соматически здорова. При гинекологическом обследовании и УЗИ патологии не выявлено. Какой метод контрацепции рекомендовать данной пациентке?

Комбинированные эстроген гестагенные низкодозированные монофазные контрацептивы Ярина, Регулон ВМС

89.На прием к гинекологу обратилась пациентка 40 лет с жалобами на обильные менструации, нуждается в надежной контрацепции. ИЗ АНАМНЕЗА: родов 2, без особенностей, 6 медицинских абортов. Три месяца назад при обследовании по поводу обильных менструаций проведена гистероскопия, гистологическое исследование соскоба эндометрия. Заключение: простая гиперплазия эндометрия без атипии. Принимала КОКи в течение 2 месяцев. Отмечает побочные эффекты в виде тошноты, головной боли. PV: шейка матки чистая, тело матки не увеличено, безболезненное. Придатки с обеих сторон не пальпируются. Какой метод контрацепции рекомендовать данной пациентке?

Барьерный

90.Пациентка 23 лет обратилась в женскую консультацию с жалобами на боли внизу живота, повышение температуры тела до 38°С, гнойные выделения из половых путей, рези при мочеиспускании. Из анамнеза: Половая жизнь с 17 лет, не замужем. Заболела остро. Объективно: ps-96 в 1 минуту живот при пальпации мягкий, болезненный в нижних отделах. Гинекологическое исследование: уретра инфильтрирована, шейка матки гиперемирована, отечна, с обширной эрозией, из шеечного канала обильные слизисто-гнойные выделения. Матка не увеличена, болезненная при пальпации, придатки с обеих сторон утолщены, болезненны, своды глубокие.DS: Свежая острая восходящая гонорея. Определите тактику врача женской консультации.

Абт, натриевая соль бензилпенициллина 6 млн однократно в\м по 3 млн в каждую ягодицу

Новокаиновая соль бензилпенициллина 4.8 млн однократно

Прокаин пенициллин G 6 млн однократно

91. Пациентка 30 лет обратилась в женскую консультацию с жалобами на отсутствие беременности в течение 3 лет в браке, зуд, жжение, болезненность при мочеиспускании (дизурия), болезненность при акте дефикации. Из анамнеза: менструальная функция не нарушена. Получала местное лечение. Исследование в зеркалах: влагалище свободное, слизистые бледно-розовые, шейка матки цилиндрической формы не эрозирована, наружный зев точечной формы, выделения слизистые. Бимануальное исследование: матка в anteversio-flexio, не увеличена, безболезненная; придатки не определяются, своды глубокие. Обследована на ИППП: выявлена ДНК Ch. trachomatis. Какой препарат целесообразно назначить для лечения данного заболевания?

Препараты тетрациклинового ряда – доксициклин азитромицин

Госпитальная хирургия (147)

Запоминание:

1. Поясничная симпатэктомия показана при: облитерирующего эндартериита и облитерирующего атеросклероза сосудов нижних конечностей.ПРОВЕРЕНО

2.Какой оперативный доступ применяется при эмболэктомии из бифуркации аорты? двусторонний бедренный.ПРОВЕРЕНО

3.Какое типичное место для дренирования плевральной полости с целью эвакуации крови при гемотораксе? 7-8 м/р ПРОВЕРЕНО

4.Что такое гнойный медиастинит? гнойное воспаление рыхлой соединительной клетчатки средостения.ПРОВЕРЕНО

5.Что определяет исход гнойного медиастинита? как быстро была проведена операция/качество дренирования.ПРОВЕРЕНО

6.С чем связана дисфагия после ваготомии? денервацией дистального отдела пищевода, периэзофагеальным воспалением, послеоперационным эзофагитом. Для лече­ния используют прокинетики — мотилиум, цизаприд (координакс), алюми­ний, содержащие антациды (альмагель, фосфалюгель, маалокс).ПРОВЕРЕНО

7.Какой из нижеперечисленных методов может быть применен при исследовании кишечных свищей? обзорная рентгенография, фистулография, ирригоскопия, пассаж бария по кишечнику.ПРОВЕРЕНО

8.Чем обусловлен истинный постхолецистэктомический синдром? Спазм/стеноз БДС. ПРОВЕРЕНО

9.Какая патология относится к надпечёночной блокаде портального кровообращения? болезнь Киари, синдром Бадда-киари, цирроз пика сердечного кровообращениятромбоз печеночных вен.ПРОВЕРЕНО

10.Какой метод применим в диагностике абсцесса печени? чаще узи и кт,радиоизотопное исследование и целиакографияредко.ПРОВЕРЕНО

11.Характерным для острого панкреатита осложнением является: разлитой перитонит, гиповолемический шок, опн, печ недост, абсцессы пж, свищи, пж некроз,острая ложная киста,плеврит и перикардит.ПРОВЕРЕНО

12.Какой из микроорганизмов наиболее часто обнаруживается в экссудате при разлитом гнойном перитоните? e.coli, enterobacteriaceae, streptococcus spp, enterococcus spp, clostridium spp staphylococcus/pseudomonas aeruginosa-

внутрибольничная.ПРОВЕРЕНО

13.В какой период разлитого гнойного перитонита характерен дефицит калия в крови?

A.Реактивный

B.Токсический

C.Терминальный

D.Начальный

E.Компенсированный

14.Какая операция показана при облитерирующем эндартериите в период развития влажной гангрены стопы, сопровождающейся отеком, лимфангитом и лимфаденитом? ампутация конечности.ПРОВЕРЕНО

15.Показание к экстренному хирургическому лечению болезни Крона: профузное кровотечение, перфорация тонкой/толстой кишки, токсическая дилятация ободочной кишки.ПРОВЕРЕНО

16.Какая патология относится к надпечёночной блокаде портального кровообращения: болезнь Киари, синдром Бадда-киари, цирроз пика сердечного кровообращениятромбоз печеночных вен.ПРОВЕРЕНО

17.Какой из микроорганизмов наиболее часто обнаруживается в экссудате при разлитом гнойном перитоните?e.coli, enterobacteriaceae, streptococcus spp, enterococcus spp, clostridium spp stafilococcus/preudomonas aeruginosa-

внутрибольничная.ПРОВЕРЕНО

18.Показание к хирургическому лечению дивертикулезе толстого кишечника: перитонит, перфорация непроходимость, абсцесс или свищ. Кровотечение безуспешное медикам лечен ПРОВЕРЕНО

Понимание

1.Пациент, 32 года, жалуется на перемежающуюся хромоту интенсивного характера, возникающую после прохождения 200 метров. Кожа стоп и голеней потеряла эластичность, сухая, шелушится, ногти утолщены, ломкие. Пульсация на артериях стопы не определяется. Для какой стадии облитерирующего эндартериита характерен этот симптомокомплекс? СТАДИЯ СУБКОМПЕНСАЦИИ ПРОВЕРЕНО

2.У больного 30 лет с деструктивным панкреатитом на 14-й день заболевания появились гектическая температура, озноб, тахикардия, сдвиг лейкоцитарной формулы влево, инфильтрат в эпигастрии. Это может быть обусловлено:

нагноившЕЙСЯся псевдокистОЙ поджелудочной железы ПРОВЕРЕНО

3.Больной 48 лет страдает варикозной болезнью нижних конечностей в течение десяти лет. Обратился с жалобами на боли по ходу варикозно расширенной вены левой голени, умеренную отечность стопы, увеличивающуюся к вечеру. Объективно: имеются варикозно расширенные вены за счет v. saphena magna в области голеней и бедер. По внутренней поверхности левого бедра в нижней трети определяется гиперемия по ходу большой подкожной вены до 5 см. При пальпации отмечается резкая болезненность в области нижней трети левой

голени, инфильтрат. Ваш диагноз острый тромбофлебит подкожных вен

ПРОВЕРЕНО

4.У больного 32 года появился резкий отек всей правой верхней конечности после резкого подъема тяжести, боль распирающего характера, не может пальцы кисти сжать в кулак. Движения, чувствительность сохранены. Верхняя конечность увеличена в объеме на всем протяжении, отек распространяется в подключичную область и передне боковую поверхность грудной клетки. Ваш предварительный диагноз? Острый тромбоз подключичной вены ПРОВЕРЕНО

5.У больной 36 лет после склерозирующего лечения варикозных вен появились распирающего характера боли в икроножных мышцах левой голени, отек стопы и голени, t=37.8ºС. Положительный симптом Мозеса. Какое осложнение развилось? Тромбоз глубоких вен голени ПРОВЕРЕНО

6.На ФАП обратилась больная 45 лет с жалобами на наличие язвы в нижней трети правой голени, боли в правой голени и стопе ломающего характера, возникающие к концу дня, отек стопы и голени, периодические судороги в нижних конечностях. При осмотре правой голени в н/3 над внутренней лодыжкой имеется язва диаметром 4 см с неровными краями, глубиной 0,5 см, дно и стенки покрыты некротическим налетом, отделяемое умеренное гнойное, зловонное. Отмечается расширение большой подкожной вены: на бедре- в виде мягкого шнура, а в н/3 бедра и по всей голениварикозное изменение. При пальпации вены и её узлов болезненности и уплотнений не обнаружено. Пульсация артерий определяется хорошо. Ваш диагноз? Трофическая язва ПРОВЕРЕНО

7.Больной 49 лет экстренно доставлен с жалобами на острую боль в левой нижней конечности, онемение, похолодание в ней. Заболел остро, внезапно. Страдает ревматизмом, состоит на учете у кардиолога. Анальгетики боль не снимают. Доставлен через 16 часов от начала заболевания, ранее жалоб со стороны нижних конечностей не было. Предварительный диагноз:

эмболия артерии Левой нижней конечности ПРОВЕРЕНО

8.Больная 74 года, обратилась к хирургу в поликлинике с жалобами на боли по ходу вен левой голени, отеки на стопах, чаще в ночное время. Из анамнеза выяснилось, что больная страдает варикозной болезнью сосудов нижней конечности в течении 3—лет. Локально: на бедрах и голени имеются варикозно-расширенные вены. По внутренней поверхности левого бедра в нижней трети определяется гиперемия по ходу v. saphena magna. пальпаторно отмечается резкая болезненность в области

нижней трети левой голени и инфильтрат. Ваш предварительный диагноз? - Острый тромбофлебит подкожных вен Проверено

9.У больного 58 лет, страдающего ревматическим митральным стенозом и мерцательной аритмией, внезапно появились боли в правой нижней конечности. При осмотре отмечается выраженный отек правой ноги. Кожные покровы конечности цианотичны с усилением подкожного венозного рисунка. Пальпация зон сосудистого пучка на бедре болезненна. Пульсация на артериях стопы определяется отчетливо. Укажите диагноз: - Острый подвздошно-бедренный венозный тромбоз Проверено

10.При рентгеноскопии легких у больного 45 лет выявлена инфильтрация средней и нижней доли правого легкого, на фоне которой отмечены множественные участки разрежения ткани с горизонтальными уровнями и тенями секвестров. Какой ваш предварительный диагноз? - Гангрена легкого Проверено

11.Больная В., 19 лет, в течение 13 дней находится на лечении в терапевтическом отделении по поводу нижнедолевой левосторонней пневмонии. В течении последних 3-х дней начала повышаться температура до 39 0С, боли в левой половине грудной клетки, одышка. На рентгенограмме грудной клетки отмечается гомогенное затемнение нижнего и среднего легочного поля, средостение смещено вправо. Диагноз? - Парапневмонический плеврит (эмпиема

плевры) Проверено

12.У больной с абсцессом правого легкого появились резкие боли в грудной клетке, одышка. На обзорной рентгенограмме органов грудной клетки признаки коллабирование правого легкого, широкий горизонтальный уровень жидкости, резкое смещение тени средостения влево. Ваш предварительный диагноз? -

Пиопневматорокс правого легкого Проверено

13.У больного, 59 лет, вовремя бужирования пищевода вслепую бужом №32 по поводу его послеожоговой стриктуры появились резкие боли за грудиной, повысилась температура тела, появилась крепитация на шее. Какое осложнение бужирования возникло у данного больного? - Повреждение пищевода медиастинит Проверено

14.Пациент, 36 лет, во время употребления в пищу жареной рыбы внезапно ощутил резкие боли за грудиной. В течение последующих 2 часов состояние больного

прогрессивно ухудшалось: повысилась до 39°C температура тела. При объективном осмотре пациента боль за грудиной усиливается при запрокидывании головы, а также при поколачивании по грудине. Ваш предположительный диагноз? - перфорация пищевода и развитие острого медиастинита Проверено

15.52-летний больной, у которого ранее был диагностирован дивертикул Ценкера, доставлен в клинику с жалобами на сильные боли за грудиной и между лопатками, возникшие в момент интенсивной рвоты. Больной в состоянии шока, температура – 39,5 С. Лейкоцитоз – 20 000. Рентгенологически – скопление воздуха и жидкости в левой плевральной полости и средостении. Ваш предварительный диагноз.- Перфорация дивертикула пищевода Проверено

16.У больной, 32 лет, при рентгенологическом исследовании грудной клетки в переднем верхнем средостении выявлено опухолевидное образование размером 8

х 7 см. О каком заболевании следует подумать? тимома Проверено

17.Мужчина 29 лет. Жалоб нет. При профилактическом флюорографическом исследовании в средостении обнаружено дополнительное образование. Объективно: состояние удовлетворительное. По органам - без особенностей. На рентгенограммах органов грудной клетки в прямой и боковой проекциях, в среднем этаже переднего средостения овальной формы образование, в толще которого выявляются более плотные включения, наружные контуры его четкие, гладкие. Прилежащие отделы легкого не изменены. При КТ исследовании в среднем этаже переднего средостения опухоль с четкими контурами. Ваш предположительный диагноз? - тератома Проверено

18.Женщина 39 лет. Жалобы на тупые боли в груди и спине. Анамнез: впервые неприятные ощущения в груди отметила полгода назад. Постепенно присоединилась боль в груди и спине. Объективно: состояние удовлетворительное. По органам без особенностей. При рентгенологическом исследовании органов грудной клетки в реберно-позвоночном углу справа на уровне Th IV-V интенсивное овальной формы образование однородной структуры, с четкими контурами, размерами 9,0х4,0 см, широким основанием тесно прилежит к телам позвонков. У верхнего и нижнего полюсов медиастинальная плевра оттеснена под тупым углом. Отмечается краевая узурация прилежащих тел позвонков и ребер, на уровне узла межреберные промежутки сзади расширены. Ваш предположительный диагноз? неврогенная

опухоль заднего средостения. Проверено

19.Больной 53 лет, жалуется на боли за грудиной, изжогу, которые усиливаются после еды и в положении лежа. В положении стоя боли уменьшаются. В анализах крови – умеренная гипохромная анемия. О каком заболевании Вы думаете? ЯЗВА ЖЕЛУДКАПроверено

20.У больной 40 лет в течение 5 лет изжога. Три месяца назад ухудшилось состояние, ночью пища из желудка стала затекать в ротовую полость, от чего больной вынужден спать полусидя. Появилось срыгивание в наклоне. Какой ваш

предварительный диагноз? ГЭРБ / грыжа пищеводного отверстия

диафрагмы

Проверено

21.Больной упал с лестницы. В процессе обследования выявлен перелом ребер. Над левой половиной грудной клетки выслушиваются перистальтические шумы. На рентгенографии органов грудной клетки обнаружена петли кишечника. Ваш предварительный диагноз: РАЗРЫВ ДИАФРАГМЫ Проверено

22.Больной, 48 лет, предъявляет жалобы на ощущение урчания, переливания в левой половине грудной клетки. Вышеописанные жалобы появились у пациента после травмы: год тому назад больной упал с дерева. В первые минуты после падения пациент ощутил интенсивную боль в левой половине грудной клетки с однократной рвотой. В течение последующих 24 часов боль в левой половине грудной клетки купировалась самостоятельно. Однако постепенно больной стал ощущать нехватку воздуха при физической нагрузке, появилось урчание в левой половине грудной клетки. Ваш диагноз? травматическая грыжа диафрагмы

Проверено

Пациент 30 лет, обратился к хирургу с жалобами на перемежающуюся хромоту интенсивного характера, возникающую при ходьбе на 200 метров. Кожа стоп и голеней потеряла эластичность, сухая, шелушится, ногти утолщены, ломкие. Пульсация на артериях стопы не определяется. Для какой стадии облитерирующего эндартериита характерен этот симптомокомплекс? Проверено

23.При обследовании хирургом поликлиники женщины 26 лет отмечены жалобы на чувство затруднения глотания жидкой пищи, периодическую регургитацию. При эзофагоманометрии выявлено уменьшение релаксации пищеводного сфинктера при глотании. Ваш предварительный диагноз? ахалазия кардии. Проверено

24.Больной 53 лет обратился с жалобами на интенсивные распирающие боли в эпигастральной области после приема пищи, ежедневную рвоту с примесью желчи, приносящую облегчение. В анамнезе резекция желудка по Бильрот II по поводу язвы двенадцатиперстной кишки. Дефицит массы более 15 кг. Ваш предварительный диагноз: Постгастрорезекционный синдром (синдром

приводящей петли 3 стадия) Проверено

25.Через 3 месяца после резекции 2/3 желудка по Бильрот II в модификации Гофмейстера — Финстерера по поводу язвы желудка больной обратился с жалобами на слабость, возникающую через 15 минут после приема пищи, сопровождающуюся чувством жара в верхней половине туловища, резким потоотделением. Через некоторое время у него начинается головокружение, шум в ушах, учащенное сердцебиение, дрожание конечностей, затем присоединяется чувство усталости, сонливости, жидкий стул, полиурия. Болей в животе нет. Приступы более выражены при приеме пищи, богатой углеводами. После операции больной не прибавляет в массе. Дефицит массы тела составляет 8 кг.

Ваш предварительный диагноз? Дэмпинг синдром, 2 стадия Проверено

26.Больной 47 лет, доставлен родственниками в стационар с жалобами на появление слабости после приема сладкой и молочной пищи, потливость, головокружение, желание лечь отдохнуть. При опросе пациента хирургом выяснили, что три месяца назад больной перенес резекцию желудка. Ваш предварительный диагноз?

Демпинг синдром Проверено

27.Больной 49 лет прооперирован по поводу кровотечения с перфорацией 5 лет назад, отмечает боли похожие на прежние, язвенные, но локализующиеся в эпигастральной области слева. Отмечает рвоты на высоте болей и похудение. Консервативное лечение не эффективно. В кале реакция Грегерсена (+++). Ваш предварительный диагноз? пептическая язва анастомоза/ постгастрорезекционный синдром , рецидив язвы

Проверено

28.Больная 19 лет, обратилась к семейному врачу с жалобами на запоры до 7 дней, общую слабость, дискомфорт в левой половине живота. Объективно состояние больной относительно удовлетворительное. Пульс и артериальное давление в норме. Локально: язык чистый, влажный. Живот обычной формы, в акте дыхания

участвует равномерно. При пальпации умеренно болезненный в левой половине живота. Симптомов раздражения брюшины нет. На обзорной рентгенограмме органов брюшной полости обнаружено удлинение сигмовидной кишки. Ваш диагноз? Долихосигма Проверено

29.Больной предъявляет жалобы на кровотечение из прямой кишки, поносы, схваткообразные боли в животе, повышение температуры тела до 38 гр., тенезмы, периодическую рвоту, а также ухудшение общего состояния и потерю веса. При ректороманоскопии видна отечность и кровоточивость слизистой оболочки толстой кишки. Ваш предположительный диагноз? Неспецифический язвенный колит Проверено

30.У больного, страдающего язвенным колитом, появилось асимметричное вздутие живота, резко ослаблена перистальтика кишечника, появилась задержка стула, повысилась температура тела. Ваш диагноз.Токсический мегаколон Проверено

31.Больной 82 года, пенсионер. Обратился в хирургический стационар с жалобами на не отхождение стула и газов. В анамнезе 4 года назад был прооперирован по поводу гангренозного аппендицита. На обзорной рентгенограмме органов брюшной полости имеются чаши Клойбера. Ваш предварительный диагноз? Спаечная кишечная непроходимость Проверено

32.Больная 22 года студентка, обратилась к хирургу с жалобами на диарею и боли в животе в течении длительного времени, вздутие живота, следы крови в каловых массах, ложные позывы к акту дефекации, снижения массы тела, слабость. При колоноскопии обнаружены линейные язвы поперечные и продольные, дающие характерный эффект «булыжной мостовой». Ваш предварительный диагноз? Болезнь Крона Проверено

33.Пациент 44 года, доставлен машиной скорой помощи. Со слов больного болеет в течении суток. Жалобы на не отхождение газов и отсутствие стула, вздутие живота, тошноту, рвоту. На обзорной рентгенограмме барий в желудке, тонкой кишке, начальном отделе толстой кишки; петли кишечника на всем протяжении резко раздуты, в мезо- и гипогастрии уровни жидкости. Больной взят на операцию. При лапаротомии сигмовидная кишка, завернулась по часовой стрелке

на 540 градусов. Какой ваш диагноз? Странгуляциленах кишечная не проходимость (заворот сигмовидной кишки) Проверено

34.Больной 40 лет две недели назад был оперирован по поводу огнестрельного ранения брюшной полости с повреждением тонкой кишки. Было выполнено ушивание ранения тонкой кишки. На 8-ые сутки послеоперационного периода по дренажу из правой подвздошной области стало выделяться кишечное содержимое. Объективно: язык суховат, живот не вздут, мягкий, болезненный в области ран. Повязка справа промокла кишечным содержимым. Появилась мацерация кожных покровов. Ваш предварительный диагноз? Наружный тонкокишечный свищ Проверено

35.На пятые сутки после операции резекции 2-3 желудка по Бильрот-2 у больного по страховочному дренажу из правого подреберья стала поступать желчь. Каким видом свища скорее всего осложняется послеоперационный период у данного больного? несформировавшийся свищ двенадцатиперстной кишки Проверено

36.У больного, оперирующего по поводу кишечной непроходимости, из тощей кишки был извлечен желчный камень. Вследствие чего желчный камень явился причиной кишечной непроходимости? Образование внутреннего желчно-

кишечного свища Проверено

37.Больной 70 лет оперирован в плановом порядке по поводу калькулезного холецистита. интраоперационной холангиографии патологии не выявлено. На 3- и сутки после операции отмечено появление желтухи, боли в области правого подреберья с иррадиацией в спину, многократная рвота. Ваш диагноз:

Послеоперационная стриктура холедоха Проверено

38.66-летний больной внезапно заболел: боли в левой подвздошной области, повышение температуры. При осмотре некоторое напряжение мышц и болезненность в левой подвздошной области. Какой диагноз вероятен? Дивертикулит сигмы Проверено

39.Больная 22 года, поступила в хирургический стационар с жалобами на диарею, следы крови в каловых массах, боли внизу живота, снижение массы тела, общую слабость. А анамнезе болеет в течении 2-х месяцев. Локально язык чистый. Живот обычной формы. При пальпации живота отмечается умеренная болезненность в нижних отделах. Симптом Щеткина Блюмберга отрицательный. При колоноскопии обнаружены линейные язвы, дающие характерный эффект «булыжной мостовой». Какой поставите диагноз данной больной? Болезнь

Крона Проверено

40.Больной 52 года, доставлен на машине скорой помощи в стационар. Жалобы больного на боли в левой половине живота, тошноту, рвоту общую слабость. Объективно состояние средней тяжести, пульс и артериальное давление в пределах нормы. Локально язык суховат, при пальпации живот болезненный в левой половине. Симптомов раздражения брюшины нет. На ультразвуковом исследовании обнаружено локальное утолщение стенки пораженного отдела кишки. Ваш предварительный диагноз? Рак ободочной кишки Проверено

41.Больной 40 лет обратился в стационар с жалобами на боли схваткообразного характера по всему животу, не отхождение стула и газов, тошноту, рвоту. Из анамнеза болеет в течении 1 суток. Объективно: состояние средней тяжести, кожные покровы обычные, тахикардия. Локально язык сухой, обложен. Живот вздут, при пальпации болезненный вокруг пупка. На обзорной рентгенограмме имеются раздутые петли кишечника с горизонтальным уровнем жидкости. Какой ваш диагноз? Кишечная непроходимость (обтурационная) Проверено

42.Больной 20 лет, обратился в центр семейной медицины с жалобами на тупую, ноющую, постоянную боль в правом подреберье, повышение температуры тела до 38С, тошноту, рвоту, сухость во рту, общую слабость. На ультразвуковом обследовании обнаружена полость в печени, заполненная жидкостью. Какой ваш диагноз?- Абсцесс печени Проверено

43.У больного, 24 лет, перенесшего в недавнем прошлом амебную дизентерию и прошедшего курс лечения с последующим выздоровлением, появились нарастающие, довольно интенсивные боли в правом подреберье, усиливающиеся при дыхании. Повысилась температура и стала носить интермиттирующий характер. Появились изнуряющие ознобы. Больной стал прогрессивно терять в массе. При обследовании определяется болезненная значительно увеличенная печень. Стул и мочеиспускание в норме. Лейкоцитов в крови 17,0 х 109 /л. Ваш диагноз?Амебный абсцесс печени Проверено

44.Больная, 69 лет, поступила в хирургическое отделение с жалобами на периодические боли в левом подреберье с повышением температуры тела до субфебрильных цифр по вечерам. В анамнезе 2 месяца назад травма левой половины туловища. При объективном обследовании отмечается значительное увеличение селезенки. При УЗИ определяется жидкостное образование 15х10 см с неоднородностью структуры в проекции селезенки. О каком диагнозе нужно подумать у больной? Абсцесс селезенки Проверено

45.Больная 80 лет. Обратилась к семейному врачу с жалобами на боли в эпигастрии, после приема жирной и жаренной пищи, тошноту, сухость во рту, общую слабость. В анамнезе вышеуказанные жалобы беспокоят в течении одного года. Локально язык суховат, обложен. Живот умеренно увеличен в объеме за счет подкожно жировой клетчатки. При пальпации живот болезненный в эпигастральной области и левом подреберье. Симптомов раздражения брюшины нет. Ваш предварительный диагноз? Хронический панкреатит Проверено

46.Больной обратился к семейному врачу с жалобами на боли во всех отделах живота, тошноту рвоту, сухость, во рту, общую слабость. Из анамнеза болеет в течении 1 суток. Локальный статус: язык сухой, обложен белым налетом. Живот при пальпации болезненный во всех отделах. Положительный симптом Щеткина Блюмберга. Ваш предварительный диагноз? Разлитой перитонит Проверено

47.Больной 25 лет доставлен в стационар на машине скорой помощи с жалобами на боли и наличие раны голени с гнойным отделяемым, повышение температуры тела до 38С, учащенное сердцебиение, общую слабость. Объективно общее состояние больного тяжелое, пульс 112 ударов в минуту, тахипноэ частота дыхания 28 в минуту. Локально на голени имеется рана с гнойным отделяемым со зловонным запахом размером 3,0х3,0 см. диагноз инфицированная раны голени. Какое осложнение развилось у данного больного? Сепсис Проверно

48.Больная 38 лет, поступила в хирургическое отделение больная. Жалобы при поступлении на боли внизу живота, больше в правой подвздошной области. Из анамнеза известно, что болеет три дня. Около 12 часов боли распространились на все нижние отделы живота. При осмотре общее состояние больной тяжелое, в сознании, кожные покровы пониженной влажности, язык сухой. Пульс до 118 в минуту. Живот умеренно вздут. При пальпации болезненный в нижних отделах, больше справа, там же напряжение мышц передней брюшной стенки. Симптом Ситковского положительный. Больной выставлен диагноз острый деструктивный аппендицит. Какое осложнение развилось у данной больной? - Разлитой

перитонит Проверено

49.Больная 19 лет поступила в приемное отделение через 3 дня от начала заболевания, когда впервые появились боли внизу живота над лоном, которые затем распространились в левую и правую подвздошную область. Боли носят постоянный характер. В сознании. Кожные покровы бледноватой окраски. Температура 38,8. Тоны сердца тахиаритмичные, пульс – до 110 ударов в минуту. В анализах сдвиг лейкоцитарной формулы в влево. Язык сухой, обложен. Живот вздут в нижних отделах, в акте дыхания участвует поверхностно. При пальпации живота резко болезненный в нижних отделах, там же напряжение мышц передней брюшной стенки. Ваш предварительный диагноз? Перитонит

(пельвиоперитонит) Проверено

50.Больному 42 лет в плановом порядке по поводу язвы желудка была выполнена резекция желудка по Гофмейстеру-Финстереру. На 3-е сутки после операции состояние больного резко ухудшилось. Появились резкие боли в эпигастральной области, которые распространились по всему животу. Дыхание поверхностное, пульс до 120 ударов в минуту. Живот вздут, при пальпации болезненный во всех отделах, напряжен. Перистальтика не выслушивается. - Послеоперационный

перитонит Проверено

51.В хирургическое отделение поступила больная 30 лет, через 4 сутоки от начала заболевания. Жалобы на боли в правой подвздошной области, над лоном. Из анамнеза известно, что боли начались в эпигастрии и через несколько часов переместились в правую подвздошную область. При осмотре общее состояние больной тяжелое, в сознании, кожные покровы пониженной влажности, язык сухой, обложен. Пульс до 110 в минуту. Живот умеренно вздут и напряжен в нижних отделах. При пальпации в нижних отделах отмечается резкая болезненность. симптомы раздражения брюшины положительные. Ваш предварительный диагноз? Острый дестр аппендицит с прорывом в брюшную полость или разлитой перитонит Проверено

52.Больной 40 лет, обратился в центр семейной медицины с жалобами на утомляемость ног, похолодание нижних конечностей, синюшность кожных покровов ног, после длительной ощущение жжения в пальцах ног. Объективно состояние удовлетворительное. Пульс 84 удара в минуту, АД 140/80 мм рт ст. Локально нижние конечности синюшного цвета, холодные на ощупь, чувствительность сохранена, пульсация на артериях нижних конечностей

ослабленная. Ваш предварительный диагноз? -Облитерирующий эндартериит

Проверено

53.Больной доставлен в хирургический стационар на машине скорой помощи с жалобами на внезапные острые боли, отечность и цианоз кожи в паховой области и верхней трети бедра. Из анамнеза со слов больного выше указанные жалобы беспокоят в течении 2 часов. При локальном осмотре правое бедро отечная, имеется выраженный цианоз кожи бедра, при пальпации отметается болезненность. Ваш предварительный диагноз? Илеофеморальный тромбоз

глубоких вен Проверено

54.Больной 36 лет, обратился в центр семейной медицины с жалобами на повышение температуры тела до 38С, озноб, кашель с трудно отделяемой мокротой, одышку, боли за грудиной, общую слабость. Со слов больного выше указанные жалобы беспокоят в течении недели, после простуды. Объективно состояние средней тяжести. Грудная клетка цилиндрической формы, в акте дыхания отстает справа, голосовое дрожание ослаблено справа, перкуторно притупление легочного звука справа. На обзорной рентгенограмме органов грудной клетки - интенсивное гомогенное затемнение правой половины грудной клетки, со смещением средостения в здоровую сторону. Ваш предварительный диагноз?

Правосторонний экссудативный плеврит \ эмпиема плевры Проверено

55.Больная 46 лет. Обратилась к терапевту в центр семейной медицины. С жалобами на умеренные боли за грудиной. После полного обследование у больной обнаружена киста средостения. Какие ваши действия, как семейный врач к какому специалисту направите больную? К хирургу Проверено

56.Больной 25 лет, обратился в центр семейной медицины с жалобами на наличие грыжевого выпячивания в боковой области живота, появление боли при поднятии тяжести. При осмотре на передней брюшной стенки, между полулунной линией (граница между мышечной и сухожильной частью поперечной мышцы живота) и наружным краем прямой мышцы имеется опухолевидное выпячивание, тугоэластической консистенции. При пальпации умеренно болезненная. Ваш диагноз? Спигелиева грыжа(Боковая грыжа (грыжа полулунной линии))

Проверено

57.Больная 48 лет. обратилась в хирургический стационар с жалобами на чувство затруднения глотания жидкой пищи, периодическую регургитацию. При

эзофагоманометрии выявлено уменьшение релаксации пищеводного сфинктера при глотании. Ваш предварительный диагноз? Ахалазия кардии Проверено

58.Больная 42 года, доставлена в хирургический стационар с жалобами на жидкий стул с примесью крови и слизи до 4-6 раз в сутки, общую слабость, потерю веса, общую слабость, сухость во рту. В анамнезе выше указанные жалобы бесопокоят в течении 1 недели. Объективно: состояние средней тяжести, кожные покровы бледноватой окраски, язык влажный, обложен белым налетом. Локально живот обычной формы, в акте дыхания участвует равномерно. При пальпации живот болезненный в левой половине живота. Ваш предварительный диагноз. Болезнь

Крона илеит Проверено

59.Больной 52 года, находится на стационарном лечении в хирургическом стационаре. На 7-е сутки после операции по поводу панкреонекроза имеется нагноившаяся обширная рана передней брюшной стенки, дном которой является петля тонкой кишки. Принятая больным пища через 20-30 минут выделяется через рану, также имеет место обильное выделение из раны кишечного сока, окрашенного желчью. Какое послеоперационное осложнение наступило у больного:Свищ Проверено

60.Пациент 44 года доставлен братом в дежурную хирургию с жалобами на боли в правом подреберье, тошноту, рвоту, повышение температуры тела до 37,5С. Объективно состояние средней тяжести. Пульс и артериальное давление в пределах нормы. Локально язык сухой обложен белым налетом. Живот при пальпации болезненный в правом подреберье. Симптомов раздражения брюшины нет. На ультрасонографии органов брюшной полости в толще печени визуализируется образование округлой формы с четкими границами. Проба Кацони положительная. Ваш предварительный диагноз? Эхинококкоз печени

Проверено

61.Больная 44 года, доставлена в хирургический стационар к хирургу с жалобами на боли в левом подреберье, тошноту, рвоту, повышение температуры тела до 38С. Из анамнеза болеет в течении недели. Локально язык сухой обложен белым налетом. Живот при пальпации болезненный в левом подреберье. При ультразвуковом исследовании в полости селезенки обнаружено полостное образование с четкими границами. Ваш предварительный диагноз?Абсцесс

селезенки Проверено

62.Больная 38 лет. Обратилась центр семейной медицины с жалобами на боли в эпигастрии, после приема жирной и жареной пищи, тошноту, сухость во рту, общую слабость. В анамнезе вышеуказанные жалобы беспокоят в течении 6 месяцев. Локально язык суховат, обложен. Живот умеренно увеличен в объеме за счет подкожно жировой клетчатки. При пальпации живот болезненный в эпигастральной области и левом подреберье. Симптомов раздражения брюшины нет. Ваш предварительный диагноз? Хронический панкреатит Проверено

63.Больной 39 лет, доставлен в хирургический стационар на машине скорой помощи с жалобами на боли во всех отделах живота, тошноту рвоту, сухость, во рту, общую слабость. Из анамнеза болеет в течении 1 суток. Объективно: состояние больного тяжелое, артериальное давление 90/60 мм рт ст, пульс 106 ударов в минуту. Локальный статус: язык сухой, обложен белым налетом. Живот при пальпации болезненный во всех отделах. Положительный симптом Щеткина Блюмберга. Ваш предварительный диагноз? Распространенный перитонит Проверено

Применение

1.Больной 28 лет, отмечает, что 2 дня назад внезапно левая конечность похолодела, онемела, боль нарастает. Из анамнеза: страдает ревматизмом. Объективно: движения в пальцах стопы отсутствуют. Локально левая нижняя конечность в пальцах стопы, голеностопном суставе движение отсутствует, чувствительность глубокая отсутствует. Голень увеличена в объеме. Кожа холодная, цвета мраморного рисунка. Пульсация отсутствует на всех уровнях. Определите

лечебную тактику. ТРОМБОЭМБОЛИЯ / ДОПЛЕР СОСУДОВ АНГИОГРАФИЯ, АМПУТАЦИЯ левой конечности Проверено

2.Больной 38 лет. Доставлен в хирургический стационар с жалобами на боли за грудиной, одышку при нагрузке, кашель, ограничение двигательной активности, повышение температуры тела до 38С, сухость во рту, общую слабость. Из анамнеза установлено, что вышеуказанные жалобы беспокоят в течении 3-х недель. Пациент дома самостоятельно принимал обезболивающие, из-за нехватки времени за медицинской помощью никуда не обращался. Со временем состояние больного ухудшилось, одышка начала беспокоить даже в покое. В стационаре была сделана обзорная рентгенограмма органов грудной - заключение абсцесс легкого осложненная эмпиемой плевры. Ваша тактика лечения? САНАЦИЯ И

ДРЕНИРОВАНИЕ С АБТ Проверено

3.У больного 42 года явления ишемии III степени на левой нижней конечности, заболел остро внезапно 3 года назад, проходит не более 30 метров. Ранее перенес инфаркт миокарда. Правая нижняя конечность не беспокоит. Шумовой симптоматики на нижней конечности нет. Пульсация на правой нижней конечности сохранена на всех уровнях, на левой – только на бедренной артерии. На ангиографии – окклюзия левой подколенной артерии с заполнением ее ветвей через коллатерали. Какая операция показана данному больному? Ауто- (алло) пластика подколеннойартерии Проверено

4.У больного 52 года, страдающего облитерирующим тромбангиитом, ранее ампутированы обе нижние конечности, страдающего остеохондрозом шейного отдела позвоночника, высоким АД, ИБС, перенес инфаркт 2 года назад, на верхних конечностях 4 месяца назад появились в области ногтевых фаланг слева на оставшихся 4, справа на всех 5 пальцах, поэтапно на одной, затем на другой вначале резкая бледность, боль, похолодание, затем цианоз, отек. Боли сильные, больной не спит. На лучевой артерии слева пульсации нет. Справа сохранена.

Тактика хирурга? грудная симпатэктомия с ампутацией пальцев в отдаленном

послеоперационном периоде не ранее чем через 2 месяца Проверено

5.Больная 56 лет обратилась в поликлинику к хирургу с жалобами на расширение подкожных вен нижних конечностей в течение 20 лет, чувство тяжести и отеки в области стоп и голеней к вечеру в течение 7-8 лет, работает продавцом. Изменение цвета кожных покровов по внутренней поверхности обеих голеней в течение 3 лет, открывалась трофическая язва. Объективно: нижние конечности бледно-розового цвета, по внутренней поверхности в нижней трети голени гиперпигментация кожи, кожа истончена, в виде «пергаментной бумаги». Здесь имеются рубец, расширенные подкожные вены на голени и бедрах за счет v. saphema magna. По ходу основного ствола и ее боковых ветвей увеличен объем голени на 2 см. Какая операция показана? Операция Троянова Проверено

6.При осмотре на дому вы диагностировали у больной острый тромбофлебит большой подкожной вены с уровнем распространения до средней трети бедра. Ваша тактика лечения? Экстренная госпитализация в хирургическок отделение Проверено

7.Больной 28 лет, находится на лечении в терапевтическом отделении с диагнозом острый абсцесс верхней доли правого легкого. На фоне общего благополучия появились острые резкие боли в правой половине грудной клетки, озноб, резкая

одышка, бледность с синюшным оттенком кожных покровов. Пульс – 130 ударов в 1 минуту. При аускультации легких справа - дыхание амфорическое. На рентгенограмме грудной клетки в правой плевральной полости определяется горизонтальный уровень жидкости до III ребра, легкое поджато на 1/2 объема, средостение смещено влево. Диагноз и тактика? Эмпиема Плевры ДРЕНИРОВАНИЕ ПО БЕЛОУ САНАЦИЯ Проверено

8.Больной 61 года, госпитализирован с жалобами на слабость, одышку, повышение температуры тела до 39 0С, кашель со слизисто-гнойной мокротой, боли в грудной клетке слева. Считает себя больным в течении двух месяцев после операции, ухудшение самочувствия в течении последней недели. Из анамнеза 2 месяца назад выполнена расширенная пульмонэктомия слева, по поводу центрального рака легкого. Рентгенологически: слева объем плевральной полости значительно уменьшен, средостение смещено влево, определяется уровень жидкости до II ребра. При пункции получен гнойный экссудат. Ваш диагноз и тактика лечения?Острая эмпиема остаточной полости слева/НАГНОЕНИЕ КУЛЬТИ ЛЕВОГО ЛЕГКОГО , Необходимо дренирование плевральной полости. Проверено

9.Больная 40 лет, обратилась в хирургический стационар с жалобами на боли за грудиной, повышение температуры тела до 38С, сухость во рту, общую слабость. В анамнезе больная болеет в течении 5 дней, дома не лечилась, за медицинской помощью никуда не обращалась. После проведенного полного обследования больной выставлен диагноз - острая тотальная эмпиема плевры. Какое лечение показано больному? САНАЦИЯ и ДРЕНИРОВАНИЕ по беллоу аб терапия Проверено

10.У больного 70 лет с острым абсцессом легкого диаметром до 10см., расположенном вблизи грудной стенки, имеет выраженные признаки тяжелой интоксикации. Какой метод лечения предпочтительнее? Дренирование абсцесса через грудную клетку Проверено

11.Больной 57 лет на протяжении последних 7 лет страдает выраженным рефлюксэзофагитом на почве диафрагмальной грыжи. Консервативное лечение неэффективно. При дуоденоскопии обнаружена язва ДПК. Тактика. Пилоропластика с ваготомией+ опер Ниссена Проверено

12.Больная 55 лет, на рентгенографии обнаружена смещение в средостение абдоминального отдела пищевода и кардиального отдела желудка и сглаживанием пищеводно-желудочной складки. Ваша тактика лечения:УСТРАНЕНИЕ ГРЫЖИ КРУРОГРАФИЯ Проверено

13.В дежурную больницу доставлен после автоаварии пациент 28 лет с клиникой острого живота. Во время экстренной лапаротомии установлено ущемление петли тонкого кишечника в щелевидном дефекте в области левого пояснично-реберного отдела диафрагмы. Такой же дефект имеется в области правого поясничнореберного отдела диафрагмы. Ваш диагноз и тактика лечение.ГРЫЖА БОХДАЛЕКА Проверено

14.Больной обратился к семейному врачу с жалобами на изжогу, боли за грудиной жгучего характера, отрыжку с примесью желудочного содержимого. В анамнезе больной страдает в течении одного года. После проведенного обследования больному выставлен клинический диагноз грыжа пищеводного отверстия диафрагмы. Ваша тактика лечения? консервативное лечение, Фундопликация по Nissen, Проверено

15.Больной 36 лет, жалуется на боли в горле, сухой кашель, ощущение инородного тела в глотке, першение. После контрастное исследования пищевода обнаружен ценкеровский дивертикул пищевода. Какое лечение должен назначить хирург больной с ценкеровским дивертикулом? УДАЛЕНИЕ ДИВЕРТИКУЛА Проверено

16.Пациент 60 лет, доставлен в стационар на машине скорой помощи с жалобами на нарушения при глотании пищи, срыгивание пищи, боли за грудиной. В анамнезе больной страдает ахалазией кардии в течении трех лет. Ваша тактика лечения?

КАРДИОДИЛАТАЦИЯ С ПОМОЩЬЮ БАЛЛОННОГО ПНЕВМАТИЧЕСКОГО КАРДИОДИЛАТАТОРА Проверено

17.Больной 47 лет прооперирован по поводу кровотечения с перфорацией 7 лет назад, отмечает боли похожие на прежние, язвенные, но локализующиеся в эпигастральной области слева. Отмечает рвоты на высоте болей и похудение. Консервативное лечение не эффективно. Какое заболевание следует подозревать в первую очередь и ваша тактика лечения? ПЕПТИЧЕСКАЯ ЯЗВА АНАСТОМОЗА Проверено

18.Больной 66 лет, обратился в хирургический стационар с жалобами на боли в верхних отделах живота, тошноту, рвоту с примесью желчи, изжогу, срыгивание пищи, снижение веса. В анамнезе установлено, что больной перенес резекцию желудка 2 года назад. После проведенного инструментального исследования выставлен диагноз синдром приводящей петли, тяжелой степени. Ваша тактика лечения: Реконструкция анастомоза по гофмейстеру-ф реконверсия

Бильрот-П в Бильрот-I или Бильрот-Н с гастроеюнальным анастомозом на выключенной по Ру петле Применяют также гастроеюнодуоденопластику

Проверено

19.У больного после селективной проксимальной ваготомии по поводу язвы ДПК через 2 года появились интенсивные боли в эпигастрии натощак, тошнота, рвота, рентгенологически выявлены признаки дуоденостаза. Выберите наиболее оптимальный метод лечения: Стволовая ваготомия, гостродуоденостомия по Джабулею гастроеюноанастомоз или дуоденоеюноанастомоз

20.Больной 70 лет доставлен в стационар на машине скорой помощи с жалобами на боли схваткообразного характера по всему животу, не отхождение стула и газов, тошноту, рвоту, общую слабость. Из анамнеза болеет в течении трех суток. Объективно: общее состояние больного тяжелое, пульс 108 ударов в минуту, артериальное давление 90/60 мм рт ст. Локально язык сухой, обложен. Живот вздут, при пальпации болезненный вокруг пупка. Симптом Щеткина Блюмберга положительный. На обзорной рентгенограмме выявлены «чаши Клойбера». Выставлен диагноз кишечная непроходимость. Ваша тактика лечения? Очистительная клизма и Экстренная операция

21.Больная В. 80 лет поступила в хирургический стационар с жалобами на неотхождение стула и газов, тошноту, рвоту каловыми массами, вздутие живота, схваткообразные боли по всему животу. Состояние больной тяжелое, тахикардия 110 ударов в минуту, АД 90/50 мм рт ст. На обзорной рентгенограмме органов брюшной полости имеются «чаши Клойбера». С диагнозом острая кишечная непроходимость больную взяли в операционную. Операционная находка – острая спаечная кишечная непроходимость с некрозом петель кишечника. Ваша тактика лечения? Рассечение спаек, резекция(40проксим,20дист) в пределах жизнеспособной кишки , анастамоз

22.Больной Ж. 82 года, пенсионер доставлен в стационар на машине скорой помощи с жалобами на не отхождение стула и газов, тошноту, рвоту каловыми массами,

вздутие живота, схваткообразные боли по всему животу. На обзорной рентгенограмме органов брюшной полости имеются арки и горизонтальный уровень жидкости. Во время операции у больного нашли опухоль ректосигмоидного отдела кишечника. Ваша хирургическая тактика? брюшноанальной резекции, или проводят операцию Гартмана , гистологическое исс.

23.Больная 79 лет оперированная по поводу опухоли сигмовидной кишки, проведена операция резекция сигмовидной кишки с опухолью, выведение колостомы, обратилась к хирургу в поликлинике. Больная жалуется на покраснения кожи вкруг свища, умеренные боли в области свища и кожный зуд. Во время перевязки в левой подвздошной области имеется наружный губовидный свищ, кожа вокруг свища гиперемирована, отечна, имеются участки мацерации, при пальпации умеренно болезненная. Ваша тактика лечения? Внебрюшинное закрытие губовидного свища (мельникова операция)

24.У больного 33 лет в области прямой кишки имеется свищ, который под влиянием консервативной терапии закрывается на короткое время, а при обострении процесса вновь открывается. Какая ваша хирургическая тактика? Выделение (иссечение)стенки кишки и ушивание двухрядным швом .

25.Больной 80 лет. В плановом порядке госпитализирован в хирургический стационар. У больного полностью сформировавшимся губовидный свищ тонкой кишки. Ваша тактика хирургического лечения: Циркулярная резекция вместе

со свищем (Внутрибрюшинная операция)

При небольших неполных губовидных свищах используют внебрюшинные способы их за­крытия. Для этого выделяют стенку кишки в зоне свища и ушивают дефект двухрядным швом. При больших неполных и при полных губовидных сви­щах показано применение внутрибрюшных способов закрытия

26.У больного после резекции ободочной кишки по поводу заворота на фоне разлитого гнойного перитонита что необходимо обязательно произвести во время операции из нижеперечисленных мероприятий? Формирование лечебного толстокишечного свища

27.Больной 50 лет, жалобы на боли в правом подреберье, желтушность кожных покровов. Из анамнеза болеет в течение 4 суток. При осмотре кожные покровы желтушные, при пальпации живот болезненный в правом подреберье. Симптом Ортнера положительный. На УЗИ обнаружен холедохолитиаз. Какая тактика лечения? Экстренная операция (холецистэктомия)

28.Больная 44 года, обратилась к хирургу с жалобами на приступообразные боли в правом подреберье, тошноту, горечь во рту, общую слабость. Из анамнеза болеет в течении 1 суток, после погрешности в диете, был прием жирной и жареной пищи. Какой инструментальный метод лечения примените для установки диагноза данной пациентки? Узи

29.Больная 40 лет. Жалобы при поступлении на тяжесть в правом подреберье, увеличение в объеме живота за счет асцита. В анамнезе больная страдает циррозом печени, осложненной портальной гипертензией в течении одного года. При инструментальном исследовании выявлено наличие жидкости в брюшной полости, патологическое увеличение селезенки в размере. Ваша тактика оперативного лечения? Спленоренальный венозный анастомоз без удаления селезенки, трансплантация печени

30.Эндоскопически установлено кровотечение из варикозных вен пищевода. Слизистая желудка не изменена. Какой из перечисленных методов гемостаза может быть применен в любом медучреждении? Тампонада зондом Блекмора, питуитрин в/в

31.Больная 43 лет поступила с жалобами на слабость, рвоту с примесью крови и мелену. При осмотре на передней брюшной стенке, выраженный венозный рисунок, пальмарная эритема. Какой метод исследования определить источник кровотечения? Эгдс

32.У больного, 15 лет по данным эзофагогастродуоденоскопии варикознорасширенные вены пищевода и признаки спленомегалии при УЗИ, в анамнезе повторные кровотечения, явления гиперспленизма. Какая операция направлена на снижение портальной гипертензии и устранение гиперспленизма? Спленэктомия и оменторенопексия

33.У больного, 46 лет, цирроз печени в течение 4-х лет. При УЗИ гепатомегалия, спленомегалия и умеренно выраженный асцит. В анамнезе эпизод кровотечения из варикозно-расширенных вен пищевода. Какое нужно применить более эффективное лечение с целью подготовки к трансплантации печени? Трансюгулярное портосистемное шунтирование

34.У больного с циррозом печени в анамнезе дважды наблюдалось кровотечение из варикозно расширенных вен пищевода. Какая операция показана с профилактической целью? ТАННЕРА, TIPS

35.Больной 40 лет, дальнобойщик обратился к хирургу с жалобами на тошноту, рвоту с кровью, головокружение, тяжесть в правом подреберье. Больной страдает циррозом печени в течении 3-х лет. В анамнезе у больного два года назад был эпизод кровотечения. Выставлен диагноз цирроз печени. Портальная гипертензия. Варикозное расширение вен пищевода осложненная кровотечением. Какая операция показана с профилактической целью? Операция таннера, TIPS или Эндоскопическое лигирование ВРВ пищевода

36.Больная 55 лет, массажистка обратился к хирургу с жалобами на тошноту, рвоту с кровью, головокружение, тяжесть в правом подреберье. Больная страдает циррозом печени в течении 6-ти лет. На гастроскопии обнаружено варикозное расширение вен пищевода осложненная кровотечением. Ваша тактика лечения? Эндоскопический Гемостаз (Зонд Блекмора), tips

37.Больная, 56 лет, поступила в хирургическое отделение с жалобами на чувство полноты в верхней половине живота. При объективном обследовании отмечается асимметрия живота за счет выбухания левого верхнего квадранта, пальпируется увеличенная селезенка. Возникло подозрение на кисту селезенки. Какой метод

инструментальной исследовании необходимо использовать при подтверждении диагноза? Узи

38.Больной 40 лет. Доставлен в стационар на боли в левом подреберье, головокружение, потемнение в глаза общую слабость. Из анамнеза больной за 1 час до поступления во время катания на велосипеде упал. На ультразвуковом исследовании выявлено разрыв селезенки, внутрибрюшное кровотечение. Ваша тактика хирургического лечения? Спленэктомия

39.Больная А., 69 лет, поступила в хирургическое отделение с жалобами на периодические боли в левом подреберье с повышением температуры тела до субфебрильных цифр по вечерам. В анамнезе 2 месяца назад травма левой половины туловища. При объективном обследовании отмечается значительное увеличение селезенки. При УЗИ определяется жидкостное образование 15х10 см с неоднородностью структуры в проекции селезенки. Выставлен диагноз абсцесс селезенки. Какая ваша тактика лечения? Спленэктомия, с ревизией б-р полости

40.Больной 30 лет обратился в хирургический стационар с жалобами на тупые боли

вэпигастрии, тошноту, рвоту, снижение веса, общую слабость. В анамнезе болеет

втечении 1 месяца. За медицинской помощью никуда не обращался. Объективно состояние больного средней тяжести. После полного обследования выставлен диагноз киста поджелудочной железы. Ваша тактика хирургического лечения? Энуклеация кисты или дистальную резекцию поджелудочной железы или операция Пьюстау-П

41.Больной обратился к семейному врачу с жалобами на выделение жидкости из послеоперационной раны. В анамнезе больной перенес операцию на поджелудочной железе. Выставлен предварительный диагноз свищ поджелудочной железы. Какой метод исследования назначите для подтверждения диагноза? Фистулография

42.Больной 50 лет обратился к семейному врачу с жалобами на боли в эпигастрии опоясывающего характера, тошноту, многократную рвоту, сухость во рту.

Выставлен предварительный диагноз панкреатит. Какой лабораторный биохимический анализ и инструментальный метод исследования назначите для уточнения диагноза? Альфа амилаза, липаза, УЗИ ПЖ

43.Больная Х. 33 года лет обратился в поликлинику к хирургу с жалобами на боли в эпигастрии опоясывающего характера, тошноту, многократную рвоту, сухость во рту. Выставлен диагноз острый панкреатит. Ваша тактика лечения? Антиферментные препараты, голодание, покой. Холод. (Некрсеквестрэктомия)

44.Больная поступила в хирургический стационар с жалобами на боли в эпигастрии опоясывающего характера, многократную рвоту, сухость во рту. Выставлен предварительный диагноз панкреатит. Какой лабораторный анализ необходимо назначить для уточнения диагноза? Альфа амилаза крови

45.Больная 40 лет, прооперирована по поводу острого гангренозного аппендицита, осложненного перитонитом. Какой метод интраоперационной желудочнокишечной декомпрессии примените у данной больной? Илеоасцендостомия (Назоинтестинальная интубация кишечника)

46.У больного установлен диагноз перитонит. После предоперационной подготовки больной взят в операционную. Какой оперативный доступ примените у данного пациента? Срединная лапаротомия

47.Больной 84 года, доставлен в хирургический стационар родственниками в тяжелом состоянии. Жалобы на боли во всех отделах живота, тошноту, рвоту, сухость во рту. Локально язык сухой, обложен. Живот при пальпации болезненный во всех отделах. Положительный симптом Щеткина Блюмберга. На ультразвуковом исследовании брюшной полости имеется жидкость во всех отделах. С диагнозом перитонит, больной взят на операцию. Операционная находка - разлитой гнойный перитонит, обусловленный перфоративной язвой

двенадцатиперстной кишки. Ваша тактика хирургического лечения? Ушивание язвы

48.В хирургический стационар обратился больной 20 лет. После полного обследования больному выставлен диагноз острый деструктивный аппендицит, осложненный распространенным перитонитом. Какой оперативный доступ примените у данного больного? Широкая (нижне) срединная лапаратомия

49.Больная 55 лет обратилась в приемное отделение хирургического стационара через 14 суток от момента заболевания, с жалобами на боли в подложечной области, затем боли переместились в правую подвздошную область, сопровождались тошнотой, сухостью во рту, рвотой. За медицинской помощью не обращалась, занималась самолечением. Локальный статус: язык сухой, обложен. Живот вздут. При пальпации болезненный во всех отделах. Положительные симптомы раздражения брюшины. Лейкоциты крови — 12,0×109/л, сдвиг лейкоцитарной формулы влево. Ваш диагноз и тактика лечения?

Перфоративный аппендицит расспространеный перитонит, Дренирование и санация брюшной полости.аппендэктомия аб терапия декомпрессия кишечника

50.Больной 43 года, страдающий язвенной болезнью двенадцатиперстной кишки, почувствовал интенсивные боли в верхних отделах живота. Обратился за медицинской помощью. Язык сухой. Живот вздут. В акте дыхания участия не принимает. При пальпации болезненный и напряженный во всех отделах. Положителен симптом Щеткина — Блюмберга. Перкуторно печеночная тупость отсутствует, определяется тимпанит. Какой метод инструментального обследования примените у данного пациента? Выполнить ЭГДС с последующим повторным рентгеновским снимком брюшной полости на свободный газ. Диагностическая лапароскопия .

51.Больной Т. 30 лет, доставлен на машине скорой помощи в дежурную хирургию. Жалобы на боли по всему животу, тошноту, рвоту, сухость во рту. Из анамнеза болеет сутки. состояние тяжелое, тахикардия 118 ударов в минуту, АД 100/60 мм рт ст. Локально язык сухой, обложен. живот при пальпации болезненный во всех отделах. Положительный симптом Щеткина Блюмберга. Выставлен диагноз

перитонит. Ваша тактика лечения? Лапаротомия с формированием лапаростомии и этапными санациями

52.Больная 55 лет, обратилась в сосудистое отделение хирургического стационара с жалобами на наличие расширенных вен на нижних конечностях, тяжесть, иногда боли в ногах, ночные судороги мышц, трофические изменения на голенях. Из анамнеза больная страдает варикозом нижних конечностей в течении 7 лет. Периодически лечилась амбулаторно. Локально при пальпации нижних конечностей пальпируются шнуровидные, болезненные уплотнения по ходу расширенных вен. После полного обследования выставлен диагноз варикозная болезнь нижних конечностей, осложнённая тромбофлебитом. Ваша хирургическая тактика лечения? Хирургическая операция (комбинированная флебэктомия) (операция Троянова-Тренделенбурга, Бэбкокка, Нарата)

53.Больная 45 лет, доставлена в хирургический стационар с жалобами на повышение температуры тела до 38С, озноб, кашель с трудно отделяемой мокротой, одышку, боли за грудиной, общую слабость. Со слов больной выше указанные жалобы беспокоят в течении 5 дней. Объективно состояние средней тяжести. Грудная клетка цилиндрической формы, в акте дыхания отстает справа, голосовое дрожание ослаблено справа, перкуторно притупление легочного звука справа. Какой инструментальный метод диагностики примените у данной больной? Рентген

54.Больной 37 лет, обратился в хирургический стационар с жалобами на повышение температуры тела до 38С, озноб, кашель с трудно отделяемой мокротой, одышку, боли за грудиной, общую слабость. Со слов больного выше указанные жалобы беспокоят в течении 3 дней. Объективно состояние средней тяжести. Грудная клетка цилиндрической формы, в акте дыхания отстает слева, голосовое дрожание ослаблено слева, перкуторно притупление легочного звука слева. На обзорной рентгенограмме органов грудной клетки - интенсивное гомогенное затемнение левой половины грудной клетки, со смещением средостения в здоровую сторону. Выставлен диагноз эмпиема плевры слева. Ваша хирургическая тактика лечения? Дренирование полости эмпиемы с проведением комплексной антибактериальной терапии, включая постоянное промывание гнойной полости растворами антисептиков.

55.Больная 74 года, доставлена родственниками в хирургический стационар. С жалобами на пульсирующую боль в груди с иррадиацией в межлопаточную область, усиление боли при надавливании на остистые отростки грудных

позвонков, усиление боли при глотании и вдохе, пастозность в области грудных позвонков, припухлость над ключицей, появление крепитации, ригидность длинных мышц спины. Больному планируется операция по поводу острого переднего гнойного медиастинита. Какой оперативный доступ примените у данной больной? Трансстернальный доступ

56.Больная 48 лет. доставлена на машине скорой помощи в хирургический стационар с жалобами на боли за грудиной, усиливающиеся после еды, изжогу, отрыжку, снижение массы тела. Из анамнеза болеет в течении четырех месяцев. После полного обследования больной выставлен диагноз Какой оперативный доступ примените у данной больной? Абдоминальный и чресплевральный доступы.

57.Больная 71 год, обратилась в хирургический стационар с жалобами на боли в эпигастральной области, тошноту, рвоту на высоте болей, черный стул и снижение массы тела. При опросе было выяснено, что больная 2 года назад перенесла операцию резекция желудка Бильрот II. Неоднократно лечилась стационарно и амбулаторно. На гастроскопии установлен диагноз пептическая язва анастомоза. Консервативное лечение не эффективно. Ваша хирургическая тактика? Селективная проксималь­ная ваготомия

58.Больная 58 лет, доставлена в хирургический стационар на машине скорой помощи с жалобами на интенсивные боли в эпигастрии натощак, тошноту, рвоту, общую слабость. Из анамнеза выяснено, что 3 года назад была проведена операция селективная проксимальная ваготомия по поводу язвы двенадцатиперстной кишки. На обзорной рентгенограмме выявлены признаки дуоденостаза. Ваша хирургическая тактика лечения? Оперативное, антрумэктомия (резекция 2/3 желудка).

59.Больная 65 лет, доставлена в хирургический стационар с жалобами на боли в верхних отделах живота, тошноту, рвоту приносящую временное облегчение. Из анамнеза болеет в течении 1 недели. Общее состояния относительно удовлетворительное. Язык суховат, обложен. Живот при пальпации мягкий, умеренно болезненный в эпигастрии. Симптомов раздражения брюшины нет. Выставлен предварительный диагноз хроническая дуоденальная непроходимость. Какое инструментальное исследование назначите для подтверждения диагноза? Фиброгастродуоденоскопия.

60.Больной 54 года, обратился в хирургический стационар с жалобами на наличие свища на передней брюшной стенке. В анамнезе больной 6 месяцев назад перенес операцию резекция кишечника с выведением колостомы. После полного

обследования выставлен диагноз: трубчатый свищ селезеночного угла толстой кишки. Ваша тактика инструментального исследования? Ирригоскопия или фистулографию

61.Пациент 62 года, доставлен в хирургический стационар с жалобами на боли в правом подреберье, желтушность кожных покровов, иктеричность склер, общую слабость. Из анамнеза болеет в течение 4 суток. При осмотре кожные покровы желтушные, при пальпации живот болезненный в правом подреберье. Симптом Ортнера положительный. На ультразвуковом обследовании выявлен камень холедоха размером 16 мм. Ваша хирургическая тактика лечения?

холецистэктомия

62.Больная 42 года обратилась в хирургический стационар к хирургу с жалобами на тошноту, рвоту с кровью, головокружение, тяжесть в правом подреберье. Больная страдает циррозом печени в течении 3-х лет. После полного обследования выставлен диагноз цирроз печени, портальная гипертензия, варикозное расширение вен пищевода осложненная кровотечением. Какая операция показана с профилактической целью у данной больной? Трансъюгулярное внутрипеченочное портосистемное шунтирование (TIPS)

63.Больной 44 года, доставлен родственниками в хирургический стационар с жалобами на тупые боли в эпигастрии, тошноту, рвоту, снижение веса, общую слабость. В анамнезе болеет в течении 1 месяца. За медицинской помощью никуда не обращался. Объективно состояние больного средней тяжести. После полного обследования выставлен диагноз киста поджелудочной железы. Ваша тактика хирургического лечения? энуклеацию кисты или дистальную резекцию

поджелудочной железы.

64.Больной 30 лет, доставлен на машине скорой помощи в хирургический стационар. После полного обследования установлен диагноз перитонит. После предоперационной подготовки больной взят в операционную. Какой оперативный доступ примените у данного пациента? срединная лапаротомия

65.Больная 34 года, доставлена сестрой в хирургический стационар с жалобами на боли в правой подвздошной области. Из анамнеза за 1 сутки до поступления боли начались в эпигастрии и через несколько часов переместились в правую подвздошную область. Общее состояние больной тяжелое, пульс 102 удара в минуту, артериальное давление 90/60 мм рт ст. Живот умеренно вздут и напряжен

в нижних отделах. Из-за напряжения мышц передней брюшной стенки исследование затруднено, пальпация в нижних отделах резко болезненна, определяются положительные симптомы раздражения брюшины. Ваш диагноз и тактика хирургического лечения данной больной? Перитонит Срединная лапаротомия

Дерматология (124)

Запоминание

1. Какой морфологический элемент относится к первичным? пятно/

узелок/

узел/

бугорок/ пузырек/ пузырь/ гнойничок/ волдырь

 

 

2.Укажите какое из нижеперечисленных заболеваний относится к стафилококковым пиодермиям. остеофолликулит,фолликулит,сикоз вульгарный,угри обыкновенные, эпидемическая пузырчатка новорожденных , + везикулопустулез, синдром стафилококковой обожженной кожи(SSSS), гидраденит,фурункул, карбункул Проверено

3.Укажите, какие рецепторы отвечают за тепловую чувствительность. терморецепторы колбы Краузе-холодов, тельца Руффини-тепловая Проверено

4.Назовите возбудителя отрубевидного лишая. fityrosporum ovals,orbiculare, malassezia fur fur. Проверено

5.Укажите возраст в котором наиболее часто встречаются плоские бородавки. Дети и подростки Проверено

6.Назовите дерматит, возникающий при воздействии на кожу концентрированных кислот. Контактный дерматит Проверено

7.Назовите первичный морфологический элемент при нейродермите. Узелок Проверено

8.Укажите лабораторный метод, который считается основным при постановке диагноза «Скрытый сифилис». Серологические реакции(РМП, ПЦР,РИФ, ИФА, РПГА) Проверено

9.Какова длительность цикла развития хламидий? 48-72 часа Проверено

10.Укажите вторичный морфологический элемент при генитальном герпесе. Трещины Эрозии, Язвы

11.Какие основные препараты применяются при лечении красной волчанки? Гидроксихлорохин, нпвс, глюкортикостероиды, иммунодепрессанты Проверено

12.Укажите основной антибиотик в терапии сифилиса. Бензилпенициллин Проверено

13.Укажите, что относится к первичным морфологическим элементам: пятно,пузырь,Волдырь,узел,пузырек,гнойничок,узелок,бугорок Проверено

14.Укажите, что относится к вторичным морфологическим элементам: 1. чешуйки,корки,эрозия,язва,трещина,вторичное пятно,рубец,лихенификация,экскориация,пигментации.

Понимание

1.Пациентка 37 лет госпитализирована в стационар с жалобами на появление высыпаний на коже туловища и конечностей. Беспокоит незначительный зуд. Объективно: на разгибательных поверхностях верхних и нижних конечностей и туловища имеются папулы различных размеров и формы, розовой окраски, покрытые серебристо-белыми чешуйками, симптом Кебнера и триада феноменов положительные. Для какого заболевания характерена вышеописанная клиническая картина? Псориаз Проверено

2.Женщина 35 лет обратилась к дерматологу с жалобами на появление пятен на коже туловища. Объективно: на коже верхней половины туловища имеются единичные пятна желтовато-бурого цвета с шелушением на поверхности. Проба Бальцера положительная. Какому диагнозу соответствует вышеописанная клиническая картина? отрубевидный лишай

3.Мужчина 40 лет, обратился в ЦСМ к дерматологу с жалобами на появление высыпаний на коже туловища. Высыпания появились 2 дня тому назад. Объективно: на коже туловища имеется сыпь, которая имеет розовую окраску, не возвышается над уровнем кожи, размером до 1 см в диаметре, при надавливании исчезает и вновь появляется на том же месте. Какой элемент описан в данной задаче? пятно

4.Женщина 62 года обратилась к врачу дерматологу с жалобами на повышение температуры, высыпания с серозным эксудатом в области губ рта, зуд, жжение. Диагноз: Простой пузырьковый лишай. Какой морфологический элемент описан при данном заболевании? пузырек

5.Женщина 50 лет обратилась к дерматологу с жалобами на высыпания на шее, болезненность при движении, повышение температуры. Болеет в течение 2-х недель. Сопутствующее заболевание: Сахарный диабет. Объективно: на коже задней поверхности шеи имеется инфильтрат застойно - синюшного цвета, с множественными пустулами и очагами некроза кожи. Из свищевых отверстий вытекает гнойно – кровянистый эксудат. У больного температура тела 39 ̊С, подчелюстные, шейные лимфатические узлы увеличены, болезненны. Выставьте правильный диагноз.Глубокая стафилодермия-карбункул

6.На приеме у дерматолога женщина 40 лет, с жалобами на появление пузырей, быстро подсыхающих в корки. Заболевание связывает с пыльной работой. При осмотре: на щеках и на лбу единичные плоские пузыри с тонкой, дряблой покрышкой и серозногнойным содержимым, до 5-10 мм в диаметре, а также большое количество светложелтых корочек на месте ссохшихся пузырей. Общее состояние удовлетворительное. Какой из нижеперечисленных предварительных диагнозов наиболее вероятен? пузырчатка( себорейная или эритематозная) Синдром Сенира-Ашера

7.В ЦСМ к дерматологу обратился мужчина 43 лет с жалобами на высыпания, сопровождающиеся вечерним и ночным зудом. Сопутствующих заболеваний нет. Из анамнеза: две недели назад ночевал в гостинице. При осмотре: на коже туловища, особенно на переднебоковых поверхностях, вокруг сосков, на верхних и нижних конечностях, в межпальцевых промежутках имеются многочисленные парные папула - везикулезные элементы, экскориации, кровянистые корочки. Симптомы Сезари, ГорчаковаАрди положительные. Субъективно: вечерний и ночной зуд. Какой из перечисленных ниже предварительных диагнозов наиболее вероятен?

чесотка Акародерматит

8.Пациенту 19 лет выставлен предварительный диагноз: Чесотка. При осмотре: на коже разгибательных поверхностей локтевых суставов имеются множественные пустулы, гнойные корочки, расчесы. Какой симптом описан в данной задаче? симптом горчакова-арди

9.Женщина 35 лет, обратилась к дерматологу с жалобами на высыпания и зуд в области стоп. Заболевание связывает с посещением бани 2 недели назад. Объективно: на коже свода стопы и ее боковых поверхностях, а также в межпальцевых промежутках на фоне эритемы и отечности имеются плоские папулы с плотной покрышкой, напоминающие саговые зерна, с шелушением и утолщением кожи на поверхности. Очаги по периферии окружены бордюром из отслаивающегося эпидермиса с мелкими пузырьками по краю, имеются трещины и мокнутия. Укажите наиболее вероятный диагноз. эпидермофития стоп

10.В ЦСМ на прием к дерматологу обратилась пациентка 30 лет, с жалобами на выпадение волос на голове. Дома содержит кошку. При осмотре на волосистой части головы определяется очаг размером 2 см в диаметре, волосы кажутся «неаккуратно выстриженными», на уровне 5-8 мм с пеньками сероватого цвета; кожа обильно шелушится, по периферии очага определяется зона легкой гиперемии. При люминесцентном исследовании обнаружено изумрудно-зеленое свечение. Укажите наиболее вероятный диагноз. Микроспория волосистой части головы

11.На прием к дерматологу обратилась женщина 35 лет, работающая дояркой, с жалобами на появление болезненных возвышений в области затылка. Объективно: в затылочной области на волосистой части головы определяются резко отграниченные от окружающей кожи островоспалительные инфильтраты, заметно возвышающиеся над уровнем кожи. Волосы в очаге склеены вследствие выделения гноя. При легком

потягивании волосы легко выдергиваются. При сдавливании очагов из волосяных фолликулов выделяется гной, симптом «медовых сот» положительный. Поставьте наиболее вероятный диагноз. Трихофития нагноительная

12.На прием к дерматовенерологу обратилась девушка 20 лет, с жалобами на высыпания в области гениталий, сопровождающиеся болезненностью, жжением, зудом. Из анамнеза: за несколько дней до появления высыпаний было переохлаждение. Объективно: на больших половых губах на фоне эритемы, сгруппированные везикулезные элементы с серозным, серозно-гнойным содержимым, корочки и эрозии. Какой из предварительных диагнозов наиболее вероятен? Генитальный герпес

13.Женщина 35 лет, обратилась к врачу в ЦСМ, с жалобами на стреляющие боли, жжение и высыпания в области спины, общее недомогание. Из анамнеза: заболевание связывает с переохлаждением, наблюдалась субфебрильная температура. Объективно: на коже туловища по ходу межреберных нервов справа имеются сгруппированные линейно расположенные пузырьки, с серозно-гнойным содержимым на эритематозном, слегка отечном фоне. Какой из предварительных диагнозов наиболее вероятен? Опоясывающий герпес

14.На прием к дерматологу обратилась девушка 20 лет, с жалобами на высыпания на руках. Из анамнеза: болеет 1 неделю. Объективно: на коже тыльной поверхности предплечий полушаровидные папулы желтовато – коричневого цвета, возвышающиеся над уровнем кожи с гладкой поверхностью размером от 3 до 5 мм в диаметре. Укажите наиболее вероятный диагноз. Простые бородавки

15.Женщина 28 лет, поступила в кожное отделение диспансера с жалобами на появление высыпаний на коже живота, сопровождающиеся зудом и жжением, повышением температуры тела. Заболевание связывает с приемом таблеток бисептола по поводу ангины. Объективно: на коже живота имеются эритематозные пятна с четкими границами, в центре очага застойно-синюшный оттенок, периферическая зона пятен слегка отечна, очертания округлой овальной формы. Какой вариант дерматита развился у пациентки? Аллергический дерматит

16.К дерматологу в ЦСМ братилась женщина 30 лет с жалобами на зуд, жжение, отек лица. Заболевание началось через 2 часа после применения нового косметического крема. Объективно: кожа лица отечна, гиперемирована. На коже всего лица на фоне

эритемы и гиперемии имеется мелкая папуло-везикулезная сыпь. Какому диагнозу соответствуют описанные выше симптомы? Аллергический контактный дерматит

17.Мужчина 40 лет госпитализирован в РЦДВ с жалобами на множественные высыпания на коже туловища, верхних и нижних конечностях. Со слов больного по поводу пневмонии принимал антибиотики, затем появилась головная боль, общая слабость, боли в суставах и высыпания на туловище, на слизистой полости рта. Объективно: на коже туловища на фоне эритемы имеются множественные крупные пузыри и обширные эрозии, при прикосновении пальцами пораженных участков кожи происходит отслойка эпидермиса, наличие обширных эрозий на слизистой полости рта. Симптом Никольского положительный. Какой синдром описан в данной задаче. Синдром Лайела

18.Мужчина 37 лет обратился к дерматологу по месту жительства. Жалобы на зуд, высыпания в вечернее время. Из анамнеза: в течение последней недели страдает ежедневными приступами зуда, усиливающимися к ночи. Волдыри возникают чаще по вечерам и к утру бесследно исчезают. Прием антигистаминных препаратов приводит к временному улучшению. Какой из нижеперечисленных диагнозов наиболее вероятен? крапивница

19.Женщина 38 лет обратилась в ЦСМ к дерматологу. Жалобы на сильный зуд кожи задней поверхности шеи. Из анамнеза: болеет в течении 5 лет. Объективно: в указанной области кожа застойногиперемирована, имеются очаги лихенификации с усиленным кожным рисунком. При пальпации очага кожа шероховатая, грубая на ощупь, уплотнена. Вокруг очага имеется зона усиленной пигментации. Укажите правильный диагноз. Органический нейродермит Видаля

20.Женщина 30 лет поступила в Республиканский центр дерматовенерологии с жалобами на высыпания на слизистой оболочке полости рта, болезненность при приеме пищи. Болеет одну неделю. При осмотре: на твердом нёбе, слизистой оболочке щек, языка и красной кайме нижней губы сливающиеся эрозии; в полости рта они имеют сочную розово-красную окраску и по периферии окаймлены обрывками беловатого эпителия; на нижней губе покрыты толстыми геморрагическими корками. Кожные покровы свободны от высыпаний. В эрозиях полости рта обнаружены акантолитические клетки. Выставьте наиболее вероятный диагноз.Пемфигус-акантолитическая вульгарная пузырчатка

21.Пациентка 37 лет поступила к врачу в ЦСМ с жалобами на внезапно возникшие высыпания на верхних и нижних конечностях ярко красного цвета. Из анамнеза: неделю назад переохладилась, наблюдалось повышение температуры до 37.5 °С, зуд и жжение в очагах. При осмотре: на конечностях имеются отечные папулы с резкими границами округлой формы диаметром 1,5 см. розово-красного цвета с ярким периферическим валиком и западением центральной части цианотичного оттенка в виде симтпома «птичьего глаза». Укажите предварительный диагноз. Многоформная экссудативная эритема

22.На прием к дерматологу обратился больной 30 лет с жалобами на появление зудящих высыпаний. Болеет 3 дня. Причину заболевания не знает. Объективно на коже туловища, на сгибательных поверхностях предплечий и голеней имеются мелкие плоские, блестящие, полигональной формы папулы с вдавлением в центре. Симптом Кебнера и сетка Уикхема положительные. Поставьте правильный диагноз. Красный плоский лишай

23.К дерматологу в ЦСМ обратилась пациентка 47 лет с жалобами на высыпания на руках и животе. Две недели назад перенесла ангину. Объективно: на разгибательной поверхности локтевых суставов имеются папулы розово-красного цвета, округлых очертаний, с четкими границами, центральная часть которых покрыта белесоватыми, легко отторгающимися чешуйками. На коже живота, в области послеоперационного рубца отмечается множество мелких аналогичных папулезных элементов. Положительный симптом Ауспитца и Кебнера. Выставьте правильный диагноз. Псориаз

24.Женщина 57 лет, обратилась в ЦСМ к дерматологу, с жалобами на появление участков уплотнения кожи, незначительный зуд, затруднения при движениях в плечевых, локтевых, голеностопных суставах. Заболевание связывает с гормональными нарушениями. Болеет в течении 6 месяцев. Объективно: кожа туловища, конечностей диффузно уплотнена, отмечаются множественные спаянные с подлежащими тканями уплотненные участки округлой формы, цвета слоновой кости с полосами депигментации. В области плечевого пояса имеется атрофия кожи, мышц. Пальпация очагов поражения безболезненная. Какой из перечисленных диагнозов наиболее вероятен? ОграническаЯ склеродермия

25.На прием обратилась 18-летняя девушка с жалобами на поражение кожи лица, спины, груди. Больна в течение 5 мес. При осмотре: кожа груди, спины повышенной сальности, выводные протоки сальных желез расширены, имеются комедоны. Кроме

того, определяются папулы и пустулы. Какой из нижеперечисленных диагнозов наиболее вероятен? Себорея

26.На прием к дерматологу обратился мужчина 32 лет с жалобами на выпадение волос, которое продолжается в течение 4 лет. Больной злоупотребляет алкоголем, жирной и острой пищей. При осмотре: кожа волосистой части головы повышенной сальности с наличием жирных желто-серого цвета чешуек. Ваш предполагаемый диагноз? Себорея

27.Пациент 22 лет, обратился к дерматовенерологу в ЦСМ с жалобами на зуд и жжение крайней плоти полового члена. Болеет в течение 1 месяца. Из анамнеза: за последний год проходил лечение в тубдиспансере с внелегочным туберкулезом, состоит на учете. Объективно: белесоватый налет на головке полового члена и крайней плоти, выделения творожистого характера. В анализах Candida аlbicans найдена. Какой из нижеперечисленных диагнозов наиболее вероятен? Урогенитальный кандидоз

28.Женщина 26 лет обратилась к венерологу с жалобами на высыпания в области туловища, которые появились неделю назад. Объективно: на коже груди и спины множество розеолезной сыпи ярко-розового цвета, мелкие, расположены симметрично. Комплекс серологических реакций резко положительный (++++) ? Какой из нижеперечисленных диагнозов наиболее вероятен Сифилис вторичный

свежий

29.В ЦСМ к дерматовенерологу обратился мужчина 30 лет с жалобами на высыпания в области слизистой рта, а также туловища. По словам пациента высыпания, на коже у него появляются не первый раз, а на слизистой полости рта впервые. Объективно: на коже туловища, преимущественно на боковой поверхности, видна розеолезная сыпь, имеющая тенденцию к слиянию бледно-розового цвета. На коже спины и груди видны папулы размером 1,0 х 1,0 см. На слизистой полости рта единичные эрозированные папулы. На задней поверхности шеи депигментированные пятна. Комплекс серологических реакций резко положительный (++++). Выставьте предполагаемый диагноз? Вторичный сифилис кожи и слизистых рецидивный

30.В военкомате при осмотре призывников был выявлен юноша с двусторонним снижением слуха и ослаблением зрения. Кроме этого отмечались дистрофическое поражение зубов, саблевидные голени. Кожные покровы свободны от высыпаний. Комплекс серологических реакций резко положительный (+++). Поставьте наиболее вероятный диагноз? поздний врожденный сифилис

31.В РЦДВ обратился мужчина 35 лет с жалобами на высыпания на коже туловища и выпадение волос, зуда нет. Из анамнеза: 3 месяца назад тоже была сыпь на коже. Объективно: на коже туловища имеется диффузно распространенная сыпь, в виде папул застойно-красного цвета, округлой формы с шелушением по периферии, в диаметре до одного сантиметра, на затылочной области волосистой части головы имеется очаг алопеции округлой формы в диаметре 3 сантиметра. Комплекс серологических реакций (++++) резко положительный. Выставте правильный диагноз. Вторичный сифилис

32.В неврологическое отделение Национального Госпиталя поступил пациент М. 52 лет, с жалобами на головную боль, резко усиливающуюся в ночное время. Объективно: кожные покровы и видимые слизистые свободны от высыпаний. Отмечается птоз и косоглазие, а также анизокория. Комплекс серологических реакций положителен (+++). Из анамнеза выяснено, что ранее болел сифилисом, занимался самолечением. Какой из нижеперечисленных диагнозов наиболее вероятен? Третичный сифилис,нейросифилис

33.К венерологу обратился пациент 34 года. Жалобы на гнойные выделения из уретры и резкую болезненность при мочеиспускании. Со слов больного, явления уретрита развились остро, через 4 дней после случайного полового контакта. Женат. При осмотре: выделения из уретры обильные, желтовато-зеленого цвета, сливкообразного характера, губки уретры воспалены. Обнаружены диплококки. Какая из нижеперечисленных инфекций наиболее вероятна? Гонорея

34.Пациент 35 лет обратился к венерологу с жалобами на боли в области правого яичка, повышение температуры до 38ºС. Из анамнеза: 1,5 месяца назад была случайная половая связь. При осмотре: наружные половые органы развиты правильно. Губки уретры не воспалены. Мошонка увеличена, кожа напряжена, гиперемирована. Придаток левого яичка увеличен, плотный, болезненный. В мазках обнаружен гонококк. Предварительный диагноз: Свежая гонорея, торпидное течение. Какое осложнение описано в данной задаче? Гонорейный эпидидимит

35.Пациент 30 лет обратился к венерологу с жалобами на выделения из уретры, частые позывы на мочеиспускание, жжение в уретре. При осмотре: наружные половые органы развиты правильно. Губки уретры незначительно гиперемированы, отечны, выделения из уретры скудные, слизистого характера. Органы мошонки без патологии. Паховые лимфоузлы не увеличены. В мазках обнаружены простейшие

микроорганизмы из семейства жгутиковых. Какая из нижеперечисленных инфекций наиболее вероятна? Трихомониаз

36.Пациент 25 лет обратился к врачу с жалобами на учащенное мочеиспускание. Из анамнеза: неделю назад был незащищенный половой акт с малознакомой женщиной. Объективно: гиперемия, отечность слизистой оболочки наружного отверстия мочеиспускательного канала, инфильтрация губок уретры, слизистые выделения. Методом ПЦР-диагностики обнаружена M. Genitalium. Какой из перечисленных диагнозов наиболее вероятен? Микоплазмоз

37.Пациентка 25 лет обратилась к венерологу с жалобами на обильные бели с неприятным запахом, возникшие после перенесенной острой респираторной инфекции. Ранее такие патологические вагинальные выделения пациентку не беспокоили. Результат микроскопии вагинального мазка окрашенного по Граму: лейкоциты - 8-10, микрофлора умеренная, единичные лактобактерии, обнаружены «ключевые клетки». Какой из нижеперечисленных предварительных диагнозов наиболее вероятен? Гарднереллез(бактериальный вагиноз)

38.Пациентка 28 лет обратилась к дерматовенерологу в ЦСМ с жалобами на высыпания в области половых органов, жжение, болезненность. Подобные высыпания появлялись и раньше. Внебрачные половые контакты отрицает. Объективно: на слизистой оболочке малых половых губ, на фоне отека и гиперемии имеются сгруппированные пузырьки с серозным содержимым и эрозии с полициклическими очертаниями. Регионарные лимфатические узлы увеличены. Лабораторные исследования не проведены. Укажите правильный диагноз. Рецидивирующий герпес

Пациентка 30 лет обратилась к дерматовенерологу в ЦСМ с жалобами на выделения слизисто-гнойного характера, периодически возникающие тянущие боли внизу живота. Болеет в течении двух недель. При осмотре в гинекологических зеркалах: отёчность, гиперемия слизистой оболочки шейки матки, слизисто - гнойные выделения из цервикального канала. При обследовании методом ПЦР найдена Chlamidia trachomatis. Какой из перечисленных диагнозов наиболее вероятен? Хламидиоз

39.В ЦСМ к дерматовенерологу обратился мужчина 50 лет, с жалобами на неприятные ощущения в области уретры, выделения слизисто-гнойного характера, нарушение потенции. Из анамнеза известно, что больного на протяжении 6 месяцев беспокоит боль и скованность в области коленного и голеностопного суставов, ахиллова сухожилия, а также периодически возникающий конъюнктивит. Методом ПЦР обнаружена ДНК С. trachomatis. О каком из осложнений хламидийной инфекции идет речь? синдром Рейтера

40.Женщина 27 лет, обратилась к врачу ГСВ с жалобами на зуд влагалища, на наличие скудных выделений. 14 дней назад была у врача оториноларинголога, по поводу хронического гайморита, в связи с чем принимала антибиотики фторхинолинового ряда. Объективно: скудные выделение из влагалища творожистого характера, слизистая влагалища отечна, гиперемирована, местами с белесоватым налетом. Какова этиология развившегося вагинита? Candida albican

41.Пациент 22 лет обратился по месту жительства к венерологу с жалобами на обильные гнойные выделения из уретры, желтого цвета, частые позывы на мочеиспускание, жжение в уретре. При осмотре: губки уретры гиперемированы, отечны, выделения из уретры обильные, желтого цвета. Органы мошонки без патологии. Паховые лимфоузлы не увеличены. Двухстаканная проба: в 1стакане – мутная, во 2 стакане – с нитями. В мазках обнаружен диплококк. Какая инфекция вызвала развитие данного заболевания? Гонорея

42.Мужчина 50 лет обратился дерматологу в ЦСМ с жалобами на появление высыпаний на коже туловища и конечностей. Беспокоит незначительный зуд. Объективно: на разгибательных поверхностях верхних и нижних конечностей и туловища имеются папулы различных размеров и формы, розовой окраски, покрытые серебристобелыми чешуйками. Триада феноменов положительные. Какой из перечисленных предварительных диагнозов наиболее вероятен? Псориаз

43.Мужчина 25 лет обратился к дерматологу в ЦСМ с жалобами на появление пятен на коже туловища, которые появились после загара. Объективно: на коже верхней половины туловища имеются единичные пятна желтовато-бурого цвета с шелушением на поверхности. Проба Бальцера положительная. Какой из перечисленных предварительных диагнозов наиболее вероятен? разноцветный лишай

44.На приеме у дерматолога в ЦСМ женщина с дочерью 5 лет. У девочки 3 дня назад на лице после расчесывания стали появляться пузыри, быстро подсыхающие в корки. После принятия душа количество высыпаний увеличилось. При осмотре: на щеках единичные плоские пузыри с тонкой покрышкой и мутным содержимым, до 5-10 мм в диаметре, а также большое количество светло-желтых корочек на месте ссохшихся пузырей. Общее состояние ребенка удовлетворительное. Какой из нижеперечисленных вариантов импетиго возможен у пациента? стрептококковое импетиго

45.К дерматологу в ЦСМ обратился мужчина 40 лет с жалобами на высыпания, сопровождающиеся вечерним и ночным зудом. При осмотре: на коже туловища, особенно на переднебоковых поверхностях, на верхних и нижних конечностях, в межпальцевых промежутках имеются многочисленные парные папуло - везикулезные элементы, экскориации, кровянистые корочки. Симптомы Сезари, Горчакова положительные. Какой из перечисленных ниже предварительных диагнозов наиболее вероятен? Чесотка

46.Женщина 35 лет обратилась к дерматологу в ЦСМ с жалобами на высыпания в области гениталий, сопровождающиеся болезненностью, жжением, зудом. Из анамнеза: за несколько дней до появления высыпаний было переохлаждение. Объективно: на больших половых губах на фоне эритемы, сгруппированные везикулезные элементы с серозным, серозно-гнойным содержимым, корочки и изъязвления. Какой из предварительных диагнозов наиболее вероятен?

генитальный герпес

47.Подросток 16 лет обратился к дерматологу в ЦСМ с жалобами на появление высыпаний на коже лица, ушных раковинах, волосистой части головы, зуд. Со слов мальчика болен 2 недели. Объективно: на коже по краю волосистой части головы, лица, ушных раковин имеются пузырьки, эрозии, корочки, трещины, местами очаги незначительного шелушения. Определите какому виду экземы соответствуют данные описанные жалобы: себорейная экзема

48.Девушка 25 лет обратилась к дерматологу в ЦСМ с жалобами на зуд кожи и появление высыпаний. Болеет с детства. Объективно: очаги поражения располагаются на коже сгибательных поверхностей предплечий, подколенных ямок, на коже лица и шеи. Кожа в очагах поражения гиперемирована с лихенификацией,

экскориациями и шелушением. Отмечается общая сухость кожи, дермографизм белый, стойкий. Какой из нижеперечисленных диагнозов наиболее вероятен? Атопический дерматит?

49.Мужчина 45 лет, поступил в городской дермато - венерологический диспансер с жалобами на распространенные высыпания на слизистой оболочке полости рта, на коже спины и груди, незначительный зуд, болезненность. Впервые эрозии появились на слизистой оболочке ротовой полости 2 месяца назад. При осмотре: на коже спины, груди имеются пузыри с тонкой дряблой покрышкой, серозным содержимым. Эрозии ярко-красного цвета, по краю эрозий обрывки эпидермиса. Кожа вокруг высыпаний не изменена. Симптом Никольского, Шеклакова, Асбо - Хансена положительные. В мазках – отпечатках обнаружены клетки Тцанка. Какой из нижеперечисленных предварительных диагнозов наиболее вероятен? Вульгарная пузырчатка

50.Мужчина с ребенком 8 лет, обратился к дерматологу в ЦСМ с жалобами на высыпания, зуд, жжение. При осмотре: на коже туловища, бедер сгруппированные пузыри, эрозии, корки. Симптом Никольского отрицательный. Симптом Ядассона положительный. Цитологическая диагностика – наличие эозинофилов в содержимом пузыря. Общий анализ крови – эозинофилия. Какой из нижеперечисленных предварительных диагнозов наиболее вероятен? Герпетиформный дерматит дюринга

51.Мужчина 28 лет обратился к дерматологу в ЦСМ с жалобами на высыпания в области туловища и заднего прохода. Объективно: на коже живота и спины единичные нумулярные папулы. На коже шеи заметна депигментация с мраморным оттенком. В области заднего прохода обширные широкие кондиломы. Не женат. КСР резко положительна. Какой вариант сифилиса имеется у пациента? вторичный

сифилис

Применение

1.Пациент 18 лет обратился к дерматологу в ЦСМ с жалобами на высыпания, зуд. Из анамнеза удалось установить, что заболевание началось после приобретения и ношения нового металлического браслета для часов. При осмотре: на коже запястья левой руки имеются эритематозные пятна с неправильными очертаниями, с нечеткими границами, мелкопапулезная и везикулезная сыпь. Диагноз: Аллергический контактный дерматит. Какие из нижеперечисленных препаратов используется местно при лечении? гкс,антигистаминные

2.Пациентка 38 лет, обратилась к врачу ГСВ с жалобами на зуд влагалища и скудные выделение творожистого характера. Из анамнеза 14 дней назад была у врача, по поводу хронического бронхита, в связи с чем принимала антибиотики фторхинолинового ряда. Какое обследование нужно провести для уточнения диагноза? Микроскопия

3.Женщина 29 лет обратилась к врачу с жалобами на незначительный зуд пораженной кожи и ощущения стягивания. Дерматоз локализуется на волосистой части головы. Волосистая часть кожи покрыта наслоениями чешуек. Волосы не поражены. Выставлен диагноз: себорейный дерматит. Какую форму местного значения наиболее целесообразно назначить в этом случае? Противогрибковые препараты(шампунь, примочки)-Кетаканазол,Низорал

4.На приеме у дерматолога женщина 30 лет, 3 дня назад на лице после укусов комаров у нее стали появляться пузыри, быстро подсыхающие в корки. Умывание горячей водой

смылом вызвало обострение. Объективно: в носогубных складках, на верхней губе, щеках и на подбородке единичные плоские пузыри с тонкой покрышкой и мутным гнойным содержимым, светло-желтые корочки. Выставлен предварительный диагноз: Стрептококковое импетиго. Какой препарат необходим для наружного лечения? Мази

сантибиотиков(эритромицин,гентамицин,левомеколь) Обрабатывание красителями

5.Пациентка 19 лет обратилась в ЦСМ к дерматовенерологу. Час назад была случайная половая связь. При гинекологическом исследовании: наружные половые органы и слизистая влагалища без признаков воспаления, внутренние половые органы без патологии. Мазок в норме. Предварительный диагноз: Здорова. Какое средство можно использовать с целью профилактики Заболеваний передающихся половым путем? Спринцевание антисептиками-хлоргексидином

6.Женщина 40 лет обратилась в поликлинику с жалобами на выпадение волос. Объективно: на волосистой части головы очаги располагаются на краевых зонах,

сливаясь друг с ругом. Воспалительные элементы покрыты асбестовидными чешуйками. Волосы обломаны на уровне 5-8 мм., тусклые, у основания одеты в белый чехол. При лабораторном исследовании обнаружены нити мицелия. Диагноз: Микроспория волосистой части головы. Какой препарат предпочтителен при лечении микроспории волосистой части головы: гризеофульвин

7.Мужчина 36 лет обратился с жалобами к дерматовенерологу на зуд и высыпания на коже спины и живота. Анамнез: болеет в течение последних 2х недель. Объективно: на коже живота и спины имеются очаги поражения в виде колец, состоящих из мелких папул. Диагноз: Микроспория гладкой кожи. Назовите диагностический метод для подтверждения данного диагноза? микроскопия на грибы, люминесцентное свечениеизумруднозеленое свечение(лампа Вуда)

8.К дерматологу обратился парень 19лет с жалобами на высыпания на теле. Из анамнеза: посещает спортивную секцию, высыпания появились месяц назад. Объективно: узелки цвета нормальной кожи в области живота, верхних конечностей, лица величиной с

булавочную головку. В центре узелков имеется пупковидное вдавление, при надавливании выявляется творожистая масса. Диагноз: Контагиозный моллюск. Выберите метод наружного лечения данного заболевания? удаление элементов,.выдавливание пинцетом со смазываним раствора йода или зелени, диатермокоагуляция, фукорцин

9.Мужчина 38 года доставлен каретой скорой помощи в приемное отделение с жалобами на появление высыпаний по всему телу, боль и жжение. Со слов сопровождающих родственников, через 3 дня после начала антибиотикотерапии у пациентки вечером резко поднялась температура тела до 38 ̊С, появилась общая слабость, головокружение. Объективно: состояние тяжелое. Кожа лица, туловища, конечностей ярко гиперемирована, в некоторых местах дряблые пузыри, эрозии. Выставлен диагноз: Синдром Лайелла. Укажите, какой препарат необходимо назначить в первую очередь. Преднизолон 1-2мг/кг

10.Мужчина 30 лет, обратился по месту жительства к дерматологу с жалобами на появление пятен на коже в области живота. При объективном осмотре на коже живота имеются множественные очаги гипо- и гиперпигментации с мелкопластинчатыми чешуйками. Симптом Бенье положителен. Предварительный диагноз: Разноцветный

лишай. Какую из нижеперечисленных проб нужно провести для подтверждения диагноза? Проба Бальцера

11.Мужчина 33 лет обратился в ЦСМ к дерматологу с жалобами на появление высыпаний на коже туловища и незначительный зуд. Болеет в течение 1 года, в прошлом году лечился амбулаторно с положительным результатом. Данное обострение заболевания связывает с нервным перенапряжением. Объективно: на коже груди и живота имеются многочисленные небольшие папулы розового цвета, покрытые серебристыми чешуйками. Псориатическая триада и симптом Кебрера положительны. Диагноз: Каплевидный псориаз. Какую мазь наиболее целесообразно применить для наружного лечения? кортикостероидные мази, салициловая,серно-салициловая, нафталановая мазь

12.Пациентка 25 лет обратилась к дерматологу с жалобами на высыпания, зуд. Из анамнеза: частота обострений – 3–4 раза в год с увеличением их продолжительности. Объективно: на коже эритематозно-сквамозные и папулезные очаги. Кожа сухая, с большим количеством экскориаций и мелкопластинчатыми чешуйками, имеются очаги лихенификации. Высыпания локализуются преимущественно на сгибательных поверхностях конечностей, тыльной поверхности кистей, передней и боковой поверхностях шеи. Диагноз: Атопический дерматит. Эритематозно-сквамозная форма, средней тяжести. Какие мази применяют для местного лечения? с гкс

13.В РЦДВ поступил мужчина 35 лет с высыпаниями на коже туловища и конечностей. У жены диагностирован вторичный свежий сифилис. Объективно: у ребенка на коже конечностей и ягодицах имеется пятнисто-папулезная сыпь, мокнущие папулы вокруг рта и заднего прохода. Какие лабораторные исследования необходимо провести для установления диагноза ребенку? ПЦР, RW

14.Женщина 38 лет, поступила на стационарное лечение в дерматологическое отделение. Жалобы на высыпания, жжение и болезненность в местах высыпаний. Болеет в течение двух лет. При осмотре: на коже пахово-бедренных складок имеются сосочковые разрастания розово-красного цвета, мягкой консистенции, покрытые сероватым налетом, серозно-гнойным отделяемым, с неприятным запахом. На слизистой оболочке полости рта - эрозии. Определяется положительный симптом Никольского. Предварительный диагноз: Вегетирующая пузырчатка. Какой из методов исследований необходимо провести для подтверждения данного заболевания?

акантолитическиеклетки в микроскопии, риф, цитологическое иследование мазка со дна эрозии

15.Пациент 27 лет, поступил на стационарное лечение. Жалобы на распространенные высыпания, сопровождающиеся сильным зудом. Болен примерно 6 лет. При осмотре: на коже верхних и нижних конечностей, туловища расположены сгруппированные папулы и пятна розово-красного цвета, пузырьки с плотной покрышкой, эрозии, экскориации и корки. В общем анализе крови эозинофилия. Симптом Никольского отрицательный. Акантолитические клетки не обнаружены. Предварительный диагноз: Герпетиформный дерматит Дюринга. Какую пробу необходимо провести для подтверждения диагноза? Содержимое пузырей на эозинофилы, йодная проба Ядасона

16.Мужчина 27 лет. Обратился к дерматовенерологу в ЦСМ с жалобами на появление сыпи, сопровождающейся умеренным зудом. Болен в течение 2-х недель, когда после простуды на коже груди заметил одно пятно. После мытья в душе появилась обильная сыпь на коже туловища. Объективно: на коже груди имеется округлое пятно красного цвета, шелушение в очаге. На коже туловища и верхних конечностей, определяется множественная сыпь в виде пятен розового цвета, неправильной формы, с шелушением по типу смятой папиросной бумаги. Диагноз: Розовый лишай Жибера. Какие ограничения должен соблюдать больной? мытье горячей водой с мочалкой и мылом запрещено,загорание,физиотерапия

17.К дерматологу в ЦСМ обратился пациент 26 лет с жалобами на зудящие высыпания. Болеет в течение недели, причину заболевания не знает. Объективно: на коже туловища, передней поверхности предплечий и голеней, высыпания представлены плоскими, мелкими блестящими папулами полигональных очертаний синюшно-розового цвета, в центре папул пупкообразное вдавление. Симптом Уикхема положительный. Выставлен диагноз: Красный плоский лишай. Какой симптом из нижеперечисленных нужно выявить подтвердить диагноз? Симптом Кебнера

18.Женщина 32 лет обратилась в ЦСМ к дерматологу. Жалобы больной: в течение 2 месяцев больную беспокоят высыпания на коже коленных суставов. Объективно: на коже разгибательной поверхности коленных суставов имеются единичные узелки розово-красного цвета, округлой формы, 0,2 – 0,5 см в диаметре. Триада феноменов, симптом Кебнера и симптом «наперстка» положительные. Диагноз: Псориаз. Какую терапию следует назначить в первую очередь? гкс,антигистаминные

19.В кожное отделение РЦДВ поступила женщина 42 лет с диагнозом: Очаговая склеродермия. Диффузное поражение кожи, мышечно-суставной синдром, хроническое течение. Жалобы на изменение цвета, чувство стягивания и уплотнение кожи, невозможно взять в складку. Болеет 3 месяца, причину сказать не может. Объективно: на коже разгибательных поверхностей коленных суставов имеются очаги уплотнения округлой формы сиреневого цвета, болезненные при пальпации. Какую терапию нужно применить в лечении данного пациента? д -пеницилламин,купренил, лидаза

20.На приеме у дерматолога мужчина 38 лет, болеет 1 год. При осмотре - на носу, щеках имеется эритема с синюшным оттенком, телеангиэктазии, ярко-розовые папулы величиной до горошины. Диагноз – розовые угри. Какие препараты необходимы в общей терапии розацеа? Метранидозол,эритромицин,ретиноиды

21.На приеме парень 19 лет с жалобами на поражение кожи лица, спины, груди, зуд волосистой части головы. Болен около 2 лет. При осмотре: кожа груди, спины повышенной сальности, имеется множество папул и пустул, узлы, рубчики. Диагноз – вульгарные угри, конглобатная форма. Какие препараты должна включать общая терапия акне? Антибиотики тетрациклинового ряда, ретиноиды

22.Мужчина 34 лет жалуется на высыпания на верхних конечностях, болезненность. Из анамнеза: при работе с аккумулятором на кожу попал электролит, содержащий концентрированную серную кислоту. При осмотре на коже в местах попадания электролита появились очаги эритемы с четкими границами, пузыри и эрозии. Диагноз: Простой контактный дерматит. Какие наружные средства нужно назначить? Кортикостероидные мази

23.На прием к дерматологу в ЦСМ обратилась девушка 20 лет с жалобами на высыпания на верхних и нижних конечностях. Анамнез: в детстве страдала аллергическим диатезом. Обострения кожных проявлений связывает с употреблением молока и яиц. При осмотре на коже разгибательной поверхности голеней и предплечий кожа ярко гиперемирована, отечна, покрыта микровезикулами с мокнутием в виде «колодцев». Диагноз: Атопический дерматит. Какая мазь применяется для местной терапии. ГКС

24.Больной М. 19 лет поступил в отделении с жалобами на удушье, нестерпимый кожный зуд, резко выраженную общую слабость, головокружение, сердцебиение. Симптомы появились после приема антибиотиков. Из анамнеза: страдает поливалентной лекарственной аллергией. Объективно: кожные покровы гиперемированы, покрыты липким потом, отек век, губ. Диагноз: отек Квинке. Какие препараты предпочтительны в лечении? Преднизолон, дексаметазон. Адреналин, антигистаминные

25.Девушка, поступила к дерматологу в ЦСМ с жалобами на геморрагическую сыпь на нижних конечностях, ягодицах и внутренней поверхности предплечий. Сыпь появилась через неделю после перенесенной острой вирусной инфекции, с тенденцией к слиянию, симметричная, не зудящая. Других патологий не выявлено. Диагноз: Геморрагический васкулит. Назначение какой терапии применимо при данном заболевании? Гепарин, антигистаминные, гкс

26.На прием к дерматологу обратился мужчина 30 лет, болен 2 недели. Беспокоит зуд кожи, особенно в ночное время. Работает шофером, живет один. Выставлен диагноз – чесотка. Какие мероприятия должны быть проведены в первую очередь? Серная мазь, полисульфидный линимент,мазь Вилькинсона, антигистаминные

27.В ЦСМ на прием к врачу обратился пациент С., 24 лет с жалобами на появление язвочки на головке полового члена, которая уже держится на протяжении 30 дней. Из анамнеза удалось выяснить, что он имел половой контакт с малознакомой женщиной более 2 месяцев назад. Объективно: в области венечной борозды имеется язва округлой формы, края ровные, дно гладкое, размер 1,5х2,0 см, в стадии эпителизации. Паховые лимфатические узлы увеличены с обеих сторон. Какое лабораторное исследование необходимо провести для подтверждения диагноза сифилис? ИФА,РПГА,ПЦР,RW

28.На приеме у дерматолога пациент 29 лет, из анамнеза - в лесу наступил на осиное гнездо. Получил укусы в левую голень. Спустя примерно 20 мин почувствовал слабость, першение и ощущение инородного тела в горле, недостаточность воздуха при дыхании, появились отечность лица, особенно губ, век, покраснения и зудящие высыпания на коже. Диагноз: Отек Квинке. Какой из препаратов назначается первостепенно? Преднизолон, дексаметазон. Адреналин, антигистаминные

29.Женщина 40 лет, обратилась к дерматологу по поводу высыпаний. Из анамнеза несколько лет жила в Узбекистане. При осмотре: на коже лба имеется язва диаметром примерно 5 см, покрыта коркой темновато-бурого цвета, выходящей за пределы язвы. Из язвы выделяется гной. Изменений со стороны внутренних органов не выявлено. Предварительный диагноз: Лейшманиоз. Препарат с содержанием какого элемента необходимо назначить? Препараты пятивалентной сурьмы

30.Пациент 20 лет в течение длительного времени страдает атопическим дерматитом. Объективно: выраженый зуд, лихенификация и сухость всего кожного покрова. Местами экскориации и расчесы. Укажите мази для местной терапии: нафтолановая,дегтярная,ихтиоловая мази

31.В ЦСМ к дерматовенерологу обратился пациент К., 25 лет с жалобами на увеличение обоих паховых лимфоузлов. Социально не адаптирован, сексуально не разборчив. Объективно: кожные покровы и видимые слизистые свободны от высыпаний, паховые лимфоузлы увеличены с обеих сторон, безболезненны, подвижны. На внутреннем листке крайней плоти видна эрозия в стадии эпителизации, размером 1,0х1,0см. КСР отрицательный. Какое дополнительное лабораторное исследование необходимо провести для подтверждения диагноза сифилис? ИФА,ПЦР,РРГА, Реакция Вассермана

32.В ЦСМ на прием к дерматовенерологу обратилась женщина Т., в возрасте 42 лет по поводу высыпаний на левом тазобедренном суставе, которые появились 3-4 недели назад. Субъективно: беспокоят боли в ногах. До этого замечала высыпания на коже туловища, ладоней и подошв. По словам пациентки, они спонтанно то появлялись, то исчезали. Объективно: на внутренней поверхности левого бедра видны сгруппированные бугорки плотные на ощупь, размером с горошину. Предположительный диагноз: Бугорковый сифилид. Какие исследования необходимо провести для уточнения диагноза? ИФА,ПЦР,РРГА, Реакция Вассермана,биопсия

33.Пациент 20 лет, с микробной экземой, обратился в ЦСМ с жалобами на высыпания на коже нижних конечностей, зуд. При осмотре отмечаются папулы, множественные везикулы с серозным, содержимым. Какой из нижеперечисленных противомикробных растворов необходимо использовать в местном лечении? Фукорцин

34.В РЦДВ мужчине

30 лет выставлен диагноз: Вторичный свежий сифилис. К препаратам

пенициллинового

ряда и макролидам отмечена непереносимость. Какой

альтернативный антибиотик может быть рекомендован в данном случае? Тетрациклин,доксициклин

35.В ЦСМ обратился пациент Н., 45 лет с жалобами на высыпания в области подошв

иповерхности кожи пальцев стоп. Из анамнеза: лечился самостоятельно мазью микозолон, без эффекта. Объективно: на коже ладоней и стоп имеются папулезные элементы округлых очертаний. На боковой поверхности туловища видна розеолезная сыпь бледно-розового цвета, немногочислена. Какие лабораторные исследования необходимо провести для уточнения диагноза?

Серологические реакции

36.В ЦСМ к дерматовенерологу обратилась женщина 25лет. Из анамнеза: женщина вступала в неоднократный половой контакт со знакомым мужчиной, у которого диагностирован сифилис. Последний контакт был 1 месяца тому назад. Объективно: кожные покровы и видимые слизистые свободны от высыпаний. Паховые лимфатические узлы не увеличены. Серологические реакции отрицательные. Назначено превентивное лечение. Какой антибиотик наиболее показан в данном случае? Тетрациклин,эритромицин

37.На прием к врачу обратился пациент М. 25 лет, с жалобами на появление болячки на головке полового члена. Из анамнеза: часто по работе бывает в командировке. Объективно: кожа головки полового члена гиперемирована, отечна, язвочка в стадии эпителизации. Паховые лимфоузлы увеличены с обеих сторон. КСР отрицательный. В материале из язвочки найдена бледная трепонема. Выставлен диагноз первичный серонегативный сифилис. Выберите антибиотик для лечения. Антибиотики пенициллинового ряда. Бензилпенициллин,цефтриаксон

38.Пациент 25 лет, обратился к дерматологу в поликлинику с жалобами на уплотнения кожи в области ладоней обеих кистей. Уплотнения появились около 3 месяцев назад. При осмотре: папулы светло - коричневого цвета, размерами 0,2-0,3 см в диаметре, безболезненные. Диагноз: Плоские бородавки. Какое средство применимо в лечении плоских бородавок? Салициловая-резорциновая мазь, серебра нитрат,криотерапи

окисью азота

39.Пациент, 33 года жалуется на высыпания на коже верхних конечностей, болезненность. Из анамнеза: во время работы на заводе на кожу попал электролит, содержащий

концентрированную серную кислоту. После попадания раствора не сразу промыл поверхность пораженной кожи. При осмотре на коже в местах попадания электролита появились очаги эритемы с четкими границами, пузыри и эрозии. Диагноз: Простой контактный дерматит. Какие средства для наружного лечения используют при данной клинической картине? Удаления раздражителя,холодные компрессы,гкс

40.Девушка 26 лет обратилась к дерматовенерологу с жалобами на скудные выделения с неприятным запахом, тянущие боли внизу живота, болезненное мочеиспускание. Объективно: слизистая влагалища гиперемирована, выделения слизисто-гнойного характера. Какой метод диагностики необходимо применить? Микроскопия мазка

41.Пациент 35 лет, жалобы на болезненность, высыпания на головке полового члена. Из анамнеза: болеет в течение 10 дней. Объективно: на головке полового члена имеются пузырьки с серозным содержимым на отечном, гиперемированном фоне, полициклические эрозии. Диагноз: Генитальный герпес. Выберите средство наружной терапии: Ацикловир

42.Девушка 22 лет обратилась к венерологу с жалобами на выделения из влагалища, рези при мочеиспускании. Из анамнеза: половая связь с постоянным половым партнером 10 дней назад. При осмотре в гинекологических зеркалах: отечность, гиперемия слизистой оболочки шейки матки, слизисто – гнойные выделения из цервикального канала. В мазках из цервикального канала и уретры обнаружены гонококки. Диагноз: Свежий подострый эндоцервицит, уретрит гонорейной этиологии. Какую группу антибиотиков необходимо назначить? Пенициллины,тетрациклины, цефалоспорины

43.Девушка 25 лет обратилась к венерологу по поводу выделений, зуда, жжения. Из анамнеза: десять дней назад была случайная половая связь. При осмотре: выделения серо-желтого цвета с неприятны м запахом и пенистого характера, отечность и гиперемия слизистой оболочки вульвы, влагалища, влагалищной части шейки матки, эрозивно-язвенные поражения слизистой оболочки наружных половых органов. При лабораторном исследовании мазка на урогенитальную инфекцию выявлен лейкоцитоз, трихомонады. Диагноз: Трихомонадный кольпит. Назначьте препарат для лечения. Метронидазол, тинидазол, орнидазол

44.Мужчина 33 лет обратился к венерологу с жалобами на выделения, зуд, жжение в мочеиспускательном канале. Из анамнеза: болеет два дня, 5 дней назад была случайная половая связь. При осмотре: выделения из уретры обильные, желтовато-зеленого цвета, сливкообразного характера, губки уретры воспалены. Органы мошонки без патологии.

Паховые лимфоузлы не увеличены. В мазках обнаружен гонококк. Диагноз: Свежая гонорея. Какую пробу необходимо провести для топической диагностики уретрита?

3хстаканную , проба Томпсона

45.Пациентка 25 лет, обратилась к дерматовенерологу в ГСВ с жалобами на зуд влагалища и на выделения творожистого характера, отечность складок влагалища. 14 дней назад была у врача терапевта, по поводу хронического бронхита, в связи с чем принимала антибиотики макролиды. Бактериоскопически обнаружены Candida albicans. Диагноз: Кандидозный вагинит. Острое течение. Какой препарат необходимо применять однократно для лечения данного заболевания по рекомендации ВОЗ? Флуконазол

46.Женщина 30 лет, обратилась к врачу дерматовенерологу в ЦСМ с жалобами на частое мочеиспускание, зуд, гиперемию и отечность больших половых губ. При осмотре между складками больших и малых половых губ имелся творожистый налет. Предварительный диагноз: Урогенитальный кандидоз. Назначьте исследование для уточнения диагноза: Посев на питательную среду,Микроскопия мазка

47.Мальчик 16 лет обратился к дерматологу в ЦСМ с жалобами на высыпания в области локтя и предплечья. При объективном осмотре обнаружены множественные округлые очаги различных размеров, бледно-розовой окраски, с мелкопластинчатым шелушением. Границы очагов четкие, с возвышающимся отечным валиком, состоящим из мелких узелковых и пузырьковых элементов. Очаги в виде фигур «кольцо в кольце». При исследовании обнаружили Microsporum canis. Диагноз: Микроспория гладкой кожи. Какой препарат применяется при лечении данной патологии? Гризеофульвин

48.Женщина 39 лет обратилась к дерматологу в ЦСМ с жалобами на опоясывающую, жгучую боль, высыпания в области левого подреберья. Из анамнеза: за несколько дней до высыпаний, после переохлаждения беспокоила стреляющая боль. Объективно: в области правого подреберье, по ходу межреберных нервов везикулы на эритематозном основании с серозным содержимым, располагающиеся линейно. Диагноз: Опоясывающий лишай. Какой из перечисленных препаратов применим при данном заболевании? Ацикловир,Ганцикловир

49.Мужчина 76 лет обратился к врачу дерматовенерологу в ЦСМ с жалобами на зуд туловища и конечностей. Из анамнеза: страдает в течение 5-х лет. Объективно: первичных морфологических элементов нет, но имеется ксероз – повышенная сухость кожи, местами с явлениями лихенификации, незначительным шелушением, множеством линейных расчесов. Диагноз: Сенильный кожный зуд. Выберите мази как средство для наружного лечения.

Мази с ментолом,бензакаин,гкс

50.На прием к дерматовенерологу в ЦСМ обратилась мать с ребенком 5 лет, у которого периодически на коже лица, туловища, конечностей, ягодиц отмечается множество рассеянных папул, точечных эрозий, кровянистых корочек на местах расчесов. Отмечается белый дермографизм. Диагноз: Атопический дерматит. Выберите препарат для лечения: ГКС

51.Женщина 70 лет обратилась к дерматологу в ЦСМ с жалобами на слабость, боль, повышение температуры тела до 37,6 С. Болеет в течение месяца. Объективно: на коже туловища вялые пузыри, эрозии, корки. При надавливании на поверхность не вскрывшегося пузыря, он увеличивается в размерах. Предварительный диагноз: Вульгарная пузырчатка. Какой из перечисленных диагностических симптомов надо провести для подтверждения данного диагноза?

Симптом Никольского,Асбо-Хансена

52.В стационар госпитализирован ребенок в возрасте 6 месяцев, со следующими клиническими проявлениями: гипотрофия 2 степени, двухсторонняя пневмония, печень и селезенка увеличены. Кожа вокруг рта инфильтрирована, имеются трещины. В области ануса мокнущие папулы. Какие лабораторные тесты необходимо провести для уточнения диагноза? СЕРОЛОГИЧЕСКИЕ

53.При профилактическом осмотре в военкомате у призывника обнаружена язвочка на головке полового члена, размером 1,0х1,0 см, в стадии эпителизации. Правый паховый лимфоузел увеличен, подвижен, безболезненный при пальпации. Из анамнеза установлено, что он имел половую связь с малознакомой женщиной 1,5 месяца назад. На появление язвочки на половом члене особого внимания не обращал. Анализ крови на КСР показал положительный результат. Выставлен диагноз первичный серопозитивный сифилис. Какой антибиотик наиболее показан в данном случае?

Экстенциллин

54.Девушка 25 лет обратилась к венерологу в ЦСМ по поводу выделений, зуда, жжения. Из анамнеза: десять дней назад была случайная половая связь. При осмотре: выделения серо-желтого цвета с неприятным запахом и пенистого характера, отечность и гиперемия слизистой оболочки вульвы, влагалища, влагалищной части шейки матки, эрозивно-язвенные поражения слизистой оболочки наружных половых органов. При лабораторном исследовании мазка на урогенитальную инфекцию выявлен лейкоцитоз, трихомонады. Диагноз: Трихомонадный кольпит. Какой из нижеперечисленных препаратов является наиболее приемлемым при лечении данного больного? Метронидазол

55.Женщина 23 г, обратилась к врачу в ЦСМ с жалобами на выделения слизисто-гнойного характера, тянущие боли внизу живота, дизурию. Беременность 20 недель. Объективно: при обследовании методом ПЦР найдена С.trachomatis. Назначение какого антибактериального препарата является наиболее целесообразным?

Джозамицин,азитромицин

56.К дерматологу в ЦСМ обратился пациент 26 лет с жалобами на зудящие высыпания. Болеет в течение недели, причину заболевания не знает. Объективно: на коже туловища, передней поверхности предплечий и голеней, высыпания представлены плоскими, мелкими блестящими папулами полигональных очертаний синюшно-розового цвета, в центре папул пупкообразное вдавление. Симптом Уикхема положительный. Выставлен диагноз: Красный плоский лишай. Какой симптом из нижеперечисленных надо провести для подтверждения диагноза? Симптом кебнера,сетка уикхема

57.Женщина 50 лет обратилась в ЦСМ к дерматологу с жалобами на высыпания на коже лица и туловища, а также нарушение болевой, тактильной, температурной чувствительности в очагах поражения. При осмотре: на коже лица, туловища, конечностей множественные узлы различных размеров, сливаются и образуют опухолевидные инфильтраты с бугристой поверхностью. Предварительный диагноз: Лепра. Какой препарат необходимо назначить? Препараты сульфонового ряда,рифампин,клофазимин,дапсон

58.Пациент 30 лет обратился к дерматологу в ЦСМ с жалобами, на высыпания по всему кожному покрову и периодически возникающий зуд. Больным себя считает в течении 3-х дней, возникновение сыпи связывает с пыльной работой на цементном заводе. Объективно: по всему кожному покрову имеются рассеянные мелкие эпидермальные папулы округлой формы красного цвета на не измененной коже. Диагноз:

Аллергический контактный дерматит. Какой препарат необходимо назначить для лечения?

Гкс,антигистаминные

Детская хирургия (115)

ЗАПОМИНАНИЕ

1.Характерными симптомами острого аппендицита у детей до 3х лет являются: боль в обл пупка, t° 38-39, интоксикации, многократная

рвота и жидкий стул. Преобладание общей симптоматики над местными

2.Весь комплекс предоперационной подготовки при перитоните у детей проводится за: в течении 2-3 часов

3.Причина врожденной низкой кишечной непроходимости: мекониевый илеус, атрезия ануса, атрезия тощей кишки проверено

a.В основе болезни Гиршпрунга лежит: аномалия развития толстой кишки врождённой

этиологии, приводящая к нарушению иннервации фрагмента кишки (врождённый аганглиоз) — проявляется упорными запорами. Проверено

4.Ранние признаки острого гематогенного остеомиелита длинных трубчатых костей: утолщение и деформация мягких тканей, окружающих кость(2-3 день заболевания)

5.Ориентировочные сроки временной нетрудоспособности после выполнения больному операции по поводу односторонней паховой грыжи (стационарный и амбулаторный этапы) 21-23 суток

6.Пороки развития центральной нервной системы: микроцефалия, анэнцефалия, гидроцефалия

7.С целью анальгезии при переломах инъекция новокаина вводится:

Через всю толщу тканей на уровне места перелома до кости 30-50 мл 1% р-ра

8.При каком виде травмы повреждается один анатомический орган, при воздействии на него двух или более различных факторов внешней среды? При комбинированном

9.Укажите, что характерно ля пороков развития глубоких вен в отличие от пороков развития поверхностных вен? Возникает из-за сдавления вен эмбриональными тяжами, добавочными мышцами( синдром КлиппеляТренога), симптомы обычно проявляются с момента как ребенок начинает ходить

10.Кровоточащий Меккелев дивертикул наиболее часто проявляется в возрасте: 10лет

11.Что такое остеобластокластома? Остеобластокластóма или гигантоклеточная опухоль

доброкачественное новообразование скелета, остеогенного происхождения

ПОНИМАНИЕ

1. На вторые сутки после аппендэктомии по поводу флегмонозного аппендицита у ребенка 5 лет резко ухудшилось состояние: нарастают боли в животе, отмечается многократная рвота, бледность кожных покровов, снижение АД, тахикардия, падение гемоглобина. Какое осложнение вы предполагаете?

Внутреннее кровотечение проверено

2. При операции по поводу острого деструктивного холецистита было обнаружено до 200 мл гнойного эксудата с примесью желчи в подвздошной ямке и малом тазу. Какая распространенность перитонита в данном случае?

Распрастраненная проверено

3. У новорожденного ребенка к концу вторых суток после рождения появились беспокойное поведение, обильные срыгивания, рвота содержимым желудка с примесью желчи, затем – с примесью кишечного содержимого. При осмотре врачомпедиатром на 3-и сутки жизни выявлено: живот вздут, контурируются растянутые петли кишок, живот болезненный при пальпации, перкуторно – пестрота звука. Стула не было, газы не отходят. При попытке постановки очистительной клизмы получена слизь в небольшом количестве. Какой вид врожденной кишечной непроходимости вы предполагаете?

Нарушение поворота кишечника/ низкая киш непр. Проверено

4. Ребенок 11 лет страдает постоянными запорами с рождения. Стул только после очистительных и масляных клизм. Последние 5 суток мать клизмы не проводила. Состояние ребенка резко ухудшилось, многократная рвота, появилось увеличение живота в объеме, при пальпации живота определяются каловые камни. В связи с чем обратились в хирургическое отделение. Каков ваш предварительный диагноз?

Болезнь Гиршпрунга проверено

5. У новорожденного к концу вторых суток после рождения появились беспокойное поведение, рвота содержимым желудка с примесью желчи, затем – с примесью кишечного содержимого. При осмотре: живот вздут, контурируются растянутые петли

кишок, перкуторно – пестрота звука. Стула не было, газы не отходят. При попытке постановки очистительной клизмы установлено отсутствие анального отверстия. На месте заднего прохода имеется кожный валик 0,5 х 0,8 см. При крике ребенка определяется выпячивание в данной области. При надавливании пальцем на промежность имеется ощущение баллотирования. Каков ваш предварительный диагноз?

Атрезия анального отверстия проверено

6. На хирургический прием поступил ребенок 3 летнего возраста. Со слов мамы после стула появляется алая кровь. Объективно ребенок, средней степени тяжести, кожные покровы бледноваты. В крови признаки анемии. Каков ваш предварительный диагноз?

Анальная трешина / полип проверено

7. В детское хирургическое отделение поступил новорожденный, со слов мамы берёт грудь, быстро утомляется и засыпает во время сосания, также во время кормления отмечается посинение носогубного треугольника. При осмотре: перкуторно определяется тупость над правой половиной грудной клетки, отсутствие дыхания справа, полное смещение органов средостения вправо. При бронхоскопии отсутствие правого главного бронха. Каков ваш диагноз?

Агенезия правого бронха и легкого проверено

8. У новорожденного 2 суток жизни внезапно ухудшилось состояние, кашель, одышка, цианоз, тахикардия до 180 в мин, слабый пульс. При перкуссии и аускультации выявлено: слева-перкуторный звук, пуэрильное дыхание, справа-тимпанит, дыхание резко ослаблено, сердце смещено влево. Ваш предварительный диагноз?

Врожденная лобарная эмфизема справа проверено

9. В хирургическое приемное отделение поступил ребенок 5 лет, который болен в течение недели ОРВИ. Получал амбулаторное лечение отхаркивающими средствами и внутримышечные введения пенициллина 2 раза в день. На 8 день заболевания состояние больного резко ухудшилось: ребенок стал вялым, гиподинамичным, стонет. Кожа стала бледной, отмечается цианоз носогубного треугольника. Появилась одышка (ЧД до 48 - 50 в мин). Температура тела – 39-40 С, лейкоцитоз - 18*109 /л. При перкуссии - слева притупление перкуторного звука с IV ребра вниз, границы сердца смещены вправо. Аускультативно - резкое ослабление дыхания слева. Каков ваш предварительный диагноз?

Острая бактериальная деструктивная левосторонняя пневмония, левосторонний экссудативный плеврит (вероятнее всего – пиоторакс) проверено

10. В хирургический стационар поступил ребенок 12 лет с жалобами на боли по задней поверхности шеи. При осмотре: в области задней поверхности шеи имеется

инфильтрат, размером 8,0х6,0 см, кожа над ним напряжена, синюшно-багрового цвета, в центре несколько гнойно-некротических стержней. Какой ваш предварительный диагноз?

Карбункул проверено

11. В хирургический стационар поступил новорожденный с жалобами на повышение температуры тела, снижение аппетита, покраснение в области спины. При осмотре в области спины кожа багрово-синюшного цвета, при пальпации определяются подкожные узелки и симптом флюктуации, по краям имеются маленькие учатски некротизированных тканей. Какое осложнение вы преполгаете? Псевдофурункулез

12.В хирургическое отделение доставлен ребенок ребенок 10 лет с жалобами на боль в колене, повышение температуры до 380С. Из анамнеза: 2 месяца назад после падения с забора сломал имеется отечность в области коленного сустава, активные и пассивные движения болезнены. Каков ваш предварительный диагноз? Постравматический артрит коленного сустава

13.В хирургическое отделение поступил новорождённый. В течение 3 недель не заживает пупочная ранка, постоянные слизистые, гнойные выделения. При осмотре: гиперемия кожи, инфильтрация мягких тканей в околопупочной области, грануляция. При зондировании пупочной ранки зонд уходит на глубину 1.5 см. Каков ваш диагноз?

Гнойный омфалит проверено

14. На консультативном приеме отец 6-летней девочки предъявляет жалобы, что девочка малоподвижна, так как ее беспокоят одышка и боль в левой нижней конечности, а также удлинение этой конечности, большой размер левой стопы на 2-3 см. Лечилась и наблюдалась по поводу врожденного периферического нервного заболевания. Эффекта нет. При осмотре левая нижняя конечность удлинена на 3 см, увеличена в объеме на всем протяжении. На ягодичной области имеется пигментное пятно до 10х 8 см, не выступающее над уровнем кож. Движения, чувствительность в полном объеме. Стопа слева больше в размере, чем правая. Каков ваш предварительный диагноз?

Врожденные артериовенозные свищи проверено

15. У ребенка после рождения в теменной области справа выявлено опухолевидное образование мягко-эластической консистенции. При беспокойстве ребенка

образование становится более напряженным, у основания определяются края костного дефекта. Ваш предварительный диагноз?

Черепно-мозговая грыжа проверено

16. В хирургическое приемное отделение поступил 6-мес. ребенок, со слов мамы, после введения I прикорма, стал крайне беспокойным, отмечалась многократная рвота. В правом подреберье - слабо болезненное подвижное продолговатое образование. При исследовании через прямую кишку - кровь на пальце. Какой диагноз вы предполагаете?

Инвагинация кишечника проверено

17.В хирургическое отделение доставлен девочка 9 лет с жалобами на приступообразные боли в животе. Больна в течение суток. Объективно: состояние средней степени тяжести. Со стороны внутренних органов без особенностей Живот мягкий, болезненный справа по латеральному каналу, симптомов раздражения брюшины нет. Справа положительный симптом 12 ребра. На экскреторной урограмме отмечается запаздывание контрастирования почки справа, дилятация ее полостной системы, мочеточник справа атоничен, прослеживается до средней трети, положительный симптом нисходящего указательного пальца. Каков ваш предварительный диагноз? Стриктура или спазм лоханочно-мочеточникого сегмента

18.В хирургический стационар поступил мальчик 4 лет. Со слов мамы у ребенка с момента рождения не определяется в мошонке правое яичко. Объективно-правая половина мошонки укорочена, яичко не пальпируется ни в мошонке, ни в области

пахового канала. При вроведении УЗИ органов брюшной полости яичко визуализируется в брюшной полости. Каков ваш диагноз?

Крипторхизм проверено

19. Ребенок 4 лет поступил в приемное отделение. Со слов родителей ребенок не двигает, не подымает левую руку. Из анамнеза: за час до поступления старший брат резко потянул за левую кисть. При осмотре: у ребенка резкие боли при попытке ротации левого предплечья. Пассивное сгибание и разгибание предплечья безболезненно. Ваш предварительный диагноз?

Подвывих головки луча у детей проверено

20. Мальчик 8 лет, обратился на приём с травмой правой верхней конечности, упал на улице с велосипеда. Локально: отёк, боли, деформация правого предплечья, резкое ограничение движения в правом локтевом суставе. На рентгенограмме – перелом диафиза локтевой кости с угловым смещением, вывих головки лучевой кости. Каков ваш диагноз?

Повреждение Монтеджи.

Это повреждение предплечья относится к переломовывихам: перелом локтевой кости в верхней трети и вывих головки лучевой кости проверено

21.Во время игры мальчик 10 лет случайно получил удар клюшкой в область грудины. Отмечается сильная боль в месте травмы при попытке глубоко вдохнуть. Объективно: в области тела грудины припухлость, гематома, ссадина. Пальпаторно – местная сильная боль, отмечается умеренная деформация в области грудины. Границы средостения незначительно расширены в зоне наибольшей болезненности. Тоны сердца обычные. Дыхание проводится с обеих сторон, но экскурсия грудной клетки неглубокая. Каков ваш предварительный диагноз? Перелом грудины

22.В травмпункт поступил ребенок 3-х лет, с жалобами на 2-х кратную рвоту, головные боли, плохой аппетит, слабость, вялость. Из анамнеза: ребенок 2 дня назад упал дома, ударился головой, отмечалась утрата сознания на несколько секунд. Объективно: при пальпации мягких тканей лобной области отмечаются боли и припухлость. На рентгенограмме черепа: без костной патологии. Каков ваш предварительный диагноз? Сотрясение головного мозга

23.На приём к врачу-педиатру обратились родители с мальчиком 4,5 лет. Установлено, что ребёнок 1 час тому назад случайно опрокинул на себя кружку с кипятком. При осмотре: на передней поверхности живота, бёдер, голеней отмечается повреждение кожи в виде отдельных очагов отёка, гиперемии с наличием эпидермальных пузырей, заполненных серозной жидкостью, часть из них вскрывшиеся. Частота дыхания — 40 в минуту. Пульс — 140 в минуту, АД — 80/55 мм рт. ст. Какой степени термический ожог имеет место? 2 степень

24.Во время оказания акушерского пособия ребенок получил травму правой верхней конечности. При осмотре рука свисает вдоль туловища, активные движения отсутствуют, болезненность в плечевом суставе. Каков ваш предварительный диагноз?

Травмы плечевого сплетения

25.Ребенок упал с дерева, ударившись левым боком о камень. Сознания не терял. Домой дошел самостоятельно. Доставлен в

хирургический стационар через 2 часа. Состояние тяжелое, АД 60/50, пульс 120 в минуту. В анализе крови Нв - 78 г/л, в общем анализе мочи эритроциты сплошь в поле зрения. Каков ваш предварительный диагноз? Гематома почки

26.Мальчик 8 лет упал с велосипеда и ударился животом о руль, сразу же почувствовал боли в животе, позывы на мочеиспускание. Доставлен в клинику через 3 часа после травмы, кожные покровы бледные, пульс 120 ударов в мин. А/Д 110-60 мм рт ст. Живот в дыхании не участвует, напряжен, резкое болезненный в нижних отделах, здесь же положительный симптом Щеткина-Блюмберга. Учитывая, что ребенок длительное время не мочился произведена катетеризация мочевого пузыря мягким катетером, получено до 5,0 мл, кровянистой жидкости. Каков ваш предварительный диагноз? Гематома почки

27.В хирургическое отделение после ДТП поступил ребенок 5 лет с множественными травмами. При осмотре: ребенок в сознании, беспокоен, кожные покровы бледные, тахикардия, тахипное. АД 90 уд.в минуту. Какая степень травматического шока наблюдается у ребенка? 1 легкая

28.На хирургический прием поступил ребенок 10 лет с жалобами на дисфагию, срыгивание непереваренной пищей и отставание в развитии. При гастроскопии обнаружено расширение просвета пищевода. Каков наиболее вероятный диагноз?

Ахалазия кардии пищевода

29.В хирургическое отделение поступил мальчик 6 лет. Заболел внезапно: появились боли в левой подвздошной области. Из анамнеза: частые запоры. При осмотре: напряжение мышц в левой подвздошной области отсутствует. Какой диагноз наиболее вероятен? Заворот сигмы

30.Девочка 13 лет поступала в хирургическое отделение с жалобами на жидкий стул с кровью в течение шести месяцев, снижение массы тела и спастические боли в животе. При исследовании выявлен воспалительный процесс в толстой кишке. Между участками поражения есть неизмененные зоны. Прямая кишка без поражений. Каков предварительный диагноз?

Болезнь крона

31.В хирургическое отделение поступил ребенок 4 лет с жалобами на боль в правой половине грудной клетки, одышку, сухой кашель. При осмотре: уменьшение дыхательной экскурсии грудной клетки и отставание правой половины при дыхании. Над очагом поражения определяется тупой перкуторный звук, дыхание резко ослаблено. На рентгенограмме определяется тотальное затемнение плевральной полости со смещением средостения в пораженную сторону. Наиболее вероятный диагноз? Плевральный выпот

32.Ребенок 5 лет поступил с хирургическое отделение с ущемленной пахово-мошоночной грыжей на 3-и сутки от начала заболевания. Температура до 39°С, гиперемия, инфильтрация и отек мошонки. Какое осложнение наблюдается у больного? Флегмона

33.У ребенка 2 лет появились явления трахеобронхита, сухой кашель, временами одышка. При рентгенологическом обследовании органов грудной клетки патологии не выявлено. При трахеобронхоскопии определяется некоторое сужение трахеи в нижней трети за счет выбухания в просвет мембранозного отдела. Каков ваш предварительный диагноз? Стеноз трахеи

34.На хирургический прием поступил ребенок 7 лет с жалобами на частые запоры в течение последнего года и выделения алой крови в конце акта дефекации в последние 2 дня. Каков ваш предварительный диагноз? Трещины

стенки заднего прохода

35. В патологии новорожденных находился ребенок 26 дней по поводу омфалита и пиодермии. За два дня до поступления в хирургическое отделение состояние ребенка ухудшилось. При осмотре: припухлость правого плечевого сустава, отсутствие активных движений в этой ручке, болезненность при пассивных движениях. Каков ваш предварительный диагноз? Инфекционный (гематогенный) остеомиелит

36.У ребенка 12 лет на 6-й день после аппендэктомии по поводу гангренозного аппендицита повысилась температура тела, появились озноб, боли внизу живота, тенезмы, боли в конце мочеиспускания, Л-14,1х10 59 0/л. Какое осложнение возникло у больного? Послеоперационный перитонит

37.На шестые сутки после аппендэктомии у ребенка 5 лет резко ухудшилось состояние: боли в животе, многократная рвота, повышение температуры тела, напряжение и болезненность брюшной стенки, положительные симптомы раздражения брюшины, гиперлейкоцитоз. Какое осложнение развилось у ребенка? Внутрибрюшное осложнение - перитонит

ПРИМЕНЕНИЕ

1. Девочка 5 лет поступила в клинику с жалобами на боли в правой подвздошной области. Из анамнеза: заболела остро за несколько часов до поступления. Появились резкие боли в области живота справа, повысилась температура тела до 37.8°С. Объективно: общее состояние ребенка средней степени тяжести. Язык у корня обложен белым налетом. При пальпации живота определяется напряжение мышц передней брюшной стенки, локальная болезненность в правой подвздошной области, положительный симптом Щеткина-Блюмберга. Общий анализ крови: Лейк -12х109/л; СОЭ – 32 мм/час. Ребенок был взят на операцию, во время которой выявлен неизмененный дивертикул Меккеля. Какова ваша дальнейшая тактика ведения?Удаление дивертикула Меккеля(не аппеэндэктомия)

2.У больного 10 лет, оперированного по поводу гангренозного аппендицита, на 6-й день после операции отмечен подъем температуры, которая приобрела гектический характер. Живот мягкий, безболезнен. Отмечено частое мочеиспускание и позывы к дефекации. Какое обследование вы назначите для дальнейшего ведения пациента? пальцевое ректальное обследование

3.Мальчик 7 лет поступил в хирургический стационар. Из анамнеза: за двое суток до поступления появились резкие боли в области живота справа. На следующий день повысилась температура тела до 37.8°С, состояние постепенно стало ухудшаться. Объективно: общее состояние ребенка средней степени тяжести. Язык у корня обложен белым налетом. При пальпации живота определяется напряжение мышц передней брюшной стенки, локальная болезненность в правой подвздошной области, положительный симптом Щеткина-Блюмберга. Общий анализ крови: Лейк -12,3х109/л; СОЭ – 40 мм/час. На операции аппендэктомии: выпот прозрачный, около отростка небольшие фибринозные наложения, инфильтрация тканей. Какова дальнейшая тактика?Одномоментное введение антибиотиков

4.Пациент 10 лет поступил в хирургическое приемное отделение с жалобами на боли в животе, рвоту, задержку стула. Из анамнеза: со слов родителей оперирован год назад по поводу аппендикулярного перитонита. Начало заболевания отмечают три дня назад, когда возникли боли в животе и задержка стула. За день до поступления появилась рвота, в связи с чем, обратились в хирургическое приемное отделение. Объективно: состояние ребенка средней степени тяжести, при осмотре живот асимметрично вздутый, прикосновения к нему усиливают приступы боли, отчетливо видна перистальтика петель кишечника. Лейк -10х109/л; СОЭ – 20 мм/час. Какое обследование вы назначите в первую очередь? Обзорная рентгенография брюшной полости(рентгеноскопия)

5.В хирургическое приемное отделение ребенок 1 мес., масса тела при рождении 3800.Беременность протекала с токсикозом первой половины, с 10 дня жизни появилось срыгивание, с 14 дня рвота после каждого кормления, затем рвота стала реже, но большей частью «фонтаном», створоженным молоком. Ребенок стал, редко мочится, появились запоры. Вес 2900. Участковый педиатр назначил атропин, но-шпу, улучшения нет. При поступлении состояние тяжелое. Кожа дряблая, складки на лице и бедрах, п/к жировой слой выражен слабо. При осмотре живота в эпигастральной области видны крупные волны перистальтики, которые переходят вправо и исчезают.

Какое обследование вы назначите для уточнения диагноза?Обзорная рентгенография

брюшной полости

6.У новорожденного через 3 часа после рождения появилась рвота с примесью желчи сразу после кормления. Через 6 часов была отмечена повторная рвота Мекония не было. Кожные покровы бледно-розовые, чистые. Живот несколько увеличен в размерах в верхних отделах и запавший в нижних. Зонд свободно проходит в желудок. Содержание последнего – створоженное грудное молоко с желчью. Какое обследование вы назначите для уточнения диагноза? Обзорная рентгенография брюшной полости

7.В хирургическое приемное отделение поступил ребенок 2 лет с диагнозом болезнь Гиршпрунга. Жалобы на появление поноса («парадоксального поноса») на фоне длительного отсутствия стула. Какова ваша тактика?сифонные клизмы с 1% раствором поваренной соли с предварительным введением в прямую кишку вазелинового масла, постоянный зонд в желудок, двусторонняя паранефральная новокаиновая блокада, дезинтоксикационная терапия

8.В хирургическое приемное отделение ребенок 1 года, со слов мамы у ребенка частые запоры, стул только после очистительной клизмы, вздутие живота. Объективно: ребенок средней степени тяжести, при осмотре: живот вздут, «симптом глины». Какое обследование вы назначите для уточнения диагноза? Ирригография/пальцевое исследование прямой кишки

9.Новорожденный доставлен в отделение хирургии каретой скорой помощи. Из анамнеза: роды в срок 39 недель, Врач–акушер заметил отсутствие анального отверстия, в связи с чем, ребенок был направлен в клинику. Объективно: ребенок вес 3 кг, рост 50см. Анальное отверстие отсутствует. Имеется ректовестибулярный свищ, по которому хорошо отходит меконий. Какова ваша тактика? ? До 3-4 месяцев ведется консервативное лечение/ 5-6 месяцев проктопластика.

10.В хирургическое приемное отделение поступил новорожденный, возраст 1 сутки. Ребенок родился от нормально протекавшей беременности и родов, в срок, с массой тела 3000г. Состояние после рождения удовлетворительное. При осмотре промежности отмечено отсутствие анального отверстия. По средней линии - кожный

валик, седалищные бугры сближены, симптом толчка отрицателен. Какое

обследование вы назначите для дальнейшего планирования тактики лечения?УЗИ

11.Пациент 4 года, поступил в клинику с жалобами на кровотечения во время дефекации. Из анамнеза: со слов родителей отмечаются постоянные запоры, дискомфорт во время акта дефекации, ощущение незавершённости процесса после посещения туалета. За день до обращения появилось кровотечение во время акта дефекации. Объективно: состояние ребенка средней степени тяжести, при ректальном обследовании обнаружен полип слизистой прямой кишки. Общий анализ крови: Нb - 110 г/л; Эр – 3,5xl012/л; Ц.п. - 0,9; Лейк - 5х109/л; п/я - 4%, с/я - 48%, л - 39%, э - 2%, м - 7%; СОЭ – 10 мм/час. Какова ваша тактика? Полипэктомия

12.Девочка 3 лет жалуется на боли при дефекации, задержка стула, в конце дефекации выделяется несколько капель алой крови. При осмотре на боковой стенке заднепроходного отверстия определяется трещина размером 0,5 см, неглубокая. Какова ваша тактика? Диета

13.Пациент 5 лет поступил в хирургический прием. Из анамнеза: до поступления в стационар у ребенка наблюдались нарушения дыхания и одышка при физических нагрузках, которые самопроизвольно исчезали. За день до обращения у ребенка появилась одышка, постепенно нарастающая, дыхание стало затрудненным. Объективно: кожные покровы цианотичного оттенка, пульс 100 уд. в минуту, ЧД 30 в минуту. АД 100/60 мм.рт.ст. При осмотре грудной клетки отмечается участие в акте дыхания вспомогательной мускулатуры, набухание вен на шее. При перкуссии правого легкого укорочение перкуторного звука над зоной поражения. На рентгенограмме грудной клетки: напряженная киста правого легкого. Какова ваша тактика лечения? Дренирование кисты

14.В хирургическое приемное отделение доставлен новорожденный. Через 40 минут после рождения наблюдалось нарастание дыхательной недостаточности, периодически наблюдались приступы тахипноэ и цианоза, выраженное втяжение уступчивых мест грудной клетки, движения правой и левой половины грудной клетки асинхронны, ЧСС – 165 ударов в минуту, верхушечный толчок сердца смещён вправо, перкуторно в нижнем отделе левого лёгкого определяется тимпанит, здесь же отсутствие дыхательных шумов. Появляется цианоз в положении на правом боку, отмечается запавший «ладьевидный» живот. Сатурация кислорода – 84 %, рН – 7,14. Какой метод обследования вы назначите для уточнения диагноза? Обзорная Рентгенография ГК

15.Состояние новорожденного тяжелое. Одышка. Дыхание слева ослаблено. Перкуторно справа коробочный звук. На рентгенограмме справа увеличение прозрачности легкого с резким обеднением рисунка. В нижнем отделе справа треугольная тень, прилегающая к тени средостения. Средостение смещено влево с снижением прозрачности левого легкого. Какой метод обследования вы назначите для подтверждения и установления диагноза?Радиоизотопное обследование легких

16.У новорожденного через 5 минут после рождения развился приступ асфиксии, переведен на искусственную вентиляцию легких. При осмотре грудная клетка вздута, дыхание с втяжением уступчивых мест грудной клетки. Аускультативно слева дыхание не проводится, справа – пуэрильное. Перкуторно слева участками определяется притупление, участками - коробочный звук. Сердечные тоны глухие, до 180 в мин., выслушиваются максимально громко по правой среднеключичной линии. Какое обследование вы назначите для уточнения диагноза? Рентгенография ГК

17.В хирургическое приемное отделение доставлен ребенок 1,5 лет. Отмечается выраженное беспокойство, учащенное дыхание, левая половина грудной клетки отстает в акте дыхания. Из анамнеза известно, что получал лечение по поводу ОРВИ в течение 5 дней. 2 часа назад резко ухудшилось состояние. Какое обследование вы назначите для уточнения диагноза? Обзорная рентгенография грудной клетки

18.В хирургическое приемное отделение поступил ребенок 7 лет. Жалобы на повышение температуры до 38-39 градусов, слабость, одышка, кашель с гной мокротой. На рентгенограмме: абсцесс правого легкого с дренированием в бронх. Какова ваша тактика? Трансторакальное дренирование по Мональди

19.Новорожденный ребёнок находится в стационаре с диагнозом некротическая флегмона в области поясницы, была проведена манипуляция:

нанесение насечек кожи в области флегмоны, через 6 часов отмечается распространение процесса на ягодицы. Какова дальнейшая тактика лечения?

Измерить площадь поражения. Сделать разрезы-насечки в шахматном порядке с

переходом на здоровые ткани. Перевязка каждые 4часа. Инфузионная и аб терапия

20.В хирургическое приемное отделение поступил новорождённый. В течении 3 недель не заживает пупочная ранка, постоянные слизистые, гнойные выделения. При осмотре: гиперемия кожи, инфильтрация мягких тканей в околопупочной области, грануляция. Какое обследование вы назначите для уточнения диагноза? бак исследование

21.Пациент 6 лет доставлен в стационар с жалобами на общее недомогание, боли в области правой голени. Из анамнеза: со слов мамы ребенок болен в течение 10 дней, начало заболевания отмечает с повышения температуры до 39°С, болей в области правой голени. Затем боли постепенно усиливались, появились припухлость и покраснение правой голени, ребенок не мог наступать на ногу, в связи с чем обратились в клинику. Была проведена диагностическая остеоперфорация, в ходе которой был получен гной. Каковы дальнейшие действия хирурга? Острый гематогенный остеомиелит. Госпитализация. Бак.посев для выделения возбудителя. АБ(цефалоспорины 2 и 3 поколения). Антикоагулянты,дезагреганты,инфузионная терапия, симптоматическая терапия

22.В хирургическое приемное отделение новорождённый, возраст 3 недели с диагнозом: Остеомиелит проксимального эпифиза бедра и обширная деструкция в метаэпифизарной зоне. Что вы предпримите для предупреждения ближайших и отдаленных осложнений? Декомпрессивная остеоперфорация

23.Новорожденный 25 дней. При осмотре врача резко заторможен. Кожа сероватожелтушного цвета, отек и гиперемия кожи вокруг пупка, пупочная ранка покрыта кровянистой корочкой, выбухает. Выше пупка пальпируется плотный тяж. Дыхание аритмично, ослабленно, мелкопузырчатые влажные хрипы непостоянной локализации. Тоны сердца глухие, ЧСС — 150 в 1 мин. Общая мышечная гипотония, гипорефлексия. Живот вздут, напряжен, печень +2 см, селезенка + 0,5 см. Стул жидкий,

спримесью белых комочков и зелени. Температура тела 35,1°С. ОАК: СОЭ -25 мм/ч. Биохимический анализ крови: билирубин — 198,6 мкмоль/л; мочевина 9,1 ммоль/л; К-

3,9 ммоль/л; Na -128 ммоль/л; СР-Б (++). IgМ- 0,6 г/л; IgА - 0,15 г/л; IgG - 5,6 г/л. Какова ваша тактика?Санация пупочной раны,дезинтоксикация,иммунокоррекция. Срочная госпитализация. Хирургическое вмешательство.

24.Пациент 2 года поступил в клинику. Из анамнеза: со слов родителей проглотил 5 сомовую монету 1 час назад. Объективно: ребенок беспокойный, состояние средней степени тяжести, отмечается саливация и дисфагия. Общий анализ крови: Нb - 120 г/л; Эр – 3,7xl012/л; Ц.п. - 0,9; Лейк - 5х109/л; п/я - 5%, с/я -

49%, л - 36%, э - 1%, м - 8%; СОЭ – 10 мм/час. Какова ваша тактика? Рентген/бронхоскопия

25.Ребенок 8 лет проглотил батарейку. На рентгенограмме она в пищеводе. Какова ваша тактика? Эзофагоскопия

26.У ребенка проникающее ранение брюшной полости в области правой доли печени. Состояние средней тяжести. Гемодинамика стабильна. Перитонеальные знаки не выражены. Свободная жидкость в брюшной полости. Что вы предпримите в первую очередь? Лапароцентез

27.Ребенок 5 лет проглотил гвоздь длиной 40 мм, два часа назад. На обзорной рентгенограмме гвоздь фиксирован в желудке. Какова ваша тактика? Гастроскопия

28.В хирургическое приемное отделение поступил ребенок 1 года. Жалобы на наличие образование в районе пупка. При осмотре: Общее состояние не нарушено. На передней брюшной стенке определяется шаровидное образование, безболезненное, расширение пупочного кольца составляет 0,5 см. Какова ваша тактика лечения?Консервативно, Герниопластика в 4-5 лет

29.В хирургическое приемное отделение ребенок 1 года с подозрением на острую форму болезни Гиршпрунга. Какое обследование вы назначите для уточнения диагноза? Пальцевое исследование прямой кишки

30.Мальчик 2 лет госпитализирован в хирургическое отделение с жалобами на рвоту с примесью кала, отсутствие стула. Из анамнеза: три недели назад перенес спленэктомию и ушивание желудка после тупой травмы живота. 2 дня назад появилась рвота, сначала с примесью желчи, затем с примесью кала. Объективно: общее состояние средней тяжести, ребенок беспокойный. При осмотре – живот вздут, через растянутую брюшную стенку видны очертания вздутых петель кишечника. Пальпация живота болезненна, перкуторно определяется тимпанит, при аускультации – ослабленные шумы перистальтики. На обзорной рентгенограмме органов брюшной полости отмечаются множественные уровни и арки. Какая лечебная тактика показана в этом случае? экстренное хирургическое вмешательство

31. В хирургическое приемное отделение поступил новорожденный. К 6-м суткам после рождения состояние ребенка ухудшилось, наросли явления токсикоза и эксикоза. Повторная рвота кишечным содержимым, при зондировании удалено до 60 мл. Кожные покровы бледно-цианотичные, мраморные, тургор снижен, акроцианоз. Живот вздут, болезненный при пальпации в правой подвздошной области. Брюшная стенка отечна. Перистальтика кишечника ослаблена. Стула нет. При осмотре из прямой кишки получен скудный стул с прожилками алой крови. Какой метод обследования вы назначите для подтверждения диагноза?

Рентгенография брюшной полости

32. У ребенка в течение 12 часов приступообразные боли в животе, задержка отхождения газов, рвота. Год назад оперирован по поводу аппендицита. Какое обследование вы назначите для уточнения диагноза?

обзорная рентгенография брюшной полости

33. Больная 9 лет поступила в хирургическое приемное отделение с жалобами на приступообразные боли в животе. Больна в течение суток. Объективно: состояние средней степени тяжести. Со стороны внутренних органов без особенностей Живот мягкий, болезненный справа по латеральному каналу, симптомов раздражения брюшины нет. Справа положительный симптом 12 ребра. На экскреторной урограмме отмечается запаздывание контрастирования почки справа, дилятация ее полостной системы, мочеточник справа атоничен, прослеживается до средней трети, положительный симптом нисходящего указательного пальца. Какие методы обследования вы назначите для подтверждения диагноза?

Радионуклидная ренография/Мультиспиральная КТ (урограмма)

34. В поликлинику обратились родители мальчика 3 месяцев, у которого ущемилась паховая грыжа. С момента ущемления прошло 12 часов. Какова ваша тактика?

экстренная операция

35.У мальчика с рождения отмечено расщепление крайней плоти. Ребенок мочится

снатуживанием тонкой струйкой. Наружное отверстие уретры точечное в области венечной борозды. Выставлен диагноз: «Гипоспадия». Какова тактика хирурга?

Рассечение наружногоотверстия уретры.Пластика уретры после года постановки диагноза

36.Ребенок 4 лет поступил в приемное отделение. Со слов родителей ребенок не двигает, не подымает левую руку. Из анамнеза: за час до поступления старший брат резко потянул за левую кисть. При осмотре: у ребенка резкие боли при попытке

ротации левого предплечья. Пассивное сгибание и разгибание предплечья безболезненно. Какова ваша тактика?

Обезболивание и вправление

37. Ребенок 10 лет поступил в травмпункт с жалобами на боли, отечность левой голени, ограничение движений. Из анамнеза: за два часа до поступления упал на левую ногу. При осмотре: выраженный отек, деформация, пальпаторно определяется крепитация. На рентгенограмме: косой перелом нижней трети обеих костей левой голени без смещения. Какова ваша тактика?

Наложение гипсовой лонгеты

38. Ребёнку 6 лет, локально: боли, отёк правого предплечья, на рентгенограмме под надкостничный перелом в типичном месте без смещения. Какова ваша тактика лечения?

Гипсовая или полимерная повязка

39. Ребенок 9 лет поступил в травмпункт с жалобами на боли, отек правого предплечья. Из анамнеза: упал на улице на правую руку за час до поступления. Объективно: выраженный отек, деформация нижней трети правого предплечья. На рентгенограмме: перелом нижней трети обеих костей правого предплечья с угловым смещением. Какова ваша тактика?

Обезболивание,репозиция,гипс

40. Ребенок 8 лет поступил в травмпункт с жалобами на боли, отечность левой голени, ограничение движений. Из анамнеза: за два часа до поступления упал на левую ногу. При осмотре: выраженный отек, деформация, пальпаторно определяется крепитация. На рентгенограмме: косой перелом нижней трети обеих костей левой голени, отмечается смещение по ширине на пол поперечника и смещение по длине на 1,5 см. Какова ваша тактика?

Обезболивание репозиция гипс

41. Ребёнку 10 лет, локально: боли, отёк левого надплечья, ограничение движения левого плечевого сустава, на рентгенограмме перелом средней треть ключицы без смещения. Какова ваша тактика лечения?

Фиксирующая повязка(шины Кузьминского,дезо,кольца дельбе)

42. Ребенок 1,5 года, поступил в травмпункт. Со слов родителей у ребенка ограничение движения левого плечевого сустава, в области левого надплечья припухлость. Из анамнеза: ребенок дома за 2 часа до поступления упал на левое плечо. На рентгенограмме: перелом средней трети левой ключицы со смещением по ширине на полный поперечник, по длине 0,2 см. Какова ваша тактика лечения:

Обезболив,репозиция наложение спицы,фиксир повязка

43. В травмпункт поступил ребенок 10 лет в сопорозном состоянии. На вопросы отвечает невнятно, с трудом. Отмечается многократная рвота. Из анамнеза: ребенок упал на улице за час до поступления, ударился головой о бордюр. Отмечалась утрата сознания до нескольких минут. Локально: обширная подкожная гематома теменной области слева. На рентгенограмме черепа: линейный перелом теменной кости слева. Какое обследование вы назначите для последующего выбора тактики лечения?

КТ/МРТ

44. В стационаре находится больной 9 лет с диагнозом: Закрытая черепно-мозговая травма, ушиб головного мозга средней степени тяжести, компрессионный перелом теменной кости справа, подкожная гематома теменной области справа. На КТ глубина импрессии 0,9 см. Какова ваша тактика при оперативном лечении?

Краниектомия,элевация декомпрессия

45. В приемное отделение поступил больной с резаной раной задней поверхности правой голени. Рана покрыта бинтовой повязкой, которую родители больного наложили самостоятельно, полностью пропитанная кровью. Рана продолжает кровить. Объективно: ребенок в сознании, кожные покровы бледно-розовой окраски, акроцианоз. Какие действия необходимо применить в первую очередь?

ПХО,остановка кровотечения, наложение швов

46. Ребенок 14 лет упал дома на правую ногу в результате чего получил открытый перелом правой бедренной кости, рана обильно кровит, из нее визуализируется кость. Была вызвана скорая помощь. Какова первая помощь врача скорой помощи в данном случае?

Остановка кровотечения ,обезболивание, наложение шины

47. В травмпункт поступил ребёнок с раной левой кисти, при осмотре рана кровоточит, размеры 1.0х3.0 см, ограничений движений левой кисти и пальцев нет. Дном раны являются сгустки крови и мягких тканей. Какой раствор вы примените для первичной хирургической обработки раны у данного больного?

Перекись водорода 3%

48. В травмпункт поступил ребенок 10 лет с раной на подошвенной области правой стопы. Из анамнеза за 2 часа до поступления ребенок во дворе дома наступил на гвоздь, который вошел в стопу примерно на 5 см. Локально: края раны небольших размеров, рана сомкнута, выделений нет. Какова ваша тактика лечения?

Обработка антисептиком,введение противостолбнячной сыворотки,наблюдени

49. В поликлинику поступил новорожденный, возраст – 2 дня. При осмотре: обширная кефалогематома слева в теменной области. Каковы ваши дальнейшие действия?

Пункция(аспирация крови),накладывают давящую повяз

50. Во время оказания акушерского пособия ребенок получил травму правой верхней конечности. При осмотре рука свисает вдоль туловища, активные движения отсутствуют, болезненность в плечевом суставе. Какова ваша тактика?

Иммобилизация повязкой Дезо и перевод в отделение хир новорожд выполняем рентгенографию,лечение:репозиция,иммобилизация на отводящей шин

51. Новорожденный ребенок, возраст 1 день, поступил в травмпункт. Из анамнеза: роды протекали в ножном предлежании. Во время родов было повреждено левое бедро. Локально: в области левого бедра отмечается припухлость, деформация, пальпаторно патологическая подвижность, крепитация. На рентген снимке: косой перелом средней трети левой бедренной кости со смещением. Какова ваша тактика лечения?

Обезболивание,репозиция,гипс

52. Пациент 10 лет был доставлен в хирургический прием каретой скорой помощи. Из анамнеза: Упал с велосипеда и ударился животом об руль за 2 часа до поступления. Объективно: кожные покровы бледные, конечности на ощупь холодные. Пульс 100 ударов в минуту, АД 100/60 мм.рт.ст. При осмотре живота – живот напряжен, при глубокой пальпации левого подреберья - резкие боли. Общий анализ крови: Нb - 108 г/л; Эр – 3,7xl012/л. УЗИ внутренних органов брюшной полости: Свободная жидкость около селезенки. Пациент был взят на операционный стол. Во время операции обнаружен массивный разрыв селезенки с переходом на область ворот органа. Продолжается активное кровотечение. Какова дальнейшая тактика хирурга?

Спленэктомия

53. У ребенка 7 лет с травмой живота перкуторно отмечено исчезновение печеночной тупости. Какое обследование вы назначите для уточнения диагноза?

Рентгенографию брюшной полости или УЗИ внутренних органов

54. Ребенок 5 лет поступил с автотравмой. Состояние тяжелое, в сознании. Жалуется на боль в низу живота, которая усиливается при позыве на мочеиспускание. При осмотре - в области промежности припухлость, пастозность тканей. При пальпации живот напряжен, болезненный, положительные симптомы раздражения брюшины. Из уретры выделятся кровь каплями. Какое обследование вы назначите для подтверждения диагноза?

Восходящая цистография

55. Ребенок 6 лет поступил в хирургическое отделение в тяжелом состоянии с жалобами на рвоту «кофейной гущей». При осмотре кожные покровы бледные, холодные. Пульс частый слабого наполнения. АД 90/50 мм.рт.ст. В анамнезе: страдает в течение года циррозом печени. Какое обследование вы назначите для подтверждения диагноза?

ЭГДС

56. Ребенок 6 лет поступил в хирургическое отделение в тяжелом состоянии с жалобами на рвоту «кофейной гущей». При осмотре кожные покровы бледные, холодные. Пульс частый, слабого наполнения. АД 90/50 мм.рт.ст. В анамнезе: страдает в течение года циррозом печени. При обследовании выявлено кровотечение из варикозных вен пищевода. Какова ваша тактика?

Зонд блэкмора

57. В хирургическое отделение поступил ребенок 3 лет с жалобами на внезапное ухудшение состояния, появление одышки. Из анамнеза: болеет пневмонией в течение 6 дней. При осмотре: кожные покровы цианотичны, частота дыхания 80 в минуту, наблюдается кряхтящее дыхание. Температура тела 37,6 градусов. Правая половина грудной клетки вздута. Перкуторно над областью поражения определяется коробочный звук, дыхание над этой зоной не прослушивается. Какова ваша тактика?

рентгенография огк

58. У ребенка 2 лет появились явления трахеобронхита, сухой кашель, временами одышка. При рентгенологическом обследовании органов грудной клетки патологии не выявлено. При трахеобронхоскопии определяется некоторое сужение трахеи в нижней трети за счет выбухания в просвет мембранозного отдела. Какое обследование вы назначите для подтверждения диагноза?

Рентгеноскопия бариевой смесью (подозрение на дивертикул пищевода?)

59. На хирургический прием поступил ребенок 7 лет с жалобами на частые запоры в течение последнего года и выделения алой крови в конце акта дефекации в последние 2 дня. Какое обследование вы назначите для подтверждения диагноза?

Пальцеводное исследование , потом ректороманоскопию

60. В хирургическое отделение поступил мальчик 10 лет с жалобами на частые приступы кашля. При осмотре: состояние относительно удовлетворительное, температура тела не повышена. Из анамнеза: впервые кашель появился 7 месяцев назад, когда ребенок был в гостях у бабушки, которая работает фермером на ржаном поле. Какое обследование вы назначите для подтверждения диагноза?

· бак посев на Aspergillus spp. “ПРОВЕРЕНО”

61. Каретой скорой помощи доставлен новорожденный возрастом 5 дней. Со слов мамы жалобы на ухудшение состояния, потеря в весе. Из анамнеза: была угроза выкидыша на 8-12 недели, роды прошли нормально, по Апгар 8-9 баллов. С первых суток сосал активно, но срыгивал с примесью желчи. Меконий отошел в первые сутки, затем - стул скудный. При осмотре: кожные покровы с мраморным оттенком, тургор снижен. Сердечные тоны приглушены, ритмичные 133 в минуту. Живот мягкий, запавший. При зондировании удалено 60 мл застойного содержимого, окрашенного желчью. Какое обследование кишечника вы назначите для подтверждения диагноза?

·Рентгенологическое обследование: обзорный снимок брюшной полости и два снимка в прямой и боковой проекции через 40 минут после дачи контрастного вещества через рот

“ПРОВЕРЕНО”

62. В хирургическое отделение поступила девочка 9 лет с жалобами на ноющие ночные боли в верхней трети левой голени. Больна в течение 2-х месяцев. Заболела после падения с велосипеда. В первые 2 недели после травмы температура тела повышалась до 37,4°. Конечность при осмотре не отличается от здоровой. При пальпации имеется небольшая болезненность в верхней трети левой большеберцовой кости. Движения в коленном суставе в полном объеме, безболезненные. На рентгенограмме обеих голеней обнаружена полость с четкими границами круглой формы диаметром 2 см в проксимальном метафизе левой большеберцовой кости. Какова ваша тактика?

· Костная киста: рекомендуется хирургическое вмешательство

“ПРОВЕРЕНО”

63. В хирургический стационар поступил новорожденный возрастом 3 недели, у которого повысилась температура тела, появился отек в области левой грудной железы. При осмотре: кожа над пораженным участком гиперемирована, пальпация железы болезненная, определяется флюктуация. Какова ваша тактика?

· хирургическое лечение (разрезы в радиальном направлении, отступя на 3-4 мм от

околососкового кружка над участком размягчения

“ПРОВЕРЕНО”

64. У мальчика 10 лет после обильной еды появились сильные схваткообразные боли в животе, многократная рвота. Стула не было, газы не отходят. Два года назад у ребенка была лапаротомия по поводу закрытой травмы живота. Мальчик

бледный, мечется от боли. Живот асимметричен, вздут в левой половине, определяется видимая перистальтика кишок, перкуторно-перемежающийся тимпанит. При обследовании прямой кишки - пустая ампула. На рентгенограмме брюшной полости - чаши Клойбера. Что вы предпримите в первую очередь?

·хирургическое вмешательство проверено

65.В течение 2 недель ребенку 13 лет проведено успешное консервативное лечение «холодного» аппендикулярного инфильтрата. Какова ваша дальнейшая тактика?

·Через 2-2,5 месяца после рассасывания инфильтрата показана плановая аппендэктомия

проверено

66.На хирургический прием поступил мальчик 3 лет. Со слов мамы жалобы на то, что у ребенка с рождения отмечено расщепление крайней плоти, мочится с натуживанием тонкой струйкой. При осмотре: наружное отверстие уретры точечное в области венечной борозды. Какова ваша тактика лечения?

Лечение гипоспадии проводится только оперативным путем проверено

Инфекционные болезни (225)

Запоминание

1.Назовите основной путь передачи вирусного гепатита С? -Парентеральный Через кровь инфицированного человека Проверено

2.Назовите маркеры острого вирусного гепатита D:

-HDV ag HD ag, анти HDV IgM, анти HDV Ig G, РНК HDV

Проверено

3.Укажите препарат для лечения трехдневной малярии?

-Хинин хлорохин Проверено

4.Сколько дней составляет инкубационный период при сальмонеллезе?

-От 2 часов до 2-3 дня. В среднем 12-24 часа. Проверено

5.Укажите чем определяется тяжесть течения ботулизма? -Острой дыхательной недостаточностью

6.Назовите заболевание при котором характерно появление мономорфной розеолезной сыпи и симптома Падалки?

-Брюшной тиф

7.Сколько месяцев составляет продолжительность инкубационного периода при вирусном гепатите В?

-6 месяцев

8.Укажите наиболее ранний симптом печеночной энцефалопатии: -Психические нарушения

9.Назовите наиболее часто встречающуюся форму чумы? -Бубонная

10.Отметьте наиболее частый симптом Бешенства?

-Гидрофобия

11.Укажите возбудителя псевдотуберкулеза? -Ерсиния псевдотуберкулезис

12.Укажите этиотропный препарат для терапии гриппа? -Осельтамивир 75 мг 2 раза в день или Занамивир

13.Назовите специфическое осложнение брюшного тифа? -Кишечное кровотечение

14.Выберите заболевание при котором отмечаются схваткообразные боли в левой подвздошной области и тенезмы?

-Дизентерия

15.Укажите основную причину смертности при ботулизме? -Острая дыхательная недостаточность

16.Укажите клинические признаки, при которых показано обследование на

ВИЧ?

-Хроническая диарея более месяца, похудание рецидивирующие инфекции дыхательных путей, Кандидоз полости рта, рецидивирующие язвы рта

17.Укажите при каком заболевании отмечается диарейный синдром?

-Дизетерия,сальмонелллез все кишечные инфекции

18.Укажите возбудителя чумы?

-Yersinia pestis

19.Укажите, что необходимо исследовать для выявления ботулотоксина? -Кровь(реакция нейтрализации)

20.Укажите, в каком отделе желудочно-кишечного тракта происходит всасывание токсина при ботулизме?

-Тонкий кишечник

21.Укажите, посев чего позволит провести диагностику тифопаратифозных заболеваний?

-Крови

22.Укажите, при каком заболевании появляется симптом Падалки?

-Брюшной тиф

Понимание:

1.Женщина 25 лет была госпитализирована в инфекционное отделение с жалобами на повышение температуры тела до 39,6С, многократную рвоту, частый стул зеленого цвета. Из анамнеза пациентка заболела остро, симптомы начались через 4 часа после употребление сырых яиц. При осмотре живот мягкий, болезненный по ходу кишечника. Выставьте данной пациентке предварительный диагноз?

Сальмонеллез проверено

2.Мужчина 29 лет был доставлен по линии скорой помощи в стационар с жалобами на сильную слабость, трудности при глотании твёрдой пищи, поперхивание при глотании воды. Туман перед глазами. Со слов пациента заболел через 29 часов после употребления в пищу маринованных огурцов. При осмотре: мягкое небо свисает на корень языка, голос гнусавый. Движение глазных яблок ограничено. Мидриаз. Выставьте данному пациенту предварительный диагноз? Ботулизм проверено

3.В хирургическое отделение был переведён мужчина из инфекционной больницы

ссимптомами раздражения брюшины. Врач приёмного отделения выставил предварительный диагноз перфорация язвы тонкого кишечника, перитонит. При каком инфекционном заболевании может возникнуть данное осложнение? Брюшной тиф

проверено

4.Женщина 36 лет обратилась к врачу с жалобами на слабость, сильную боль в животе, многократную рвоту, однократный жидкий стул и повышение температуры тела выше 37,6°С. Из эпидемиологического анамнеза заболела остро, через час после того, как ела салат с майонезом. Срочно пациентке был промыт желудок, после чего её состояние улучшилось. Выставьте данной пациентке предварительный диагноз? ПТИ

( пищевая токсикоинфекция) проверено

5.В приёмное отделение по линии скорой помощи были доставлены 6 человек с одинаковыми симптомами. Пациенты предъявляли жалобы на повышение температуры тела, многократную рвоту, боли в животе. Со слов пациентов стул был зелёного цвета. Из анамнеза: вместе кушали в кафе блюдо из курицы. О каком наиболее вероятном диагнозе идёт речь? Сальмонеллез проверено

6.Мужчина 40 лет, поступил по линии скорой помощи в приёмное отделение с жалобами на боли в животе, лихорадку выше 38,50С. При осмотре: живот болезненный при пальпации слева, сигмовидная часть толстого кишечника спазмирована. Со слов

пациента при дефекации боли. Стул частый, но скудный. Выставьте данному пациенту предварительный диагноз? Шигеллез проверено

7.Мужчина 42 лет, обратился к врачу с жалобами на боли в животе схваткообразного характера, слабость, головную боль, частый стул со слизью и прожилками крови. Из анамнеза данные симптомы начались остро ночью. При осмотре: температура при поступлении 38°. живот при пальпации болезненный в левой подвздошной области. Лабораторно: в общем анализе крови: лейкоциты—14, лимфоциты— 26%, моноциты 13%, СОЭ—19 мм/час. Выставьте данному пациенту предварительный диагноз? Шигеллез проверено

8.В поликлинику обратился мужчина 49 лет с жалобами на боли в нижней части живота, больше в левой подвздошной области, схваткообразного характера. Частый скудный стул с примесью слизи и крови, ложные позывы на низ, боли при акте дефекации, повышение температуры до 39С. Из анамнеза: болеет 2-й день. Выставьте данному пациенту предварительный диагноз? Дизентерия (шигеллез) проверено

9.Мужчина 40 лет был доставлен в приёмное отделение инфекционного стационара с жалобами на обильный жидкий стул более 10 раз, многократную рвоту, жажду, снижение аппетита, слабость. Из анамнеза: заболел остро, симптомы начались ночью. Из эпидемиологического анамнеза: прилетел из Таиланда 2 дня назад. При осмотре состояния тяжёлое, пациент сопорозный, температура тела нормальная, АД 100\60, пульс 98 ударов в минуту. Ваш предположительный диагноз? Холера проверено

10.По скорой помощи в приёмное отделение был доставлен пациент жалобами на головные боли, повышение температуры тела до 39°С и боли в животе. При осмотре пациент заторможён, на вопросы отвечает медленно. Из анамнеза болеет 9 дней. На коже скудная розеолёзная сыпь. Печень увеличена на 2 см ниже края реберной дуги. При перкуссии живота – укорочение перкуторного звука справа. Выставьте данному пациенту предварительный диагноз? Брюшной тиф проверено

11.Женщина 46 лет обратилась в приёмное отделение с жалобами на повышение температуры тела до 38,7 С, обильную сыпь по всему телу, головные боли, слабость, боли в животе. Со слов пациентки болеет 6 день, сыпь появилась на 5 день болезни. Из эпидемиологического анамнеза живёт в селе работает на птицеферме. При осмотре: зев гиперемирован, на коже сыпь обильная розеолезно-папулезная по всему телу, живот мягкий, безболезненный. Выставьте данной пациентке предварительный диагноз? Паратиф проверено

12.Мужчина 50 лет был госпитализирован в инфекционное отделение с жалобами на повышение температуры до 38-39 °С, слабость, снижение аппетита. Со слов пациента болеет в течение 8 дней. При осмотре состояние средней тяжести, температура 38,4 °С, пациент вялый, адинамичный. Кожа бледная, сухая, на коже живота выявлена розеолезная единичная сыпь. В лёгких без патологии. Тоны сердца приглушены, АД – 90/60 мм рт. ст., Пульс – 68 уд/мин. Язык обложен густым серым налётом у корня, на боковых поверхностях отпечатки зубов. Живот вздут, при пальпации мягкий, безболезненный. Пальпируются печень и селезёнка. Поставьте данному пациенту предварительный диагноз? Брюшной тиф проверено

13.По скорой помощи в отделение инфекционного стационара был доставлен пациент с жалобами на осиплость голоса, снижение остроты зрения, сильную слабость

итрудности при дыхании. Из анамнеза: данные симптомы начались утром. Вчера ел домашнюю консервацию. При осмотре пациент в сознании, возбужден, гнусавит, одышка. Зев спокоен, живот мягкий и безболезненный. Зрачки расширены. Выставьте данному пациенту предварительный диагноз? Ботулизм проверено

14.Дежурный врач был вызван в отделение к пациенту, которому резко стало плохо. Из анамнеза – пациент лежит в инфекционном стационаре 4 дня с диагнозом брюшной тиф. Со слов медсестры у пациента на фоне лихорадки 39С резко снизилось артериальное давление, температура упала до 36С. Пациент стал бледным и вялым. При осмотре: у пациента сопорозное сознание. Кожа бледная. АД 80/60, ЧСС 110. Живот мягкий. Какое неотложное состояние развилось у данного больного?

Септический шок проверено

15.Мужчина 48 лет был доставлен в приёмное отделение по скорой помощи. Пациент п оступил с жалобами на головную боль, инверсию сна, вялость. Со слов родственников пациента болеет 6 дней. Ранее отмечал рвоту, боли в животе и многократный жидкий стул. При осмотре состояние пациента тяжёлое, пациент адинамичный, сознание нарушено, бредит. Живот при пальпации мягкий, безболезненный. На коже редкая единичная розеолезная сыпь. Врач выставил предварительный диагноз Сальмонеллёз. О каком клиническом варианте течения данного заболевания идет речь?

тифоподобном проверено

16.Мужчина 42-х лет обратился в приёмное отделение инфекционного стационара. С жалобами на лихорадку, рвоту, тошноту, головные боли, снижение артериального давления, розеолёзную сыпь на животе и конечностях. Из анамнеза

болеет 8 день. Врач заподозрил тифопаратифозные заболевания. При осмотре врач отметил укорочение перкуторного звука в подвздошной области справа. О каком симптоме идет речь?

симптом падалки проверено

17.Пациентка 25 лет обратилась к врачу ЦСМ с жалобами на боли и дискомфорт в нижней части живота, больше справа, жидкий стул со слизью и кровью, ложные позывы на стул, снижение аппетита. Со слов пациентки данные симптомы отмечаются в течении 2 месяцев. Из эпидемиологического анамнеза была 3 месяца назад в Индии. При осмотре живот мягкий, справа в проекции толстого кишечника болезненность. Лабораторно: в копрограмме большое количество стекловидной слизи с кровью. О каком заболевание можно думать?

амебиаз проверено

18.Мужчина 40 лет был в приёмное отделение больницы в сопорозном сознании. Со слов родных ранее пациент жаловался на частый водянистый стул, многократную рвоту. Заболел 1 день назад, после возвращения из Бангладеша. Объективно: сопорозный, температура тела нормальная, АД 100\60, пульс 100 ударов в минуту, кожная складка расправляется медленно, живот мягкий, безболезненный. Выставьте данному пациенту предварительный диагноз:

холера проверено

19.Мужчина 54 лет поступил в приемное отделение с жалобами на высокую температуру, головные боли, боли в животе, сыпь на теле. Со слов пациента данные симптомы отмечаются в течение 8 дней. При осмотре пациент беспокоен, язык обложен налетом серым, живот при пальпации болезненный в илеоцекальной области, на теле скудная мономорфная розеолезная сыпь, печень и селезенка увеличены. Выставьте данному пациенту предварительный диагноз?

Брюшной тиф проверено

20.Мужчина 28 лет, обратился к врачу с жалобами на повышение температуры до 38,8ᵒ С, спастические боли в левой половине живота, частый скудный стул до 15 раз

спрожилками крови. Объективно: кожа бледная, живот при пальпации мягкий, болезненный по ходу толстого кишечника слева, сигма спазмирована. Пульс 90 в минуту, АД 120/80 мм. рт. ст. Врач поставил предварительный диагноз дизентерия. Какая форма дизентерии у данного пациента?

гастроэнтероколитическая проверено

21.Женщина 66 лет обратилась в поликлинику с жалобами на общую слабость, озноб, повышение температуры до 390С, головную боль, боли внизу живота, больше слева, частый скудный стул до 10 раз в сутки со слизью и прожилками крови. Данные симптомы отмечает в течении дня. Из эпидемиологического анамнеза: ела клубнику с огорода несколько дней назад. Объективно: состояние средней тяжести. Живот мягкий, при пальпации болезненность в левой подвздошной области. Выставьте данной пациентке предварительный диагноз?

дизентирия проверено

22.Мужчина 35 лет, доставлен в хирургическое отделение в бессознательном состоянии с диагнозом кишечное кровотечение. Со слов родственников заболевание началось постепенно с появления общей слабости, головной боли, снижения аппетита, повышения температуры до 39,2 С, с вздутием живота и появлением сыпи на теле. При осмотре состояние сопорозное, АД -100/60. ЧСС 110. Кожа бледная с мраморным оттенком. На теле единичная розеолёзная сыпь. При каком заболевании развивается данное осложнение?

брюшной тиф проверено

23.Больной 35 лет, обратился к терапевту с жалобами на появление эпизодов удушья с затрудненным выдохом, кашель с трудно отделяемой мокротой. Приступы удушья возникают 2-3 раза в неделю чаще ночью и проходят самостоятельно. Считает себя больным в течении трех месяцев. Объективно: состояние удовлетворительное. При аускультации легких: сухие, рассеянные хрипы. Живот мягкий болезненный в подвздошной области. Печень, селезенка не увеличены. На рентгене легких – пятна Леффлера. Выставьте данному пациенту клинический диагноз?

Эозинофильная пневмония проверено

24.Девушка 20 лет, обратилась в ЦСМ с жалобами на слабость, повышенную утомляемость, зуд в промежности, который самопроизвольно проходит, но через две недели появляется вновь. Объективно: на кожных покровах следы расчесов, покрытые корочками. Лабораторно в общем анализе крови: эозинофилия. Поставьте диагноз данной больной?

Энтеробиоз проверено

25.Мужчина 48 лет в тяжёлом состоянии был доставлен в инфекционную больницу. С жалобами на лихорадку, мышечные боли, слабость, отёк век. Из анамнеза: 5 дней назад ел солёное свиное сало, которое купил на рынке. При осмотре больного, учитывая анамнез, врач попросил доставить в лабораторию

больницы остатки пищи (сало) для исследования. При микроскопии анализ показал наличие паразита. Какой паразит был обнаружен в сале?

свининой цепень проверено

26.Больная 40 лет, обратилась в ЦСМ по поводу постоянных, усиливающихся болей в животе, потерю веса. Последние три месяца жалобы на неоформленный стул, зуд промежности. Потерю веса. В общем анализе крови: Гемоглобин 89 г/л, эозинофилы – 7%. Выставьте данной пациентке предварительный диагноз?

Аскаридоз проверено

27.Мужчина 55 лет, обратился к врачу с жалобами: на понижение аппетита, тошноту, боли в животе, неустойчивый стул, выпадение волос, кожный зуд. При опросе выявлено, что в течение двух дней с калом отходят гельминты веретенообразной формы, беловато-желтого цвета, длиной около 30 см. Выставьте предварительный диагноз данному пациенту?

Аскаридоз проверено

28.Женщина 38 лет обратилась к врачу с жалобами на повышение температуры тела, тошноту, сыпь, кашель, одышку, боли в животе, жидкий стул, похудание. Дома содержат собаку. При осмотре: бледность кожных покровов, эритематозная сыпь, трофические изменения кожи, ногтей, волос, гепатомегалия. Ваш предварительный диагноз?

Токсокароз проверено

29.Мужчина 50 лет обратился к врачу с жалобами на быструю усталость и утомляемость, кожный зуд. Из анамнеза: был потребителем инъекционных наркотиков 15 лет назад. При осмотре: кожа бледная, склеры слегка иктеричны. Живот мягкий, безболезненный, печень увеличена на 2 см. При инструментальных обследованиях: на УЗИ печени выявлены диффузные изменения паренхимы печени. Лабораторно: антитела к HCV-IgG положительно.Какой вирусный гепатит у данного больного?

ВГС

30.Мужчина в возрасте 32 лет обратился в инфекционную больницу с жалобами на плохой аппетит, ноющиеmm боли в правом подреберье, потемнение мочи, желтушную окраску кожи и склер. Анамнез болезни: заболел 6 дней назад, когда повысилась температура до 38°С. Со слов пациента желтушность кожи и потемнение мочи заметил день назад. Эпидемиологический анамнез: живёт в студенческом общежитии, в комнате 4 человека. Объективно: общее состояние средней степени тяжести. Температура 37,5 °С. Кожные покровы и склеры умеренно желтушны, сыпи

нет. Живот не вздут, мягкий, безболезненный. Печень выступает на 2 см из-под края рёберной дуги, чувствительная при пальпации. Лабораторно – ИФА anti-HAV-IgM положительно. Выставьте данному пациенту клинический диагноз?

ВГА Острый вирусный гепатит А(анти-HAV IgM+), типичный, среднетяжелая форма, острое течение, неосложненный.

31.Девушка 20 лет, обратилась к врачу в связи с тем, что заметила желтушное окрашивание склер и кожи. Никаких других симптомов болезни не отмечает. Самочувствие хорошее. Из анамнеза: раньше гепатитом не болела. Никаких парентеральных вмешательств в течение полгода не было. Недавно болел 8 летний младший брат. При осмотре: отмечается легкая желтушность склер и кожи, печень у края реберной дуги, чувствительная при пальпации и перкуссии. Лабораторно: общий билирубин - 150 мкмоль/л, прямой билирубин - 100 мкмоль/л, АЛТ 400 ед., АСТ 450 ед., тимоловая проба -19 ед., ИФА – anti-HAV-IgМ положительный. Какой вид вирусного гепатита у данной больной?

ВГА

32.Мужчина 50 лет обратился в инфекционный стационар с жалобами на пожелтение склер, кожи, тёмную мочу, слабость, тошноту. Со слов пациента считает себя больным 15 дней. Все эти дни держалась высокая температура тела. болели крупные суставы, аппетит снизился. В последние 2 дня появилась тёмная моча, желтушность склер и кожи. Дома принимал парацетамол. Из анамнеза: 3 месяца назад получал курс внутривенных и внутримышечных уколов. При осмотре: состояние средней тяжести. Температура 36,8 °С. Склеры и кожа умеренно желтушны. ЧСС 52 уд/мин, АД – 110/60 мм рт. ст. Живот мягкий, слегка болезненный в правом подреберье. Печень выступает из-под края рёберной дуги на 3 см. Моча темно-коричневого цвета. ИФА –HbsAg положительный. Выставьте данному пациенту клинический диагноз?

ВГВ Острый вирусный гепатит В (HBsAg+), желтушная форма, средней степени тяжести, острое течение

33.Мужчина в возрасте 22 лет обратился к врачу с жалобами на слабость, повышенную температуру тела пожелтение кожи, кожный зуд. Из анамнеза: в течение 5 дней головная боль, температура 38°С, недомогание. На 7 день болезни появилась тёмная моча и окружающие заметили желтушность кожи и склер. Из эпидемиологического анамнеза: на месте учебы так же заболели несколько человек с подобными симптомами. При осмотре: иктеричность кожи и видимых слизистых, адинамия. Тоны сердца приглушены, пульс – 60 уд/мин, АД – 100/60 мм рт. ст. Живот чувствительный при пальпации в области правого подреберья. Печень увеличена на 2 см ниже края рёберной дуги. Лабораторно: anti-HAV-IgM положительный. Укажите какой вирусный гепатит у данного пациента?

ВГВ ВГА

34.Мужчина в возрасте 50 лет обратился в инфекционный стационар с жалобами на пожелтение склер, кожи, тёмную мочу, слабость, тошноту. Со слов

пациента считает себя больным 15 дней. Все эти дни держалась высокая температура тела. болели крупные суставы, аппетит снизился. В последние 2 дня появилась тёмная моча, желтушность склер и кожи. Дома принимал парацетамол. Из анамнеза: 3 месяца назад получал курс внутривенных и внутримышечных уколов. При осмотре: состояние средней тяжести. Температура 36,8 °С. Склеры и кожа умеренно желтушны. ЧСС 52 уд/мин, АД – 110/60 мм рт. ст. Живот мягкий, слегка болезненный в правом подреберье. Печень выступает из-под края рёберной дуги на 3 см. Моча темно-коричневого цвета. ИФА –HbsAg положительный. Укажите какой вид вирусного гепатита у данного больного?

ВГВ

35. По санавиации был вызван врач инфекционист для консультации в Чуйскую областную больницу к пациенту с диагнозом острый вирусный гепатит. Из анамнеза пациент находится в стационаре на лечении в течении 14 дней. В динамике состояние улучшалось, температура нормализовалась, и пациент готовился к выписке. Сегодня состояние пациента резко ухудшилось, температура вновь повысилась, желтушность усилилась. Печеночные тесты ухудшились. ИФА IgHDV положительный. Укажите какой вирусный гепатит у данного больного?

ВГВ ВГД

36.Мужчина 48 лет обратился к врачу с жалобами на желтушность кожи, склер, боли в суставах, зуд кожи, снижение аппетита. Из анамнеза: полгода назад имплантировал зубы. При осмотре: язык обложен, расчесы в области груди и конечностей, живот мягкий, печень +5,0 см ниже края реберной дуги, чувствительная при пальпации. Пульс 60/мин, АД-110/60 мм рт. ст. Лабораторно: лейкоциты- 4,0*109/л, СОЭ - 8 мм/час. Общий билирубин -170 мкмоль/л, прямой билирубин -100 мкмоль/л, тимоловая проба -3,5 ед., АЛТ 400 ед., АСТ 450 ед. ИФА – HbsAg положительно. Укажите какой вид вирусного гепатита у данного больного?

ВГС ВГВ

37.Мужчина 40 лет поступил по линии скорой помощи в отделение реанимации в бессознательном состоянии. Со слов родственников больного болеет 7 день, в течение которого была высокая лихорадка до 38-390C. Из эпидемиологического анамнеза: 2 месяца назад перенес полостную операцию по поводу перитонита. Получал внутривенную инфузию свежезамороженной плазмы во время операции. При осмотре: больной в тяжелом состоянии, без сознания. Кожа и склеры ярко желтушны. Печень +2,0 см ниже края реберной дуги. Температура тела 38,7С. пульс 90 в мин. Лабораторно: Лейкоциты - 7,2*109/л, СОЭ – 17 мм/час, билирубин общий - 300 мкмоль/л, Прямой билирубин - 250 мкмоль/л. HbsAg положительно. АЛТ -1500 ед., АСТ 2000 ед. Укажите какой вирусный гепатит у данного больного?

ВГС ВГВ

38.Больной 50 лет поступил по линии скорой помощи в отделение реанимации

вбессознательном состоянии. Со слов родственников больного болеет 7 день, в течение

которого была высокая лихорадка до 38-390C. Из эпидемиологического анамнеза: 2 месяца назад перенёс оперативное вмешательство по поводу перитонита. При осмотре: больной в тяжелом состоянии, без сознания. Кожа и склеры ярко желтушны. Печень +2,0 см ниже края реберной дуги. Температура тела 38,7С. пульс 90 в мин. Лабораторно: Лейкоциты - 7,2*109/л, СОЭ – 17 мм/час, билирубин общий - 300 мкмоль/л. Прямой билирубин - 220 мкмоль/л, АЛТ -1800 ед., АСТ 2000 ед. Протромбиновый индекс 55%. Какое осложнение развилось у данного пациента?

Перитонит Фульминантное течение ОВГВ (массивный некроз печени) Печеночная кома 1

39.Женщина 35 лет, доставлена в инфекционную больницу без сознания, с резко выраженной желтухой. Из анамнеза: 3 месяца назад больная была прооперирована по поводу аппендицита. Со слов родственников пациентки желтуху они заметили у нее вчера. В течение недели была лихорадка и боли в суставах. При осмотре: кожа ярко желтушна, пациентка без сознания. Ощущается «печеночный» запах изо рта. При перкуссии нижний край печени определяется на 2,0 см выше реберной дуги по среднеключичной линии справа. Лабораторно: Общий билирубин 260, прямой билирубин 180, АСТ -2200 Ед/л, АЛТ – 2500 Ед/л. Выставьте данной пациентке предварительный диагноз?

фульминантное течение ОВГВ. Печеночная кома 1

40.Больная 46 лет, доставлена в инфекционную больницу без сознания, с резко выраженной желтухой. Из анамнеза: 2 месяца назад больная перенесла холецистоэктомию. Со слов родственников пациентки желтуху они заметили у нее вчера. Перед потерей сознания у пациентки была рвота с кровью. В течение недели лихорадила до 38-39 С. При осмотре: кожа ярко желтушна, пациентка без сознания. Носовое кровотечение. При перкуссии нижний край печени определяется на 2,0 см выше реберной дуги по среднеключичной линии справа. Лабораторно: Протромбиновый индекс 55%. Какое состояние развилось у данной пациентки?

парентеральный гепатит Массивный некроз печени (фульминантное течение ОВГВ)

41.Мужчина 57 лет обратился в приемное отделение инфекционного стационара с жалобами на периодическую слабость, быструю утомляемость, нарушение сна, незначительную желтушность склер и кожный зуд. Со слов пациента данные симптомы отмечает в течение последних 12 месяцев. Из эпидемиологического анамнеза: в молодости употреблял инъекционные наркотики, но сейчас зависимости нет. При осмотре: склеры слегка желтушны. На коже следы расчесов на коже. Лабораторно – ПЦР на НСV качественный положительно. Укажите какой вид вирусного гепатита у данного больного?

ВГС (хронизация?)

42.Мужчина 40 лет был доставлен в инфекционную больницу в тяжёлом состоянии. Со слов родных: пациент болен в течение 2х дней. Дома получал лечение

парацетамолом эффекта не было. Дома пациент жаловался на сильную головную боль, многократную рвоту, не приносящую облегчение, температура тела повышалась до 39С. При осмотре пациент сопорозный, на вопросы отвечает плохо. Пациент заторможен. Менингеальные симптомы положительные. На нижних конечностях петехиальная геморрагическая сыпь, неправильной формы, при надавливании не исчезает. Какой вид менингита у данного больного?

БАКТЕРИАЛЬНЫЙ МЕНИНГИТ Комбинированные гематогенногенерализованные формы (менингит+менингококцемия)

43.Мужчина 27 лет, обратился в ЦСМ, с жалобами на повышение температуры тела, гнойное отделяемое из глаз, боли в горле, жидкий стул. Работает в школе учителем. Со слов пациента болеет второй день. При осмотре: инъекция сосудов склер конъюнктив, гнойное отделяемое, гиперемия зева, налетов нет, пальпируются периферические лимфоузлы, гепатоспленомегалия. Выставьте данному пациенту предварительный диагноз? Аденовирусная инфекция(Фарингоконъюнктивальная лихорадка,Фолликулярный конъюнктивит, диарея), средней степени тяжести, неосложненная.

44.Девушка 19 лет, обратилась к врачу инфекционисту с жалобами на повышение температуры тела до 39,5°C, головные боли, боли в мышцах, чувство саднения за грудиной, сухого болезненного кашля. При осмотре: инъекция сосудов склер, слизистая оболочка мягкого и твердого неба гиперемирована, зернистость задней стенки глотки. О каком диагнозе следует думать?

грипп, типичный, неосложненный

45.Женщина 30 лет поступила в приёмное отделение с жалобами на головные боли, температуру, рвоту, сыпь на теле. Со слов больной болеет в течение 1 дня, когда появились данные симптомы. Сыпь появилась с нижних конечностей. При объективном осмотре состояние тяжелое. Менингеальные симптомы положительные. На нижних конечностях сыпь.Выставьте данной пациентке предварительный диагноз?

ревматизм Менингококковая инфекция. Менингокоцемия+менингит

46.Мужчина 40 лет поступил в приёмное отделение инфекционной больницы,

сжалобами на сильные головные боли, сыпь на теле, многократную рвоту, высокую температуру. Данные симптомы наблюдались в течение 1 дня. Со слов больного на работе был контакт с коллегой с похожей симптоматикой. Объективно: состояние тяжелое, лихорадка 39,30С. Сыпь крупно звездчатая на конечностях, туловище, ригидность шейных мышц. О какой комбинированной форме менингококковой инфекции идет речь? Менингококцемия+менингит

47.В отделение был вызван врач инфекционист для консультации. У пациента, который получает лечение в стационаре вдруг появилась сыпь и повысилась

температура тела и началась рвота, не приносящая облегчения. Так же пациент жаловался на сильные головные боли. На момент осмотра пациент в оглушённом состоянии. Отвечает на вопросы медленно. Пациент лихорадит, повторная рвота, ригидность шейных мышц. Лабораторно: ликвор вытекал под давлением, цитоз 1000 клеток. Бакпосев ликвора высеян – n. meningitidis. Какая сыпь возникает при данном заболевании? Геморрагическая сыпь, неправильной звездчатой формы, с

некрозами в центре.

48.Мужчина 26 лет доставлен в приемное отделение инфекционного стационара с жалобами на повышение температуры в течение 2 дней. Сегодня появилась головная боль, тошнота, рвота. Объективно: ригидность шейных мышц. Лабораторно: ликвор вытекал под давлением, клетки единичные в поле зрения, белок 0,033г/л, сахар 2,2 ммоль /л, реакции Панди и Нонне-Апельта отрицательные. Ваш наиболее вероятный диагноз?

О:менингизм

49.Женщина в возрасте 33 лет, обратилась в поликлинику с жалобами на повышение температуры тела до 370 С, сухой кашель, осиплость голоса, першение в горле. Данные симптомы беспокоят 2 день. Объективно: состояние относительно удовлетворительное, при осмотре: зев гиперемирован, налетов на миндалинах нет. О каком наиболее вероятном диагнозе идёт речь?

О:Острый назофарингит(/Парагрипп

50.Мужчина 27 лет, доставлен по линии скорой помощи в инфекционную больницу с жалобами на, сильные головные боли, многократную рвоту, не приносящую облегчения. Повышение температуры до 39,6 С. Из анамнеза пациент контактировал с коллегой на работе, у которого были подобные симптомы, которые сопровождались геморрагической сыпью. Объективно: заторможен, лежит на боку, все менингеальные знаки резко положительны. Какой менингит у данного больного?

Менингококковый

51.Мужчина в возрасте 44 лет получает лечение в инфекционной больнице с диагнозом менингококковый менингит тяжёлой формы в течение 10 дней. Состояние больного на фоне проводимой антибактериальной терапии улучшилось, появился аппетит. Головные боли, тошнота и рвота прошли. Отметьте правильный критерий отмены антибиотикотерапии для данного пациента?

О:Критерии отмены антибиотиков (санация CМЖ, цитоз менее 100 клеток, 80% из них представлены лимфоцитами)

52.К женщине 35 лет вызвана бригада скорой помощи. Со слов родственников пациентка заболела остро 1 день назад, когда температура резко повысилась до 39,8С. Была рвота и сегодня появилась сыпь. Эпид. анамнез: работает в школе где были госпитализированы пациенты с подобной симптоматикой. Объективно: состояние тяжёлое, кожа бледная. На коже в области ягодиц, бедер, голеней обильная геморрагическая сыпь. Менингеальных и очаговых знаков нет. Выставьте данной пациентке предварительный диагноз:

О:Менингококцемия

53.Мужчина 40 лет обратился в инфекционное отделение с жалобами на повышение температуры тела до 39,7С, головной боли, заложенности носа, сыпь на теле. Объективно: кожные покровы бледные, на поверхности ягодиц, голеней, стоп имеется обильная сыпь неправильной звездчатой формы. Менингеальные симптомы отрицательные. АД 90/50 мм. рт. ст., ЧСС 110 в мин. Выставьте данному пациенту предварительный диагноз?

О:Менингококцемия (+назофарингит)

54.Женщина 80 лет, поступила в приемное отделение с жалобами на сильную головную боль, повышение температуры тела до 39°С. Больна 6-й день. При осмотре: возбуждена, лицо гиперемировано, склеры инъецированы, кровоизлияния в переходной складке конъюнктив. Розеолезно-петехиальная сыпь на коже. Пальпируется селезенка. Из анамнеза жизни: ранее в молодости перенесла сыпной тиф, вирусный гепатит. Выставьте данной пациентке клинический диагноз?

О:Болезнь Брилла-Цинссера

55.Мужчина 50 лет, обратился к врачу с жалобами на потрясающий озноб, головную боль, повышение температуры тела до 40° С. Со слов пациента заболел спустя 7 дней после возвращения из Гвинеи, где был в двухмесячной командировке. Личную химиопрофилактику хинином проводил нерегулярно. При осмотре: кожа с желтушным оттенком. инъецированность склер, потемнение мочи виде "черного пива”. В моче обнаружили гемосидерин и уробилин. Выставьте данному больному предварительный диагноз?

О:Малярия( тропическая , 3х дневная)

56.Мужчина 50 лет, обратился к врачу с жалобами на потрясающий озноб, головную боль, повышение температуры тела до 40° С. Со слов пациента заболел спустя 7 дней после возвращения из Гвинеи, где был в двухмесячной командировке. Личую химиопрофилактику хинином проводил нерегулярно. При осмотре: кожа с желтушным

оттенком. инъецированность склер, потемнение мочи виде "черного пива”. В моче обнаружили гемосидерин и уробилин. Чем осложнилось данное заболевание? . О:Гемоглобинурийная лихорадка

57.Женщина 39 лет обратилась к врачу инфекционисту с жалобами на озноб, сильную головную боль, резкую общую слабость. Заболела 3 дня тому назад, почувствовала озноб, который продолжался 2 часа, затем жар, головную боль, ломоту во всем теле. Температура 40,5С держалась в течение нескольких часов, затем понизилась до 36,2С с обильным потоотделением. На третий день вновь поднялась температура до 39,5С с ознобом. Ваш предварительный диагноз?

О:Трёхдневная малярия

58.Женщина, прибывшая из Индии, жалуется на пароксизмальные приступы озноба, обильное потоотделение, лихорадку. Эти приступы длятся 1 или 2 дня и снова появляются через каждые 36-48 часов. При исследовании окрашенных мазков крови выявлено наличие нескольких кольцевидных форм внутри эритроцитов. Назовите возбудителя данного заболевания?

О:P. Falciparum (тропическая)

59.Мужчина 39 лет поступил в инфекционный стационар с жалобами на повышение температуры до 39,5° С, которая держалась в течение 3 х дней, а затем критически снизилась до субфебрильных цифр. Снижение температуры сопровождалось обильным потоотделением. Через 12 часов отмечено повторное повышение температуры до 40°С, сопровождающееся ознобом, общей слабостью, головной болью, мышечными болями. Из эпиданамнеза возвратился из Бангладеша. При осмотре: вялый, кожные покровы бледные, склеры иктеричны. Какой вид желтухи развивается при данном заболевании?

О:Надпеченочная желтуха(гемолитическая)

60.60. Мужчина в возрасте 41 лет, обратился к врачу инфекционисту с жалобами на высокую температуру до 38,5С, головная боль, мышечные боли. Из анамнеза: работает зоопарке. При осмотре врач обнаружил гиперемию лица с инъекцией конъюнктивы и склер. Врач заподозрил орнитоз. Какой наиболее вероятный путь передачи для данного заболевания?

О:аэрогенный путь (воздушно-пылевой)

61.Женщина 37 лет, обратилась к врачу с жалобами на повышение температуры тела 38,3С, кашель, одышку. Из анамнеза: содержит голубей и домашних

птиц. Объективно: в легких жесткое дыхание, рассеянные сухие хрипы; в нижнем отделе доли правого легкого дыхание ослаблено, выслушиваются влажные мелкопузырчатые хрипы, гепатоспленомегалия. Рентгенологическое исследование органов грудной клетки: признаки интерстициальной пневмонии в нижних отделах правого лёгкого. Выставьте данной пациентке предварительный диагноз?

О:Орнитоз типичная(пневмоническая форма)

62.Мужчина 55 лет обратился в поликлинику с жалобами на слабость, недомогание, боль в коленных суставах. Из анамнеза: работает ветеринаром. Болеет около 4 месяцев, когда стал отмечать слабость, подъем температуры тела до 3800С, потливость и боли в коленных суставах. Выставьте данному пациенту предварительный диагноз?

О:Подострый бруцеллез. Локомоторная форма

63.Мужчина 47 лет был доставлен в приемное отделение инфекционной больницы с жалобами на повышение температуры тела до 390С, головных болей, бессонницы, зрительные и слуховые галлюцинации. Со слов родственников, заболел пять дней назад. Объективно: пациент возбужден, на сгибательных поверхностях конечностей и боковых поверхностях туловища обильная розеолезно-петехиальная сыпь, увеличение печени и селезенки. Выставьте данному пациенту предварительный диагноз?

О:Сыпной тиф период разгара

64.Мужчина 30 лет обратился в приемное отделение инфекционного стационара c жалобами на высокую температуру, головную боль, повторное носовое кровотечение, боли в икроножных мышцах. Со слов пациента данные симптомы длятся

втечении 4х дней. Из эпиданамнеза неделю назад ездил купаться на пруд. Объективно: состояние тяжелое, умеренная желтуха склер и кожи, гепатоспленомегалия, олигурия. Ваш предварительный диагноз?

О:Лептоспироз(б. Васильева Вейла, инф желтуха),тяжёлая форма

65.Мужчина 37 лет поступил в инфекционную больницу в тяжелом состоянии, температура 40С, сознание спутанное, психомоторное возбуждение, спастические гемипарезы конечностей. Анамнез болезни: заболел 2 недели назад, когда остро поднялась температура до 38,5С, беспокоили сильные головные боли, мышечные боли, тошнота и рвота. Затем температура нормализовалась, но через неделю опять повысилась температура и появились вышеперечисленные симптомы. В эпиданамнезе: 1 месяц назад был укус клеща. Какая форма клещевого энцефалита у данного больного:

О:Двухволновая молочная лихорадка(2хволновой менингоэнцефалит)

66.Женщина 58 лет поступила в приемное отделение с жалобами на повышение температуры тела до 38,7°С сухой кашель. Лечилась ампициллином– без эффекта. Со слов больной: «В семье все болеют. Внук госпитализирован в больницу с пневмонией, у дочери тоже высокая температура и кашель. Недавно внуку купили попугая». При осмотре: кожа чистая. В легких некоторое укорочение перкуторного звука в нижних отделах справа, мелкопузырчатые хрипы, гепатоспленомегалия.

О:Орнитоз типичная (пневмон-я) форма

67.Мужчина 38 лет, обратился к врачу по поводу жалоб на длительную лихорадку, увеличение печени, потерю веса, анемию. Из анамнеза работает в Танзании геологом, где часто были укусы москитами. Объективно: кожа бледная, на ногах трофические язвы, при пальпации безболезненные, печень и селезенка увеличена на 2 см. Лабораторно: при микроскопическом исследование пунктата костного мозга обнаружены внутри макрофагов паразиты с голубой цитоплазмой, красным ядром и кинетопластом. Ваш предположительный диагноз:

О:лейшманиоз висцеральная, кожная формы

68.Женщина 40 лет обратилась к врачу ЦСМ с жалобой на появление на язвы на коже руки. Из анамнеза выяснено, что язва появилась 7 месяцев назад, которая не заживает. При объективном осмотре на коже правой кисти имеется язва кратерообразной формы, а по краям отмечается толстый инфильтрат, язва при надавливании безболезненная. Эпидемиологический анамнез – пациентка в течение года работала в Индии переводчиком, где неоднократно отмечала укусы москитами. Выставьте данной пациентке предварительный диагноз?

О:Кожный лейшманиоз

69.Мужчина 70 лет, обратился в приемное отделение инфекционной больницы по поводу длительной лихорадки. В течение 6-ти дней наблюдалось повышение температуры до 37,8 - 38,6ºС и беспокоила головная боль. Со слов пациента 40 лет назад перенес сыпной тиф. При осмотре на боковых поверхностях грудной клетки обнаружены единичные розеолы. Дыхание везикулярное. Тоны сердца глухие. Пульс 100 в минуту. АД - 100\90 мм рт. ст. Язык не изменен. Печень на уровне реберной дуги.

Окаком наиболее вероятном диагнозе идет речь?

О:Болезнь Брилла.Период разгара

70. Мужчина в возрасте 54 лет, обратился в приемное отделение инфекционного стационара на 5-й день болезни с жалобами на высокую температуру,

резкую головную боль, бессонницу. Со слов пациента живет на теплотрассе и не имеет дома. Объективно: температура 39,5ºС. Пульс 130 в минуту, АД - 90\65 мм рт. ст. Возбужден, говорлив. Лицо гиперемировано. Склерит. На туловище обильная розеолезно-петехиальная сыпь. Положительный симптом Говорова-Годелье, печень и селезенка увеличены. Выставьте данному больному предварительный диагноз?

О:Сыпной тиф период разгара

71.Женщина 50 лет была доставлена в приемное отделение инфекционного стационара с жалобами на повышение температуры тела до 39-40ºС, головную боль, слабость. Из анамнеза симптомы начались 4 дня назад. Объективно: пациентка возбуждена, эйфорична. Гиперемия лица, на переходной складке конъюнктивы петехии, на коже груди, живота, конечностей обильные розеолезно-петехиальные высыпания. ЧСС 120. тремор языка. Печень и селезенка увеличены на 2 см ниже края реберной дуги. Выставьте данной пациентке предварительный диагноз?

О:Сыпной тиф

72.Мужчина в возрасте 33 лет, обратился к врачу с жалобами повышение температуры до 38-39 градусов, бессонницу, выраженные боли в спине и икроножных мышцах. Со слов пациента данные симптомы отмечаются в течение 3 дней. Объективно: состояние тяжелое. Гиперемия лица, конъюнктивит, склерит. Отмечается желтушность склер. Печень увеличена на 2 см ниже реберной дуги. На коже верхних конеччччностей после наложения манжеты появилась геморрагическая сыпь. Стал реже мочиться. Стул не изменен. Выставьте данному пациенту предварительный диагноз?

О:Лептоспироз, тяжелое течение.

73.Женщина 27 лет, была доставлена в приемное отделение инфекционного стационара с жалобами на повышение температуры тела, рвоту, пожелтение глаз и темную мочу. Из анамнеза болезни: болеет в течении 4-х дней. Эпиданамнез: контакт

вшколе с учениками у которых были данные симптомы ранее. Объективно: состояние среднетяжелое. Температура 37,6 °С. Кожа и склеры желтушные. Живот мягкий, печень выступает из-под края рёберной дуги на 3 см. Анализ крови: лимфоциты— 45 %, моноциты. — 9 %; СОЭ-22мм/ч. Лабораторно: АЛТ -1000 ЕД/л, АСТ -1200 ЕД/л, общий билирубин -75 мкмоль/л, прямой 50 мкмоль/л. В крови обнаружены: anti-HAV IgM положительно. Какой вирусный гепатит развился у данной пациентки?

О:Вирусный гепатит А

74.Мужчина 29 лет обратился к врачу поликлиники с жалобами на слабость, боли в суставах, потливость и снижение аппетита. Со слов пациента данные симптомы

продолжаются в течение последнего месяца. Из эпиданамнеза: работает на рынке мясником. Объективно: увеличение подчелюстных, подмышечных и паховых лимфатических узлов. Увеличение печени и селезенки. Лабораторно: реакция Райта - 1:800. Выставьте данному пациенту предварительный диагноз?

О:Острый бруцеллёз,септическая форма

75.Женщина 60 лет повторно обратилась к врачу поликлинику с жалобами на повышение температуры тела, гиперемию лица справа, отечность, боли и жжение в области очага воспаления на лице. Со слов пациентки ранее 1 месяц назад получала лечение по поводу данного воспаления в инфекционном стационаре. Объективно: в области правой щеки имеется разлитая гиперемия кожи с неправильными контурами. На месте гиперемии имеется отек ткани. При пальпации поражённого участка умеренно-выраженная болезненность. Выставьте данной пациентке диагноз?

О:Рожистое воспаление(эритематозная форма)

76.Женщина 27 лет, была доставлена в приемное отделение инфекционного стационара с жалобами на повышение температуры тела, рвоту, пожелтение глаз и темную мочу. Из анамнеза болезни: болеет в течении 4-х дней. Из эпиданамнеза: три месяца назад сделала себе татуировку. Объективно: состояние тяжелое. Температура 38,6 °С. Кожа и склеры желтушные. Живот мягкий, печень выступает из-под края рёберной дуги на 4 см. Анализ крови: лейкоциты — 6,5–10в/л; лимфоциты— 45 %, моноциты. — 9 %; СОЭ-22мм/ч. Лабораторно: АЛТ -1800 ЕД/л, АСТ -1500 ЕД/л, общий билирубин - 150 мкмоль/л, прямой 120 мкмоль/л. Какой вид гепатита развился у данной пациентки?

О:Острый гепатит В

77.Больная 55 лет, жалуется на головную боль, озноб, тошноту, головокружение, чувство жжения в области левой щеки. При осмотре в 1-й день болезни температура 39,5°С, на левой щеке пятно гиперемии с четкими границами по ходу естественных складок кожи, кожа в области пораженного участка лоснится, напряжена, горячая на ощупь. Наблюдается выраженный отек подкожной клетчатки щеки и нижнего века, болезненность при пальпации периферии эритемы. На 2-й день болезни на фоне эритемы появилась единичная небольшая везикула, наполненная прозрачным содержимым, пальпируются подчелюстные лимфатические узлы слева. Ранее подобного не наблюдалось. Какая форма рожистого воспаления развилась у данной пациентки? эритематозно-булезная форма

78.Женщина 40 лет была доставлена в тяжелом состоянии в стационар. Из анамнеза болезни: жалобы на частный жидкий, обильный, водянистый стул без

патологических примесей, который сначала имел каловый характер, затем приобрел вид «рисового отвара». Присоединилась обильная рвота. Из эпиданамнеза вернулась вчера из Индии. Объективно: температура тела 35,6 °С. Кожные покровы синюшны, холодные, сухие, тургор снижен. Наблюдались судороги мышц нижних конечностей. Тоны сердца глухие, ритмичные. Пульс — 135 уд/мин, АД — 60/30 мм рт. ст. Язык сухой, обложен коричневым налетом. Живот мягкий, безболезненный при пальпации. Выставьте данной пациентке предварительный диагноз?

О:Холера,тяжёлая степень

79.Больной 20 лет, обратился в ЦСМ с жалобами на сильную головную боль, боли в мышцах, чувство жара, повышение температуры тела до 39°С. Эпиданамнез: обмывал лицо и руки в яме с водой. При объективном обследовании: кожные покровы обычной окраски, язык густо обложен, гиперемия лица, слизистой ротоглотки. конъюнктивит, Менингеальных явлений нет. Мышцы икроножные болезненные при пальпации. Гепатоспленомегалия. ЧСС100 в минуту, АД – 110/70 мм. рт. ст. Диурез снижен. Лабораторно: в крови выделены leptospira. Ваш предварительный диагноз? Лептоспироз

80.Мужчина 40 лет обратился к врачу с жалобами на повышение температуры тела до 38,0°С, озноб, головную боль, слабость, кашицеобразный стул. Объективно: сознание сохранено, отвечает с неохотой, односложно. Кожа бледная, на передней брюшной стенке единичные бледно-розовые элементы сыпи. Язык увеличен, густо обложен толстым грязно-коричневым налетом, сухой, с отпечатками зубов. Пульс 64 уд. в мин. АД – 90/50 мм рт. ст. Тоны сердца приглушены. Дыхание жестковатое, без хрипов. Живот вздут, при пальпации мягкий, безболезненный. Положительный симптом Падалки. Гепатоспленомегалия. Кашицеобразный стул без примесей. Ваш предварительный диагноз? Брюшной тиф

81.Мужчина 27 лет обратился к врачу с жалобами на озноб, слабость, схваткообразные боли внизу живота. Жидкий стул до 20 раз в сутки с примесью слизи

икрови. Из эпиданамнеза часто есть в кафе, не всегда соблюдает правила гигиены. При объективном осмотре: язык влажный, обложен серо-грязным налетом, живот при пальпации мягкий, болезненный по ходу толстого кишечника, сигма уплотнена, спазмирована. Общий анализ крови: лейкоциты — 12,4*10/9л, эозинофилы— 0 %, палочкоядерные — 35 %, сегментоядерные. — 56 %, лимфоциты. — 19 %, моноциты.

— 2 %, СОЭ — 25 мм/ч. Копрограмма: эритроциты — 8, лейкоциты 5–8 в поле зрения, слизь +++. Ваш предварительный диагноз?

О:Дезинтерия

82.Женщина 50 лет обратилась в инфекционный стационар, с жалобами на чувство тяжести и тупые боли в эпигастрии, сухость во рту, тошнота, двукратная рвота, один раз кашицеобразный стул. Из эпиданамнеза: ела дома маринованные грибы домашнего приготовления. Объективно: при поступлении температура 37 °С. Состояние тяжелое. Больная вялая. Сознание ясное. Резкая сухость слизистых оболочек рта. Двусторонний птоз, мидриаз, отсутствие реакции зрачков на свет, нарушение конвергенции, речь гнусавая. Тоны сердца приглушены, ритмичные, брадикардия. Живот вздут, стул задержан. Ваш предварительный диагноз?

О:Ботулизм

83.Мужчина 48 лет поступил в инфекционный стационар на 6 день болезни с жалобами на повышение температуры тела до 40ᵒС, головную боль, мышечные боли, озноб и потливость. При сборе анамнеза врач установил, что повышение температуры отмечается через день в течение последней недели. Из эпиданамнеза: больной 2 недели назад вернулся из командировки из Индии. Ваш предварительный диагноз?

О:Лейшманиоз

Малярия p.knowlesi?

84.В реанимационное отделение инфекционной больницы поступила больная 23 года с жалобами на спутанность сознания, туман перед глазами, затруднённое дыхание. Эпидемиологический анамнез: накануне употребляла консервированные огурцы. На момент осмотра отмечается невнятная речь, нарушение глотания, осиплость голоса, мидриаз со снижением реакции на свет. Выставьте данной пациентке предварительный диагноз?

О:Ботулизм среднетяжелый, неосложненный

85.Мужчина 23 лет, поступил в больницу с жалобами на высокую температуру, озноб, слабость, желтушность склер. Болеет 10 день. Дома принимал ципрофлоксацин, но без улучшения. На 4-день болезни отмечался сильный озноб, температура повысилась до 40ºС, держалась 6 часов и снизилась с обильным потоотделением. Подобные приступы лихорадки повторялись несколько раз через каждые сутки. Из эпиданамнеза выяснено, что пациент выезжал в Пакистан на паломничество. При осмотре: лицо гиперемировано, склеры желтушны. Пульс 110 уд/мин, АД 90/60мм рт. ст. Печень и селезенка увеличены. Лабораторно: из крови выделен plasmodium falciparum. Какой вид малярии развился у данного пациента?

О:Тропическая малярия

86.Мужчина 34 лет, обратился к врачу с жалобами на боли в суставах, повышение температуры тела, повышенную потливость, слабость и снижение аппетита, на боли в паху. Со слов больного данные симптомы отмечает в течение 15 дней. Из эпидемиологического анамнеза: пациент работает продавцом коров и овец на базаре. Объективно: не лихорадит, зев спокоен, суставы не изменены, в легких жесткое дыхания, отечность и воспаление мошонки. Лабораторно: реакция Райта: 1-400. Выставьте данному пациенту клинический диагноз?

О:Острый бруцеллёз,септико-метастатическая форма

87.Женщина 50 лет, обратилась в стационар с жалобами на озноб, чувство тяжести в эпигастрии, рвоту, повышение температуры тела. Боли в животе приобрели разлитой характер, появился обильный водянистый стул с зеленоватым оттенком. Из эпидемиологического анамнеза: ел в пищу сырое куриное яйцо. При осмотре в приемном покое: состояние тяжелое, выраженная слабость. Кожные покровы бледные, цианоз губ, судорожное сведение икроножных мышц. Дыхание везикулярное. Пульс126/мин. АД-90/40 мм.рт.ст. Язык сухой, густо обложен коричневым налетом. Живот болезненный в эпигастрии и мезогастрии. Пальпируется печень на 1 см ниже реберной дуги. Лабораторно: высеян Sal. Enteritidis. Поставьте данной пациентке клинический диагноз? Сальмонеллез, гастроинтестинальная форма, гастроэнтеритический

вариант, тяжелое течение,осложненное ГВШ II ст. ПРОВЕРЕНО

88.Девушка 18 лет, поступила в стационар с жалобами желтушное окрашивание кожи, повышение температуры тела, рвоту, потемнение мочи. Со слов пациентки: ранее желтухой не болела. Никаких парентеральных вмешательств в течение 6 месяцев не было. Объективно: наблюдается желтушность склер и кожи, печень увеличена на 2-3см ниже края реберной дуги, чувствительная при пальпации. Общий билирубин крови - 120шш мкмоль/л, прямой билирубин - 100 мкмоль/л. аланинаминотрансфераза (АлАТ) - 400,0 ЕД/л, тимоловая проба -18 ед. ИФА – anti HAV IgM положительный. Поставьте данной пациентке клинический диагноз? Острый

вирусный гепатит А(анти-HAV IgM+), типичный, среднетяжелая форма, острое течение, неосложненный.

ПРОВЕРЕНО

89.Больной 40 лет, обратился к врачу с жалобами на желтушность кожи, склер, зуд кожи. Из анамнеза: в контакте с желтушными больными не был. Объективно: Аппетит снижен, язык обложен белым налетом, живот мягкий, печень +3 см, болезненная при пальпации. Пальпируется край селезенки и беспокоит тошнота. Температура нормальная. Пульс 66/мин., ритмичный, АД-110/60 мм.рт.ст., спит плохо из-за зуда кожи. Лабораторно: билирубин общий - 210 мкмоль/л. прямой билирубин -

130 мкмоль/л, тимоловая проба -3,5 ед., холестерин крови - 7.5 мкмоль/л, АлАТ - 450 ЕД/л., ИФА - HBsAg положительный. Какой вирусный гепатит развился у данного больного? Вирусный гепатит Б

ПРОВЕРЕНО

90.Мужчина 22 лет обратился к врачу-терапевту участковому на 2 день болезни с жалобами на сильную головную боль в лобной области, боли в глазных яблоках, мышцах и суставах, общую слабость, отсутствие аппетита, частый сухой кашель, заложенность носа и незначительные выделения из носа, чувство першения в горле. Анамнез заболевания: заболел 2 два дня назад. Из эпидемиологического анамнеза: за 2 дня до заболевания навещал друзей в общежитии, среди которых были лица с подобными симптомами. Поставьте данному пациенту клинический диагноз? -

Грипп (ринофаринготрахеит), типичная форма, средняя степень тяжести, период разгара

ПРОВЕРЕНО

91.Мужчина 40 лет, был доставлен в инфекционное отделение с жалобами на повышение температуры до 39°С, упорную головную боль, тошноту, рвоту, снижение аппетита. Болеет 4 дня, начало острое. Принимал антигриппин, без улучшения. Из эпиданамнеза: две недели назад был на Иссык-Куле, ходил на экскурсии по горной местности, снял с себя присосавшегося клеща. Объективно: на месте укуса клеща нет изменений. Кожа лица и шеи гиперемированы. Лицо одутловато. Зев гиперемирован. Пульс 120 уд/мин, АД 90/60 мм.рт.ст.. Печень и селезенка не увеличены. Отмечается выраженная ригидность затылочных мышц. Поставьте предварительный диагноз данному пациенту? - Клещевой вирусный энцефалит, менингеальная форма, средней

степени тяжести, острое течение

ПРОВЕРЕНО

Применение

1.В приёмное отделение инфекционного стационара по линии скорой помощи был доставлен мужчина с подозрением на холеру. Из эпидемиологического анамнеза: прилетел из Индии 1 день назад. При осмотре состояние тяжелое. Рвота многократная, стул жидкий без примесей. Назначьте лабораторное исследование для постановки клинического диагноза?

-бакпосев кала и рвоты на 1% щелочно-пептонной воде или среде Монсура проверено

2.Женщина 65 лет поступила в приемное отделение инфекционной больницы с жалобами на поперхивание при глотании воды, опущение век, сильную слабость, одышку. Со слов больной данные симптомы начались утром, в течении дня состояние ухудшилось. Из эпидемиологического анамнеза употребляла в пищу консервированные огурцы домашнего приготовления. Врач приемного отделения поставил предварительный диагноз: пищевой ботулизм. Назначьте данной пациентке антибактериальную терапию?

- левомицетин 0,5 г 4 раза в сутки 5 дней или цефтриаксон 5-7 дней

3.Мужчина в возрасте 33лет обратился в приемное отделение инфекционного стационара с жалобами на лихорадку, рвоту, тошноту, головные боли, снижения давления, сыпь на животе. Из анамнеза болеет 8 день. При осмотре: пациент в сознании, АД 90/60, ЧСС 60, на коже живота розеолёзная редкая сыпь. Врач выставил предварительный диагноз брюшной тиф. Назначьте данному пациенту антибактериальную терапию?

-ципрофлоксацин 0,5 г внутрь 2 р в сутки перорально до 10го дня нормальной температуры

4.Девушка 25 лет, обратилась в приемное инфекционное отделение с жалобами на боли в животе схваткообразного характера в левой части живота, головную боль, рвоту, частый жидкий стул со слизью и прожилками крови, При осмотре: температура 38,5, живот при пальпации мягкий, болезненный в левой подвздошной области. Анализ крови: лейкоциты—15. Назначьте антибактериальную терапию данной пациентке?

- ципрофлоксацин 0,5 г 2 р в сутки в теч 5дней

5.Женщина 47 лет поступила в приемное отделение инфекционной больницы с жалобами на поперхивание, трудности при глотании, опущение век, сильную слабость, одышку. Со слов больной данные симптомы начались утром, в течении дня состояние ухудшилось. Из эпидемиологического анамнеза употребляла в пищу консервированные огурцы домашнего приготовления. При осмотре в сознании, одышка, мидриаз. Врач выставил предварительный диагноз: пищевой ботулизм. Какую дозу противоботулинистической сыворотки необходимо назначить данной пациентке?

- 50 тыс МЕ(2 лечебные дозы по 25тыс МЕ)

6.Мужчина поступил в приёмное отделение инфекционного стационара на 9 день болезни с жалобами на сильную головную боль, общую слабость, плохой аппетит; постоянную высокую температуру тела. Сегодня почувствовал сильную давящую боль

вправой половине живота, Объективно кожные покровы бледные, сухие, горячие, язык утолщен, покрыт бурым налетом. Печень и селезенка увеличены. Отмечаются слабо

выраженные перитонеальные явления. Врачом был заподозрен Брюшной тиф. Какова будет дальнейшая тактика врача приемного отделения?

- госпитализировать

7.Мужчина в возрасте 32 лет обратился в приемное отделение с жалобами на боли

вживоте слева, температуру, многократный стул с кровью, ложные позывы в туалет. Со слов пациента данные симптомы наблюдаются в течение 2х дней. При осмотре – пациент в сознании, температура 37,5 0С. Живот при пальпации мягкий, сигмовидная часть толстого кишечника спазмирована. Какой антибиотик необходимо назначить данному пациенту?

- дизентерия: ципрофлоксацин 0,5 г 2 р в день

8.Женщина 68 лет обратилась в приёмное отделение инфекционного стационара с жалобами на общую слабость, озноб, повышение температуры до 39ᵒС, головную боль, боли в животе слева частый скудный стул до 10 раз в сутки с прожилками крови. Данные симптомы отмечает в течении одного дня. Объективно: состояние средней тяжести. Живот мягкий, при пальпации болезненность и спазмированность в левой подвздошной области. Назначьте данной пациентке антибиотикотерапию?

- ципрофлоксацин 0,5 г 2 р в день

9.Мужчина, 35 лет, обратился в поликлинику с жалобами на жидкий стул, нерезкие боли в животе, небольшое похудание. Началось через месяц после возвращения из командировки в Уганде. В течение 10 дней отмечалось повышение температуры до 37,1ᵒС, послабление стула. В последующие дни стул стал обильным до 5­6 раз в сутки. Кал имел вид «малинового желе». После присоединились боли в животе, усиливающиеся при дефекации. При осмотре отмечается небольшое увеличение печени. Врач поставил предварительный диагноз «Амебиаз». На какое исследование необходимо собрать кал?

- микроскопическое исследование кала для выявления вегетативных форм

(трофозоитов) и цист

10.Мужчина, 48 лет, поступил в приемное отделение инфекционного стационара на 2 день болезни с жалобами на жидкий стул, рвоту без предшествующей тошноты, слабость, вялость, судороги икроножных мышц. Из эпидемиологического анамнеза 2 дня назад прилетел из Индии. При общем осмотре больной беспокоен, видимые слизистые сухие, губы потрескавшиеся. Температура тела 36ᵒС. Вес пациента 60 кг. Кожа сухая, тургор снижен. ЧД=26 в мин. Пульс слабого наполнения, ЧСС=110 в мин., АД=60/40 мм рт. ст. Живот мягкий, безболезненный при пальпации. Стул до 10 раз в день в виде «рисового отвара». Диурез снижен. Рассчитайте объем жидкости для регидратационной терапии данного пациента?

6000мл\кг в течение 3 часов

11.Мужчина 46 лет, обратился в ЦСМ с жалобами на повышение температуры до 38,8ᵒ С, боли в животе слева, частый стул до 15 раз в сутки в виде «ректального плевка» с прожилками крови. Объективно: кожа бледная, живот при пальпации мягкий, болезненный по ходу толстого кишечника, сигма спазмирована. Пульс 90 в минуту, умеренного наполнения, АД 120/80 мм рт. ст. Мочеиспускание свободное. Какое исследование кала нужно произвести для постановки диагноза?

Копрограмма(Копроцитоскопия), бактериологический посев кала

12.Женщина 38 лет, обратилась к врачу поликлиники с жалобами на повышение температуры тела, болями в левой половине живота, многократным жидким стулом с прожилками крови. Объективно: кожа бледная, живот при пальпации мягкий,

сигма спазмирована.

 

АД 120/80 мм рт.ст

ЧСС 78 в

минуту,. Врач

выставил

предварительный

диагноз

Дизентерия.

Назначьте

данной

пациентке

 

 

 

антибиотикотерапию?

Ципрофлоксацин 500мг 2р\сут - 5дн

 

 

 

13.Мужчина 50 лет, обратился в инфекционную больницу с жалобами на повышение температуры, озноб, схваткообразные боли в животе, головную боль, боли во время акта дефекации, стул до 15 раз в день. При осмотре состояние средней тяжести, кожа бледная, сыпи нет. Пульс 90 в минуту. Живот мягкий, болезненный слева, сигмовидная кишка уплотнена. Мочеиспускание свободное. Стул в виде слизистогнойного комка с прожилками крови. Назначьте данному пациенту антибиотикотерапию?ю веса. Началось через месяц после возвращения из командировки в Ин

Ципрофлоксацин 500мг 2р\сут - 5дн

14.Мужчина в возрасте 48 лет, обратился в поликлинику с жалобами на жидкий стул, боли в животе, потердии. В течение последних 7 дней отмечалось повышение температуры до 37ᵒС. В последующие дни стул стал обильным до 10 раз в сутки. Кал имел вид «малинового желе», присоединились боли в животе, усиливающиеся при дефекации. При осмотре отмечается небольшое увеличение печени. Врач выставил предварительный диагноз Амебиаз. Назначьте лечение данному пациенту?

Метронидазол перорально 500-750 мг 3р в день в теч 7-10дн

Тинидазол 2г перорально 1р\д в течение 3дн

15. Женщина, 40 лет, обратилась в поликлинику с жалобами на жидкий стул, боли и вздутие живота, слабость, ухудшение аппетита. Страдает дисфункцией кишечника последние 2 года. Последние 6 месяцев самочувствие ухудшилось, стала терять массу тела и беспокоили тянущие боли внизу живота, тошнота, отрыжка. Стул

жидкий до 5 раз в сутки, иногда со слизью и кровью, с гнилостным запахом. При микроскопии кала обнаружены цисты и крупные простейшие, тела которых покрыты ресничками. Назначьте данной пациентке лечение?

Метронидазол 500 мг 3 р/сут( или мономицин150-250- тыс.ЕД4 р/день,или паромицин)

Диагноз:Балантидиаз

16. Больной 45 лет обратился в районную больницу с жалобами на повышение температуры, озноб, сильные боли в животе, частый жидкий стул с кровью и с гнилостным запахом. Из анамнеза выяснилось, что периодически в течение последних 6 месяцев отмечались подобные симптомы. Эпиданамнезе: проживает в сельской местности и занимается свиноводством. Врачом был выставлен диагноз Балантидиоз. Назначьте лечение данному больному?

метронидазол 500 мг 3 раза в сутки( или мономицин150-250- тыс.ЕД4 р/день 2мя курсами по 5 дней с интервалом в 5- 6 дней, или паромицин; или окситетрациклин

500мг 4 раза/день-10 дней)

17.Женщина в возрасте 53 лет поступила в приемное отделение инфекционной больницы с жалобами на двоение, сетку перед глазами, и снижение остроты зрения, рвоту, слабость. Со слов больной данные симптомы начались днем. Из эпидемиологического анамнеза употребляла в пищу маринованные огурцы домашнего консервирования. При осмотре: в сознании, состояние среднетяжёлое, дыхание не нарушено, ЧД 16, зрачки расширены. Врач выставил предварительный диагноз – пищевой ботулизм. Назначьте дозу противоботулинистической сыворотки в международных единицах данной пациентке?

1леч.доза – 50 тыс.МЕ в/м

18.Мужчина 40 лет в тяжелом состоянии был доставлен по линии скорой помощи в реанимационное отделение инфекционной больницы. Со слов родных дома употреблял маринованные грибы. При осмотре: состояние тяжелое, дышит тяжело, дыхание учащенное поверхностное, пациент возбужден, беспокоен, зрачки расширены, слизистые сухие. Врач выставил предварительный диагноз: Ботулизм. Назначил противоботулинистическую сыворотку. По какому методу необходимо вводить данную сыворотку?

По методу Безредко

19. Мужчина 40 лет в тяжелом состоянии был доставлен по линии скорой помощи в реанимационное отделение инфекционной больницы. Со слов родных дома употреблял маринованные грибы. При осмотре: состояние тяжелое, дышит тяжело, дыхание учащенное поверхностное, пациент возбужден, беспокоен, зрачки расширены, слизистые сухие. Врач выставил предварительный диагноз: ботулизм и назначил противоботулинистическую сыворотку. Назначьте дозу противоботулинистической сыворотки в международных единицах данному пациенту?

75тыс.МЕ

20.Мужчина 50 лет поступил в приемное отделение инфекционного стационара с жалобами на высокую температуру до 38,8, боли в животе, головные боли, сыпь на теле. Из анамнеза: данные симптомы отмечаются в течении 8 дней. При осмотре: пациент в сознании, дезориентирован, беспокоен, язык обложен налетом серым, по краям следы от зубов, живот при пальпации болезненный в илеоцекальной области, при перкуссии притупление в эпигастрии, на теле скудная мономорфная розеолезная сыпь, печень и селезенка увеличены. Лабораторно: реакция Видаля 1-200. Назначьте данному пациенту диетический стол, учитывая его диагноз?

Диагноз: Брюшной тиф

Стол № 13 А (гипоаллерг,на пару,протертое)

21. Мужчина 50 лет поступил в приемное отделение инфекционного стационара с жалобами на высокую температуру до 38,80С, боли в животе, головные боли, сыпь на теле. Из анамнеза: данные симптомы отмечаются в течении 8 дней. При осмотре: пациент в сознании, дезориентирован, беспокоен, язык обложен налетом серым, по краям следы от зубов, живот при пальпации болезненный в илеоцекальной области, при перкуссии притупление в эпигастрии, на теле скудная мономорфная розеолезная сыпь, печень и селезенка увеличены. Лабораторно: реакция Видаля 1-200. Назначьте данному пациенту стартовую антибактериальную терапию?

Диагноз: Брюшной тиф

Фторхинолоны: ципрофлоксацин 500 мг внутрь 2 р/сутки(или офлоксацин 400 мг) в течение 5-7 дней.

Азитромицин,цефтриаксон-резерв.

22. Мужчина 63 лет поступил в приемное отделение инфекционного стационара с жалобами на высокую температуру до 38,8С, боли в животе, головные

боли, сыпь на теле. Из анамнеза: данные симптомы отмечаются в течении 8 дней. При осмотре: пациент в сознании, дезориентирован, беспокоен, язык обложен налетом серым, по краям следы от зубов, живот при пальпации болезненный в илеоцекальной области, при перкуссии притупление в эпигастрии, на теле скудная мономорфная розеолезная сыпь, печень и селезенка увеличены. Лабораторно: реакция Видаля 1-200. Врач назначил строгий постельный режим из-за опасности развития кишечного кровотечения. У пациента аллергия на фторхинолоны. Назначьте данному пациенту антибактериальную терапию?

Цефтриаксон 1-2 г/сутки в/м или в/в(или азитромицин)

23. Мужчина 40 лет обратился в приемное отделение инфекционного стационара с жалобами на повышение температуры до 39°С, снижение аппетита, слабость. Со слов пациента болеет в течение 9 дней. При осмотре пациент в сознании, состояние средней тяжести, температура тела 38,7 °С. Кожа бледная, сухая, на коже живота выявлена редкая розеолезная сыпь. При пальпации живот мягкий безболезненный. Живот слегка вздут. Печень увеличена в размерах. Врач выставил заподозрил тифопаратифозные заболевания. Назначьте исследование для выявления бактерионосительства?

РНГА с Vi-АГ (Vi-агглютинация с Vi-эритроцитарным диагностикумом(титр

1:80,1:160));ИФА

24.Мужчина 40 лет был доставлен в приемный покой в тяжелом состоянии. Жалобы на боли в животе, рвоту, частый жидкий стул зеленого цвета, боли в суставах

иголовные боли, высокую температуру тела. Из анамнеза часто употребляет в пищу сырые домашние яйца. На момент осмотра состояние тяжелое. В сознании. Зев спокоен, кожа и слизистые сухие, живот мягкий, болезненный по ходу кишечника. Врач выставил предварительный диагноз: Сальмонеллез тяжелая форма. Назначьте данному пациенту антибиотикотерапию?

Ципрофлоксацин по 0.5 г 2 раза в сутки

25.Мужчина 40 лет был доставлен в приемный покой в тяжелом состоянии. Жалобы на боли в животе, рвоту, частый жидкий стул зеленого цвета, боли в суставах

иголовные боли, высокую температуру тела. Из анамнеза часто употребляет в пищу сырые домашние яйца. На момент осмотра состояние тяжелое. В сознании. Зев спокоен, кожа и слизистые сухие, живот мягкий, болезненный по ходу кишечника. Врач выставил предварительный диагноз: гастроэнтероколит возможно сальмонеллезной

этиологии. Назначьте лабораторное исследование для подтверждения данного диагноза?

Бактериологическое исследование испражнений, рвотных масс,промывных вод желудка

+Серодиагностика:РА, РНГА, ИФА с целью выявления антител

26. Мужчина 55 лет был доставлен в приемное отделение по линии скорой помощи. Пациент поступил с жалобами на головную боль, на инверсию сна, вялость, резкую слабость, бред и галлюцинации. Со слов родственников пациента дома болел 6 дней. Ранее отмечал рвоту, боли в животе и многократный жидкий стул. При осмотре состояние пациента тяжелое, пациент адинамичный сознание нарушено, бредит. Живот при пальпации мягкий, безболезненный. На коже редкая единичная розеолезная сыпь. Врач заподозрил сальмонеллёз. Учитывая тяжесть состояния назначьте данному пациенту антибиотикотерапию?

*ципрофлоксацин по 0.5 г 2 р/сутки(или офлоксацин 0.4-0.8 г/сутки) внутрь или парентерально

*или цефтриаксон до 4 г/сут в течение 5-7дней

27. Мужчина 40 лет, обратился к врачу с жалобами на повышение температуры до 38,8ᵒ С, появление спастических болей в левой половине живота, частый стул до 20 раз в сутки в виде «ректального плевка» с прожилками крови. Объективно: кожа бледная, язык влажный, обложен сероватым налетом, живот при пальпации мягкий, болезненный по ходу толстого кишечника, сигма спазмирована. Пульс 90 в минуту, умеренного наполнения, АД 120/80 мм рт. ст. Мочеиспускание свободное. Врач поставил предварительный диагноз дизентерия. На какое исследование необходимо отправить кал для подтверждения диагноза?

Бактериологическое исследование-бакпосев-исследование на Shigella(капрокультура)

28. Мужчина 40 лет по линии скорой помощи был доставлен в инфекционный стационар. При осмотре пациент ведёт себя не адекватно. На вопросы врача отвечает невпопад. Из анамнеза болеет хроническим вирусным гепатитом В+D в течении 5 лет. При физикальном осмотре: кожа ярко желтушна. Носовое кровотечение. ЧСС 110. Повторная рвота. Лабораторно: Общий билирубин 200 мкмоль/л, прямой 140 мкмоль/л. Какой лабораторный анализ необходимо назначить в первую очередь?

ПТИ(протромб.индекс)

29. В гепатологическое отделение инфекционного стационара вызван ночной дежурант к пациенту, который ведет себя неадекватно. На момент осмотра пациент эйфоричен, дезориентирован. На вопросы врача отвечает невпопад. При физикальном осмотре кожа ярко желтушна, слизистые иктеричны. На местах уколов гематомы. Живот мягкий, печень увеличена в размере. Болезненная при пальпации. ЧСС 110. АД 140/90. Протромбиновый индекс 40%. Дежурный врач перевел пациента в отделение реанимации с диагнозом острый вирусный гепатит, фульминантное течение. Назначьте данному пациенту противовирусную терапию?

Тенофовир, 300 мг, 1 раз в сутки, внутрь, ежедневно не менее 3х мес,

или Энтекавир, 0,5 мг, внутрь, ежедневно-не менее 3х мес

30.30.Женщина 44 года, обратилась к врачу с жалобами на потерю аппетита, выраженную слабость, боль в правом подреберье, повышение температуры тела и желтушность глаз. При опросе больной выяснилось, что цвет мочи стал темный. Для постановки клинического диагноза назначьте обследование?

Лаб.исслед. ОАК,Биох.(АЛТ,АСТ,Тимоловая проба) Серология (ИФА на маркеры вирусного гепатита,пцр)фракции билирубина

31.Девушка 20 лет, обратилась к врачу в связи с тем, что заметила желтушное окрашивание склер и кожи. Из анамнеза: раньше гепатитом не болела. Никаких парентеральных вмешательств в течение полгода не было. Недавно болел 10 летний младший брат. При осмотре: отмечается легкая желтушность склер и кожи, печень у края реберной дуги, чувствительная при пальпации и перкуссии. Общий билирубин крови - 140 мкмоль/л, прямой билирубин - 105 мкмоль/л, АЛТ 350 ед., АСТ 400 ед., тимоловая проба -19 ед. Врач выставил предварительный диагноз: острый вирусный гепатит. Назначьте данной пациентке терапию?

Базисная терапия: дезинтоксикация,режим, диета стол 5

32.Мужчина 55 лет обратился к врачу с жалобами на пожелтение склер, кожи, тёмную мочу, слабость, тошноту. Со слов пациента считает себя больным в течение 10 дней. Из анамнеза: 4 месяца назад была операция по протезированию коленного сустава. При осмотре: состояние средней тяжести. Температура 36,8 °С. Склеры и кожа умеренно желтушны. ЧСС 66 уд/мин, АД – 110/60 мм рт. ст. Живот мягкий, слегка болезненный в правом подреберье. Печень выступает из-под края рёберной дуги на 3 см. Моча темно-коричневого цвета. Врач выставил диагноз: острый вирусный гепатит. Назначьте терапию данному пациенту?

Базисная терапия как при ВГА

Тенофовир 300 мг 1 р в сут внутрь ежедневно не менее 3 х мес или

Энтекавир 0.5 мг ежедневно не менее 3 х мес

33. Мужчина 49 лет обратился к врачу инфекционисту с жалобами на слабость, желтушность кожи и слизистых, боли в суставах, боли в проекции печени. Со слов пациента страдает хроническим вирусным гепатитом В в течении 4 х лет. Получал лечение тенофовиром, но бросил лечение так как тяжело его переносил. При осмотре: кожа желтушна, склеры иктеричны. На коже следы расчесов. Живот мягкий, печень в пределах нормы. Назначьте данному пациенту противовирусную терапию?

34. Женщина 40 лет обратилась к врачу с жалобами на быструю утомляемость, расстройство пищеварения, тошноту. Со слов пациентки данные симптомы наблюдаются у нее последние 2 месяца. Из эпидемиологического анамнеза – три года назад имплантировала зубы. При осмотре: Живот мягкий, печень не увеличена. ИФА на HCV положительно. Врач выставил предварительный диагноз – хронический вирусный гепатит С. Какое лабораторное обследование позволит выбрать правильную схему лечения?

ПЦР-количественный , генотипирование

34.Больная 35 лет, доставлена в инфекционную больницу без сознания, с резко выраженной желтухой. Со слов родственников пациентки желтуху они заметили у неё два дня назад. Перед потерей сознания у пациентки была рвота с кровью. При осмотре: кожа ярко желтушна, пациентка без сознания. При перкуссии нижний край печени определяется на 2,0 см выше реберной дуги по среднеключичной линии справа. Из анамнеза: 3 месяца назад больная была прооперирована: по поводу аппендицита. Лабораторно: Протромбиновый индекс 55%. Какой вид внутривенной инфузии необходим данной больной?

переливание СЗП

35.Мужчина 50 лет обратился в врачу инфекционисту с жалобами на периодическую слабость, быструю утомляемость, нарушение сна, незначительную желтушность склер и кожный зуд. Со слов пациента данные симптомы отмечает в течение последних 12 месяцев. Из эпидемиологического анамнеза: в молодости употреблял инъекционные наркотики. При осмотре: склеры слегка желтушны. На коже следы расчесов на коже. Лабораторно – ПЦР на НСV качественный положительно. Тест APRI выше 3. Врач заподозрил цирроз печени. Назначьте инструментальный метод исследования для уточнения степени фиброза печени?

Динамическая эластография(фибросканирование)

36. Больной 49 лет обратился к врачу с жалобами на быструю утомляемость, нарушение сна, незначительную желтушность склер и кожный зуд. Со слов пациента данные симптомы отмечает в течении последних 10 месяцев. Из эпидемиологического анамнеза: переносил 10 лет полостную операцию на животе по поводу перитонита. При осмотре: склеры желтушны. Лабораторно – ПЦР на НСV качественный положительно. Лабораторно: Тромбоциты 75 тысяч, АЛТ 150 Ед/л, АСТ 170 Ед/л., общий билирубин 21 мкмоль/л, прямой билирубин 17 мкмоль/л. Врач заподозрил цирроз печени. Какой неинвазивный тест позволит определить риск развития цирроза печени?

APRI, Fib-4

37.Женщина 50 лет обратилась к врачу инфекционисту с жалобами на утомляемость, нарушение сна, кожный зуд. Со слов пациентки данные симптомы отмечает в течении последних 8 месяцев. Из эпидемиологического анамнеза: кесарево сечение 20 лет назад. При осмотре: склеры слегка желтушны. На коже следы расчесов на коже. На УЗИ печени отмечается увеличение ее эхогенности, диффузная неоднородность. Лабораторно: Тромбоциты 75 тысяч, АЛТ 120 Ед/л, АСТ 100 Ед/л., общий билирубин 22 мкмоль/л, прямой билирубин 17 мкмоль/л. ИФА HCV положительно. Назначьте данной пациентке противовирусную терапию?

Даклатасвир 60 мг/ Софосбувир 400 мг 12 нед

38.Женщина 47 лет обратилась к врачу с жалобами на желтушность кожи, тёмную мочу, слабость, тошноту. Из анамнеза: 6 месяцев назад перенесла холецистоэктомию. При осмотре: состояние средней тяжести. Температура 37°С. Кожа умеренно желтушна. АД – 120/60 мм рт. ст. Живот мягкий, болезненный в правом подреберье. Печень выступает из-под края рёберной дуги на 2,5 см. Лабораторно: общий билирубин 110 мкмоль/л, прямой 70 мкмоль/л. АСТ 540 ЕД/л., АЛТ 600 ЕД/л. Какой диетический стол необходимо назначить данной пациентке?

Стол 5

39.Мужчина 40 лет обратился к врачу инфекционисту с жалобами на желтушность кожи и склер, повышение температуры тела, тошноту и боли в правой части живота. Из анамнеза 2 месяца назад удалял зуб. При осмотре: состояние средней тяжести, кожа ярко желтушна, склеры иктеричны. Живот мягкий, печень увеличена в размере на 3 см. Назначьте лабораторный анализ для постановки клинического диагноза данному пациенту?

ИФАHbsAg,anti HB-core, HBeAg

АлТ, АсТ, тимоловая проба

40. Мужчина 38 лет обратился к врачу, с положительным ПЦР анализом на вирусный гепатит В. Из анамнеза 3 года была операция на колене. При опросе: в последние 4 месяца периодически бывают боли в правом подреберье, стал быстрее утомляться и периодически болят суставы. Лабораторно: ПЦР количественный на вирусный гепатит В – 1,5 млн копий в 1 мл. Назначьте данному пациенту противовирусную терапию?

Тенофовир 300 мг 1 р в сут внутрь ежедневно не менее 3 х мес или

Энтекавир 0.5 мг ежедневно не менее 3 х мес

41. Мужчина 37 лет обратился к врачу инфекционисту с положительным ИФА анализом на вирусный гепатит В. Со слов пациента недавно женился и планировал с женой ее беременность. У жены ПЦР на вирусный гепатит В отрицательный. При

опросе: в последние 5 месяцев боли в правом подреберье, периодически болят суставы. Назначьте профилактические меры для жены пациента?

Вакцинация против гепатита В

42.В инфекционную больницу был доставлен пациент в тяжелом состоянии. Со слов родных: пациент болен в течении 3х дней. Дома получал парацетамол эффекта не было. Дома пациент жаловался на сильную головную боль, многократную рвоту, не приносящую облегчение, температура тела повышалась до 39С. При осмотре пациент оглушенный, на вопросы отвечает плохо. Пациент заторможен. Менингеальные симптомы положительные. Врач выставил предварительный диагноз менингит. Для уточнения данного диагноза назначьте дополнительные исследования?

анализ СМЖ

43.Мужчина 44 года, обратился к врачу с жалобами на боль в глазных яблоках, головную боль, ломоту в теле, повышение температуры тела до 390С, сухой кашель, заложенность носа. Со слов пациента заболел вчера. Объективно: состояние тяжелое, склеры инъецированы, при осмотре ротоглотки: миндалины увеличены, зев гиперемирован. В легких везикулярное дыхание. Врач приемного отделения поставил диагноз – Грипп. Назначьте противовирусную терапию данному пациенту?

Осельтамивир по 75 мг 2 р в день или

Занамивир 2 ингаляции по 5 мг 2 р в день в теч 5 дней

44.К больной 35 лет вызвана бригада скорой помощи. Заболела остро, температура повысилась до 400С, озноб, однократная рвота, на следующий день заметила сыпь. Эпид. анамнез: работает в детском саду, где подобное заболевание отмечалось у ребенка. Объективно: состояние тяжелое, бледная. На коже в области ягодиц, бедер, голеней обильная геморрагическая сыпь. Менингеальных и очаговых знаков нет. Лабораторно: лейкоциты – 17,5 10^9/л, гемоглобин 120 г/л, тромбоциты 250 тыс., СОЭ-17. Для постановки клинического диагноза назначьте исследование?

бактериоскопия крови, бакпосев крови

45.Мужчина 30 лет, обратился к врачу с жалобами на повышение температуры тела до 39,60С, головную боль, артралгию, заложенность носа. Объективно: кожные покровы бледные, на поверхности ягодиц, голеней имеется обильная сыпь неправильной звездчатой формы, размерами от петехий крупных сливных очагов, диаметром от 3-6 см. Менингеальные симптомы отрицательные. АД 90/50 мм. рт. ст., глухость сердечных тонов, аритмия, ЧСС 110 в мин. Назначьте данному пациенту антибактериальную терапию?

Цефтриаксон, цефотаксим

(пенициллин)

46. Женщина 50 лет поступает по линии скорой помощи в инфекционную больницу с жалобами на повышение температуры тела до 390С-40С, головную боль, многократную рвоту, не приносящую облегчения. Из анамнеза: болеет 1

день. При осмотре: состояние тяжелое, гиперемия щек, сухие губы, гиперемия и зернистость зева, менингеальные знаки положительные. Врач выставил предварительный диагноз менингит. Была проведена спинномозговая пункция: белок 1,0 г/л, цитоз 1000 клеток. Бактериологический посев ликвора – высеян Neisseria meningitidis. Назначьте данному пациенту антибактериальную терапию?

Бензатин бензилпенициллин Цефтриаксон, цефотаксим

47. Больная 44 года, обратилась в приемное отделение инфекционного стационара с жалобами на головную боль, боль в глазных яблоках, ломоту в теле, повышение температуры тела до 390С, сухой кашель, заложенность носа. Заболела накануне вечером. Объективно: состояние тяжелое, склеры инъецированы, при осмотре ротоглотки: миндалины увеличены, зев гиперемирован. В легких везикулярное дыхание. Врач приемного отделения поставил диагноз – Грипп. Назначьте противовирусную терапию данной пациентке?

Осельтамивир по 75 мг 2 р в день или

Занамивир 2 ингаляции по 5 мг 2 р в день в теч 5 дней

48. Беременная женщина 30 лет, обратилась в приемное отделение инфекционного стационара с жалобами на головную боль, боль в глазных яблоках, ломоту в теле, повышение температуры тела до 390С, сухой кашель, заложенность носа. Со слов пациентки заболела день назад. Беременность 33 неделя. Объективно: состояние тяжелое, склеры инъецированы, при осмотре ротоглотки: миндалины увеличены, зев гиперемирован. В легких везикулярное дыхание. Врач приемного отделения поставил предварительный диагноз – Грипп. Назначьте этиологическую противовирусную терапию данной пациентке?

Осельтамивир по 75 мг 2 р в день или

Занамивир 2 ингаляции по 5 мг 2 р в день в теч 5 дней

49. Больной 19 лет, доставлен по скорой помощи в инфекционную больницу с жалобами на внезапное повышение температуры до 39,50С, сильные головные боли, многократную рвоту, не приносящую облегчения. Из анамнеза пациент проживает в общежитии, где были недавно госпитализированы подобные больные. Объективно: заторможен, лежит на боку, все менингеальные знаки резко положительны. Лабораторно ликвор – белок 1,1 г/л, цитоз без счета, представлен нейтрофилами. Бактериологический посев ликвора: высеян Neisseria meningitidis. Назначьте данному пациенту антибиотикотерапию?

Бензатин бензилпенициллин

Цефтриаксон, цефотаксим

50.Мужчина 55 лет обратился к врачу на 7 день болезни с жалобами на высокую температуру в течение 6 дней, головную боль, слабость, кашель. Сегодня почувствовал одышку. При объективном осмотре: Больной в сознании. Температура тела – 38,9 С, ЧД 26 в мин, SpO2 90, АД 110/70 мм. рт. ст., ЧСС 100 уд/мин,

аускультативно над легкими ослабление дыхания в нижних отделах. Врач выставил предварительный диагноз: внебольничная пневмония. Какой вид терапии на данный момент необходимо назначить пациенту?

Оксигенотерапия

51.Больной 66 лет обратился к врачу в приемное отделение инфекционного стационара на 6 день болезни с жалобами на высокую температуру в течение 5 дней, головную боль, слабость, кашель, одышку. При объективном осмотре: температура тела– 38,90С, ЧД 23 в мин, SpO2 91, А/Д 110/70, ЧСС 88, больной в сознании, аускультативно над легкими ослабление дыхания в нижних отделах. Лабораторно: ПЦР тест на COVID-19 положительный. Определите тактику врача?

Госпитализация, изоляция, RОГК, дезинтоксикация, оксигенотерапия

52.Женщина 61 год была доставлена в инфекционный стационар по линии скорой помощи с жалобами на сильную головную боль, слабость, повышение температуры тела, снижение аппетита, сухой кашель, потерю обоняния, жидкий стул. Из эпидемиологического анамнеза: муж болеет Коронавирусной инфекцией. Объективно: состояние тяжелое, отмечается одышка при физической нагрузке. Вес 50 кг. Лабораторно: фибриноген 600 г/л, D -димер 1,5 мг/л. Назначьте антикоагулянтную терапию данной пациентке?

эноксапарин 0,8 2 р в день каждые 12 часов п/к

53.Мужчина 70 лет был госпитализирован в стационар с диагнозом среднетяжелая форма COVID19 на 7 день болезни. С жалобами на слабость, повышение температуры тела, сухой кашель. В общем анализе крови лейкоцитоз с нейтрофильным сдвигом, СОЭ38 мм. рт. ст., Прокальцитонин- 1,0. Назначьте лечение данному пациенту?

АБТ незамедлительно

54.Девушка 29 лет обратилась к врачу ЦСМ с жалобами на повышение температуры тела до 37,50С, сухой кашель, першение в горле, осиплость голоса. Со слов пациентки данные симптомы беспокоят 3 день. Объективно: состояние относительно удовлетворительное, при осмотре: зев гиперемирован, налетов на миндалинах нет. Врач выставил предварительный диагноз ОРВИ – парагрипп. Какой вид терапии необходимо назначить данной пациентке?

симптоматическая терапия

55.Мужчина 48 лет, обратился в приемное отделение инфекционного стационара с жалобами на периорбитальные боли, сильную головную боль, ломоту в

теле и боли в мышцах. Повышение температуры тела до 390С, сухой кашель, заложенность носа. Со слов пациента болеет второй день. Объективно: состояние тяжелое, склеры инъецированы, при осмотре ротоглотки: миндалины увеличены, зев гиперемирован. В легких везикулярное дыхание. Врач выставил предварительный диагноз – Грипп. Для постановки клинического диагноза назначьте исследование?

Вирусологическое исследование носоглоточного отделяемого, ИФА, ПЦР

56.В военной части среди солдат произошла вспышка менингококковой инфекции, заболевшие были госпитализированы в инфекционный стационар. Врач инфекционист осмотрел часть солдат, которые не заболели, но были в контакте. Среди осмотренных был выявлен пациент с назофарингитом. У него был взят мазок из носоглотки. Назначьте солдату с назофарингитом профилактическую антибактериальную терапию?

Ципрофлоксацин 500 мг х 1 раза в сутки внутрь

57.Мужчина 48 лет обратился к врачу с жалобами на приступы лихорадки, которые сопровождаются периодами ремиссии, увеличение печени и ее болезненность. Из анамнеза установлено, что несколько дней назад он возвратился с Индии. При объективном обследовании больного увеличение печени и селезенки. Лабораторно: Hb88г/л. Для поставновки клинического диагноза назначьте лабораторное исследование?

Микроскопия окрашенной ,,толстой капли" крови

58.Женщина 34 года обратилась в приемное отделение инфекционного стационара с жалобами на озноб, сильную головную боль, резкую общую слабость. Заболела 3 дня тому назад, почувствовала озноб, который продолжался 2 часа, затем жар, головную боль, ломоту во всем теле. Температура 40,5С держалась в течение нескольких часов, затем понизилась до 36,2С с обильным потоотделением. На третий день вновь поднялась температура до 39,5С с ознобом. Для постановки клинического диагноза назначьте метод исследования?

Микроскопия окрашенной ,,толстой капли" крови

59.Женщина 34 года обратилась к врачу инфекционисту с жалобами на озноб, сильную головную боль, резкую общую слабость. Заболела 3 дня тому назад, почувствовала озноб, который продолжался 2 часа, затем жар, головную боль, ломоту во всем теле. Температура 40,5С держалась в течение нескольких часов, затем понизилась до 36,2С с обильным потоотделением. На третий день вновь поднялась температура до 39С с ознобом. Лабораторно: тонкий мазок – plasmodium vivax. Назначьте данной пациентке купирующую терапию?

делагил 0.5 г, повторяют через 6, 24, 48 часов

60.Женщина 34 года обратилась к врачу инфекционисту с жалобами на озноб, сильную головную боль, резкую общую слабость. Заболела 3 дня тому назад, почувствовала озноб, который продолжался 2 часа, затем жар, головную боль, ломоту во всем теле. Температура 40,5С держалась в течение нескольких часов, затем

понизилась до 36,2С с обильным потоотделением. На третий день вновь поднялась температура до 39С с ознобом. Назначьте лечение данной пациентке?

делагил 0.5 г, повторяют через 6, 24, 48 часов

61.Мужчина 35 лет обратился к врачу с жалобами на лихорадку в течение двух недель, головную боль, боли в глазных яблоках, озноб, кашель, боли в груди при дыхании. Из анамнеза: пациент приехал из Узбекистана, где работал на упаковке хлопка. Среди работников предприятия заболели еще несколько человек. Заболевание началось остро. При объективном исследовании картина бронхопневмонии и гепатолиенальный синдром. Врач выставил предварительный диагноз: Ку – лихорадка. Назначьте данному пациенту антибактериальную терапию?

доксициклина однократно 200 мг перорально, затем по 100 мг перорально два раза в день, пока у пациента не наступит улучшение и температура не нормализуется на протяжении не менее 5 дней, длительность терапии при этом должна составить не менее 7 дней

62.Женщина 34 года обратилась к врачу инфекционисту с жалобами на озноб, сильную головную боль, резкую общую слабость. Заболела 3 дня тому назад, почувствовала озноб, который продолжался 2 часа, затем жар, головную боль, ломоту во всем теле. Температура 40,5С держалась в течение нескольких часов, затем понизилась до 36,2С с обильным потоотделением. На третий день вновь поднялась температура до 39С с ознобом. Лабораторно: тонкий мазок – plasmodium vivax. Назначьте данной пациентке радикальную терапию?

Артесунат в/в капельно 1 доза 4 мг/кг сутки, далее 2е-7е сутки по 2 мг/кг сутки.

63.Мужчина 37 лет, обратился в инфекционный стационар с жалобами на повышение температуры тела 38,9С, кашель, одышку. Из анамнеза: содержит голубей. Объективно: в легких жесткое дыхание, рассеянные сухие хрипы; в нижнем отделе доли правого легкого дыхание ослаблено, выслушиваются влажные мелкопузырчатые хрипы, гепатоспленомегалия. Рентгенологическое исследование органов грудной клетки: признаки интерстициальной пневмонии в нижних отделах правого лёгкого. Для постановки клинического диагноза назначьте исследование?

РСК с орнитозным антигеном

64. Больной 37 лет, обратился в инфекционный стационар с жалобами на повышение температуры тела 38,9С, кашель, одышку. Из анамнеза: содержит голубей. Объективно: в легких жесткое дыхание, рассеянные сухие хрипы; в нижнем отделе доли правого легкого дыхание ослаблено, выслушиваются влажные мелкопузырчатые хрипы, гепатоспленомегалия. Рентгенологическое исследование органов грудной клетки: признаки интерстициальной пневмонии в нижних отделах правого лёгкого. Назначьте антибактериальную терапию для данного заболевания?

Тетрациклин по 0,3—0,5 г 4 раза в сутки до 4—7-го дня нормальной температуры, либо доксициклин.

65.Женщина 45 лет ообратилась к врачу с жалобами на повышение температуры тела, слабость, вялость, снижение аппетита. Повышенную потливость. Объективно: увеличены подмышечные лимфоузлы размером до 0,5 см в диаметре, умеренный общий гипергидроз, суставы внешне не изменены, сердце и легкие без патологии, печень и селезенка не увеличены. Врач поставил предварительный диагноз острый бруцеллез. Назначьте лабораторный анализ для постановки клинического диагноза?

Реакция агглютинации Райта

66.Мужчина 55 лет, обратился к врачу с жалобами на озноб, сильную головную боль в лобно-теменной области, боли в мышцах спины и конечностей. Из анамнеза заболел 3 дня назад. Из эпиданамнеза: несколько дней тому назад проводил уборку помещения, в котором гнездились дикие голуби. Объективно: над нижними долями легких укорочен перкуторный звук, выслушиваются сухие и влажные хрипы, печень и селезенка увеличена на 2 см. Врач выставил диагноз орнитоз. Назначьте данному больному антибактериальную терапию?

Тетрациклин по 0,3—0,5 г 4 раза в сутки до 4—7-го дня нормальной температуры, либо доксициклин.

67.Женщина 37 лет, обратилась к врачу инфекционисту с жалобами на периодическое повышение температуры тела, озноб, потливость, нарастающую слабость. Из эпидемиологического анамнеза: последние полгода жила с семьей в Пакистане. Объективно: состояние средней тяжести. Печень и селезенка увеличены. Врач приемного отделения выставил предварительный диагноз «Малярия». Для постановки клинического диагноза назначьте лабораторное исследование?

Микроскопия окрашенной ,,толстой капли" крови

68.Женщина 40 лет, поступила в инфекционный стационар с жалобами на приступообразные подъемы температуры через каждые 48 часов, озноб, потливость, нарастающую слабость. Из эпиданамнеза установлено, что последние полгода жила с семьей в Индии, отмечал укусы комарами. Объективно: состояние средней тяжести. Печень и селезенка увеличены. Назначьте лабораторное исследование для постановки клинического диагноза?

Микроскопия окрашенной ,,толстой капли" крови

69.Женщина 55 лет, обратилась в поликлинику с жалобами на боли в суставах, периодическую лихорадку, повышенную потливость, слабость и снижение аппетита. Со слов пациентки данные симптомы отмечает в течении 1 месяца. Из эпидемиологического анамнеза: пациентка живет в селе, содержит дома крупно и мелко рогатый скот. Объективно: состояние удовлетворительное, не лихорадит, зев спокоен, суставы не изменены, в легких жесткое дыхание. Для постановки клинического диагноза назначьте лабораторное исследование?

Кожно аллергическая проба Кацони

70.Мужчина 47 лет обратился к врачу инфекционисту с жалобами на повышение температуры тела и боли в суставах, потливость. Из анамнеза; болеет в течение 1 месяца. Из эпидемиологического анамнеза: работает мясником на рынке. Объективно: зев спокоен, суставы не изменены, не лихорадит. Врач приемного отделения выставил диагноз «Острый бруцеллез». Назначьте данному пациенту антибактериальную терапию?

Доксициклин 100 мг 2 р\д - 45 дней

71.Мужчина 40 лет был доставлен в приемное отделение инфекционного стационара с жалобами на повышение температуры тела до 39-40ºС, головную боль, слабость. Из анамнеза болеет 3 день. Объективно: пациент возбужден, лицо гиперемировано, на переходной складке конъюнктивы петехии, на коже груди, живота, конечностей обильные розеолезно-петехиальные высыпания. ЧСС 110. Отмечается тремор языка. Печень и селезенка увеличены. Назначьте данному пациенту антибактериальную терапию?

Брюшной тиф среднетяжелая степень. Цефтриаксон по 1г два раза в день внутримышечно. Плюс симптоматическая терапия.

72.Женщина 30 лет, обратилась к врачу семейной медицины с жалобами на повышение температуры тела до 38, потливость, периодические боли в суставах. Со слов пациентки болеет в течение 1 месяца. Пациентка находится на 22 неделе беременности. Работает в селе ветеринаром. Лабораторно: реакция Райта 1:200, реакция Хеддельсона резко положительная. Учитывая беременность назначьте данной пациентке антибиотикотерапию?

Рифампицин 300 мг\сут х 3 р/д per os – 45 дней

73.Женщина 43 года, обратилась к врачу с жалобами на повышение температуры тела, слабость, потливость. Из анамнеза: данные симптомы отмечаются в течении 1 месяца, со слов пациентки температуру 38,2С переносит легко. Работает на мясокомбинате. Объективно: увеличены все группы периферических лимфатических узлов, суставы внешне не изменены, но отмечается болезненность в тазобедренном и коленном суставах. Увеличены размеры печени и селезенки. Назначьте данной пациентке антибактериальную терапию?

Доксициклин 100 мг 2 р\д 45 дней + рифампицин 300 мг 2-3 р\д – 45 дней

74.Женщина 40 лет поступила на 4-й день болезни с жалобами на повышение температуры тела до 390С, боли в суставах, снижение аппетита, кашель, рвоту, сыпь по телу на шее, руках и ногах. При осмотре: гиперемия кожных покровов в виде «капюшона», «перчаток и носков». Сыпь мелкоточечная. Лимфоузлы увеличены, гепатоспленомегалия. Назначьте антибиотик для лечения данной пациентки?

Левомицетин по 0,5 г 4 раза в сутки в течение 10 14 дней.

75. Женщина 30 лет, поступила в инфекционный стационар на 3 день болезни. Пациентка жалуется на высокую температуру тела, сильные боли в икроножных

мышцах, головную боль, желтушность кожи и глаз. Из анамнеза: пациентка работает на рыболовной ферме. Объективно: состояние средней тяжести, лицо гиперемировано. гепатоспленомегалия. Для постановки клинического диагноза назначьте лабораторное исследование?

Бактериоскопическое и бактериологическое исследование крови, антитела в парных сыворотках, ИФА, ПЦР

76.Мужчина 44 года обратился к врачу инфекционисту с жалобами на множественные язвы на руках, которые долго не заживают. Из анамнеза: данные язвы не заживают в течении длительного времени, так же пациент отмечает, что часто болеет простудными заболеваниями. После лабораторного исследования врач диагностировал кожный лейшманиоз тяжелой формы. Какой антибиотик необходимо назначить данному пациенту?

Мономицин по 250 тыс. ЕД трижды в день, 10 млн ЕД на курс, паромомицин

77.Мужчина 46 лет был доставлен в приемное отделение инфекционного стационара на 4 день болезни. Состояние больного тяжёлое. Со слов родственников: пациент жаловался на высокую температуру, головную боль, повторное носовое кровотечение, боли в икроножных мышцах. Объективно: умеренная желтуха склер и кожи, гепатоспленомегалия, олигурия. Назначьте этиотропное лечение данному пациенту? Пенициллин 6-12млн

78.Женщина 48 лет обратилась к врачу ЦСМ с жалобами на ограничение движения в правом коленном суставе. Из анамнеза 3 года назад перенесла бруцеллез. При осмотре: температура тела в пределах нормы, аппетит не нарушен. Движения в

коленном суставе ограничены. Назначьте терапию данной пациентке? НПВС,

физиотерапия, лечебная физкультура, массаж

79.Мужчина 60 лет обратился к врачу инфекционисту с жалобами на озноб, головную боль, повышение температура тела до 380С. Из анамнеза 5 месяцев назад вернулся из командировки в Индии, где переболел трехдневной малярией и был пролечен. Объективно: живот мягкий, пальпируется увеличенная печень и селезенка. Выставлен диагноз: рецидив 3-дневной малярии. Назначьте препарат для профилактики рецидивов 3х дневной малярии? Примахин 15 мг 14 дней

80.Мужчина 28 лет был доставлен в приемное отделение инфекционного стационара с жалобами на лихорадку до 38-39С, анемию, головные боли, желтушность кожи и склер. Из анамнеза: прибыл в Бишкек из Кении 2 дня назад. При осмотре: состояние тяжелое, сознание сохранено, бледен. Пульс — 120/мин. АД — 90/50 мм.рт.ст. гепатоспленомегалия. Лабораторно: гемоглобин 85, общий билирубин 150 мкмоль/л, не прямой 120 мкмоль/л. В «толстой капле» крови обнаружены плазмодии. Врач приемного отделения выставил диагноз: тропическая малярия. На хинин у

пациента аллергия. Назначьте данному пациенту этиотропное лечение? Мефлохин

1,250 мг 1 день Артемизинин-комбинированные препараты: Артеметер 20 мг + Лумефантрин

120 мг ) по 4 таблетки 2 р в сут 3 дня, Артесунат (50мг)+ сульфадоксинпириметамина (250мг)

81. Мужчина 29 лет обратился к врачу с жалобами на повышение температуры тела до 40-41°С, озноб, желтушное окрашивание кожи и склер, боли в поясничной области, снижение выделения мочи. Из анамнеза: 5 дней назад прилетел из Индии. Болеет 3 день. Объективно: температура 39,9°С. Кожные покровы желтушные. Инъекция сосудов склер. Гепатоспленомегалия. Симптом XII ребра положительный. За сутки выделил 250 мл мочи. Лабораторно: гемоглобин - 80 г/л, общий билирубин 120 мкмоль/л, не прямой 90 мкмоль/л. Из крови выделен – plasmodium falciparum. Назначьте данному пациенту этиотропную терапию?

Хинина дигидрохлорид 30 % - 2 мл (600 мг) парентерально, затем

Хинин 650 мг 3 р в день 7 дней + доксициклин 100 мг в день 7 дней

82.Девушка 26 лет, прилетела из Анголы. Заболела остро с озноба и повышения температуры тела до 40-41°С, появилось желтушное окрашивание кожи и склер, присоединились боли в поясничной области, резко уменьшилось количество выделяемой мочи. При поступлении на 8-й день болезни: температура 39,9°С. Кожные покровы желтушные. Инъекция сосудов склер. Гепатоспленомегалия. Симптом XII ребра положительный. За сутки выделил 250 мл мочи. Назначьте лабораторное исследование для постановки диагноза данной пациентке?

Обнаружение возбудителя при микроскопии крови (тонкий мазок и толстая капля), креатинин сыворотки крови, узи почек

83.Женщина 47 лет была доставлена по линии скорой помощи в приемное отделение инфекционной больницы с жалобами на повышение температуры тела до 3940ºС, головные боли, слабость. Из анамнеза: женщина без определенного места жительства, живет в плохих санитарных условиях. Скорая помощь подобрала пациентку на улице. При осмотре: пациентка в сознании. На вопросы отвечает медленно, пациентка эйфорична. Лицо гиперемировано, на коже груди, живота, конечностей обильные розеолезно-петехиальные высыпания. ЧСС 100. Выраженный треморязыка. Печень увеличена на - 2 см. Назначьте данной пациентке этиотропную терапию? тетрациклин 300-400 мг 4 раза в сутки или доксициклин по 100 мг 2 раза

всутки, левомицетин по 500 мг 4 раза в сутки до 2-го дня нормальной температуры

84.Мужчина 60 лет поступил в инфекционный стационар с жалобами на головную боль, высокую температуру сыпь на теле. Объективно: больной слегка заторможен и эйфоричен. Кожа лица гиперемирована, конъюнктивы гиперемированы. Губы сухие. На мягком небе имеется энантема и точечное кровоизлияние у основания язычка. При попытке высунуть язык больной высовывает его толчками, отмечаются фибриллярные подергивания языка. На боковых поверхностях груди, сгибательных

поверхностей предплечий, на спине розеолезно-петехиальная сыпь. Для постановки клинического диагноза назначьте лабораторное исследование?

РНГА 1:1000, РСК 1:160. Выявление в ИФА антител класса IgM или IgG.

85.Женщина 53 года обратилась в приемное отделение инфекционного стационара с жалобами на озноб, головную боль, повышение температуры до 38ᵒС, покраснение, отечность и болезненность левой голени. Из анамнеза: 5 раз переболела рожей в течение последних 3х лет. Последний эпизод отмечался 1,5 месяца назад. Назначьте препарат для профилактики рецидива рожи?1,5 млн ЕД бициллина-5 в/м -

2-3 года

86.Женщина 53 года обратилась к врачу с жалобами на озноб, головную боль, повышение температуры до 38ᵒС, покраснение, отечность и болезненность нижней трети правой голени. Из анамнеза: 5 раз переболела рожей в течение последних 3 лет. Последний эпизод отмечался 4 месяца назад. Назначьте данной пациентке антибиотик для профилактики рожи и ее длительность? 1,5 млн ЕД бициллина-5 в/м -2-3 года

87.В реанимационное отделение в изолятор госпитализирован больной 25 лет

сдиагнозом столбняк. Объективно можно увидеть, как больной выгибается на постели в дугообразное положение, опираясь только пятками и затылком, все мышцы напряжены. На лице больного своеобразное выражение: морщинистый лоб, суженные глазные щели, растянутые губы, опущенные уголки рта. Назначьте дозу антитоксической противостолбнячной лошадиной сыворотки для лечения данного больного?100 000–150 000 МЕ

88.Женщина 48 лет, доставлена скорой помощью в приемное отделение с жалобами на повышение температуры тела до 39С, боли в мышцах, озноб, тошноту, также на болезненность, покраснение и отечность в области правой голени. При осмотре: в области правой голени в средней и нижней ее трети яркая гиперемия, резко ограниченная от здоровой кожи, края ее неровные, в виде зубцов. Паховые лимфоузлы справа увеличены до 1,5см, болезненны. Назначьте стартовую антибактериальную терапию данной пациентке?пенициллин (в/м по 4-6 млн ЕД/сут) 7-10 дней

89.Женщина 48 лет находится на стационарном лечении с диагнозом рожистое воспаление. На момент поступления отмечалось повышение температуры до 39С. На правой голени отмечались яркая эритема с четкими границами и неровными краями, выраженная отечность. На фоне эритемы отмечались буллы с геморрагическим содержимым. При пальпации местно отмечалась гипертермия и болезненность. На 4-й день пребывания в стационаре температура не снизилась, отек нарос, болезненность и

местная гипертермия сохраняются. Отмечаются участки некроза пораженного участка и здоровой кожи. Определите дальнейшую тактик у врача?

Госпитализация в хир отделение

90. Женщина 70 лет, была доставлена по линии скорой помощи, в стационар с жалобами на сильные боли в животе слева, высокую температуру, частый стул с кровью. Со слов пациентки данные симптомы наблюдаются в течение 1 дня. При осмотре – пациентка в сознании, температура 38,8 0С. Живот при пальпации мягкий, болезненный по ходу толстого кишечника, больше слева. Сигмовидная кишка спазмирована. Выберите антибиотик для лечения данной пациентки?

Ципрофлоксацин по 500 мг 2 раза в день в течение 5 дней.

91.Пожилой мужчина 72 лет поступил в тяжелом состоянии в отделение с жалобами на повышение температуры тела выше 38-39 в течение последних 7-8 дней. Также жалобы на боли в животе, падение давления. Из анамнеза: проживает в регионе неблагоприятном по тифопаратифозным инфекциям. При осмотре: пациент в сознании, но дезориентирован, живот при пальпации болезненный в илеоцекальной области, при перкуссии притупление в эпигастрии, на теле скудная мономорфная розеолезная сыпь, печень и селезенка увеличены, АД -90/60, ЧСС -60 уд. в мин. Назначьте данному пациенту антибактериальную терапию? ципрофлоксацин по 500 мг 2 раза в сутки

перорально до 10-го дня нормальной температуры. При непереносимости ципрофлоксацина или отсутствии эффекта (сохранение лихорадки более 4 дней): цефтриаксон 1-2 г в сутки в/м, в/в до 10-го дня.

92.Женщина 48 лет была доставлена скорой помощью приемное отделение инфекционного стационара с жалобами на затруднение дыхания. Пациентка с трудом делает вдох, и принимает вынужденное положение сидя. Из анамнеза: врач скорой помощи сообщил, что пациентка дома принимала в пищу консервированные баклажаны. Объективно: одышка выражена, с участием вспомогательной мускулатуры, ортопноэ. Пациентка не может глотать, голос осиплый. Полуптоз на обоих глаз. ЧД 26. Врач приемного отделения госпитализировал больную в реанимационное отделение. Какой вид неотложной помощи необходимо оказать пациентке? Зондовое промывание желудка,

небольшими порциями жидкости. Специфическая терапия ПБС

93.Мужчина 40 лет обратился в приемное отделение инфекционного стационара с жалобами на повышение температуры до 39°С, снижение аппетита, слабость. Со слов пациента болеет в течение 9 дней. При осмотре пациент в сознании, состояние средней тяжести, температура тела 38,7 °С. Кожа бледная, сухая, на коже живота выявлена редкая розеолезная сыпь. При пальпации живот мягкий безболезненный. Живот слегка вздут. Печень увеличена в размерах. Врач заподозрил тифопаратифозные заболевания.

Выберите исследование, что бы выявить бактерионосительство? реакция Vi-

гемагглютинации,(диагностический титр 1 : 80, 1 : 160).

Проверено

94.Мужчина неизвестного возраста в тяжелом состоянии был доставлен в реанимационное отделение инфекционного стационара по скорой помощи. Пациент заторможен, на вопросы отвечает с опозданием, периодически рвота. При осмотре: сознание сопорозное, менингеальные симптомы выражены, на нижних конечностях обильная геморрагическая сыпь звездчатой формы, температура тела 39С. Выберите исследование для постановки диагноза? 1. Спиномозговая пункция.

2.Бактериологический метод, Бактериоскопичейский метод (ликвор, кровь) (100% поставит) но для этого по-любому придется делать пункцию, поэтому наверно пункция.

Проверено

95.Мужчина неизвестного возраста в тяжелом состоянии был доставлен в реанимационное отделение инфекционного стационара по скорой помощи. Пациент заторможен, на вопросы отвечает с опозданием, периодически рвота. При осмотре: сознание сопорозное, менингеальные симптомы выражены, на нижних конечностях обильная геморрагическая сыпь неправильной звездчатой формы, температура тела 39С. Назначьте антибактериальную терапию данному больному? пенициллин в суточной

дозе для взрослых 20-30 млн. Ед. в 6 введений в/м; цефтриаксон в суточной дозе до 2-4 г, в 1-2 введения в/в; цефотаксим до 12 г в сутки, в 3-4 введения, в/в;

меропенем до 6 г в сутки, в 3 введения, в/в.

(насколько помню пенициллины препараты выбора)

Проверено

96.Больная 38 лет, обратилась в приемное отделение инфекционного стационара с жалобами на головную боль, боль в глазных яблоках, ломоту в теле, повышение температуры тела до 39С, сухой кашель, заложенность носа. Заболела накануне вечером. Объективно: состояние тяжелое, склеры инъецированы, при осмотре ротоглотки: миндалины увеличены, зев гиперемирован. В легких везикулярное дыхание. Лабораторно: ПЦР мазок из носоглотки – H1-N5 – положительный. Назначьте противовирусную терапию для данной пациентки? H1N5 это грипп А.птичий грипп. (лекция инфекц болезни 5 курс)

осельтамивир по 75 мг 2 раза в день

занамивир: 2 ингаляции по 5 мг 2 раза в день в течение 5 дней (к амантадин и ремантадину устойчивы)

Проверено

97.Мужчина 40 лет, был доставлен в приемное отделение в тяжелом состоянии с жалобами на головную боль, сильную слабость, боли в суставах и теле, температуру тела до 39С, одышку. Со слов пациента болеет 3 день. Объективно: состояние тяжелое, одышка выражена, с участием вспомогательной мускулатуры. SPO2-89%, ЧД-23. В легких в нижних отделах ослабление дыхания. Какова тактика врача приемного отделения?

Экстренная госпитализация в инфекционное отделение

Проверено

98.Беременная женщина 30 лет, обратилась в приемное отделение инфекционного стационара с жалобами на головную боль, боль в глазных яблоках, ломоту в теле, повышение температуры тела до 39С, сухой кашель, заложенность носа. Со слов пациентки заболела день назад. Беременность 33 неделя. Объективно: состояние тяжелое, склеры инъецированы, при осмотре ротоглотки: миндалины увеличены, зев гиперемирован. Выберите правильный противовирусный препарат для данной пациентки?

Осельтамивир

(римантадин и амантадин противопоказаны)

Проверено

99.Больной 19 лет, доставлен по скорой помощи в инфекционную больницу с жалобами на внезапное повышение температуры до 39,5С, сильные головные боли, многократную рвоту, не приносящую облегчения. Из анамнеза пациент проживает в общежитии, где были недавно госпитализированы подобные больные. Объективно: заторможен, лежит на боку, все менингеальные знаки резко положительны. Лабораторно ликвор – белок 1,1 г/л, цитоз без счета, представлен нейтрофилами. Бактериологический посев ликвора: высеян Neisseria meningitidis. Назначьте антибиотикотерапию данному пациенту?

пенициллин в суточной дозе для взрослых 20-30 млн. Ед. в 6 введений в/м; цефтриаксон в суточной дозе до 2-4 г, в 1-2 введения в/в;

цефотаксим до 12 г в сутки, в 3-4 введения, в/в;

меропенем до 6 г в сутки, в 3 введения, в/в.

(насколько помню пенициллины препараты выбора)

Проверено

100. Женщина 40 лет обратилась в приемное отделение инфекционного стационара с жалобами на тошноту, рвоту, потерю аппетита, повышенную

нервную возбудимость, раздражительность, неустойчивый стул, похудание, боли

вживоте. Со слов пациентки данные симптомы беспокоят в течение шести месяцев. Отмечала самостоятельное выползание члеников из ануса. Лабораторно:

вобщем анализе крови анемия, относительный лимфоцитоз ускоренная СОЭ. Врач заподозрил тениаринхоз. Выберите препарат, применяемый при данном заболевании?

празиквантель 10-15 мг/кг/сут во время еды 1 день

никлозамед 2 гр/сут 1 раз на ночь, разжевывая и запивая небольшим количеством воды за 15 мин

Проверено

101. Женщина 60 лет обратилась в приемное отделение к врачу с жалобами на слабость, потерю аппетита, боли в животе, потемнение мочи. На момент осмотра склеры иктеричны, желтушность кожных покровов, размеры печени по Курлову 10-9-7 см, болезненность при пальпации. Лабораторно: билирубин общий 85 мкмоль/л, билирубин прямой 59 мкмоль/л, АЛТ 400 ЕД. Предварительный диагноз вирусный гепатит В. Назначьте данной пациентке исследование для постановки клинического диагноза?

Серологические показатели ВГВ (ИФА):

анти-HBsAg

анти-HВcAg

анти-HBeAg

ПЦР (ДНК ВГВ) качественный и количественный анализ

Проверено

102.Мужчина 70 лет обратился к врачу инфекционисту с жалобами на повышение температуры тела, сыпь на теле. Из анамнеза болезни: болеет в течении 5 дней. Эпиданамнез: со слов пациента 50 лет назад перенес

103.тиф. При осмотре: пациент в сознании, адекватен, на вопросы отвечает правильно. Сыпь на коже – розеолезно-петехиальная. В анализе крови: лейкопения, ускоренное СОЭ. Назначьте лабораторное исследование для постановки диагноза?

Серологические реакции (обнаружение антител) РНГА 1:1000, РСК 1:160

ИФА: Ig M и G Проверено

104. Мужчина 70 лет обратился к врачу инфекционисту с жалобами на повышение температуры тела, сыпь на теле. Из анамнеза болезни: болеет в течении 5 дней. Эпиданамнез: со слов пациента 50 лет назад перенес тиф. При осмотре: пациент в сознании, адекватен, на вопросы отвечает правильно. Сыпь на коже – розеолезно-

петехиальная. В анализе крови: лейкопения, ускоренное СОЭ. Назначьте данному пациенту антибактериальную терапию?

тетрациклин внутрь по 300-400 мг 4 раза в сутки доксициклин по 100 мг 2 раза в сутки левомицетин по 500 мг 4 раза в сутки

Проверено

105. Женщина 60 лет повторно обратилась к врачу поликлинику с жалобами на повышение температуры тела, гиперемию лица справа, отечность, боли и жжение в области очага воспаления на лице. Со слов пациентки ранее 1 месяц назад получала лечение по поводу данного воспаления в инфекционном стационаре. Объективно: в области правой щеки имеется разлитая гиперемия кожи с неправильными контурами. На месте гиперемии имеется отек ткани. При пальпации поражённого участка умеренно-выраженная болезненность. Назначьте лечение данной пациентке?

последовательная двухкурсовая антибиотикотерапия:

вслед за курсом ß-лактамных (пенициллина в/м по 4-6 млн ЕД/сут, цефтриаксон, цефотаксим по 1 г 1-2 раза в сутки в\м) после 2-3-дневного перерыва назначают линкомицин по 0,6 г 3 раза в день в/м курсом 7 сут. (рецидивирующая форма), а так пенициллины, цефалоспорины.

Проверено

106.Женщина 60 лет повторно обратилась к врачу поликлиники с жалобами на повышение температуры тела, гиперемию лица справа, отечность, боли и жжение в области очага воспаления на лице. Со слов пациентки ранее 1 месяц назад получала лечение по поводу данного воспаления в инфекционном стационаре. Объективно: в области правой щеки имеется разлитая гиперемия кожи с неправильными контурами. На месте гиперемии имеется отек ткани. При пальпации поражённого участка умеренно-выраженная болезненность. Назначьте данной пациентке препарат для профилактики рецидива?

Бициллинопрофилактика ежемесячно по 1,5 млн ЕД бициллина-5 в/м в течение 3- 4 мес (в сезон) или круглогодично

107.Женщина 27 лет, была доставлена в приемное отделение инфекционного стационара с жалобами на повышение температуры тела, рвоту, пожелтение глаз и темную мочу. Из анамнеза болезни: болеет в течении 4-х дней. Объективно: состояние тяжелое. Температура 38,6 °С. Кожа и склеры желтушные. Живот мягкий, печень выступает из-под края рёберной дуги на 4 см. Анализ крови: лейкоциты — 6,5–10в/л; лимфоциты— 45 %, моноциты. — 9 %; СОЭ-22мм/ч. Лабораторно: АЛТ -1800 ЕД/л, АСТ -1500 ЕД/л, общий билирубин -150 мкмоль/л, прямой 120 мкмоль/л. Какой вид терапии необходим данной пациентке?

Базисная терапия: режим полупостельный или постельный. Диета-стол No 5 по Певзнеру.

Дезинтоксикационная терапия; жирорастворимые витамины А и Е, энтеросорбенты, препараты урсодезоксихолевой кислоты по 10–15 мг/кг в сутки

108. Женщина 27 лет, была доставлена в приемное отделение инфекционного стационара с жалобами на повышение температуры тела, рвоту, пожелтение глаз и темную мочу. Из анамнеза болезни: болеет в течении 4-х дней. Объективно: состояние тяжелое. Температура 38,6 °С. Кожа и склеры желтушные. Живот мягкий, печень выступает из-под края рёберной дуги на 4 см. Анализ крови: лейкоциты — 6,5х109/л; лимфоциты— 45 %, моноциты. — 9 %; СОЭ-22мм/ч. Лабораторно: АЛТ -1800 ЕД/л, АСТ -1500 ЕД/л, общий билирубин -150 мкмоль/л, прямой 120 мкмоль/л., Назначьте исследование для постановки клинического диагноза?

Серологические (ИФА) определение антигенов и антител к вирусам гепатитов А,

B, D, C

анти- HAV-IgM; антиHAV- IgG

анти-HBsAg; анти – HВcAg

анти-HDV IgM или IgG

анти-HСV IgM, анти-HСV IgG

109.Женщина 65 лет была доставлена в приемное отделение инфекционного стационара с жалобами на одышку, повышение температуры тела до 39С, сухой кашель. Эпиданамнез: контакт с сестрой у которой отмечались подобные симптомы. Объективно: состояние тяжелое, одышка выраженная, с участием вспомогательной мускулатуры, в легких в нижних отделах ослабленное. ЧД-26, SPO2-88%. Анализ крови: лейкоциты. — 6,4х109/л, лимфоциты – 7%. Врач заподозрил коронавирусную инфекцию. Назначьте лабораторное исследование для постановки диагноза?

ПЦР Выявление РНК SARS-CoV-2

110.Женщина 65 лет была доставлена в приемное отделение инфекционного стационара с жалобами на одышку, повышение температуры тела до 39С, сухой кашель. Эпиданамнез: контакт с сестрой у которой отмечались подобные симптомы. Объективно: состояние тяжелое, одышка выраженная, с участием вспомогательной мускулатуры, в легких в нижних отделах ослабленное. ЧД-26, SPO2-88%. Анализ крови: лейкоциты. — 6,4–9/10л, лимфоциты – 7%. Врач заподозрил коронавирусную инфекцию. Какова тактика врача приемного отделения?

Госпитализация в инфекционный стационар или перепрофилированное отделение. Консультация врача ОРИТ

111.Женщина 65 лет была доставлена в приемное отделение инфекционного стационара с жалобами на одышку, повышение температуры тела до 39С, сухой кашель. Эпиданамнез: контакт с сестрой у которой отмечались подобные симптомы.

Объективно: состояние тяжелое, одышка выраженная, с участием вспомогательной мускулатуры, в легких в нижних отделах ослабленное. ЧД-26, SPO2-88%. Анализ крови: лейкоциты. — 6,4х109/л, лимфоциты – 7%. Врач заподозрил коронавирусную инфекцию. Назначьте инструментальное исследование для постановки диагноза?

Рентген органов грудной клетки (подозрение на пневмонию)

112. Женщина 65 лет была доставлена в приемное отделение инфекционного стационара с жалобами на одышку, повышение температуры тела до 39С, сухой кашель. Эпиданамнез: контакт с сестрой, у которой отмечались подобные симптомы. Объективно: состояние тяжелое, одышка выраженная, с участием вспомогательной мускулатуры, в легких в нижних отделах ослабленное. Анализ крови: лейкоциты. — 6,4–9/10л, лимфоциты – 7%. Фибриноген – 700 г/л, АЧТВ – 65 сек., Д-димер 5,0 нг/м. Врач заподозрил коронавирусную инфекцию. Какой вид терапии необходим данной пациентке? ПАТОГЕНЕТИЧЕСКАЯ ТЕРАПИЯ: ЗАМЕСТИТЕЛЬНАЯ

ИНФУЗИОННАЯ ТЕРАПИЯ СЗП; СИМПТОМАТИЧЕСКАЯ ТЕРАПИЯ: жаропонижающие препараты; Антибиотикотерапия (?)

Кардиология (130)

ЗАПОМИНАНИЕ

1. Как называется ургентное жизнеугрожающее состояние, характеризующееся синдромом тяжелой артериальной гипертензии со стойким повышением диастолического артериального давления, в подавляющем большинстве случаев достигающего 140 мм. рт. ст. и более с тяжелым поражением эндотелия сосудов органов-мишеней, особенно часто проявляющееся на глазном дне отеком диска зрительного нерва, ватообразными экссудатами и ретинальными кровоизлияниями

Осложненный гипертонический криз

Проверено

2. Как называется спектр изменений развивающийся при артериальной гипертензии и проявляющийся тоногенной дилатацией с последующей концентрической гипертрофией миокарда левого желудочка и развитием сердечной недостаточности?

Гипертоническое сердце

Проверено

3. Как называется рентгеноконтрастный метод исследования, который является наиболее точным и достоверным способом диагностики коронарной болезни сердца, позволяя точно определить характер, место и степень сужения коронарной артерии?

Коронароангиография

Проверено

4. Как называется хирургическое катетерное вмешательство, с использованием специальных катетеров, пункционно введенных в полость сердца через сосуды, и радиочастотной энергии, направленное на устранение нарушений сердечного ритма Радиочастотная катетерная аблация предсердия

Проверено

5.Какие нарушения выявляют при внутрисердечном электрофизиологическом исследовании сердца? Нарушение ритма и проводимости, АV блокады,желудочковые,наджелудочковые тахикардии,блокады ножек пучка Гиса.

Проверено

6.При каком из пороков сердца, увеличиваются левые камеры сердца?Недостатчность аортального клапанаПроверено

7.Для какого заболевания характерны патологические сердечные шумы, напоминающие «хруст снега»?Перикардит,трение перикарда Проверено

8.К поздним осложнения острого инфаркта миокарда относится:Кардиосклероз,тромбоэмболия, синдром дресслера,ХСН,аневризма

Проверено

9.Как называется комплекс мероприятий, обеспечивающих наилучшее физическое и психическое состояние, позволяющий больным с хроническими или перенесенными острыми сердечно-сосудистыми заболеваниями, благодаря собственным усилиям, сохранить или восстановить свое место в обществе, социальный статус и вести активный образ жизни: Реабилитация

Проверено

10.В норме площадь митрального отверстия: 4-6 см2Проверено

11.Что характерно для мерцательной аритмии?

-

отсутствие зубцов Р

-

наличие мелких f волн разной формы, беспорядочные, нерегулярные

-

интервалы R-R разной продолжительности

-

Дефицит пульса

Проверено

12.Какой из ниже перечисленных препаратов уменьшает риск развития фибрилляция желудочков при ОИМ? Бета-адреноблокаторы

Проверено

13.К ранним осложнениям острого инфаркта миокарда относится:

-отек легких

-кардиогенный шок

-нарушения ритма и проводимости

-аневризма

-разрыв сердца

-перикардит

-тромбоэмболия

Проверено

14.При инфекционном эндокардите у наркоманов типичным является поражение:

трикуспидального клапана, т.к. чаще поражаются правые отделы сердца

Проверено

15.Какой из перечисленных рентгенологических признаков является общим для миокардита и экссудативного перикардита: кардиомегалия

Проверено

16.Какой основной фактор обусловливает нарушение гемодинамики при гипертрофической кардиомиопатии: гипертрофия миокарда, нарушение

диастолической функции миокарда

Проверено

17.Какой метод исследования используют для определения функционального класса стенокардии напряжения? нагрузочные пробы (тредмил-тест, ВЭМ и тд.)

Проверено

18.Какая аускультативная картина сердца наиболее характерна для пациентов с выраженной сердечной недостаточностью? протодиастолический ритм “галопа”

Проверено

ПОНИМАНИЕ

1.Больная Б., 58 лет, поступила в кардиологическое отделение с жалобами на сильные головные боли в затылочной области пульсирующего характера. За медицинской помощью никуда не обращалась, самостоятельно принимала каптоприл 1 таб под язык при повышении АД. Объективно: состояние средней тяжести. Отеки ног. Перкуторно левая граница сердца-на 1,5 см кнаружи от левой срединно-ключичной линии, аускультативно: акцент II тона над аортой. АД - 180/110 мм.рт.ст., ЧСС - 84 уд/мин. На ЭКГ признаки ГЛЖ с наршением процессов реполяризации. На Эхо-КГ: ТМЖП-1,2 см, ТЗСЛЖ-1,3 см. Диффузный гипокинез стенок ЛЖ. Лечащий врач расценил это состояние как гипертоническое сердце. Какая стадия данного осложнения имеется у этой пациентки?

III стадия

2.Больной Г., 40 лет, обратился к участковому терапевту с жалобами на головные боли, головокружение, снижение остроты зрения, стойкое повышение АД до 210/120 мм.рт.ст., не поддающееся коррекции антигипертензивными препаратами. В течение 5 лет страдает артериальной гипертензией. При осмотре: состояние относительно удовлетворительное. При осмотре: левая граница относительной сердечной тупости сердца смещена влево, при аускультации акцент II тона над аортой. ЧСС - 76 уд/мин. АД на правой руке 195/120 мм.рт.ст., на левой руке 190/120 мм.рт.ст. Артерии глазного дна сужены, извиты, определяется участок кровоизлияния в сетчатку и отек зрительного нерва. На ЭКГ - признаки ГЛЖ с нарушением процессов реполяризации. Ваш предварительный

диагноз? ГБ III стадия, 3 степень, очень высокого риска. Гипертоническое

сердце, гипертоническая ретинопатия.

Проверено

3.Пациентка 32 лет, страдающая митральным пороком, жалуется на сухой кашель, сердцебиение, одышку при незначительной физической нагрузке. Объективно: бледность кожи, акроцианоз губ, щек, отеки нижних конечностей. В легких дыхание жесткое, выслушиваются влажные мелкопузырчатые хрипы в нижних отделах. Тоны сердца аритмичные, ЧСС – 110 уд/мин, АД – 110/80 мм.рт.ст. Печень на 3 см ниже реберной дуги. Были назначены бета-блокаторы. Каков механизм действия бета-блокаторов при лечении хронической сердечной недостаточности? увеличения площади сокращающейся зоны( уменьшение

числа «спящих» кардиомиоцитов), снижение ЧСС, уменьшение степени ишемии миокарда в покое и при физической нагрузке, снижение числа жел аритмий, антифибриллярное действие и уменьшение вероятности внезапной сердечной смерти

Проверено

4.При назначении какой группы препаратов пациентам с коронарной болезнью сердца в сочетании с сердечной недостаточностью и ХОБЛ повышается уровень брадикинина в крови, что обусловливает появление кашля, имитируя обострение хронического обструктивного бронхита ? и-АПФ Проверено

5.Больная Х., 25 лет обратилась врачу с жалобами на появление обморочных

состояний, головокружение, незначительную одышку. При осмотре - «пляска каротид». Куполообразный верхушечный толчок смещён вниз и влево. Перкуторно определяется значительное увеличение границ сердца влево. Аускультативно: I тон ослаблен на верхушке, диастолический шум убывающего характера на аорте и в т. Боткина-Эрбо. ЧСС 60 уд/мин АД170/30 мм рт.ст. Биохимичсекие анализы крови без особенностей. На ЭКГ – гипертрофия левого желудочка с нарушением процессов реполяризации. Какой дигноз у больной?

Недостаточность аортального клапана.

Проверено

6.Пациент А., 26 лет. Обратился к врачу с жалобами на повышение температуры тела до 380С, сопровождающиеся ознобами, отеки голеней, тяжесть в правом подреберье. Из анамнеза: в течение последних 3-х лет употребляет наркотики. Объективно: t°-37,7 0С, кожные покровы бледные с

синюшным оттенком, отеки стоп и голеней. Тоны сердца ритмичные, ЧСС 110 уд./мин, отмечается ослабление первого тона над мечевидным отростком. Пальпируется край селезенки. Из обследований: ОАКHb 100,0 г/л, лейк. 15 х 109/л, СОЭ 55 мм/час. На ЭКГ: ритм синусовый, ЧСС 110/мин. ЭхоКГ: отмечается увеличение размера правого предсердия до 4,5 см, правого желудочка до 2,7 см, трикуспидальная регургитация II-III степени, множество дополнительных эхосигналов на трехстворчатом клапане, “лохматые створки”. Определите диагноз?-

Первичный инфекционный эндокардит правых отделов сердца, острое течение. Недостаточность трикуспидального клапана. Проверено

7.У больного С., 35 лет, на фоне высокой лихорадки, появилась одышка, учащенное сердцебиение. Из анамнеза: употребляет наркотики. При объективном исследовании: цвет кожных покровов – “кофе с молоком”, мелкоточечные петехиальные высыпания на теле, выявлено увеличение границ сердечной тупости влево, тахикардия, диастолический шум в точке Боткина. АД 120/60 мм рт. ст., гепатоспленомегалия. О каком заболевании идет речь? инфекционный эндокардитПроверено

8.Женщина К., 32 лет обратилась к врачу по месту жительства с жалобами на одышку при физической нагрузке, сердцебиение, перебои в работе сердца. Считает себя больной около 6 мес, когда после простуды, появились прокалывающие боли в сердце, чувство «замирания» в сердце. Последние 2 мес появилась одышка, которая в последнее время стала беспокоить при незначительной физической нагрузке, в связи с чем она обратилась к врачу. Объективно: кожные покровы бледные, акроцианоз. Верхушечный толчок в 5 межреберье, разлитой, ослаблен. Левая граница сердца – на 1,0 см кнаружи от левой СКЛ. Тоны аритмичные, за 1 мин 20 внеочередных ударов, приглушены. 1 тон на верхушке ослаблен. Какой вариант миокардита у больной? - Аритмический миокардит Проверено

9.Больной К., 40 лет, Обратился к врачу с жалобами на ноющие боли в области

сердца на протяжении 2 дней, сердцебиение и одышку при физической

нагрузке. 2 нед назад переболел острым респираторным заболеванием. В ОАК - лейкоцитов 10,3х109/л, СОЭ 38 мм/ч, гаммаглобулины 23 %, СРБ - «+++», фибриноген 5,3 г/л, АСТ – 51 Е/л, АЛТ – 53 Е/л. На ЭКГ – ритм не синусовый, неправильный, мерцательная аритмия, ЧСС – 112уд/мин. На ЭхоКГ - дилатация предсердий и желудочков, диффузный гипокинез миокарда, ФВ - 32 %, митральная регургитация II степени. Какой из нижеперечисленных методов исследований является золотым стандартом диагностики данного заболевания?- Эндомиокардиальная биопсия

Проверено

10. - Больной Е., 24 лет. Обратился к врачу с жалобами на тупые боли в области сердца с одышкой, появившиеся в последние 2 нед. Менее месяца назад перенес ОРВИ. Ранее считал себя здоровым человеком, уровень артериального давления не контролировал. Наличие вредных привычек (курение, злоупотребление алкоголем) отрицает. При осмотре: Частота дыхательных движений - 22 в минуту. При аускультации лёгких - дыхание везикулярное, хрипов нет. Тоны сердца глухие, пульсовое давление снижение до 25 мм рт.ст. во время глубокого вдоха. АД 100/60 мм.рт.ст., ЧСС - 102 в минуту. На ЭКГ-зубец (-) Т в отведениях V1-V4.

О какой патологии можно думать? экссудативный (выпотной) перикардит

Инфекционный миокардит

11.У больной 34 лет во время эпидемии гриппа повысилась температура до 390С и держалась в течение суток, появились небольшие катаральные явления. Через неделю от начала заболевания присоединились боли в области сердца, сердцебиение, выраженная одышка, отеки голеней. Госпитализирована в тяжелом состоянии с нарушением кровообращения в большом и малом круге, увеличением сердца. В крови лейкоциты 15 х 109/л, СОЭ 19 мм/час, тропонин 3,5 нг/мл. На ЭКГ блокада левой ветви пучка Гиса. Для постановки диагноза какой метод диагностики Вы рекомендуете?

Ответ:ЭХО-КГ// Эндомиокардиальная биопсия Проверено

12.Больной Т., 48 лет. Обратился к врачу с жалобами на чувство стеснения в груди, боли в области сердца, лихорадка до 38 градусов, одышка, дважды перед приездом врача была рвота. Болен в течение недели: появилась и нарастала одышка, особенно в горизонтальном положении. Объективно: отечность лица, шейные вены набухшие. Больной сидит на краю кровати,

склонившись вперед. Кожные покровы бледные с цианотичным оттенком. В легких дыхание везикулярное, хрипов нет. ЧДД-20 в мин. Пульс частый, малого наполнения, верхушечный толчок не определяется. Сердце расширено во все стороны, АД – 120/80 мм.рт.ст. Печень увеличена на 2 см. Какой Ваш предварительный диагноз?

Ответ: Экссудативный перикардит

Проверено

13.Больной Х. 45лет, поступил в поликлинику с жалобами на чувство сдавливания в области сердца, одышку, слабость. В анамнезе состоит на учете в тубдиспансере. Объективно: цианоз, асцит, отеки на ногах, пульсация вен шеи. В легких дыхание ослаблено в нижних отделах. Пульс слабого наполнения и напряжения, на вдохе наполнение пульса уменьшается. Верхушечный толчок не определяется. Границы сердца не расширены. Тоны сердца приглушены. Шумов нет. АД90/60 мм.рт.ст., Печень увеличена на 5 см. Венозное давление – 30 мм.вод.ст, общий белок плазмы крови20г/л. Рентгенологически- в плевральных полостях выпот. Выбухание верхней полой вены, увеличена 1-я дуга по правому контуру сердца. Присутствуют очаги кальцификации в перикарде. Какой Ваш предварительный диагноз?

Ответ:Констриктивный перикардит с обызествлением (панцирное сердце)

Проверено

14.Больной Д., 32 лет, жалуется на дискомфорт в грудной клетке, одышку, сердцебиение, приступы головокружения, частые эпизоды потери сознания. Объективно: при пальпации определяется двойной верхушечный толчок, при аускультации - поздний систолический шум над верхушкой сердца. ЭКГ: в отведениях V4-6 определяется патологический зубец Q, R V5> RV4. ЭхоКГ: отношение толщины межжелудочковой перегородки к нижней стенке ЛЖ составляет 2,3; систолическое прилегание передней створки митрального клапана к межжелудочковой перегородке, среднесистолический шум наступает через, 14 с после открытия клапана аорты, определяется субаортальный градиент систолического давления. Какой вероятный диагноз у больного?

Ответ : Асимметричная Гипертрофическая кардиомиопатия Проверено

15.Больной Т., 16 лет, госпитализирован с жалобами на выраженную одышку, сердцебиение, увеличение живота, общую слабость. Из анамнеза жизни: 8

месяцев назад внезапно умер его 17-летний брат. Объективно: значительное отставание в физическом развитии, кожа бледная, при аускультации в легких ниже углов лопаток дыхание отсутствует; тоны сердца ослаблены, выслушиваются систолический шум с эпицентром в точке Боткина, ЧСС - 130 за 1 мин; асцит. Рентгенологически: признаки кардиомегалии, имеется жидкость в обеих плевральных полостях. ЭхоКГ: фракция выброса - 27%, диаметр левого желудочка~9см, толщина задней стенки левого желудочка - 0,65 см. Поставьте диагноз:

Ответ : Дилатационная кардиомиопатия. Экссудативный плеврит. Асцит.

Проверено

16.Пациентка П., 40 лет, обратилась к врачу с жалобами на одышку, приступы удушья, приступообразные интенсивные боли за грудиной при незначительной физической нагрузке. Из анамнеза известно, что перенесла ревмокардит в 13 лет, по поводу чего получала бициллинопрофилактику. Объективно: резкая бледность кожных покровов. Заметно сотрясение головы в такт систоле, положительный капиллярный пульс, пульсация сонных артерий. В легких ослабленное везикулярное дыхание. Пульс скорый, высокий. Выслушивается мягкий диастолический шум над аортой и в точке Боткина-Эрба, ослабление II тона над аортой. АД 140/40 мм рт. ст. Решено провести ЭхоКГ исследование. Какие результаты Вы ожидаете получить?

Ответ:Диастолическое дрожание передней створки митрального клапана,

Несмыкание створок аортального клапана в диастолу, аортальная регургитация.Проверено

17.Больной Е., 45 лет. Жалуется на тупые, колющие боли в прекардиальной области, с иррадиацией в трапециевидную мышцу, усиливающие после кашля, при глубоком вдохе. Объективно: положение ортопноэ. В легких дыхание ослаблено в нижних отделах. Перкуторно границы сердца не расширены. АД90/60 мм рт.ст. ЧСС 80/мин. Пульс слабого наполнения и напряжения, на вдохе наполнение пульса уменьшается. Верхушечный толчок не определяется. Границы сердца не расширены. Тоны сердца приглушены. Печень увеличена на 5 см. Венозная давления – 30 мм.вод.ст, общий белок плазмы крови20г/л. Больного направили на ЭКГ. Какие изменения Вы увидите на ЭКГ при данном болезни?

ответ :Фибринозный перикардит: Конкордантная элевация R-ST почти во всех отведениях, инверсия зубца Т во многих отведениях, нет патологического зубца

Q.Проверено

18.Больной К., 40 лет, Обратился к врачу с жалобами на ноющие боли в области

сердца на протяжении 2 дней, сердцебиение и одышку при физической нагрузке. 2 нед. назад переболел острым респираторным заболеванием. В ОАК лейкоцитов - 10,3х109/л, СОЭ 38 мм/ч, гаммаглобулины 23 %, СРБ - «+++», фибриноген 5,3 г/л, АСТ – 51 Е/л, АЛТ – 53 Е/л. На ЭКГ – ритм не синусовый, неправильный, мерцательная аритмия. ЧСС – 112 уд/мин. Депрессия сегмента ST и отрицательный зубец Т в отведениях I, aVL, V4-6. Больной был направлен на эхокардиографию, где выявлена дилатация полостей. Какие еще эхокардиографические показатели характерны для данного заболевания?

ответ:Небольшое увеличение КДО, КСО ЛЖ, снижение ФВ, локальные нарушения сократимости, признаки относительной недостаточности митрального клапана с регургитацией, в/ы внутрисердечные тромбы, и утолщение стенок ЛЖ за счет отека интерстиция

Гипокинезия участвков миокарда,снижение сократительной функции миокарда,Снижение ФВ, увеличение КДО,КСО

19.Больной 52 лет поступил в кардиологическую клинику с диагнозом: острый инфаркт миокарда задней стенки левого желудочка. На 3-й сутки наблюдения внезапно стала нарастать одышка, появились боли и чувство распирания в правом подреберье, отеки на ногах. Объективно: Состояние тяжелое. Кожные покровы бледные, акроцианоз. В легких хрипов нет. Число дыханий 26 в мин. Тоны сердца глухие, ритмичные. Выслушивается пансистолический шум по парастернальной линии, определяется систолическое дрожание. ЧСС 92 ударов в мин. АД 100/60 мм рт. ст. Печень увеличена на 4 см. Ваш предварительный диагноз:

ответ:Раннее осложнение ОИМ. Разрыв межжелудочковой перегородки.

ОПЖН.

Проверено

20.Мужчина 56 лет, 10 дней назад поступил в отделение ургентной кардиологии (УК) с диагнозом: «Острый инфаркт миокарда». На 2-е сутки после стабилизации гемодинамических параметров пациент был переведен с отделения УК в палату кардиологического отделения больницы. В настоящее время самочувствие пациента относительно удовлетворительное. Объективно: АД – 110/70 mmHg,

ЧСС – 58/мин. Ангинозных болей нет. Какую ступень физической реабилитации Вы разрешите пациенту на данном этапе:

ответ Ступень I

Проверено

21.Девушка 24 лет, обратилась к врачу по месту жительства с жалобами на прогрессирующее ожирение, характеризующееся отложением жира в области лица, верхней части туловища и живота, гирсутизм, нарушение менструального цикла, боли в поясничной области. Объективно: АД - 150/90 мм рт. ст. При обследовании: сахар крови натощак 7,5 ммоль/л. УЗИ внутренних органов: выявлено увеличение левого надпочечника. Ваш предварительный диагноз:

ответ:Синдром Иценко-Кушинга. Вторичная АГ 1степени. Нарушение

толерантности к глюкозе.

Проверено

22.Женщина 65 лет поступила в стационар с жалобами на головные боли в затылочной области, рвоту, головокружение, мелькание мушек перед глазами, судороги. Из анамнеза: повышение уровня АД в течении 5-х лет, максимальные цифры 210/100 мм рт. ст. Постоянно принимала индапамид, эналаприл с положительным эффектом, но в течении последнего месяца отменила гипотензивную терапию, стала много употреблять в пищу соленой пищи. Объективно: Пульс твердый, напряженный, 82 в минуту. Тоны сердца приглушены, акцент 2-го тона на аорте. АД 220/140 мм рт.ст. Какой вариант осложненного гипертонического криза у пациентки:

Ответ:Церебральный Проверено

23.Парень 20 лет на приеме у кардиолога по месту жительства жалуется на одышку при умеренных физических нагрузках. Анамнез: с детства частые ангины. Объективно: Кожные покровы обычной окраски, отеков нет. Сердечные тоны ритмичные, на верхушке I тон ослаблен, систолический шум. ЧСС – 78 уд/мин. АД – 120/80 мм рт. ст. Ваш предварительный диагноз:

Ответ: Хроническая ревматическая болезнь сердца. Недостаточность митрального

клапана.

Проверено

24.Мужчина 39 лет, обратился к врачу по месту жительства с жалобами на одышку при незначительных физических нагрузках, учащенное сердцебиение, приступы удушья по ночам, слабость. В анамнезе: частые ангины. Объективно: Эпигастральная пульсация. Верхушечный толчок усилен, разлитой, пальпируется на 2 см кнаружи от СКЛ. Границы сердца: правая – на 2 см кнаружи от правого края грудины, верхняя – ІІ ребро, левая – на 2 см кнаружи от СКЛ. Тоны сердца приглушенные, ритмичные, ЧСС 92 в минуту, на верхушке І тон усилен, систолический и диастолический шум, акцент ІІ тона на легочной артерии АД 110/70 мм рт. ст. Ваш предварительный диагноз:

1.Недостаточность митрального клапана. Стеноз левого AV отверст

2.Недостаточность аортального клапана. Стеноз устья аорты

3.Стеноз левого AV отверстия. Недостаточность аортального клапана

4.Стеноз устья аорты. Стеноз левого AV отверстия

5.Недостаточность митрального клапана. Недостаточность аортального клапана

25.Мужчина А. 48 лет обратился к врачу по месту жительства с жалобами на головные боли, головокружение. В анамнезе страдает артериальной гипертонией в течение 2 лет с максимальным повышением АД до 160/100 мм.рт.ст., курит, имеет отягощенную наследственность по КБС. При физикальном исследовании рост 173 см, масса тела 100 кг, окружность талии 106 см. ЧСС75 уд/ мин., АД – 160/90 мм.рт.ст. По органам и системам без отклонений от нормы. В лабораторных анализах липидного спектра: общий холестерин 7,23 ммоль/л, холестерин липопротеидов высокой плотности (ЛПВП) 1,2 ммоль/л, холестерин липопротеидов низкой плотности 4,03 ммоль/л. Какой целевой уровень холестерина липопротеидов низкой плотности следует рассматривать для данного пациента?

Ответ:< 1.8 ммоль/л

Проверено

26.Больной В., 50 лет, обратился к врачу с жалобами на приступы болей сжимающего характера, возникающие за грудиной, иррадирующие в левую руку, под левую лопатку. Продолжительность болей 3-5 мин. Боли возникают при ходьбе и подъеме на два этажа, в покое проходят. Беспокоят около 3 месяцев. Объективно: температура 36,60С. Общее состояние удовлетворительное. Кожа чистая. Подкожно-жировая клетчатка развита избыточно, ИМТ – 32 кг/м2. Дыхание везикулярное, ЧДД 16 в мин. Тоны сердца ритмичные, чистые, ЧСС 84 в мин., АД 120/80 мм.рт.ст. Абдоминальной патологии не выявлено. В анализах крови уровень общего холестерина – 4,2 ммоль/л. Какой модифицируемый фактор риска КБС выявлен у пациента?

Ожирение 1 степени

Проверено

27.Женщина К. 56 лет обратилась к врачу по месту жительства с жалобами на давящие боли за грудиной при ходьбе, купирующиеся приемом нитроглицерина или прекращением движения. Болеет 5 лет. В посленее время отмечает, что приступы загрудинных болей стали частыми. Объективно: Состояние удовлетворительное. АД = 150/80 мм рт ст. Границы сердца в норме.Тоны сердца ясные. Дыхание везикулярное. Печень по краю реберной дуги. Рекомендовано снять ЭКГ при болях. Какой должна быть на ЭКГ продолжительность специфической горизонтальной или косонисходящей депрессии сегмента ST от точки J в одном и более ЭКГ отведении? 80мс ( 0.08 с.)

Проверено

28.Больной К. 48 лет, жалуется на сжимающие боли за грудиной, иррадиирующие в левое плечо, возникающие при ходьбе через 100 метров, купирующиеся приемом одной таблетки нитроглицерина через 3 минуты. Положительный результат опросника Роуза в течение 5 лет. Ухудшение состояния последние 10 дней. Приступы болей за грудиной стали беспокоить в покое, в ночное время. Курит около 20 лет. Наследственность отягощена. Общее состояние удовлетворительное. Дыхание везикулярное. Границы сердца в норме. Тоны сердца приглушены, ритм правильный, акцент II тона над аортой. ЧСС – 82 удара в минуту. АД - 135/80 мм рт. ст. О каком варианте стенокардии идет речь?

Нестабильная стенокардия, прогрессирующее течение

Проверено

29.Больной А. 58 лет доставлен бригадой скорой помощи в отделение реанимации с жалобами на боли в эпигастральной области, сопровождающиеся слабостью, потливостью, тошнотой. Ранее боли не беспокоили, считал себя здоровым. Попытка купировать боли раствором соды облегчения не принесла. После приема Нитроглицерина под язык боли уменьшились, но полностью не прошли. Сохранялись тошнота, слабость, потливость. На снятой ЭКГ выявлен глубокий зубец Q в III и aVF отведениях; сегмент ST в этих же отведениях приподнят над изолинией, дугообразный, переходит в отрицательный зубец Т. Лечащим врачом выставлен диагноз острого инфаркта миокарда. Какая локализация данного заболевания у пацитента? Заднедиафрагмальный (нижний) ИМПроверено

30.Пациент А. 56 лет поступил в стационар с жалобами на давящие боли за груиной, он перенес неосложненный КБС Не Q-волновой инфаркт миокарда 2 месяца назад. Для решения вопроса трудоспособности, было назначено велоэргометрическое исследование с физической нагрузкой. Больной жалоб не

предьявлял. Исходные показатели гемодинамики были следующие: АД-120/80 мм.рт.ст., ЧСС-74 удара в минуту. Во время проведения теста появились признаки, которые явились показанием для прекращения проведения последнего. Укажите один из этих признаков:

приступ стенокардии;

снижение систолического давления более чем на 10 мм рт. ст. от исходного уровня или отсутствие его повышения при возрастании нагрузки;

АД более чем 220/130 мм рт. ст.;

выраженная одышка, приступ удушья;

головокружение, тошнота, сильная головная боль, сбивчивые ответы на вопросы, бледность, цианоз;

отказ пациента от дальнейшего проведения пробы (боязнь или чувство дискомфорта);

достижение возрастной субмаксимальной ЧСС (ЧСС = 207 − 0,7 × × возраст)

горизонтальное, косонисходящее, корытообразное снижение сегмента ST на 1 мм и более;

медленное косовосходящее снижение ST со снижением точки j на 2 мм при Q–x > 50 % от соответствующего QT.

подъем сегмента ST на 1,5 мм.

снижение ST в восстановительном периоде (признается не всеми авторами);

частая экстрасистолия (4 : 40), пароксизмальная тахикардия, мерцательная аритмия;

развитие нарушений атриовентрикулярной и желудочковой проводимости;

изменение QRS-комплекса, в том числе переход QRS в QS; появление отсутствовавшего до нагрузки зубца Q шириной 0,03 с и амплитудой 1 мм и более;

Проверено

31.Мужчина 58 лет, поступил с жалобами на головную боль, головокружение, боли давящего характера в прекардиальной области, возникающие при повышении уровня АД. Из анамнеза: страдает сахарным диабетом; отец умер от инсульта. Объективно: Повышенного питания. Границы сердца - правая по правому краю грудины, верхняя II ребро, левая на 2 см кнаружи от левой средне-ключичной линии. АД – 180/90 мм Нg, Тоны сердца ритмичные. ЧСС78уд. в мин. Из обследований: ОАМреакция кислая, белок 50мг, лейкоциты 2-3 в п/зр, эритроциты неизмененные 1-2 в п/зр, гиалиновые цилиндры 1-2 в п/зр. На ЭКГ признаки ГЛЖ. Ваш предварительный диагноз?

ГБ 3 стадии, 3 степени, очень высокого риска. Гипертоническое сердце. Гипертоническая нефропатия. СД. Ожирение.

Проверено

32.Мужчина 45 лет, обратился к терапевту с жалобами на головную боль, преимущественно в затылочной области, мелькание мушек перед глазами после перенапряжения. Объективно: Вес 98 кг, рост 175 см. При пальпации сердца – верхушечный толчок разлитой, усиленный, приподнимающийся. Акцент II тона над аортой при аускультации. ЧСС 72 уд/мин. АД 180/110 мм рт.ст. В анализах крови: сахар крови – 5,5 мкмоль/л, креатинин - 105 мкмоль/л, калий – 4,5 ммоль/л. Пациенту рекомендовано пройти ЭКГ. Какие изменения на ЭКГ, наиболее характерны для больных этим заболеванием?

Гипертрофия ЛЖ (зубец R в V5(V6) > 16 мм,

RV6>RV5>RV4,

зубец S в V1, V2 >12 мм,

RV5 (V6) +SV1> 28 мм)

Проверено

33.Больной А. 45 лет, инженер-технолог обратился в стационар с жалобами на редкие приступы стеснения в груди, возникающие при ходьбе через 500-1000 метров, ощущение перебоев в работе сердца. Объективно: дыхание везикулярное, тоны сердца звучные. Пульс 72 уд в мин., ритмичный. АД 130/80 мм рт.ст. ЭКГ: ритм синусовый, неправильный, за счет появления единичных внеочередных комплексов QRS, зубец P отсутствует перед комплексом QRS, полная компенсаторное пауза. Какое нарушение ритма сердца возникло у данного пациента? Желудочковая экстрасистолия Проверено

34.Пациент К., 60 лет, поступил в отделение ургентной кардиологии в связи с появлением интенсивных загрудинных давящих болей общей продолжительностью более 30 мин, не купирующихся приемом нитроглицерина, покрывался холодным липким потом. На снятой ЭКГ - монофазный подъем

сегмента ST в II, III, aVF отведениях. Пациент подключен к круглосуточному кардио-мониториру. При регистрации каких изменений на ЭКГ будет эффективна электрическая дефибрилляция:1. Крупноволновых осцилляций правильной форма,фибрилляция желудочков Проверено

35.Больная А. 65 лет обратилась к врачу с жалобами на приступы головокружения с кратковременными потерями сознания. В анамнезе: перенесла инфаркт миокарда нижней стенки ЛЖ два года назад. Год назад впервые возник приступ потери сознания в течение нескольких минут, с непроизвольным мочеиспусканием. В последний месяц аналогичные приступы участились. Объективно: сознание ясное. Тоны сердца глухие, аритмичные, ЧСС - 42 ударов в минуту, Ps - 42 в мин. АД - 110/65 мм рт. ст. На снятой ЭКГ отмечается – постепенное увеличение интервала P–Q с последующим выпадением желудочкового комплекса QRS при сохранении зубца Р. Назовите отклонения от нормы, выявленные на ЭКГ, и сформулируйте ЭКГ-заключение?AV блокада 2 степень мобиц 1

Проверено

36.Пациентка А. 32 лет, обратилась к врачу по месту жительства с жалобами на сухой кашель, сердцебиение, одышку при незначительной физической нагрузке. В анамнезе частые ангины. Ухудшение состояния в течение месяца. Объективно: бледность кожи, акроцианоз губ, щек, отеки нижних конечностей. В легких дыхание жесткое, выслушиваются влажные мелкопузырчатые хрипы в нижних отделах. Тоны сердца: ритм неправильный, мерцательная аритмия, ЧСС – 100 уд/мин, на верхушке сердца - I тон усилен, тон открытия митрального клапана, там же мезодиастолический шум. АД – 120/80 мм.рт.ст. Живот мягкий, печень пальпируется на 3 см ниже правой реберной дуги. Лечащим врачом выставлен диагноз ревматической лихорадки. Какой порок сердца явился причиной развития застойной сердечной недостаточности?Стеноз митрального клапана

Проверено

37.Мужчина А. 62 лет находился в отделение реанимации с клиникой ОИМ переднеперегородочной области ЛЖ. На второй день пребывания в стационаре внезапно стал отмечать выраженное удушье, кашель с отделением пенистой мокроты с примесью крови. Объективно: положение ортопноэ, акроцианоз, в легких – перкуторно легочной звук, масса мелко-среднепузырчатых влажных хрипов. I тон ослаблен, систоло–диастолический шум с пресистолическим усилением на верхушке сердца. ЧСС – 120 в мин., АД 110/70 мм.рт.ст. Какое осложнение ОИМ

развилось у данного больного? Острая левожелудочковая сердечная недостаточность(сердечная астма или отек легких)

Проверено

38.Женщина 72 лет, поступила в реанимацию с жалобами на интенсивные давящие боли за грудиной, с иррадиацией в левую руку, длительностью в течение часа. Боли прошли только после внутривенного введения морфина. Объективно: состояние тяжелое. В легких ослабленное везикулярное дыхание, хрипы не выслушиваются. ЧД 16 в мин. Границы сердца не расширены, тоны приглушены, пульс ритмичный, 60 уд/мин, АД 140/80 мм рт.ст. На ЭКГ патологический зубец Q в V3-4, там же подъем сегмента ST до 2 мм. Лечащим врачом выставлен диагноз инфаркта миокарда. Поражение какой стенки ЛЖ и какая стадия заболевания имеется у данной пациентки:1. Передняя стенка ЛЖ,острая стадия

Проверено

39.Женщина 77 лет, длительное время страдающая гипертонической болезнью с повышением АД до 200/100 мм.рт. ст., госпитализирована в реанимационное отделение с диагнозом инфаркт миокарда нижней стенки. На 3-и сутки у больной внезапно наросла одышка, отмечено снижение АД до 80/50 мм.рт.ст., при аускультации сердца выслушивается пансистолический шум в области абсолютной тупости сердца. Какое вероятное осложнение инфаркта миокарда развилось у данной пациентки:

Разрыв сердца

Проверено

40.Мужчина 56 лет, на приеме у кардиолога отмечает головные боли в затылочной области, головокружение, снижение зрения. Анамнез: вышеуказанные жалобы беспокоят в течении 3 лет, по поводу чего к врачам не обращалась. Наследственность отягощена. Стаж курения 15 лет по 4 штуки в день. Объективно: ИМТ 33 кг/м2, дыхание везикулярное, границы относительной тупости сердца в норме, тоны сердца ритмичные, ЧСС 80 в мин, АД 170/100 мм рт.ст. Ваш предварительный диагноз: ГБ 2 стадии, 2 степени, высокий риск.

Проверено

41.Юноша 17 лет, направлен на обследование из военкомата. Жалоб не предъявляет. В анамнезе частые ангины. Объективно: состояние удовлетворительное. В легких везикулярное дыхание. Границы относительной сердечной тупости незначительно увеличены вверх и влево. Тоны сердца ритмичные, ослабление I тона и систолический шум на верхушке. В анализе крови без патологии. На электрокардиограмме – ритм синусовый, правильный, частота сердечных

сокращений 72 в минуту, признаки гипертрофии левого желудочка. Ваш предварительный диагноз: Недостаточность митрального клапана

Проверено

42.Женщина 38 лет, экономист, обратилась к кардиологу с жалобами на головокружение, одышку при незначительных физических нагрузках. В анамнезе: в детстве частые ангины. Объективно: заметно сотрясение головы в такт систоле, положительный капиллярный пульс, пульсация сонных артерий. Пульс скорый, высокий. АД 140/50 мм.рт.ст. Отмечается усиление и смещение верхушечного толчка влево и вниз. При аускультации выслушивается мягкий диастолический шум над аортой и в точке Боткина-Эрба, ослабление II тона над аортой. Ваш предварительный диагноз: 1. Недостаточность клапанов аорты

(аортальная недостаточность)Проверено

43.Мужчина 33 лет, госпитализирован в стационар с жалобами на лихорадку (38оС) в течение 3 месяцев с ознобами и артралгиями, одышку в покое. В анамнезе в течение 10 лет принимает наркотики. При осмотре: температура тела – 39,2оС. Геморрагическая сыпь на голенях, изменение ногтевых пластин по типу “часовых стекол”. Дыхание ослаблено в нижних отделах с обеих сторон. ЧДД - 22 в минуту. При аускультации сердца систолический шум на трикуспидальном клапане. АД 104/70 мм рт. ст. ЧСС – 104 в минуту. Печень выступает из-под края реберной дуги на 4 см. Пальпируется нижний полюс селезенки. Ваш предварительный диагноз: Подострый Инфекционный эндокардит? СН ФК 4 ( акцент на

иньекционный анамнез, сыпь) 44.Проверено

45.Мужчина 32 года, после ОРВИ стал отмечать одышку, тяжесть в правом подреберье. Затем присоединились приступы удушья в ночное время, отеки на голенях. В последние месяцы отмечает эпизоды кровохарканья. Объективно: в нижних отделах легких незвучные мелкопузырчатые хрипы. Границы сердца расширены влево, тоны сердца глухие. ЧСС – 90/мин. На верхушке систолический шум. ЭКГ: мерцательная аритмия, тахисистолическая форма. ЭхоКГ: левое предсердие – 4,4; КДР левого желудочка – 6,0см. Правый желудочек – 3,2см. Фракция выброса – 40%. Диффузная гипокинезия. Ваш предварительный диагноз: Дилятационная кардиомиопатия (ДКМП) СН ( ФК

3-4? Не написано про физ нагрузки, но по клинике это уже запущенная стадия ЛЖ СН, и еще признаки ПЖ СН есть)

46.У мужчины 43 лет, программиста, без каких-либо жалоб, проходящего диспансеризацию, при эхокардиографическом исследовании получены следующие результаты: ЛП = 3,8 см, КДР = 5,2 см, КСР = 3,4 см, ТМЖП = 1,9 см, ТЗСЛЖ = 1,1 см, ПЖ= 2,4 см. Клапанный аппарат интактен, насосная и

сократительная функция в пределах нормы. По данным Д-ЭХОКГ: патологических токов не обнаружено. Ваш предварительный диагноз:

ГКМП

Гипертоническая болезнь (гипертрофия лж?) увеличены только ТМЖП. Все остальное в норме.

47.Мужчина 49 лет, на приеме у кардиолога жалуется на боли за грудиной с иррадиацией в левое плечо, лопатку, возникающие при быстрой ходьбе, сопровождаются легким головокружением, проходят в покое. Из анамнеза страдает повышением артериального давления около 3 месяцев. Объективно: границы сердца расширены влево. На верхушке ослаблен 1 тон, акцент II тон над аортой. АД - 170/100 мм рт. ст. Пульс аритмичен, 56 в мин, несколько напряжен. ЭКГ: ритм синусовый, неправильный, определяется постепенное от одного комплекса к другому увеличение длительности интервала P-Q (R) с последующим выпадением комплекса QRST при сохранении зубца Р. Ваш предварительный диагноз: КБС. Стенокардия напряжения стабильная. ФК 1. ГБ 2 стадии, 2

степени (риск неизвестно). АВ блокада 2 степени Мобиц 1 Проверено

48.Мужчина 70 лет поступил в реанимацию с жалобами на отеки ног, одышку в покое. Анамнез: год назад перенес инфаркт миокарда. Объективно: Положение ортопноэ. В легких дыхание ослабленно везикулярное, застойные влажные мелкопузырчатые хрипы в нижних базальных отделах. Тоны сердца приглушены, на верхушке ритм галопа, акцент II тона 2 м/р слева от грудины. ЧСС – 100 уд/мин. АД 90/60 мм рт.ст. Печень выступает на 3 см ниже края реберной дуги. Рентгенография органов грудной клетки: расширение сердца в поперечнике, КТИ 60%. Какой класс хронической сердечной недостаточности выявлен у пациента:

ФК 4, стадия ХСН 3 Проверено

ПРИМЕНЕНИЕ

1. Пациентка А.,1938 гр., обратилась в поликлинику с жалобами на одышку при незначительной физической нагрузке, перебои в работе сердца, отеки нижних конечностей, тяжесть в правом подреберье. В анамнезе в течение ряда лет нарушения ритма по типу мерцательной аритмии. Резкое ухудшение состояния в течение 2 месяцев. При осмотре: акроцианоз губ, отеки на нижних конечностях, в легких дыхание жесткое, мелкопузырчатые влажные хрипы в нижних отделах, ЧД-22 в мин. Границы сердца перкуторно увеличены влево, тоны сердца приглушены, мерцательная аритмия, ЧСС – 110 уд/мин, PS – 100 в 1 мин. АД –100/70 мм.рт.ст. Печень пальпирукется ниже края реберной дуги на 2 см, край плотный. Какой мочегонный препарат лучше назначить пациентке?

Верошпирон

Проверено

2. Больной Д., 58 лет, поступил с жалобами на одышку при обычной физической нагрузке, проходящую в покое. Из анамнеза: в возрасте 51 года перенес инфаркт миокарда. В течение последнего года отметил появление одышки. При осмотре: состояние средней тяжести. Кожные покровы обычной окраски. Акроцианоз губ. При аускультации над легкими выслушивается жесткое дыхание. Границы сердца увеличены влево. Аускультативно тоны сердца приглушены, систолический шум на верхушке. ЧСС - 94 удара в минуту, АД 125/80 мм.рт.ст. Живот мягкий, безболезненный. Назначение какого анализа крови подтвердит наличие сердечной недостаточности? Мозговой натрийуретический фермент

Проверено

3. Женщина 60 лет, обратилась к семейному врачу с жалобами на одышку и сердцебиение при умеренных физических нагрузках, отеки нижних конечностей. Анамнез: 4 месяца назад перенесла ОИМ. Объективно: Кожные покровы обычной окраски, отеки голеней. Дыхание жесткое, влажные незвучные мелкопузырчатые хрипы ниже углов лопаток с обеих сторон. ЧД-22 в минуту. Тоны сердца приглушены, ЧСС100 уд/мин. АД – 110/80 мм. рт. ст. Печень увеличена на 2см, умеренно болезненна. Был назначен верошпирон в суточной дозе 25 мг. Уровень какого микроэлемента необходимо контролировать в процессе лечения данным препаратом? Калия

Проверено

4. Мужчина 56 лет жалуется на сжимающие боли за грудиной, возникающие при ходьбе в среднем темпе через 200-300 метров. Вышеуказанные симптомы отмечает в течение нескольких месяцев. Обращался к врачу по месту жительства. На снятой ЭКГ вне приступа - без изменений. Для определения дальнейшей тактики ведения, какое исследование показано? Функциональная диагностика ВЭМ,тредмил тест

Проверено

5. Мужчина А., 55 лет, вызвал скорую помощь, когда возникли интенсивные боли жгучего, давящего характера за грудиной, распространяющиеся в подлопаточную область, сопровождающиеся потливостью, длительностью 25 минут, нитроглицерин под язык – без эффекта (принял 3 таблетки), боли купированы врачом скорой помощи наркотическими анальгетиками. При осмотре: состояние средней тяжести. ЧСС 58 в мин, ритм правильный, АД 110/60 мм рт.ст. ЭКГ: ритм синусовый, правильный. Подъем

сегмента ST в II, III, АVF на 3 мм, горизонтальное смещение сегмента ST вниз на 2 мм в отведениях V4-V6. Госпитализирован в отделение реанимации. Какое исследование следует провести для дальнейшей тактики ведения? селективная

коронароангиография

Проверено

6. Больной 60 лет, в течение 10 лет страдает артериальной гипертензией с периодическим повышением АД до 180/110мм.рт.ст. 3 года назад перенес ОИМ. При осмотре: состояние относительно удовлетворительное, АД - 160/110 мм.рт.ст., ЧСС - 68 уд/мин. При обследовании: левая граница относительной сердечной тупости сердца смещена влево, во втором межреберье справа от грудины - грубый систолический шум, проводящийся на сонные артерии. Артерии глазного дна сужены, извиты. Какой метод инструментального исследования назначите для визуализации гипертонического сердца? ЭхоКг

Проверено

7. Больной А., 55 лет, с жалобами на головные боли, шум в ушах, мелькание мушек перед глазами, обратился в поликлинику по месту жительства. В течение 12 лет страдает артериальной гипертензией с периодическим повышением АД до 190/110 мм.рт.ст. 2 года назад перенес Оcтрый инфаркт миокарда. При осмотре: состояние относительно удовлетворительное. При осмотре левая граница сердца смещена влево, АД - 180/110 мм.рт.ст., ЧСС - 70 уд/мин. На Эхо-КГ: ТМЖП-1,2 см, ТЗСЛЖ-1,3 см, ФВ

– 40%. Артерии глазного дна сужены, извиты. Какой из нижеследующий препарат является оптимальным для дальнейшего лечения?БАБ

Проверено

8. У женщины, 38 лет, со сроком 2-й беременности 36 недель, внезапно повысилось АД до 180/100 мм.рт.ст. Появились сильное головокружение, слабость, головная боль. Машиной скорой помощи доставлена в родильный дом. При обследовании у пациентки в общем анализе мочи выявлена протеинурия. Какой из ниже перечисленных препаратов показан для лечения артериальной гипертензии во время беременности? Метилдопа

Проверено

9. Мужчина 56 лет жалуется на сжимающие боли за грудиной, возникающие при ходьбе в среднем темпе через 200-300 метров. Вышеуказанные симптомы отмечает в течении нескольких месяцев. Обращался к врачу по месту жительства. На снятой ЭКГ

вне приступа - без изменений. Какой метод исследования будет наиболее информативен для уточнения диагноза? Функциональная диагностика ВЭМ,тредмил тест Проверено

10.Пациент А. 50 лет обратился к врачу по месту жительства с жалобами на боли давящего характера в области верхней трети грудины, возникающие при ходьбе на 500-

600метров, которые продолжаются около 5 мин, проходят самостоятельно или после приема нитроглицерина. В анамнезе страдает ХОБЛ. Курит на протяжении 30 лет по 20 сигарет в день. При осмотре АД - 130/80 мм рт. ст., пульс 100 удара в минуту. При обследовании: на ЭКГ в покое патологических изменений не выявлено. При проведении ВЭМ пробы выявлены признаки коронарной недостаточности ФК II. Какой антиангинальный препарат необходимо назначить больному? БМКК(амлодипин,нефедипин)

Проверено

11.У пациента А. 66 лет, перенесшего инфаркт миокарда год назад, появились одышка, отеки голеней и стоп, слабость, вялость. В легких выслушиваются влажные хрипы в нижних отделах. На ЭКГ синусовая тахикардия и очаговые изменения переднеперегородочной области ЛЖ. На Эхо-КГ полости расширены, ФВ-30%, Сист.ЛАД-40 мм.рт.ст, в перикарде-жидкость. Лечащий врач назначил мочегонные препараты, бетаблокаторы. Какую группу лекарственных препаратов необходимо добавить в качестве патогенетической терапии этого заболевания? ИАПФ

Проверено

12. Больная П., 40 лет, обратилась к врачу с жалобами на одышку, приступы удушья. Из анамнеза известно, что перенесла ревмокардит в 13 лет, по поводу чего получала бициллинопрофилактику. Объективно: резкая бледность кожных покровов. Заметно сотрясение головы в такт систоле, положительный капиллярный пульс, пульсация сонных артерий. В легких ослабленное везикулярное дыхание. Пульс скорый, высокий. Отмечается усиление и смещение верхушечного толчка влево и вниз, выслушивается мягкий диастолический шум над аортой и в точке Боткина-Эрба, ослабление II тона над аортой. АД 140/40 мм.рт.ст. Какой метод исследования подвердит диагноз? ЭхоКГ

Проверено

13. Больная, 42 лет, наблюдается онкологом по поводу рака левой молочной железы; в течение последнего года трижды проводились курсы лучевой терапии. В течение последнего месяца прогрессирует одышка, усиливающаяся при незначительной нагрузке, появились признаки асцита. Объективно: в нижних отделах легких с обеих сторон крепитирующие влажные хрипы, верхушечный толчок не определяется.

Перкуторно границы сердца умеренно расширены во все стороны. АД 110/70 мм рт.ст. ЧСС 80/мин, пульс парадоксальный. ЭКГ: низкий вольтаж зубцов, отрицательный зубец Т практически во всех отведениях. Какими препаратами следует начать терапию данного заболевания? Диуретики

Проверено

14. Больная Ж., 30 лет. Обратилась к врачу с жалобами на одышку, резкие боли в эпигастрии слева, слабость. Объективно: цвет кожи – «кофе с молоком», единичные геморрагии на предплечье. Температура субфебрильная. Сердце увеличено в размерах влево. Диастолический шум во 2 межреберье справа и в точке Боткина. Пальпируется увеличенная печень и селезенка. Анализ крови: Нb 60 г/л, эритр 2,8*1012/л, лейк 4,0*109/л, СОЭ 60 мм/час. Анализ мочи: относительная плотность 1020, белок 0,66%, в осадке единичные свежие и выщелоченные эритроциты, единичные лейкоциты. При бактериальном посеве крови высеяли золотистый стафилококк. Сочетаниям каких антибиотиков при лечениии вы отдадите предпочтение? Пенициллин(или препарат группы пенициллина)+гентамицин, Ванкомицин

Проверено

15. Больная Х., 30 лет. Обратилась к врачу с жалобами на одышку, резкие боли в эпигастрии слева, слабость. Объективно: цвет кожи – «кофе с молоком», единичные геморрагии на предплечье. Температура субфебрильная. Сердце увеличено влево. Диастолический шум во 2 межреберье справа и в точке Боткина. Печень увеличена на 2 см. Селезёнка увеличенная, болезненная. Анализ крови: Нb 60 г/л, эритр 2,8*1012/л, лейк 4,0*109/л, СОЭ 60 мм/час. Анализ мочи: относительная плотность 1020, белок 0,66%, в осадке единичные свежие и выщелоченные эритроциты, единичные лейкоциты. Выявили возбудителя: зеленящий стрептококк. С каких препаратов необходимо начинать антибактериальную терапию? Ванкомицин

Проверено

16. Больная Х., 35 лет. Обратилась к врачу с жалобами на повышение температуры, сопровождающиеся проливными потами, одышку при незначительной физической нагрузке, боль в правом подреберье. Два месяца тому назад после экстракции зуба стала эпизодически повышаться температура тела. При осмотре: состояние больной средней тяжести. Анализ крови: Нв 100 г/л, лейк 12,6*109/л, нейтрофилы 86%, СОЭ 56 мм/час, СРБ +++, титры противострептококковых антител не повышены. Общий белок 82 г/л, альбумины 40%, глобулины 60%, α110%, α212%, β- 14%, - 22%. При посеве крови дважды высевался зеленящий стрептококк, чувствительный к пенициллину, ампициллину, линкомицину. Какие препараты будете назначать? Пенициллин+гентамицин

Проверено

17. У мужчины 58 лет, находящегося в кардиологическом отделении по поводу трансмурального переднего распространённого инфаркта миокарда, на 3-й неделе заболевания, после выписки из стационара появились боли в грудной клетке, повышение температуры тела повысилась до 38 С. При осмотре: Над сердцем в области абсолютной тупости выслушивается шум трения перикарда. В ОАК: лейкоциты 17,2 х 109/л, эозинофилы- 8%, СОЭ – 34 мм/час. Предварительный диагноз: Синдром Дресслера. Препараты какой группы необходимо добавить к лечению данного пациента: ГКС,Преднизолон

Проверено

18.Больной 49 лет поступил в реанимацию с диагнозом: острый задний инфаркт миокарда. В первые сутки наблюдения внезапно потерял сознание. Пульс и давление не определяются. На ЭКГ: синусовый ритм с переходом в асистолию желудочков. Ваша тактика:Адреналин

Проверено

19.В реанимацию поступил больной 70 лет с жалобами на интенсивную боль за грудиной, длительностью 6 часов, холодный липкий пот, чувство страха смерти. Больной в ступоре, кожа с землистым оттенком. АД 60/40 мм рт.ст, пульс нитевидный. Предварительный диагноз: Кардиогенный шок. Ваша тактика лечения: Обезболивание,оксигенотерапия, инотропные средства(допамин,добутамин)

Проверено

20.Мужчина 68 лет госпитализирован в кардиореанимацию с диагнозом острый инфаркт миокарда. Жалобы при поступлении на одышку при незначительных физических нагрузках, приступы удушья при переходе в горизонтальное положение, кашель с отхождением розовой мокроты. Объективно: Положение ортопноэ. В легких дыхание жесткое, выслушиваются влажные мелкопузырчатые хрипы. Какой из мочегонный препаратов необходимо назначить пациенту учитывая развитие острой левожелудочковой сердечной недостаточности: : Фуросемид

Проверено

21.Мужчина 63 лет, 10 дней назад перенес острый трансмуральный инфаркт миокарда. Находится на стационарном лечение в кардиологическом отделение. Объективно: состояние тяжелое. Кожные покровы: акроцианоз губ. Аускультативно: тоны сердца

приглушены. Левая граница сердца отклонена влево на 1 см кнаружи от СКЛ и вниз VI м/р. ЧСС 58 уд. АД 100/60. На ЭХОКГ – фракция выброса 30%. Для предотвращения периферического тромбомбооброзования, на данном этапе реабилитации, что необходимо применить: Клопидогрел+аспирин

массаж нижних конечностей

Проверено

22. Мужчина 19 лет поступил в клинику по направлению военкомата. При обследовании были выявлены высокие цифры АД 200/120 мм рт.ст. пациент отмечает, что и раньше определялись высокие цифры АД. Объективно: хорошо развиты мышцы верхней половины туловища. Усиленная пульсация сосудов верхней половины туловища. Тоны сердца ритмичные. Над областью сердца и со спины между лопатками выслушивается систолический шум. АД на руках 190/110 мм рт.ст. на ногах 105/60 мм рт.ст. ЧСС 84 в минуту. Пульсация на руках отчетливая, на ногах ослабленная. Предварительный диагноз: коартация аорты. Какой метод диагностики необходим для постановки диагноза: ЭхоКГ

Проверено

23. Мужчина 64 лет обратился к врачу по месту жительства с жалобами на головные боли, головокружение, шум в ушах, повышение АД до 210/120 мм рт. ст. Из анамнеза: высокие цифры АД в течение 2 лет. На фоне постоянного приема метопролола, гипотиазида понижения АД не отмечалось. Объективно: Липидная дуга роговицы, ксантомы на лице. Над легкими: дыхание везикулярное, хрипов нет. Тоны сердца приглушены, ритмичные. Акцент II тона над аортой. Пульс - 76 уд. в мин. АД - 196/116 мм рт. ст. Над брюшной аортой выслушивается систолический шум. Предварительный диагноз: Вазоренальная артериальная гипертензия. Выберите наиболее информативный инструментальный метод диагностики: рентгеноконтрастная ангиография, ультразвуковое дуплексное исследование почечных артерий

Проверено

24. Женщина 59 лет поступила в стационар с жалобами на удушье, стеснение в груди, сухой кашель. Из анамнеза: 4 года страдает гипертонической болезнью, систематически не лечится. Вышеперечисленные жалобы появились впервые. Объективно: состояние тяжелое. Положение ортопное, разлитая бледность кожных покровов, акроцианоз. Дыхание шумное, ЧД 40 в мин. В легких по всем полям разнокалиберные влажные хрипы. Пульс 120 уд. в мин., ритмичен, напряжен. Левая граница сердца смещена влево на 2 см. Тоны сердца ритмичные. АД 180/120 мм рт. ст. У больной гипертонический криз, осложненный острой сердечной недостаточностью.

Какой из нижеперечисленных гипотензивных парентеральных препаратов предпочтительнее использовать в данном случае? Фуросемид,эналаприлат +нитроглицерин

Проверено

25. У мужчины 28 лет отмечается постоянно высокое артериальное давление с частыми гипертоническими кризами, когда АД повышается до 230/110 мм. рт. ст. Криз развивается внезапно, появляется сердцебиение, перебои в работе сердца, боль за грудиной, слабость, дрожание рук, потливость, головная боль, боль в животе, частое обильное мочеиспускание в конце криза. Каким препаратом необходимо внутривенно купировать гипертонический криз, который связан с феохромоцитомой: Фентоламин,доксазозин

Проверено

26. Мужчина 30 лет на приеме у врача по месту жительства отмечает сердцебиение и одышку при умеренных физических нагрузках. Анамнез: с детства частые ангины. Объективно: Кожные покровы обычной окраски, отеков нет. Сердечные тоны ритмичные, на верхушке I тон хлопающий, диастолический шум. ЧСС – 90 д в мин. АД

– 110/70 мм рт. ст. Какую площадь митрального отверстия вы ожидаете увидеть на ЭхоКГ, учитывая что у пациента стеноз левого a-v отверстия: менее 4 см, симптомы при стенозе менее 2см

Проверено

27. Молодой человек, 26 лет обратился к врачу по месту жительства с жалобами на одышку и сердцебиение при умеренной физической нагрузке. Анамнез: с детства частые ангины. Объективно: Дыхание жесткое, хрипов нет. Верхушечный толчок в Vмежреберье на 2 см кнаружи от средне-ключичной линии слева. Аускультативно: I тон на верхушке ослаблен, систолический шум. Предварительный диагноз: Недостаточность митрального клапана. Какой метод диагностики необходимо назначить для уточнения диагноза: ЭхоКг

Проверено

28. Мужчина 48 лет обратился к врачу по месту жительства с жалобами на кратковременные потеря сознания, боли в области сердца давящего характера с

иррадиацией в левую руку, возникающие при физической нагрузке. Из анамнеза: в детстве частые ангины. Аускультация: во II м/р справа II тон ослаблен, там же выслушивается систолический шум, который проводится на сосуды шеи. Во II м/р справа также выслушивается диастолический шум, который проводится в точку Боткина-Эрба. АД 120/70 мм рт.ст. Предварительный диагноз: Стеноз устья аорты в сочетании с недостаточностью аортального клапана. Какой метод инструментальной диагностики необходимо назначить для уточнения диагноза: ЭхоКг

Проверено

29. Мужчина 56 лет, страдает артериальной гипертензией. Из анамнеза: последние 2 года периодически повышалось АД до максимальных цифр 180/90 мм рт.ст. При физической нагрузке периодически возникал дискомфорт в области сердца, одышка. Курит более 30 лет. Анамнез: отец умер от инфаркта миокарда в 60 лет. При поступлении проведена коронароангиография, обнаружена окклюзия передней межжелудочковой артерии, выполнено чрезкожное вмешательство: стентирование передней межжелудочковой артерии. В анализах: общий холестерин - 6,36 ммоль/л, ЛПНП - 3,69 ммоль/л, ЛПВП - 1,25 ммоль/л, триглицериды - 3,26 ммоль/л. Какую группу лекарственных препаратов необходимо назначить пациенту для нормализации уровня холестерина? Статины

Проверено

30. Больной А. 40 лет обратился к врачу по месту жительства с жалобами на сжимающие боли в грудной клетке, возникающие как при физических, так и при эмоциональных нагрузках. В анамнезе с юношеского возраста страдает сахарным диабетом I типа. В течение последнего полугода стали беспокоить вышеперечисленные жалобы. На серии снятых электрокардиограмм зафиксирована депрессия сегмента ST в отведениях V4-V6 глубиной 1,0мм, продолжительностью 0,08 сек., проба с нитроглицерином положительная. Выберите препараты, которые бы вы рекомендовали больному для лечения данного заболевания? Нитраты длительного действаи, БАБ

Проверено

31. Больной А., 59 лет, обратился к врачу с жалобами на приступы болей давящего характера, возникающие за грудиной, отдающиеся в левую руку, под левую лопатку, в левую ключицу. Продолжительность болей 10 мин. Боли возникают при ходьбе по комнате, при дефекации, при переедании, проходят после приема нитроглицерина через 3-5 мин. Беспокоят около 3 месяцев. Объективно: температура 36,60С. Общее состояние удовлетворительное. Кожа чистая. Подкожно-жировая клетчатка развита избыточно. Дыхание везикулярное, ЧД 16 в мин. Тоны сердца ритмичные, выслушивается систолический шум над аортой. ЧСС 84 в мин., АД 130/80 мм.рт.ст. Какие анализы

крови необходимо назначить для уточнения диагноза? Липидный спектр, свертываемость, тропонины

Проверено

32. Больной 58 лет, строитель, жалуется на жгучие боли в области сердца с иррадиацией в левую руку, возникающие при ходьбе медленным шагом на расстояние 500 метров, из-за чего он вынужден останавливаться. Боли проходят через 1-2 мин. после приема НТГ. Из анамнеза: болен в течение 12 лет, принимал НТГ, курит по 1 пачке в день 20 лет. Объективно: повышенного питания. Тоны приглушены, систолический шум на верхушке, ритм правильный, ЧСС 60 уд/мин, АД 130/80 мм Нg. Из обследований: ОАК-

HB 150 г/л, эр. 5,1х10^9/л, ЦП 0,9, лейкоциты 10,2х10^9/л, п-2%, с-62%, л-32%, м-4%,

СОЭ- 7 мм/ч. Общий холестерин крови- 6 ммоль/л. Сахар крови натощак - 5,2 моль/л. Какой метод исследования будет наиболее информативен для уточнения диагноза? Тесты функциональной диагностики, экг

Проверено

33. Больной Г., 48 лет, жалуется на резкие сжимающие боли за грудиной, которые распространяются в левое плечо и руку. Боли длятся 40 мин. Подобные боли возникли впервые, по дороге на работу. Курит. Объективно: кожные покровы бледные, влажные. Цианоз губ. Дыхание везикулярное, хрипов нет. Пульс 92 уд./мин., ритмичный, удовлетворительного наполнения. АД – 150/80 мм рт. ст. Границы сердца: правая и верхняя в норме, левая – на 1 см кнаружи от левой срединно-ключичной линии. Тоны сердца приглушены, шумов нет. Живот мягкий, безболезненный. Печень и селезенка не пальпируются. На снятой ЭКГ субэндокардиальная ишемия передне-перегородочной области ЛЖ. В какой дозе необходимо назначить клопидогрель в плановом порядке в качестве антиагрегантной терапии при данном заболевании? 75 мг

Проверено

34. Мужчина С. 59 лет доставлен бригадой скорой помощи в отделение кардиологии ЦРБ с жалобами на сильные боли в области сердца давящего характера, иррадирующие в левую руку, под левую лопатку, чувство жжения за грудиной. Приступ возник 2 часа назад. Заболевание связывает со стрессовой ситуацией на работе. Объективно: общее состояние средней тяжести, сознание ясное. Кожные покровы и видимые слизистые оболочки бледные. Дыхание везикулярное, ЧД 20 в мин. Тоны сердца ритмичные, приглушены. ЧСС 92 в мин. АД 110/70 мм рт.ст. Абдоминальной патологии не выявлено. Какое обследование является «золотым стандартом» диагностики данного заболевания? коронароагиография, исследование тропонинов в сыворотке крови

Проверено

35. Женщина П. 65 лет поступила в реанимацию в связи с интенсивными болями за грудиной, удушьем. Год назад перенесла инфаркт миокарда передней стенки левого желудочка. Чувствовала себя относительно удовлетворительно. Данные жалобы возникли внезапно среди ночи. Объективно: состояние тяжелое. Положение ортопноэ, акроцианоз. Дыхание жесткое, выслушиваются разнокалиберные влажные хрипы. Набухание шейных вен. ЧД =26 в мин., АД-110/60 мм рт ст., ЧСС120 уд/мин. Тоны сердца глухие, ритмичные. ЭКГ: патологический зубец Q в I, AVL, V1 - V3, подъем ST в I, V1 — V6 до 4 мм. Какое исследование необходимо провести с целью дифдиагностики между ОИМ и постинфактным кардиосклерозом? Тропонин

Проверено

36. Больной А. 56 лет поступил в клинику с жалобами на сжимающие боли за грудиной, сердцебиение, одышку. Из анамнеза: 15 лет страдает КБС, ухудшение самочувствия отметил около 40 минут назад. Объективно: состояние тяжелое. В легких дыхание везикулярное, хрипов нет. Границы сердца смещены влево: на 1,5 см кнаружи от срединно-ключичной линии. Тоны сердца ослаблены, ритм правильный. ЧСС -120 уд/мин. АД — 90/60 мм рт. ст. Живот мягкий, безболезненный. Печень не пальпируется. На ЭКГ: комплекс QS с элевацией сегмента ST в I, II стандартных отведениях, aVL, V1V6. Каким препаратом необходимо купировать болевой синдром при этом заболевании? Морфин, фентанил

Проверено

37. Мужчину 59 лет доставили в реанимацию в связи с интенсивными давящими болями за грудиной. Боль появилась 40 минут назад, в покое, не купировалась 2 дозами Изокет. Отмечает слабость, потливость. Ранее боли за грудиной не беспокоили, нагрузку переносил хорошо. Периодически повышалось АД до 160/100 мм рт. ст. Курит 25 лет. Объективно: состояние средней тяжести. Кожа влажная. ЧД - 18 в мин. Дыхание везикулярное. Тоны сердца приглушены, ритмичные, ЧСС – 70 уд/мин, АД – 160/100 мм рт. ст. Анализ крови на тропонин положительный. На ЭКГ острый период острого инфаркта миокарда передне-перегородочной области ЛЖ. Введение каких препаратов является наиболее эффективным для прогноза при инфаркте миокарда с подъемом сегмента ST в первые 12 часов: тромболитическая терапия

Проверено

38. Больной К., 56 лет, поступил с жалобами на дискомфорт за грудиной. В день поступления возникла интенсивная сжимающая боль за грудиной при быстрой ходьбе и не купировалась приемом нитроглицерина. В связи с чем вызвал бригаду скорой помощи, которая купировала болевой синдром промедолом и доставила пациента в стационар. При осмотре: состояние средней тяжести. Ритм сердца правильный. ЧСС 80 уд/мин. АД 132/76 мм рт. ст. В анализах крови: эритроциты 4,5х1012/л, лейкоциты

10,8х109/л, СОЭ 6 мм/час. Биохимия крови: холестерин 7,0 ммоль/л, тропонин Т – пол. На ЭКГ субэндокардиальная ишемия передне-перегородочной области ЛЖ. На фоне лечения в стационаре у больного появились сильные распирающие головные боли, в связи с чем он стал отказываться принимать все лекарства. Вероятнее всего, данные жалобы могут быть вызваны приемом: Нитроглицерина

Проверено

39. Больной Р.59 лет жалуется на жгучие боли за грудиной с иррадиацией в нижнюю челюсть. Прием 1т НТГ боль не купировал. Боль продолжалась 30 минут. 10 лет артериальная гипертония, АД до 170/90 мм рт. ст. Курит 20 лет, наследственность отягощена. Впервые отметил появление загрудинных болей после интенсивной нагрузки, боли прошли в покое. При осмотре: состояние тяжелое. ИМТ32 кг/м2. Кожные покровы бледные. Дыхание везикулярное. ЧД-22. Тоны сердца приглушены, акцент II тона на аорте, АД – 160/ 90 мм рт. ст. ЧСС – 92 уд/мин. На ЭКГ субэндокардиальная ишемия передне-перегородочной области ЛЖ. Тропониновый тест положительный. Какой антикоагулянтный препарат должен быть назначен пациенту при данном заболевании? Гепарин

Проверено

40. Больной К., 57 лет, поступил в стационар с жалобами на жгучие боли за грудиной с иррадиацией в левое плечо, продолжительностью 1,5 ч, не снимающиеся нитроглицерином, резкую слабость, липкий пот. Накануне поработал на даче. В анамнезе 5 лет приступы сжимающих болей за грудиной во время быстрой ходьбы, длящиеся 3–5 мин., проходящие в покое и приема нитроглицерина. Объективно: кожные покровы бледные, влажные, акроцианоз. Пульс 96 уд./мин., АД – 90/60 мм рт. ст. Границы сердца расширены влево. Тоны сердца глухие. Дыхание везикулярное. Какое обследование необходимо провести в первую очередь для уточнения диагноза?

ЭКГ

41. Женщина 44 лет, обратилась с жалобами на головную боль, головокружение, шум в ушах, постоянную слабость, иногда тошноту и «мелькание мушек» перед глазами. Объективно: Гиперемия лица. Отѐков не выявлено. Границы относительной сердечной тупости: правая - на 1,5 см кнаружи от правого края грудины, верхняя – III ребро, левая - по левой СКЛ. Ритм сердца правильный, тоны ослаблены, акцент II тона на аорте. ЧСС =90 уд/мин. АД 180/100 мм рт.ст. На ЭКГ: признаки гипертрофии левого желудочка. Какой дополнительный метод обследования вы назначите данной пациентке для уточнения диагноза? ЭХОкг

Проверено

42.Больная К. 52 лет с жалуется на головную боль в затылочной области, в основном

вутренние часы, периодически тошноту, потемнение в глазах, сердцебиение и одышку при выполнении физической работы, по утрам небольшие отеки на лице. Подобные проявления были у ее матери, которая умерла 4 года назад от инсульта. Объективно: состояние удовлетворительное. Кожные покровы несколько бледноваты, веки пастозны. ЧД 20 в минуту, пульс 80 ударов в минуту, полный ритмичный. АД 160/110 мм рт.ст. Тоны сердца ритмичны, несколько приглушены. Левая граница сердца на 1 см кнаружи от левой среднеключичной линии. Пастозность голеней. Больной назначен эналаприл. Какой препарат лучше добавить для достижения хорошего гипотензивного эффекта? Тиазидные диуретики

Проверено

43.Мужчина М. 32 лет жалуется на жгучие боли в области сердца, возникающие при физической нагрузке, одышку, перебои в работе сердца. До 28 лет активно занимался спортом. В семье имелись случаи внезапной смерти среди родственников в молодом возрасте. Какое исследование показано для постановки окончательного диагноза? ЭХОКГ

Проверено

44. Мужчина М. 40 лет поступил в стационар с клиникой тромбоэмболии легочной артерии. При обследовании выявлены: увеличение размеров сердца, тоны глухие, ритм неправильный, мерцательная аритмия, ЧСС 100-120 в минуту, дефицит пульса 25 в минуту, систолический шум на верхушке, гепатомегалия, отеки на стопах и голенях. На ЭхоКГ: выраженная дилатация полостей сердца, клапаны интактны, диффузный гипокинез всех стенок ЛЖ. Предварительно поставлен диагноз: идиопатическая дилатационная кардиомиопатия. Какое обследование показано для дифференциальной диагностики с КБС? Рентгеноконтрастирование

Проверено

45. Пациенту при обследовании выставлен диагноз: идиопатическая гипертрофическая кардиомиопатия. Какая группа лекарственных препаратов является средством выбора в лечении данного больного? БАБ(бисопролол,атенолол,пропроналол)

Проверено

46. Больной А. 34 лет доставлен в стационар с жалобами на учащенное сердцебиение, возникшее внезапно, сопровождающееся выраженной слабостью. Объективно: больной бледен. Аускультативно: тоны сердца резко ослаблены, ЧСС - 200 уд.в мин. АД - 120/80

мм рт. ст. На ЭКГ: ширина комплексов QRS - 0,08 мс, интервалы RR одинаковы 0,3 с. С какого препарата целесообразнее начать неотложную помощь? Аденозин

47.У больного А. 65 лет в первые сутки острого инфаркта миокарда отмечается частая политопная желудочковая экстрасистолия, АД 100/60 мм рт.ст. Препаратом выбора в данной ситуации является?БАБ

48.У больного К. 58 лет, который страдает стенокардией напряжения, появились перебои в работе сердца. В анамнезе год назад перенес инфаркт миокарда. Объективно: ЧСС 86 в мин., АД 160/100 мм рт. ст. На ЭКГ зарегистрированы периодическое появление внеочередных деформированных расширенных комплексов QRS по типу тригеминии. Какой группе антиаритмических средств следует отдать предпочтение при длительном лечении больного?БАБ

Проверено

49. Пациент С., 46 лет, находится в отделении ургентной кардиологии с диагнозом: «Острый инфаркта миокарда». Пациент подключен к круглосуточному кардиомонитору. В случае возникновения осложнений и регистрации фибриллации желудочков на ЭКГ мониторе, с какой силой тока необходимо проводить кардиоверсию? начинать с 200-360дж

Проверено

50. Больной В. 69 лет доставлен в приемное отделение больницы с жалобами на жгучие боли за грудиной, сопровождавшиеся чувством нехватки воздуха, холодным липким потом. Со слов родных выяснено, что вышеперечисленные жалобы возникли внезапно час назад. Во время осмотра врача внезапно потерял сознание. Объективно: Кожные покровы бледные, акроцианоз. Дыхание отсутствует. На крупных сосудах отсутствует пульсация. Расширены зрачки. АД не определяется. ЭКГ: хаотически неправильный ритм, комплексы QRS и зубцы Т отсутствуют. Что необходимо провести для оказания помощи пациенту?

1.Радиочастотную аблацию аритмогенных очагов

2.Внутриаортальную баллонную контрпульсацию

3.Имплантацию временного электрокардиостимулятора

4.чреспищеводную электрическую стимуляцию предсердий

5.Экстренную кардиоверсию

51. Больной А. 75 лет на приеме у врача предъявляет жалобы на приступы головокружения, иногда с кратковременной потерей сознания, участившиеся в течение

последнего месяца. Анамнез: больным себя считает около 10 лет, когда впервые появились сжимающая боль в области сердца с одышкой при ходьбе до 200 м, боль купирующаяся Нитроглицерином. Год назад впервые возник приступ потери сознания в течение нескольких минут, с мочеиспусканием. В последний месяц аналогичные приступы участились. Объективно: сознание ясное. Тоны сердца глухие, ритмичные. Временами выслушивается громкий (пушечный) I тон. ЧСС - 34 удара в минуту. АД - 180/100 мм рт. ст. Какой метод лечения данного состояния является наиболее эффективным?

пост ЭКСпроверено

52.Женщина А. 73 лет обратилась к врачу по месту жительства с жалобами на общую слабость, утомляемость, головные боли, головокружение, периодические потери сознания. Из анамнеза: данные симптомы беспокоят в течение месяца, ранее не обследовалась, не лечилась. Объективно: ЧСС 36 ударов минуту. АД – 100/60мм.рт.ст. На ЭКГ: полная A-V блокада III cтепени, дистальный тип. Какой будет ваша тактика лечения?пост ЭКС

53.Больной А. 65 лет поступил в клинику с диагнозом: «Острый инфаркт миокарда нижней стенки ЛЖ». На ЭКГ на фоне типичных признаков ОИМ нижней стенки ЛЖ выявлено полное разобщение предсердного и желудочкового ритмов, интервалы P-P и R-R постоянны, ЧСС предсердий – 72 уд/мин, ЧСС жел. – 40 уд/мин. Какой препарат показан в данной ситуаци Атропин,реперфузионная терапия, при неэффективности

> 7днейвременная экс проверено

54.Женщина А. 63 лет, поступила в отделение реанимации с жалобами на одышку и кашель с отделением розовой пенистой мокроты. Объективно: вынужденное положение тела, цианоз, над всей поверхностью легких выслушиваются влажные хрипы. I тон ослаблен, систоло–диастолический шум с пресистолическим усилием на верхушке сердца. ЧСС – 130 в мин., АД 115/80 мм.рт.ст. Какой дополнительный метод обследования вы назначите данной пациентке? Рентген гк, экг, эхо-кг проверено

55.Мужчина А. 68 лет поступил в отделение реанимации с жалобами на удушье, кашель с отделением розовой пенистой мокроты. В анамнезе страдает гипертонической болезнью в течение 20 лет с максимальным повышением АД до 200/100 мм.рт.ст. В последнее время лечения не получал. Данное состояние развилось внезапно после психо-эмоционального стресса. Объективно: положение ортопноэ, акроцианоз, над всей поверхностью легких выслушиваются множественные средне-крупнопузырчатые влажные хрипы. Тоны сердца приглушены, ритм правильный, ЧСС – 125 в мин., АД 240/120 мм.рт.ст. Применение какого мочегонного препарата необходимо для купирования острой сердечной недостаточности? Фуросемид проверено

56. Мужчина 48 лет на приеме у семейного врача жалуется на появление сжимающих болей за грудиной, с иррадиацией в левую руку, возникающие при ходьбе на 200-300 метров в спокойном темпе. Боли проходят при прекращении нагрузки. Из анамнеза: Вышеуказанные жалобы беспокоят в течение недели. Факторы риска: курение, артериальная гипертензия. Какой метод инструментальной диагностики подтвердит Ваш диагноз: функциональные нагрузочные пробы(велоэргометрия,

тредмил,проба с добутамином, эргометрином, дипирадомоловый тест); чпэс; экг по холтеру

Проверено

57.Мужчина 65 лет, находящийся на стационарном лечении 2 недели назад по поводу подтвержденного инфаркта миокарда, обратился с жалобами на субфебрилитет, боль в грудной клетке колющего характера, которая связана с дыхательными движениями и усиливается на высоте вдоха. Объективно: состояние средней тяжести. При аускультации легких - без патологии, сердца – шум трения перикарда. ЧСС-80 в мин. АД 120/80. Живот мягкий, безболезненный. На ЭКГ – подъем сегмента ST до 1,5 мм в отведениях V1-V5. Какую группу препаратов необходимо назначить данному пациенту:

-ацетилсалициловая кислота (150–350 мг 4 раза в сутки)

-другие НПВС (ортофен, ибупрофен, диклофенак, мовалис и др.) - глюкокортикоиды (преднизолон, метипред и др.)проверено

58.Больной 53 лет, поступил в отделение с жалобами на головную боль в теменнозатылочной области, головокружение, при контакте с пыльцой растений приступы удушья. Болен в течение 2-х месяцев. Из анамнеза: курит с 20 лет, состоит на учете у терапевта по поводу бронхиальной астмы. Объективно: повышенного питания. В легких дыхание жесткое. Пульс - 90 в 1 мин., ритмичный, напряженный. АД на обеих руках - 160/100 мм рт. ст. Тоны сердца на верхушке приглушены, акцент II тона на аорте. Со стороны органов брюшной полости - без патологических изменений. Какой гипотензивный препарат противопоказан пациенту:БАБ, ИАПФ (вызывают кашель)

Проверено

59.Мужчина 26 лет, аспирант, обратился к врачу с жалобами на головные боли. В 19летнем возрасте впервые случайно было зафиксировано повышение АД до 180/100 мм рт.ст. Анализы крови и мочи, УЗИ почек и надпочечников, УЗИ щитовидной железы без патологии. Объективно: АД = 200/110 мм рт.ст. ЧСС 78 уд/мин., ритмичные. При аускультации выслушивается систолический шум с максимумом в межлопаточной области. АД на руках 160/100 мм рт.ст., АД на ногах 110/70 мм рт. ст. ЭКГ: отклонение ЭОС влево, признаки ГЛЖ. ЭхоКГ: патологических токов не выявлено. Какой метод исследования подтвердит диагноз:

рентгенография ОГК проверено

60.Мужчина 40 лет обратился к врачу с жалобами на одышку при умеренных физических нагрузках, общую слабость. Из анамнеза: в детстве частые ангины. При осмотре отмечается facies mitralis. Периферических отеков нет. В легких везикулярное дыхание, хрипов нет. Границы сердца смещены вверх и вправо. Ритм сердца

правильный, ЧСС 86 в 1 мин, на верхушке «хлопающий» I тон, там же диастолический шум, акцент II тона на легочной артерии. Какой метод диагностики необходимо назначить для уточнения диагноза: эхо-кг Проверено

61.Мужчина 38 лет на приеме у кардиолога, предъявляет жалобы на слабость, потливость, учащенное сердцебиение. Считает себя больным около месяца. Страдает тонзиллитом с частыми обострениями. 2 месяца назад удалил 2 зуба. При осмотре: Кожные покровы бледные влажные. Температура 37.0, ЧД - 20 в мин. ЧСС 110 в мин. АД 150/40 мм рт.ст. Аускультативно: во II межреберье справа резко ослаблен 2 тон, убывающий диастолический шум cлева от грудины. ОАК: Эр. - 4,5*1012 /л, Лейк. – 11*109 /л, п/я -8. с/я –57% , СОЭ - 43 мм/час. СРБ ++. Какой инструментальный метод диагностики подтвердит диагноз: эхо-кг на наличие вегетаций (м-модальное

исследование)

Проверено

62.Мужчина 40 лет обратился к врачу с жалобами на ноющие боли в области сердца на протяжении 2 дней, сердцебиение и одышку при физической нагрузке. 2 недели назад переболел острым респираторным заболеванием. ОАК: 10,3х109/л, СОЭ 38 мм/час, гаммаглобулины 23 %, СРБ - «+++». ЭКГ – ритм несинусовый, неправильный. ЧСС 112 в минуту. ЭхоКГ: дилатация желудочков и предсердий, диффузный гипокинез миокарда, ФВ - 32 %, митральная регургитация II степени. Какой из нижеперечисленных методов исследований является “золотым стандартом” диагностики для подтверждения диагноза:

эндомиокардиальная биопсия (ЭМБ)проверено

63.Мужчина 45 лет на приеме у кардиолога жалуется на тупые, колющие боли в прекардиальной области, с иррадиацией в трапециевидную мышцу, усиливающие после кашля, при глубоком вдохе. Объективно: В легких дыхание ослаблено в нижних отделах. Верхушечный толчок не определяется. Границы сердца расширены во все стороны. Пульс слабого наполнения и напряжения, на вдохе наполнение пульса уменьшается. Тоны сердца приглушены. АД90/60 мм рт.ст. ЧСС 80/мин. Печень увеличена на 5 см. Какое исследование показано для подтверждения диагноза:

эхо-кг, пункция полости перикардапроверено

64.Мужчина 45 лет, поступил в стационар с жалобами на сердцебиение, боли в сердце. Заболел остро: за 3 часа до поступления возник приступ жгучих болей за грудиной, вскоре появилось учащенное сердцебиение. Больной был госпитализирован. Объективно: состояние тяжелое, бледен, небольшой цианоз губ. Конечности холодные. Тоны сердца глухие, ритм правильный, тахикардия - 180 ударов в минуту, пульс слабый, дефицита пульса нет, АД 90/60 мм рт.ст. По остальным органам и системам без изменений. На ЭКГ: ритм несинусовый, правильный, ширина комплекса QRS 0,09 сек, ЧСС 180 в мин. Какой препарат необходимо назначить для неотложной помощи:

Аденозин 6-12 мг в/в болюсно

65.Женщина 64 лет поступила в стационар с жалобами на одышку при подъеме на

3этаж. Из анамнеза: в возрасте 55 лет перенесла острый инфаркт миокарда. Объективно: Дыхание везикулярное, хрипов нет. Левая граница сердца в V межреберье на 1,5 см кнаружи от левой СКЛ. При аускультации сердца тоны ослаблены, шумов нет. ЧСС - 86 удара в минуту, АД 125/80 мм рт.ст. Живот мягкий, безболезненный. Печень

не увеличена. При обследовании: ЭхоКГ: КДРЛЖ – 6,0 см. Фракция выброса 42%. Зоны акинеза в области перенесенного инфаркта. Какой препарат необходимо назначить в качестве базисной терапии сердечной недостаточности:БАБ Проверено

ЛОР

Запоминание:

1. Что используют для исследования обонятельной функции носа?

Одоривекторы, ольфактометрия

2. Пальцевое исследование носоглотки производят при?

Аденоидных вегетациях

3. Какая операция является основной при отосклерозе?

Стапедопластика

4. Крупной артерией полости носа является?

Клиновидно-небная артерия

5. Через какое время необходимо удалить задний тампон при носовых кровотечениях?

48-72 часа (3-4 сутки)

6.Ангиома носа наиболее часто локализуется?

Вобласти носовой перегородки

7.Где производится пункция гайморовой пазухи?

Внижнем носовом ходе, отступя от ее переднего конца на 1,5-2см

8.Укажите доброкачественные новообразования носоглотки?

Юношеская ангиофиброма

9. В каком квадранте барабанной перепонки делается парацентез?

Нижним заднем квадранте

10. Укажите, какой хрящ является фундаментом гортани?

Перстневидный

11. Чем заполнен перепончатый канал?

Эндолимфой

12. Какое анатомическое образование отвечает за звуковосприятие?

Кортиев орган (улитка)

Понимание

1. Больной М. 32 года обратился к ЛОР клинику с жалобами на распирающие боли в носу и затруднение носового дыхания. Со слов 3 дня назад получил удар в область носа. При передней риноскопии носовые ходы резко ссужены за счет воспалительного отека слизистой оболочки в передних отделах носовой перегородки с обеих сторон, носовое дыхание затруднено. При зондировании отмечается симптом флюктуации, болезненность. Какой из перечисленных диагнозов повреждения перегородки носа наиболее вероятен?

Гематома перегородки носа. Либо уже абсцесс

2. Больная В. 35 лет, обратилась в приёмный покой с жалобами на боли и припухлость в области правой ушной раковины. Со слов больной пять дней назад случайно поцарапала ушную раковину булавкой. Через три дня, появилось покраснение кожи ушной раковины, она увеличилась в размерах, опухла, появилась головная боль. Состояние больной постепенно ухудшалось, появилась температура. При осмотре кожа ушной раковины резко гиперемирована, инфильтрирована. Процесс распространяется на околоушную область, кожу слухового прохода. Гиперемированные участки окружены линией демаркации.

Какой из перечисленных диагнозов наиболее вероятен?

Рожистое воспаление

3. Больная Г. 30 лет, обратилась к ЛОР врачу в поликлинику по месту жительства с жалобами на снижение слуха, шум в правом ухе после насморка. При отоскопии барабанная перепонка втянута, серого цвета, рукоятка молоточка укорочена по ходу его определяется инъекция сосудов. Слизистая полости носа гиперемирована, отёчна, нижние носовые раковины гипертрофированы. Шепотная речь на правое ухо 2 метра. Опыт Ринне отрицательный, латерализация звуков в больное ухо.

Какой из перечисленных диагнозов наиболее вероятен?

Острый тубоотит справа синоним евстахиит

4. Больная М. 30 лет, обратилась в ЛОР клинику с жалобами на снижение слуха и шум в ушах. Со слов больной следует, что слух снижался постепенно, но после родов снижение слуха стало более ощутимым. Отмечает странное явление, когда в шумной обстановке слышит лучше. При осмотре ЛОР органов видимой патологии не обнаружено. При отоскопии слуховые проходы широкие, серных масс не выявлено, барабанные перепонки с обеих сторон бледные как бы истончены. Слуховая функция нарушена с обеих сторон, но субъективно больная ощущает большее снижение слуха справа. Какой из перечисленных диагнозов наиболее вероятен?

Отосклероз тимпональная форма

5. Больной С. 25 лет обратился в ЛОР клинику с жалобами на гнойные выделения из уха, периодические головные боли. Страдает хроническим отитом с детства, отмечает периодические обострения, лечился не регулярно. К врачу не обращался. При отоскопии правого уха в слуховом проходе небольшое количество гноя. После туалета обнаружен краевой дефект барабанной перепонки в задне-верхнем квадранте, заполненный грануляциями. Барабанная перепонка утолщена, мутная, серого цвета, с небольшим краевым дефектом, и утолщенным, инфильтрированным краем. Со стороны других ЛОР органов, выраженной патологии не выявлено.

Какой из перечисленных диагнозов наиболее вероятен?

Хронический гнойный эпитимпанит

6. Ребенок 10 лет доставлен родителями в ЛОР стационар с жалобами на гноетечение из уха, боли и припухлость в заушной области справа. Жалобы появились около 3-х недель назад. Несмотря, на проводимое лечение гноетечение сохранялось, но боли купировались. При осмотре, в правом слуховом проходе обильное гнойное отделяемое, выявлена перфорация в мезотимпануме, определяется нависание задневерхней стенки слухового прохода в костном его отделе. При перкуссии заушной области справа отмечает болезненность.

Какой из перечисленных диагнозов у больного наиболее вероятен?

Острый гнойный средний отит, осложненный мастоидитом

7. Больная И. 30 лет обратилась в приёмный покой с жалобами на резкие, нестерпимые боли в правом ухе. Со слов больной заболела около 4 - 5 дней назад, заболевание началось с переохлаждения. Общее состояние не страдает, активна, температура не повышалась. При осмотре ушная раковина не изменена, но отмечается резкая болезненность при попытке оттянуть ушную раковину. Слуховой проход сужен, в

просвете его - слизисто-гнойное отделяемое. Больной с трудом удалось завести в правый слуховой проход только узкую воронку.

Какой из перечисленных диагнозов у больной наиболее вероятен?

Диффузный наружный отит? Да

8. Больной М. 40 лет, обратился к ЛОР врачу в поликлинику с жалобами на снижение слуха, звон и шум в ушах. В анамнезе перенёс ОРВИ и пневмонию, по поводу которой получил антибактериальную терапию. После лечения появился периодический звон и шум в ушах, позже - отметил снижение слуха. Отоскопия: барабанные перепонки серого цвета, слегка втянуты, опознавательные знаки сохранены. При исследовании слуха шёпотная речь на оба уха 3 метра, страдает разборчивость. Акуметрия: проба Ринне положительная, проба Швабаха укорочена с обеих сторон. латерализация в пробе Вебера отсутствует.

Какой из перечисленных диагнозов у больного наиболее вероятен?

Двухсторонняя сенсоневральная тугоухость

9. Больной 19 лет, обратился в ЛОР клинику с жалобами на головную боль, усиливающуюся при наклоне головы, гнойные выделения из носа в течении 3 недель. Объективно: болезненность при пальпации клыковой ямки, а также при перкуссии зубов, припухлость щеки справа. При риноскопии – отек и гиперемия слизистой оболочки полости носа, а также гнойные выделения в среднем носовом ходу справа. На R- грамме околоносовых пазух определяется диффузное понижение прозрачности гайморовой пазухи справа. На R-графии верхней челюсти отмечается - воспаление альвеолярного отростка и 6 зуба, в области дна верхнечелюстной пазухи справа.

Какой из перечисленных диагнозов наиболее вероятен?

Одонтогенный гайморит справа

10. Больной Т. 28 лет, обратился в приемный покой с жалобами на постоянную заложенность носа, которая купируется только после закапывания сосудосуживающих капель в нос. Капли использует в течение года. В последнее время с небольшим эффектом. При осмотре носовая перегородка по средней линии, патологического отделяемого в полости носа нет. Отмечается гиперемия, отек, гиперплазия и инфильтрация слизистой нижних носовых раковин, в связи с чем носовые ходы резко ссужены, носовое дыхание затруднено. Адреналиновая проба отрицательная. На рентгенограмме околоносовых пазух патологии не выявлено. Предположите, какой ринит у больного наиболее вероятен?

Хронический гипертрофический ринит

11. Больной Н. 23 лет, обратился в ЛОР клинику с жалобами на затруднённое носовое дыхание и головную боль. В анамнезе - получил травму носа около недели назад. Кровотечения из носа не было. Сознания не терял. Заложенность носа наросла постепенно, в течение нескольких часов после полученной травмы носа. Общее состояние больного удовлетворительное. При осмотре спинка носа по средней его линии пальпаторно определялась умеренная болезненность, однако крепитации не отмечалось. При передней риноскопии носовая перегородка симметрично утолщена в области преддверия носа, раковины и полость носа необозримы, носовое дыхание у больного отсутствует. Предположите, какой из перечисленных диагнозов у больного наиболее вероятен?

Гематома носовой перегородки

12. Ребенок 5 лет обратился в клинику с родителями, со слов материзатруднённое открывание левого глаза, головная боль, заложенность носа, недомогание в течение четырёх дней. Из анамнеза: заболел остро, после переохлаждения появились боли в проекции лобной пазухи и припухлость левого верхнего века слева. Обьективно: отёк и гиперемия верхнего века слева, глазная щель сужена, при пальпации левой половины лица - болезненность, при передней риноскопии слизистая оболочка левой половины носа отечная, гиперемированная, в носовых ходах - обильное гнойное отделяемое. На рентгенограмме гомогенное затемнение левой гайморовой пазухи, в левой лобной пазухе отмечается уровень жидкости.

Оцените характер риногенного глазничного осложнения у ребенка.

Реактивный отек клетчатки глазницы и век

13. На консультацию к ЛОР врачу в поликлинику обратилась женщина 25 лет с жалобами на головные боли, временами боли опоясывающего характера в области затылка, гнойные выделения из носоглотки, особенно по утрам. Жалоб на затрудненное носовое дыхание не отмечает. Объективно: слизистая оболочка полости носа умеренно гиперемирована, раковины не увеличены, носовое дыхание свободное. При осмотре глотки - обильное гнойное отделяемое по задней стенке. На обзорной рентгенограмме прозрачность околоносовых пазух сохранена. На боковой рентгенограмме околоносовых пазух определяется понижение прозрачности основной (клиновидной) пазухи. Предположите, какой из перечисленных ниже диагнозов у больной наиболее вероятен?

Гнойный сфеноидит

14. Больной 30 лет, обратился к ЛОР врачу с жалобами на снижение слуха и шум в левом ухе, а также периодические слизисто-гнойные выделения из него. Гноетечением

из уха страдает много лет. Вначале выделения из уха были постоянными, затем стали периодическими и слух не улучшился. Объективно: общее состояние удовлетворительное. Отоскопия: A/S –стойкий дефект в pars tensa барабанной перепонки. Камертональные исследования: слева опыт Ринне отрицательный, латерализация звука в левое ухо, опыт Швабаха удлинен.

Поставьте наиболее вероятный диагноз?

Хронический гнойный мезотимпатинт, краевая форма

15. Больная Г. 35 лет обратилась в ЛОР клинику с жалобами на затруднённое носовое дыхание. В анамнезе, по месту жительства перенесла операцию – полипотомию носа. Носовое дыхание восстановилось, но через три месяца полипы рецидивировали. На рентгенограмме и КТ придаточных пазух носа выявлено затемнение практически всех придаточных пазух носа. При передней риноскопии - полость носа обтурирована полипами. Носовое дыхание практически отсутствует. Предположите, какой из перечисленных ниже диагнозов у больной наиболее вероятен?

Полипозный риносинусит? Да

16. Больной М. 14 лет обратился в приемный покой с жалобами на постоянную заложенность носа, затруднённое носовое дыхание. Периодически у больного наблюдаются спонтанные носовые кровотечения, причину которых не может указать. При осмотре выявлена умеренно отёчная слизистая полости носа, с цианотичным оттенком, без патологического отделяемого. При осмотре носоглотки обнаружено бугристое образование, прикрывающее хоаны. Образование имеет темно-багровый цвет. Предположите, какой из перечисленных ниже диагнозов у больного наиболее вероятен?

Юношеская ангиофиброма носоглотки

17. Больной А. 25 лет, обратился в ЛОР клинику с жалобами на охриплость голоса, чувство першения, саднения и сухости в голе. Начало заболевания связывает с приемом холодного напитка шесть дней назад. Со слов больного заболевание началось с появления сухого кашля, затем кашля с мокротой. Самочувствие не страдало, постоянно ощущал дискомфорт при глотании. При ларингоскопии, выявлена гиперемия слизистой гортани, более выраженная в области истинных голосовых складок, с участками точечных кровоизлияний. В просвете гортани небольшое количество вязкой мокроты, местами переходящей в корки. Предположите, какой из перечисленных ниже диагнозов у больного наиболее вероятен?

Острый катаральный ларингит

18. Больной С. 30 лет, доставлен в ЛОР клинику с жалобами на резкую боль при глотании, попёрхивание жидкой пищей, повышение температуры до 38ºС. Со слов больного заболел остро, после ОРЗ, который перенёс на ногах. При непрямой ларингоскопии: слизистая гортани резко гиперемирована, отёчна, инфильтрирована в области черпало-надгортанных складок и надгортанника. За счёт инфильтрации надгортанник ограничен в подвижности. Голосовые складки утолщены, инфильтрированы, ограниченны в подвижности при фонации. Выявлена припухлость региональных лимфоузлов. Предположите, какой из перечисленных ниже диагнозов у больного наиболее вероятен?

Флегманозный ларингит.

19. Больной Д. 34 лет, доставлен в приёмный покой машиной скорой помощи, с жалобами на боли при глотании, затруднённый приём пищи, поперхивание и даже невозможность глотания слюны. В анамнезе, неделю назад, перенёс травму глотки мясной костью, после чего появились боли при глотании, которые постепенно нарастали, присоединилась охриплость голоса, появилась одышка, поднялась температура. При непрямой ларингоскопии надгортанник резко инфильтрированн, утолщен, практически неподвижен. На язычной поверхности надгортанника имеется участок выраженного выпячивания, с флюктуацией в центре. Предположите, какой из перечисленных ниже диагнозов у больного наиболее вероятен?

Абсцесс надгортанника

20. Ребёнок 5 летнего возраста, доставлен родителями в ЛОР стационар. Со слов родителей ребенок заболел остро, после перенесенного ОРВИ, внезапно среди ночи, проснулся от приступообразного лающего кашля. Ребёнок беспокоен, мечется в постели, дыхание свистящее, резко затруднённо, выражена инспираторная одышка. Определяется втяжение мягких тканей ярёмной ямки, над и подключичных пространств, эпигастральной области. Подобное состояние длилось в течение часа, после чего, появилась обильная потливость, дыхание улучшилось и ребёнок заснул. Утром проснулся почти здоровым, но сохранилась некоторая охриплость голоса. Предположите, какой из перечисленных ниже диагнозов наиболее вероятен?

Подскладочный ларингит или ложный круп

21. Больной 45 лет обратился к ЛОР врачу в поликлинику с жалобами на охриплость голоса. Является курильщиком с большим стажем, отмечает частые обострения ларингита, который развивается после переохлаждения местного (холодное питьё) и общего. При осмотре гортани выявлены участки гипертрофии слизистой оболочки в меж-черпаловидном пространстве в виде поперечного валика с зазубренными краями, выступающими в просвет гортани. Голосовые складки утолщены, подвижность их сохранена, но ограниченна, что проявляется в неполном смыкании их при фонации и

при произношении высоких звуков. Предположите, какой из перечисленных ниже диагнозов у больного наиболее вероятен?

Пахидермический ларингит (гиперпластический )

22. Больная 43 лет, обратилась в ЛОР клинику с жалобами на охриплость голоса, одышку при физической нагрузке, чувство дискомфорта в гортани. В анамнезе перенесла струмэктомию. При непрямой ларингоскопии голосовая щель свободна. Слизистая гортани не изменена, патологического отделяемого в просвете гортани нет. При фонации левая голосовая складка неподвижна, справа - подвижность сохранена. Какой из перечисленных ниже диагнозов наиболее вероятен?

Паралич левого возвратного гортанного нерва

23. Больная 25 лет, доставлена скорой помощью в приёмный покой, с жалобами на затруднённое дыхание, слабость. В анамнезе перенесла, какоето вмешательство на гортани эндоларингеально, какое точно - не знает. В настоящее время голос грубый, хриплый. Имеются проявления дыхательной недостаточности первой степени. Предварительный диагноз обострение хронического ларингита. При эндоскопии: бледно серые образования в виде тутовой ягоды на широком основании в области передней комиссуры и передней трети голосовых складок. Предположите, какой из перечисленных ниже диагнозов у больной наиболее вероятен?

Папиллома голосовых складок?

24. Больная 45 лет обратилась к терапевту в поликлинику с жалобами на боли в горле при глотании. Больная высоко лихорадит, жалуется на общую слабость, головную боль, ознобы, потливость бессонницу. При осмотре в зеве разлитая гиперемия, миндалины гипертрофированы, гнойных налетов не обнаружено. Кожные покровы и видимые слизистые бледные. Умеренно увеличена печень, селезёнка, подмышечные лимфатические узлы. В анализе крови - лейкоцитоз 112 х 109/л, преобладают миелобласты, лимфобласты, гемоцитобласты, эритроциты 1,5 х 10 1/л, гемоглобин 33 г/л, цветной показатель 1, СОЭ 20 мм/ч. Предположите, какой из перечисленных ниже диагнозов наиболее вероятен?

Миелобластный тонзиллит. Или миелолейкоз

25. Ребенок 13 лет доставлен родителями в ЛОР стационар с жалобами на частые ангины, слабость, быструю утомляемость, боли в области сердца и суставах. При осмотре: застойная гиперемия задней стенки глотки, края небных дужек гиперемированные, отечные, инфильтрированные, небные миндалины увеличенные, рыхлые, спаяны с дужками, устья лакун расширенные, в лакунах - казеозные пробки. На электрокардиограмме - нарушение межжелудочковой проводимости. Предположите, какой из перечисленных ниже диагнозов наиболее вероятен?

Хронический тонзиллит. Декомпенсированная форма

26. Больной Г. 45 лет, обратился к ЛОР врачу в поликлинику с жалобами на чувство заложенности уха слева, снижение слуха, шум в ушах (низкочастотного характера), чувство переливания жидкости в ухе, аутофонию. При отоскопии: AS - слуховой проход свободный, барабанная перепонка серого цвета слегка выбухает. Шепотную речь слышит с 3-х метров. Риноскопия: слизистая носа отечная, застойная, носовая перегородка искривлена влево, в виде костного гребня на уровне нижнего носового хода. Носовые ходы ссужены, носовое дыхание затруднено. Предположите, какой из перечисленных ниже диагнозов наиболее вероятен?

Острый экссудативный средний отит слева. Мукозный ?

27. Больная 31года, обратилась в приемный покой с жалобами на сильные боли в правом ухе, снижение слуха, временами шум. Считает себя больной в течение 3-х дней, после перенесенной острой респираторно-вирусной инфекции. При отоскопии: AD – барабанная перепонка гиперемирована, отечна, покрытая кровяными пузырьками заполненными геморрагическим содержимым. Какой из перечисленных ниже диагнозов наиболее вероятен?

Острый гриппозный средний отит справа

28. Больная 28 лет, обратилась в приемный покой с жалобами на сильную боль в ухе слева, иррадиирущую в височную и теменную область, усиливающуюся при жевании, на повышение температуры до 37,7 С. При отоскопии: AS - на передней стенке слухового прохода определяется конусовидное возвышение, кожа на его поверхности гиперемированна. В центре образования - гнойная головка, просвет слухового прохода сужен, барабанная перепонка не визуализируется. Какой из перечисленных ниже диагнозов наиболее вероятен?

Фурункул наружного слухового прохода

29. Больная 20 лет, обратилась в ЛОР клинику с жалобами на сильную боль в правом ухе, иррадиирущую в висок, заложенность уха, головную боль, повышение температуры тела до 38 градусов. Из анамнеза - больна 2 дня. Начало заболевания связывает с насморком. При отоскопии: ADслуховой проход свободен, кожа не изменена, барабанная перепонка ярко гиперемирована, инфильтрирована, выбухает в просвет наружного слухового прохода, опознавательные пункты отсутствуют. Слух на правое ухо снижен ШР2м, опыт Ринне отрицательный, латерализация звуков вправо. Какой из перечисленных ниже диагнозов наиболее вероятен?

Острый гнойный средний отит справа, доперфоративная стадия

30. Больной 40 лет, обратился с жалобами к ЛОР врачу в поликлинику на боли в правом ухе, невозможность открыть рот, резкую болезненность в ухе при жевании и припухлость за ухом, понижение слуха. Болен 5 дней, вначале был зуд в ухе, затем появились боли, которые постепенно нарастали. Стало трудно раскрывать рот, повысилась температура, снизился слух. Обьективно: общее состояние больного удовлетворительное, t – 37,2ºС, рот открывает с трудом, болезненность при надавливании на козелок и потягивании за ушную раковину. Отоскопия: A/D – наружный слуховой проход резко сужен за счет воспалительного инфильтрата на задней стенке его. Какой из перечисленных диагнозов наиболее вероятен?

Фурункул задней стенки правого наружного слухового прохода

31. Больной А. 25 лет, обратился в ЛОР-стационара с жалобами на боли в горле, повышение температуры тела до 39 градусов, выраженную слабость. Со слов больного заболел остро, начало заболевания связывает с приемом холодного напитка. В анамнезе: частые ангины. При фарингоскопии: зев ассиметричен, выражена инфильтрация задней небно-глоточной дужки, небная миндалина слева смещена кнутри и кпереди. Какой из перечисленных ниже диагнозов наиболее вероятен?

Задний паротонзилит слева

32. Больная Н. 25 лет, обратилась к ЛОР-врачу на частые ангины, субфебрилитет, периодические артралгии и боли в области сердца. При фарингоскопии: небные миндалины увеличенные, разрыхленные, спаяны с небными дужками, края дужек отечны, инфильтрированы, гиперемированы, устья лакун расширенные, в них - казеозные пробки. В подчелюстной области пальпируются увеличенные, болезненные лимфоузлы. Какой из перечисленных ниже диагнозов наиболее вероятен?

Хронический тонзиллит компенсаторная форма

33. Больной А. 45 лет, обратился к ЛОР-врачу поликлиники с жалобами на периодическую заложенность носа, которая появилась после травмы носа. Больной предъявлял достаточно характерные жалобы: если лежал на правом бокузакладывало правую половину носа, еслина левом боку, то закладывало левую половину носа. При риноскопии: слизистая оболочка полости носа отечная, застойная, местами определяется пятнистость - сизые участки с белесоватыми пятнами Воячека. Носовые ходы резко ссужены, носовое дыхание затруднено.

Какой ринит у больного наиболее вероятен?

Хронический ( нейровегетативный) вазомоторный ринит

34. Больной А.42 года, обратился к ЛОР клинику с жалобами на затрудненное носовое дыхание больше слева. При детальном осмотре выявлено сохраненное дыхание

справа, полное отсутствие дыхания слева. При передней риноскопии в левой половине носа слизистая оболочка полости носа бледно розовая, в носовых ходах - слизистое отделяемое. При задней риноскопии определяется округлое образование белесоватого цвета с гладкой поверхностью слизистой консистенции, обтурирующее левую половину носоглотки.

Какой из перечисленных ниже диагнозов наиболее вероятен?

Зональный полип носа (хоанальный)

35. Пациент 9 лет доставлен родителями в ЛОР стационар с жалобами на боль в горле больше справа. Болен 5 дней. Была диагностирована ангина, проводилось лечение. Состояние тяжелое. Температура тела 39,6. Кожные покровы бледные. При фарингоскопии: тризм жевательной мускулатуры, смещение и выбухание правой миндалины к средней линии и кпереди, отмечается гиперемия и инфильтрация передней дужки и мягкого неба справа, увеличенные и болезненные подчелюстные лимфатические узлы. Какой из перечисленных ниже диагнозов наиболее вероятен?

Паротонзилит справа либо паротонзилярный абсцесс( абсцесс правильнее, но его не ставят до вскрытия и получения гноя, но это точно он)

36. Больной 35 лет, обратился к ЛОР-врачу поликлиники с жалобами на: обильную риноррею, приступы чихания и затрудненное носовое дыхания. При осмотре: слизистая полости носа бледная с синюшным оттенком, на поверхности нижних носовых раковин определяются светлые пятна, раковины застойные. Смазывание слизистой полости носа сосудосуживающими каплями не привела к сокращению носовых раковин и спровоцировала приступ чихания. Со слов больного приступы чихания наблюдаются в течении всего года и провоцируются: домашней пылью, шерстью животных, перьями подушек. Какой из перечисленных ниже диагнозов наиболее вероятен?

Хронический вазомоторный аллергический ринит

37. Больной П., 19 лет, обратился в ЛОР клинику с жалобами на постоянную заложенность носа, которая купируется только после закапывания нафтизина. Капли использует в течение года. В последнее время с небольшим эффектом. При осмотре носовая перегородка по средней линии, патологического отделяемого в полости носа нет. Отмечается гиперемия, отек и инфильтрация слизистой нижних носовых раковин, в связи с чем носовые ходы резко сужены, носовое дыхание затруднено. Адреналиновая проба отрицательная. На рентгенограмме пазух носа патологии не выявлено. Предположите, какой из перечисленных диагнозов у больного наиболее вероятен?

Хронический гипертрофический ринит

38. Больной Ш. 17 лет, обратился в ЛОР клинику с жалобами на затруднённое носовое дыхание и головную боль. В анамнезе - получил травму носа 2 дня назад. Кровотечения из носа не было. Сознания не терял. Заложенность носа наросла постепенно, в течение нескольких часов после полученной травмы носа. Общее состояние больного удовлетворительное. При осмотре: спинка носа по средней линии, пальпаторно определялась умеренная болезненность, однако крепитации не отмечалось. При передней риноскопии носовая перегородка симметрично утолщена в области преддверия носа, раковины и полость носа необозримы. Предположите, какой из перечисленных диагнозов у больного наиболее вероятен?

Гематома носовой перегородки

39. Больной Т., 35 лет обратился к ЛОР врачу с жалобами на постоянную заложенность носа, зуд в носу, обильные водянистые выделения, чихание, слабость. Болен в течение двух недель. Аналогичное состояние отмечал три года подряд в это же время года (весна). При осмотре слизистая полости носа бледная, застойная, в носовых ходах с обеих сторон обильное водянистое отделяемое, носовые раковины отёчные, застойные, гнойного отделяемого нет. Предположите, какой из перечисленных ниже диагнозов у больного наиболее вероятен?

Хронический аллергический вазомоторный ринит. Сезонная форма

40. Больная А., 45 лет обратилась в ЛОР клинику с жалобами на затруднённое носовое дыхание. В анамнезе, по месту жительства перенесла операцию – полипотомия носа. Носовое дыхание восстановилось, но через три месяца полипы рецидивировали. На КТ придаточных пазух носа выявлено затемнение практически всех придаточных пазух носа. Полость носа обтурирована полипами. Носовое дыхание практически отсутствует. Предположите, какой из перечисленных ниже диагнозов у больной наиболее вероятен?

Рецидивирующий полипозный риносинусит

41. Больной А., 25 лет, обратился в ЛОР клинику с жалобами на охриплость голоса, чувство першения, саднения и сухости в голе. Начало заболевания связывает с приемом холодного пива шесть дней назад. Со слов больного заболевание началось с появления сухого кашля, затем кашля с мокротой. Самочувствие не страдало, постоянно ощущал дискомфорт при глотании. При ларингоскопии: гиперемия слизистой гортани, более выраженная в области истинных голосовых складок, с участками точечных кровоизлияний. В просвете гортани небольшое количество вязкой мокроты, местами переходящей в корки. Предположите, какой из перечисленных ниже диагнозов у больного наиболее вероятен?

Острый катаральный ларингит

42. Ребёнок 5 летнего возраста, доставлен в приемный покой бригадой скорой помощи с жалобами на затруднение дыхания, одышку, после перенесенного ОРВИ. Внезапно среди ночи, проснулся от приступообразного лающего кашля. Ребёнок был беспокоен, метался в постели. Дыхание свистящее, резко затруднённое, выражена инспираторная одышка. Определяется втяжение мягких тканей ярёмной ямки, над- и подключичных пространств, эпигастральной области. Подобное состояние длилось в течение часа, после чего, появилась обильная потливость, дыхание улучшилось и ребёнок заснул. Утром проснулся почти здоровым, но сохранилась некоторая охриплость голоса. Предположите, какой из перечисленных ниже диагнозов у больного наиболее вероятен?

Подскладочный ларингит или ложный круп

43. Больная К., 28 лет, обратилась в приемный покой с жалобами на сильную боль в ухе слева, иррадиирущую в височную и теменную область, усиливающуюся при жевании, на повышение температуры до 37,7 С. При отоскопии: AS - в слуховом проходе - на передней стенке определяется конусовидное возвышение, кожа на его поверхности гиперемирована. В центре образования - гнойная головка, просвет слухового прохода сужен, барабанная перепонка не визуализируется. Предположите, какой из перечисленных ниже диагнозов у больной наиболее вероятен?

Фурункул наружного слухового прохода

44. Больная 20 лет, обратилась в ЛОР клиникус жалобами на сильную боль в правом ухе, иррадиирущую в висок, заложенность уха, головную боль, повышение температуры тела до 38ºС. Из анамнеза - больна 2 дня. Начало заболевания связывает с насморком. При отоскопии: ADслуховой проход свободен, кожа не изменена, барабанная перепонка ярко гиперемирована, инфильтрирована, выбухает, опознавательные пункты отсутствуют. Слух на правое ухо снижен ШР2м, опыт Ринне отрицательный, латерализация звуков вправо. Какой из перечисленных ниже диагнозов у больной наиболее вероятен?

Острый гнойный средний отит справа, доперфоративная стадия

45. Больной обратился к ЛОР-врачу поликлиники с жалобами на: обильную риноррею, приступы чихания и затрудненное носовое дыхание. При осмотре: слизистая полости носа бледная с синюшным оттенком, на поверхности нижних носовых раковин определяются светлые пятна, раковины застойные. Смазывание слизистой полости носа сосудосуживающими каплями не привело к сокращению носовых раковин и спровоцировало приступ чихания. Со слов больного приступы чихания наблюдаются в течение всего года и провоцируются домашней пылью, шерстью животных, перьями подушек. Предположите, какой из перечисленных ниже диагнозов у больного наиболее вероятен?

Хронический вазомоторный аллергический ринит, круглогодичный

46. Больная А. 30 лет обратилась в приемный покой с жалобами на шум в ушах и снижение слуха с обеих сторон. Из анамнеза: после перенесенного гриппа две недели назад появились вышеперечисленные жалобы. Отоскопия в норме. Слуховой паспорт свидетельствует о двухстороннем понижении слуха по типу нарушения системы звуковосприятия.

Поставьте наиболее вероятный диагноз?

Острая двухсторонная сенсоневральная тугоухость

47. Больной 16 лет доставлен машиной скорой помощи в ЛОР клинику с жалобами на резкую слабость, недомогание, головную боль, боль в горле. Болен второй день, около недели назад был в контакте с больным ребенком, который госпитализирован в инфекционное отделение с диагнозом дифтерия. Состояние больного средней тяжести, ослаблен. Температура 38, 6 ºС, пульс 82 в мин. Слизистая оболочка миндалин гиперемирована, покрыта грязно-серым налетом, который распространяется на дужки, налет снимается с трудом, подлежащая ткань кровоточит. В подчелюстной области - отечность мягких тканей шеи. Какие микроорганизмы являются причиной развития данного заболев

Corynebacterium diphtheria/ бацилла Лефлера/ дифтерийнаятпалочка

ПРИМЕНЕНИЕ

1. Ребенок 3-х лет, доставлен родителями в ЛОР стационар с жалобами на затрудненное носовое дыхание, частые простудные заболевания и головная боль. Со слов матери ночью дышит с открытым ртом, при осмотре – лицо одутловатое, отвисшая нижняя челюсть, неправильный прикус зубов, высокое «готическое» небо, носовое дыхание осслаблено, при пальцевом исследовании определяются аденоидные вегетации III степени. Выберите наиболее оптимальный метод лечения?

Аденотомия

2. Больной 32 года обратился к ЛОР клинику с жалобами на распирающие боли в носу и затруднение носового дыхания. Со слов больного - 3 дня назад получил удар в области носа. При передней риноскопии носовые ходы резко сужены за счет воспалительного отека слизистой оболочки в передних отделах носовой перегородки с обеих сторон, носовое дыхание затруднено. При зондировании отмечается симптом флюктуации, болезненность. Выберите наиболее оптимальный метод лечения у данного больного?

Вскрытие и пункция гематомы перегородки носа, далее передняя тампонада Носа

3. Ребенок 3-х лет в сопровождении матери обратился в детскую поликлинику с жалобами на наличие инородного тела в носу. Со слов матери ребенок накануне, засунул в себе в нос округлый предмет от детского конструктора. Что из перечисленного является наиболее эффективным для удаления инородного тела из полости носа?

Крючок Воячека

4. У больной 35 лет, обратившейся в ЛОР клинику с жалобами на значительное снижение слуха на правое ухо после перенесенного тяжелого гриппа, заподозрен острый неврит слухового нерва. Выберите какие аудиометрические данные подтвердят этот диагноз?

Сенсоневральная тугоухость. Снижение костной и воздушной проводимости, отсутствие костно воздушного интервала

5. Больная 27 лет обратилась в ЛОР клинику с жалобами на снижение слуха, шум в ушах, больше справа. В шумной обстановке (во время езды в трамвае, троллейбусе) периодически отмечает некоторое улучшение слуха. Причины заболевания указать не может. Снижение слуха и шум в ушах значительно усилились год назад во время беременности. Объективно: ушные раковины и области сосцевидных отростков внешне не изменены, при пальпации безболезненны. Слуховые проходы широкие. Барабанные перепонки серого цвета, блестящие, с выраженными опознавательными знаками. Больной после обследования был выставлен диагноз отосклероз. Выберите, какое оперативное вмешательство нужно провести больной?

Стапедопластика

6. Больной 32 лет обратился в ЛОР клинику с жалобами на понижение слуха на оба уха, чувство заложенности ушей, аутофонию, ощущение переливания жидкости в ушах при перемене положения головы. Болен в течение месяца после перенесенного острого респираторного заболевания. Отоскопия: барабанные перепонки серого цвета, мутные, опознавательные пункты не определяются, подвижность барабанных перепонок ограничена. Проходимость слуховых труб III степени. ШР на правое ухо — 2 м, на левое ухо — 2,5 м, РР — 5 м на оба уха. Был выставлен диагноз: экссудативный

средний отит. Выберите из ниже перечисленного метод продувания слуховых труб, позволяющий ввести лекарство в барабанную полость:

Катетеризация слуховых труб

7. У ребенка 2 –х лет, родившегося в асфиксии, родители заметили отсутствие реакции на звуки. Из анамнеза выяснено, что мать перенесла корьевую краснуху в первой половине беременности. На аудиограмме – нейросенсорная глухота. На КТ височных костей - все анатомические образования наружного, среднего и внутреннего уха сохранены.

Какова ваша тактика лечения для данного больного?

Слухопротезирование либо кохлеарная имплантация слухового нерва

8. Больная 35 лет обратилась в ЛОР клинику с жалобами на периодически возникающие среди полного здоровья приступы системного головокружения на фоне нарастающего шума и снижения слуха на левое ухо, сопровождающиеся тошнотой, рвотой, расстройством равновесия. Каждая попытка движения и изменения положения головы в пространстве усиливает головокружение и вегетативные расстройства. Приступ длится до 4-6 часов, возникает и проходит спонтанно. На аудиограмме – нейросенсорная тугоухость слева 3-4 степени, четко выражена флуктуация слуха. Больной выставлен диагноз – болезнь Меньера. Выберите, какое оперативное вмешательство целесообразнее провести данной больной?

Перерезка барабанной струны (хордоплексустомия) либо вскрытие мешочков преддверия

9. В ЛОР - отделении находится больной 56 лет с диагнозом "Отечно-инфильтративный ларингит, сахарный диабет первого типа, средней тяжести". Несмотря на проводимую антибактериальную и противоотечную терапию состояние больного ухудшилось: появились и стали нарастать боли в области шеи, затруднение дыхания и глотания, усилились явления дисфонии. Температура тела фебрильная, в крови лейкоцитоз. При непрямой ларингоскопии слизистая оболочка гортаноглотки и гортани гиперемирована, инфильтрирована, за счет чего вестибулярный отдел гортани сужен до 5 мм, голосовые складки не обозримы. Дыхание затруднено, инспираторная одышка в покое до 24 в 1 мин, дисфония. Какое лечение наиболее оптимально в данном случае?

Вскрытие флегмоны шеи. Хирургическое лечение. АБ терапия

10. Ночью врач скорой помощи вызван к ребенку 2 лет с жалобами на внезапно наступивший приступ удушья, кашель, шумное дыхание, насморк, высокую температуру тела. Заболевание началось 3 дня назад с вялости, сонливости, головной боли, снижения аппетита. На другой день появились насморк и лающий кашель. Ночью ребенок проснулся от удушья и грубого кашля. Объективно: состояние средней

тяжести, температура тела 37,90 С., инспираторная одышка в покое, в акте дыхания участвует вспомогательная мускулатура. ЧД – 30 в мин., РS – 100 в мин., голос чистый, кашель лающий. Какой дополнительный метод исследования необходимо применить для подтверждения диагноза?

Непрямая ларингоскопия на отек подскладочного пространства. Бак посев на отсутствие дифтерийной палочки

11. Больная 70 лет обратилась в ЛОР клинику с жалобами на боли в области шеи у верхнего края щитовидного хряща справа, боли усиливающиеся при глотании. Прием жесткой пищи затруднен. Перечисленные жалобы больная связывает с приемом в пищу курицы за 5 дней до обращения к врачу, когда сначала появилось ощущение инородного тела, а затем боли. Общее состояние удовлетворительное, температура тела 37,50 С, пальпация шеи в области верхнего края щитовидного хряща справа умеренно болезненна. При непрямой ларингоскопии в правом грушевидном синусе отмечается симптом "слюнного озера", инородное тело визуально не обнаружено. Назначьте дополнительное обследование для установки диагноза

Рентгенография пищевода с барием

1. У больного 48 лет, доставленного ургентно в ЛОР клинику - при люмбальной пункции выявлены менингеальные знаки: плеоцитоз за счет нейтрофилов в спинномозговой жидкости. Со слов родственников известно, что состояние резко ухудшилось 3 дня тому назад, после перенесенной ОРВИ. С детства страдает хроническим гнойным эпитимпанитом. Диагностирован отогенный гнойный менингит. Выберите какой метод лечения предпочтителен для больного ?

Хирургическая элиминация гнойного очага в ухе и санирующая операция на височной кости

2. Больная 40 лет обратилась в ЛОР клинику с жалобами на снижение слуха на оба уха в течение 2 недель, после перенесенного ОРВИ, шум и звон в ушах Боли в ушах не отмечает. Объективно: отоскопически -барабанные перепонки серого цвета, слегка втянуты, опозновательные знаки барабанной перепонки выражены ясно.

Выберите какое исследование следует произвести для объективной оценки состояния слухового анализатора пациентки?

Аудиометрия

14. Больной 44 лет, поступивший в стационар в экстренном порядке с повышенной до 38, 3°С температурой, с детства отмечает периодическое гноетечение из уха. При последнем обострении появилась разлитая головная боль, тошнота и рвота, не

связанные с приемом пищи. Определяется ригидность мышц затылка, симптом Кернига. Заподозрен отогенный менингит.

Выберите какое дополнительное исследование необходимо для уточнения диагноза и прогнозирования тактики лечения у больного?

Рентгенография височной кости, МРТ головного мозга, люмбальная пункция

15. У больной 36 лет, доставленной в ЛОР стационар машиной скорой помощи, при очередном обострении появилась головная боль в височной области, субфебрильная температура. В анамнезе – больная длительное время наблюдается у ЛОР врача по поводу эпитимпанита (от оперативного лечения отказывалась), Невропатологом выявлена очаговая симптоматика, окулистом – застой вен на глазном дне. На КТ черепа определяется очаг в височной доле мозга. Диагностирован отогенный абсцесс мозга. Оцените алгоритм лечения больного в данной ситуации?

Хирургическая элиминация гнойного очага в ухе и санирующая операция на височной кости. Антибиотикотерапия, дезинтоксикационная терапия,передача к нейрохирургу для вскрытия абсцесса мозга

16. У больного 45 лет, обратившегося в ЛОР клинику с гнойным эпитимпанитом, резко ухудшилось общее состояние: появилась разлитая интенсивная головная боль, помутнение сознания, температура тела повысилась до 40°С. При обследовании выявлены менингеальные знаки. В цереброспинальной жидкости выявлен высокий плеоцитоз за счет нейтрофилов. Диагностирован отогенный гнойный менингит.

Какую врачебную тактику необходимо применить в данном конкретном случае?

Хирургическая элиминация гнойного очага в ухе и санирующая операция на височной кости. Антибиотикотерапия, дезинтоксикационная терапия,передача к нейрохирургу. НУ НЕ ЛЕЧАТ ЛОР ВРАЧИ ОТОГЕННЫЕ ОСЛОЖНЕНИЯ, НУ ЧЕ СПРАШИВАТЬ.

17. Больная 38 лет обратилась в ЛОР клинику с жалобами на расстройство координации, снижение слуха и гноетечение из правого уха. При осмотре: субтотальный дефект правой барабанной перепонки, гнойное отделяемое с неприятным запахом и кариес кости в барабанной полости. Компьютерная томография выявила абсцесс мозжечка.

Какую врачебную тактику необходимо использовать в данном случае?

Хирургическая элиминация гнойного очага в ухе и санирующая слухосохроняющая операция на височной кости, вскрытие абсцесса мозжечка. Антибиотикотерапия, дезинтоксикационная терапия,передача к нейрохирургу

18. Больной 53 лет обратился в ЛОР клинику с жалобами на резкое снижение слуха, гной с неприятным запахом из левого уха. В анамнезе - в течение многих лет страдает эпитимпанитом, от оперативного лечения категорически отказывается, лечится самостоятельно народными методами. Обратился к врачу в связи с появившимся во время последнего обострения «перекосом» лица. При обследовании выявлен парез лицевого нерва слева. Выберите план госпитализации и врачебной тактики в данном конкретном случае?

Срочная госпитализация, санирующая слухосохроняющая операция среднего уха, ГКС, витамины группы В, возможны блокады лицевого нерва

19. Больной В. 35 лет доставлен в больницу. Температура 38,7°С. Со слов жены, в течение 2 лет практически не прекращалось гноетечение из правого уха, неделю тому назад стал жаловаться на головную боль, 2 часа назад сознание стало спутанным. Компьютерная томография выявила абсцесс височной доли мозга. Решите, в какое отделение следует госпитализировать больного?

Нейрохирургия , с вызовом оперирующего лор врача

20. В детскую больницу обратилась мать, у которой у ребенка 8 лет после перенесенного отогенного менингита развилась тугоухость IV степени.

Выберите каковы меры и где может обучаться этот ребенок?

Обучение в общеобразовательной школе, пользование слуховым аппаратом

21. Больная 43 лет обратилась в поликлинику к ЛОР врачу с жалобами на головную боль в височной области, субфебрильную температуру, снижение слуха. В анамнезе больная длительное время наблюдается по поводу эпитимпанита, очередное обострение отмечает после перенесенного несколько дней назад охлаждения. Окулистом выявлен – застой вен на глазном дне. Невропатологом - очаг в височной доле мозга. Диагностирован отогенный абсцесс мозга. Выберите основной метод диагностики отогенного абсцесса височной доли головного мозга:

МРТ головного мозга и Рентгенография височной кости

22. У больной 44 лет, обратившейся в ЛОР клинику отмечаются жалобы на ощущение жжения, першение в горле. Температура нормальная. В области верхнего полюса левой нёбной миндалины отмечается поверхностное изъязвление с неровными краями, покрытое сероватым налетом легко кровоточащее при дотрагивании. При пальпации вокруг язвы определяется деревянистой плотности инфильтрат, переходящий на мягкое небо и корень языка. На шее слева пальпируется конгломерат увеличенных лимфоузлов, размером 5x6 см, плотный, ограниченно смещаемый. Назначение какого метода исследования является целесообразным, чтобы установить точный диагноз?

Реакция Вассермана на сифилис, проба манту на туберкулёз

Биопсия (петлевая)

23. В ЛОР стационар родителями доставлен ребенок 4 лет, который плохо дышит носом, часто страдает респираторными заболеваниями, плохо спит, вскрикивает во сне. Объективно отмечается бледность кожных покровов, полуоткрытый рот, сглаженность носогубных складок, одутловатое лицо, узкий нос, высокое готическое небо, неправильный прикус зубов. Какой из перечисленных ниже методов обследования является наиболее целесообразным?

Пальцевое исследование носоглотки, эпифарингоскопия(задняя риноскопия),эндоскопия носоглотки.

24. Ребенок 5 лет с аденоидными вегетациями доставлен в ЛОР стационар с выраженным затруднением носового дыхания. Ребенок, по словам матери, все время дышит ртом, беспокойно спит ночью, вскрикивает во сне, часто болеет респираторными заболеваниями. Ребенок бледен, лицевой череп вытянут в вертикальном направлении. При задней риноскопии определяются розового цвета дольчатые образования, прикрывающие хоаны на 2/3.

Какой из перечисленных ниже методов лечения является наиболее целесообразным?

Аденотомия

25. Больной 25 лет обратился в поликлинику к ЛОР врачу с жалобами на легкую боль в горле слева. Общее состояние не нарушено. Температура З6,6 0 С. Изменений со стороны внутренних органов не определяется. У верхнего полюса левой миндалины сероватый налет, после удаления которого обнаружена довольно глубокая язва с неровным сальным дном. Со стороны других ЛОР-органов отклонений от нормы не определяется. Предполагаемый диагноз сифилис – твердый шанкр 1 ст. Какой из перечисленных ниже методов исследования крови является наиболее целесообразным для подтверждения диагноза?

Реакция Вассермана, ИФА на антитела к бледной трепонеме

26. Больной 16 лет доставлен в инфекцинную больницу на носилках с жалобами на резкую слабость, недомогание, головную боль, боль в горле. Болен второй день, около недели назад был в контакте с больным ребенком, который госпитализирован в инфекционную больницу с диагнозом дифтерия. Состояние больного средней тяжести, температура 38, 6 ºС, пульс 82 в мин. Слизистая оболочка миндалин гиперемированна,

покрыта грязно-серым налетом, который распространяется на дужки, мягкое небо, налет снимается с трудом, подлежащая ткань кровоточит. В подчелюстной области - отечность мягких тканей шеи.

Какой из перечисленных ниже методов обследования является наиболее оптимальным?

Бак посев на выявление палочки Лефлера

27. Пациент Б., 30 лет, обратился в ЛОР клинику с жалобами на затруднение дыхания, осиплость голоса, шумное дыхание. Со слов пациента, вышеуказанные жалобы появились после употребления в пищу меда.

Объективно: при орофарингоскопии слизистая оболочка ротоглотки отечная, отмечается стекловидный отек язычка. При непрямой ларингоскопии определяется выраженный отек слизистой оболочки гортани, голосовая щель 2-3 мм. Выставлен диагноз: Аллергический отек гортани. Стеноз III стадии. Выберите наиболее оптимальную тактику лечения у больного:

ГКС. Трахеостомия

28. Больная 22 лет, обратилась к ЛОР врачу в поликлинику с жалобами на сильную боль в горле, t -38 0 С, слабость, вялость, после перенесенного охлаждения. Общее состояние больной средней тяжести, кожные покровы бледные, пальпируются увеличенные регионарные лимфатические узлы глотки. Фарингоскопически определяется гиперемия и инфильтрация мягкого неба, нёбных дужек, отёчные и гиперемированные небные миндалины, на поверхности которых многочисленные круглые, слегка возвышающиеся желтые точки. Какой из перечисленных ниже методов лечения является наиболее целесообразным?

Антибиотикотерапия, туалет полости рта, постельный режим

29. Больной 36 лет обратился к ЛОР врачу в поликлинику с жалобами на сильную боль в горле при глотании, озноб, t 39 0 С, слабость, боли в суставах, гнилостный запах изо рта. Объективно: общее состояние тяжелое, кожные покровы сероватые, пальпируются увеличенные регионарные лимфатические узлы глотки. Фарингоскопически определяется гиперемия и инфильтрация мягкого неба и нёбных дужек, увеличение и гиперемия небных миндалин, гнойно-фибринозный налет на поверхности миндалин. В общем анализе крови выявлен сдвиг лейкограммы влево.

Какой из ниже перечисленных методов обследования является наиболее оптимальным?

Мазок на флору с нёбных миндалин

30. Больной 46 лет, обратился к ЛОР врачу в поликлинику с жалобами на сильную боль в горле при глотании, озноб, t 39 0 С, слабость, боли в суставах, гнилостный запах изо рта. Объективно: общее состояние тяжелое, t 39,6 0 С, кожные покровы сероватые, пальпируются увеличенные регионарные лимфатические узлы глотки. Фарингоскопически определяется гиперемия и инфильтрация мягкого неба и нёбных дужек, увеличение и гиперемия небных миндалин, гнойно-фибринозный налет на поверхности миндалин. В общем анализе крови выявлен сдвиг лейкограммы влево.

Какой из ниже перечисленных методов лечения является наиболее оптимальным?

Антибиотикотерапия местная и общая, дезинтоксикационная терапия

31.Больной 16 лет обратился к ЛОР врачу в поликлинику с жалобами на сильную боль в горле, озноб, t 39 0 С, слабость, боли в суставах. Горло болит 4 дня, до этого в течение недели беспокоит слабость, вялость недомогание субфебрильная температура. Объективно: общее состояние тяжелое, t 39,6 0 С, кожные покровы сероватые, пальпируются увеличенные шейные, подмышечные, паховые лимфатические узлы; печень и селезенка увеличены. Фарингоскопически определяется гиперемия и инфильтрация мягкого неба и нёбных дужек, увеличенные и гиперемированные небные миндалины, покрыты фибринозным налетом. В общем анализе крови выявлены в большом количестве атипичные мононуклеары. Какая терапия из ниже перечисленных является наиболее вероятной? Госпитализация в инфекционное отделение, полоскание антисептиками, общеукрепляющая терапия

32.Больной 28лет обратился к ЛОР врачу в поликлинику с жалобами на сильную боль в горле, больше справа, t 38,2 0 С, слабость. Болен 5-ий день, лечился полосканием горла. Объективно: голос гнусавый, кожные покровы бледные, t 38,6 0 С, пальпируются увеличенные подчелюстные лимфатические узлы, резко болезненные справа. Фарингоскопически: рот открывается шириной на один палец, асимметрия мягкого неба, за счет инфильтрации и отека правой передней небной дужек, небные миндалины увеличенные и гиперемированные, в лакунах миндалин гнойное скопление, правая миндалина смещена к центру, отмечается отек язычка.

Какой из ниже перечисленных методов лечения является наиболее правильным?

Вскрытие паратонзилярного пространства, антибиотикотерапия, дезинтоксикационная терапия

33. Ребенок 9 лет, доставлен бригадой скорой медицинской помощи в приемный покой с жалобами на затрудненное дыхание, сильную боль в горле, t 38,2 0 С, слабость. Со слов родителей известно, что болен 3-й день. Объективно: состояние тяжелое, дыхание стридорозное, голос хрипловатый, кожные покровы бледные, t 38,6 0 С, пальпируются увеличенные, резко болезненные регионарные лимфатические узлы, голова наклонена влево. Фарингоскопически асимметрия, гиперемия и выпячивание задней стенки глотки, больше слева. При ощупывании тупым зондом определяется флюктуация.

Какой из ниже перечисленных методов лечения является наиболее оптимальным?

Вскрытие заглоточного абсцеса, антибиотикотерапия и дезинтоксикационная терапия

34. Мужчина 57 лет обратился к ЛОР врачу в поликлинику с жалобами на постоянный зуд в наружных слуховых проходах, который иногда становится нестерпимым. Манипуляции в ушах различными предметами (спичками и т.д.) не приносят облегчения. Более того, дважды, после подобных манипуляций, возникал острый диффузный наружный отит (диагноз врача поликлиники) с резкими болями в ушах и высокой температурой. Выставлен предварительный диагноз: Отомикоз. Выберите, какие дополнительные диагностические методы исследования необходимо использовать для подтверждения диагноза?

Отоскопия, бак посев из уха

35. Больной А. 56 лет обратился в ЛОР клинику с жалобами на охриплость, усиливающуюяся к вечеру, данное ощущение беспокоит в течение полугода, последний месяц охриплость сопровождается незначительными болевыми ощущениями. Ларингоскопически определяется гиперемия и инфильтрация правой голосовой складки по всей длине, при фонации подвижность правой голосовой складки ограничена. Больной был выставлен диагноз: Парез гортани справа. Какие методы исследования необходимо провести больному, чтобы подтвердить диагноз?

Ямрт органов шеи, стробоскопия гортани,

36. Больная 18 лет обратилась в ЛОР клинику по поводу носового кровотечения. Такие кровотечения, более или менее обильные, возникают у нее часто в предменструальный период. В момент осмотра кровотечения нет. Кожные покровы и видимые слизистые оболочки бледные. Пульс ритмичен - 88 уд/мин. В носовых ходах - кровянистые сгустки. Слева в передненижнем отделе перегородки носа сосуды резко расширены,

видны следы кровотечения. Какой метод лечения является наиболее целесообразным в этом случае?

Холод на переносицу, прижать пальцем крылья носа, передняя тампонада, гемостатическая терапия. Консультация гинеколога и эндокринолога. (Но если вариантов не будут таких, то прижигание сосудов перегородки – радикальные меры )

37. Больной В. 35 лет поступил в ЛОР отделение с жалобами на затруднение дыхания через естественные дыхательные пути. Считает себя больным после перенесенной вирусной инфекции. Объективно: при непрямой ларингоскопии определяется стекловидный отек слизистой оболочки гортани. Голос сиплый. Голосовая щель 6-7 мм. Выставлен диагноз: Аллергический отек гортани. Стеноз I стадии. Какой метод лечения является наиболее оптимальным для больного?

ГКС, антигистаминные препараты, диуретики

38. Больной 25 лет обратился в ЛОР клинику с жалобами на охриплость, ощущение инородного тела в горле. Около года назад больная перенесла закрытую ЧМТ по поводу чего находилась в реанимационном отделении. В течении двух недель проводилась инжекционная вентиляция легких (ИВЛ). При осмотре гортани в задней трети обеих голосовых складок определяются образования округлой формы, с гладкой поверхностью, бледно розового цвета, размером 3 и 2 мм. Больной был выставлен диагноз: гранулема гортани. Выберите тактику лечения данного больного?

Эндоларингиальное удаление образования

39. Ребенок 6 лет доставлен родителями в ЛОР стационар с жалобами на затруднение дыхания. В анамнезе: часто болеет респираторными заболеваниями, один раз в течение последних трех лет болел ангиной. При фарингоскопии у ребенка определяется резкое увеличение небных миндалин, которые почти смыкаются по средней линии., слизистая оболочка бледно-розового цвета, влажная. При приеме пищи ребенок отмечает затруднение. Налетов и пробок в миндалинах нет. Признаки хронического тонзиллита отсутствуют. Какой из ниже перечисленных методов лечения является наиболее оптимальным?

Тонзилотомия

40. Ночью врач скорой помощи вызван к ребенку 2 лет с жалобами со слов родителей, на внезапно наступивший приступ удушья, кашель, шумное дыхание, насморк, высокую температуру тела. Заболевание началось 3 дня назад с вялости, сонливости, головной боли, снижения аппетита. На другой день появились насморк и лающий кашель. Ночью ребенок проснулся от удушья и грубого кашля. Страдает пищевой аллергией (клубника, цитрусовые). Объективно: состояние средней тяжести, температура тела 37,90 С.

Акроцианоз, инспираторная одышка в покое, в акте дыхания участвует вспомогательная мускулатура. ЧД – 30 в мин., РS – 100 в мин., голос чистый, кашель лающий. Слизистая оболочка носа и глотки гиперемирована. Какой метод исследования гортани необходимо применить для постановления диагноза?

Гипофарингоскопия

41. Мужчина 28 лет обратился к ЛОР врачу в поликлинику с жалобами на внезапное повышение температуры до 39°, озноб, ломящую боль в суставах, мышцах. Больной принял препарат «Фервекс». Состояние больного улучшилось, однако на следующий день температура вновь поднялась до 37,5°, появилась стреляющая, затем распирающая боль и шум в левом ухе, понизился слух. Вызванный на дом терапевт поставил диагноз «грипп» и направил больного к сурдологу, который после обследования выставил диагноз: Острый средний отит, осложненный кохлеоневритом слева. Выберите какой метод диагностики, из ниже перечисленных был использован в данном случае?

Аудиометрия

42. Ребёнок 5 лет, доставлен родителями в ЛОР-стационар с жалобами на стреляющие боли и припухлость в области левого уха. При осмотре: левая ушная раковина оттопырена. В заушной области определяется отёк и инфильтрация мягких тканей кожи, распространяющееся на теменную, височную и околоушную области, а также флюктуирующее выпячивание величиной с куриное яйцо. Пальпация заушной области резко болезненна. Наружный слуховой проход имеет щелевидную форму за счёт инфильтрации кожи задне-верхней стенки. Барабанная перепонка плохо обозрима. Больному был выставлен диагноз: Мастоидит. Выберите какое из нижеперечисленных вмешательств целесообразнее применить в данной ситуации у больного?

Мастоидит отличия, санация гнойного очага

43. У девочки 10 лет, доставленной родителями в ЛОР стационар, в разгар ОРВИ появилось ощущение заложенности в левом ухе, которое вскоре сменилось пульсирующей рвущей болью, усиливающейся при глотании. Компресс и вливание капель в ухо несколько улучшили состояние ребёнка. Однако через 3 часа боль в ухе усилилась, приобрела распирающий характер, в связи с чем, машиной скорой помощи ребёнок доставлен к ЛОР врачу. Выставлен диагноз: Острый средний отит, стадия доперфоративного гнойного воспаления слева. Выберите какой из нижеперечисленных вариантов лечения наиболее адекватен в данной ситуации?

Парацентез барабанной перепонки слева, антибиотикотерапия

44. Больной 28 лет обратился в ЛОР клинику с жалобами на боль в горле, усиливающуюся при глотании, больше справа, боль в правом ухе, затруднение при

глотании и открывании рта, общее недомогание. Заболел 5 дней назад после перенесенной катаральной ангины. Рот открывает на один поперечный палец. В глотке определяется гиперемия слизистой оболочки, инфильтрация околоминдаликовой области справа, правая миндалина сдвинута к средней линии. Язычок резко отечен и смещен несколько влево. Уплотнены, увеличены и болезненны при пальпации подчелюстные лимфоузлы справа. Выберите какой из ниже перечисленных методов лечения является для больного наиболее оптимальным?

Вскрытие паротонзилярной ниши справа либо паротонзилотомия

45. Больной 25 лет обратился в ЛОР клинику с жалобами на частые ангины, 5-6 раз в год, протекающие с повышением температуры до 38-39-С, болями в суставах, сердце. Проводимое в последнее время консервативное лечение – без должного эффекта. При мезофарингоскопии: края небных дужек гиперемированны, отечны, инфильтрированны, миндалины рубцово изменены, плотные, выступают за края дужек, с гнойными пробками в лакунах. Подчелюстные ретромандибулярные и верхние шейные лимфоузлы увеличены, безболезненны. У больной выявлены тахикардия и изменение на электрокардиограмме, а также положительные ревмотесты. Какой из ниже перечисленных методов лечения является наиболее оптимальным для больной?

Тонзилоэктомия

46. У ребенка 7 лет жалобы на боль в горле, головная боль, температура до 39,2 С, рвота, выраженная слабость. При осмотре слизистая оболочка глотки гиперемирована, на небных миндалинах, небных дужках, на слизистой оболочке щек имеются розоватобелые пузырьки, отмечаются увеличенные и болезненные подчелюстные лимфатические узлы, в крови - лейкоцитоз - 10. Какая терапия, из ниже перечисленных является для больного наиболее оптимальной?

Ацикловир ( противовирусная терапия интерферон + симптоматическое)

47. Больная 35 лет, обратилась к ЛОР врачу поликлиники по поводу обострения правостороннего хронического гнойного среднего отита. Считает себя больной в течение 5 лет. Обострения бывают 3-4 раза в год, провоцируются респираторновирусной инфекцией. При отоскопии в правом наружном слуховом проходе - небольшое количество гнойного экссудата. Барабанная перепонка тусклая, опознавательные признаки стушеваны. В передне-верхнем квадранте определяется перфорация, просвет которой заполнен серо-белыми бесструктурными массами. Какие дополнительные способы исследования, которые могут быть применены в данном случае?

Рентген височной кости гистология масс, аудиометрия

48. У больного с охриплостью и затруднением дыхания не удается осмотреть гортань при непрямой ларингоскопии. Какой другой метод осмотра - вы назначите, чтобы увидеть просвет гортани и оценить подвижность голосовых складок?

Прямая ларингоскопия, УЗИ гортани

49. У больной 70 лет жалобы на боли в области шеи у верхнего края щитовидного хряща справа, боли усиливаются при глотании. Прием жесткой пищи затруднен. Перечисленные жалобы больная связывает с приемом в пищу курицы за 5 дней до обращения к врачу, когда сначала появилось ощущение инородного тела, а затем боли. Общее состояние удовлетворительное, температура тела 37,5ºС, пальпация шеи в области верхнего края щитовидного хряща справа умеренно болезненна. При непрямой ларингоскопии в правом грушевидном синусе отмечается симптом "слюнного озера", инородное тело визуально не обнаружено. Назначьте дополнительное обследование пищевода для установления диагноза.

Рентгенография пищевода с барием

50. У больной 44 года, обратившейся в ЛОР клинику жалобы на ощущение жжения, першения в горле. Температура тела нормальная. В области верхнего полюса левой нёбной миндалины отмечается поверхностное изъязвление с неровными краями, покрытое сероватым налетом легко кровоточащее при дотрагивании. При пальпации вокруг язвы определяется деревянистой плотности инфильтрат, переходящий на мягкое небо и корень языка. На шее слева пальпируется конгломерат увеличенных лимфоузлов, размером 5x6 см, плотный, ограниченно смещаемый. Назначение какого метода исследования является целесообразным, чтобы установить точный диагноз?

Реакция васермана, бак посев на бледную трепонему

51. Ребенок 4 года плохо дышит носом, часто страдает респираторными заболеваниями, плохо спит, вскрикивает во сне, отмечается ночное недержание мочи. Объективно: бледность кожных покровов, полуоткрытый рот. При осмотре сглаженность носогубных складок, одутловатое лицо, узкий нос, высокое готическое небо, неправильный прикус зубов. Какие из перечисленных ниже методов обследования являются наиболее целесообразными?

Пальцевое исследование носоглотки, эпифарингоскопия (задняя риноскопия),

52. У ребенка аденоидные вегетации 5 лет, резко затруднено дыхание через нос. Ребенок, по словам матери, дышит ртом, беспокойно спит ночью, вскрикивает во сне, часто болеет респираторными заболеваниями, перенес пневмонию. Ребенок бледен, лицевой череп вытянут в вертикальном направлении. При задней риноскопии

определяются розового цвета дольчатые образования, прикрывающие хоаны на 2/3. Какой из перечисленных ниже методов лечения является наиболее целесообразным?

Аденотомия

53. Больной 25 лет обратился в поликлинику к ЛОР врачу с жалобами на легкую боль в горле слева. Общее состояние не нарушено. Температура З6,6º С. Изменений со стороны внутренних органов не определяется. У верхнего полюса левой миндалины сероватый налет, после удаления которого обнаружена довольно глубокая язва с неровным сальным дном. Со стороны других ЛОР-органов отклонений от нормы не определяется. Предполагаемый диагноз: сифилис – твердый шанкр. Какой из перечисленных ниже методов исследования является наиболее целесообразным для подтверждения диагноза?

Реакция Вассермана, бак посев на бледную трепонему

54. Пациент 45 лет доставлен машиной скорой помощи в ЛОР клинику с жалобами на выраженное затруднение дыхания, слышное на расстоянии, максимальное участие в дыхании вспомогательной мускулатуры, появление чувства страха, периодически спутанное сознание, психоэмоциональное возбуждение. У больного холодный пот, руки, губы, нос холодные, нарастает распространенный цианоз, дыхание частое и поверхностное, нарастает тахикардия, артериальное давление падает, пульс слабого наполнения и напряжения, усугубляется ацидоз. При непрямой ларингоскопии: вход в гортань свободный, надгортанник в виде развернутого лепестка. Голосовые складки гиперемированы, отечны с обеих сторон, голосовая щель резко сужена, подголосовое пространство не просматривается. Оцените данные и выберите связку, которую необходимо рассечь врачу:

Коническая связка

55. Ребенок 10 лет, доставлен бригадой скорой медицинской помощи в ЛОР клинику с жалобами на затрудненное дыхание, сильную боль в горле, t тела 39ºС, слабость. Со слов родителей известно, что ребенок болен 3-ий день. Объективно: состояние тяжелое, дыхание стридорозное, голос хрипловатый, кожные покровы бледные, пальпируются увеличенные, резко болезненные регионарные лимфатические узлы, голова наклонена влево. При фарингоскопии: асимметрия, гиперемия и выпячивание задней стенки глотки, больше слева. При ощупывании тупым зондом определяется флюктуация. Какой из ниже перечисленных методов лечения является наиболее оптимальным?

Вскрытие заглоточного абсцеса, антибиотикотерапия

56. У ребёнка 9 лет, за последний год 3 раза диагностирован правосторонний острый средний отит. Дважды лечился амбулаторно, последний раз отит протекал тяжело, пришлось делать парацентез в стационаре. При исследовании слуха выявлено двустороннее снижение слуха по типу нарушения звукопроведения. Из анамнеза удалось выяснить, что ребёнок часто и длительно болеет простудными заболеваниями, долгое время у него нарушено носовое дыхание, он плохо учится. Выберите какие дополнительные способы исследования из ниже перечисленных необходимо применить для уточнения диагноза?

Аудиометрия, Рентген височной кости, передняя риноскопия?

57. Больной 54 лет обратился к ЛОР врачу в поликлинику с жалобами на охриплость в течение последних двух лет, усиливающаяся при простудах. Больной курит 25 лет по 1- 1,5 пачки в день. При ларингоскопии - определяются ворсинчатые выросты слизистой оболочки в области межчерпаловидного пространства. Голосовые складки инфильтрированы, умеренно гиперемированы, при фонации неполное смыкание по средней линии, подвижность не ограничена, дисфония, дыхание свободное, голосовая щель до 1,7 см. Больной выставлен диагноз: хронический гиперпластический ларингит. Выберите, какой метод профилактики заболевания вы назначите в первую очередь?

Отказ от курения и соблюдение голосового режима

58. Больной 68 лет обратился в ЛОР клинику с жалобами на выраженное затруднение дыхания и охриплость. Объективно: выражена инспираторная одышка, голос хриплый. Умеренная воспалительная инфильтрация голосовых складок, голосовая щель узкая, подвижность правой голосовой складки ограниченна, а левая голосовая складка неподвижна и находится в срединном положении. Через час после проведенного лечения состояние больного улучшилось, исчезла одышка, голосовая щель стала шире. Однако спустя 4 часа дежурный персонал обнаружил больного лежащим на спине без сознания. Спонтанного дыхания нет, непроизвольное мочеиспускание, зрачки расширены, кожные покровы цианотичны, пульс нитевидный. Произвести интубацию трахеи для искусственной вентиляции легких не удалось, ввиду сужения голосовой щели. Что нужно сделать для оказания первой помощи?

Коникотомия

59. В детскую поликлинику обратилась мать с 3-х летним ребенком, с жалобами на наличие у него инородного тела в носу. Со слов матери ребенок накануне, засунул в себе в нос округлый предмет от детского конструктора. Что из перечисленного является наиболее эффективным для удаления инородного тела из полости носа?

Крючок Воячека

60. В ЛОР клинику обратился пациент 67 лет с жалобой на носовое кровотечение. Попытка остановить кровотечение наложением холодного предмета на переносицу и прижатием крыльев носа к перегородке не принесла эффекта. Артериальное давление на момент осмотра 120/80 мм рт. ст. Источник кровотечения – зона Киссельбаха справа. Какой метод остановки кровотечения будет наиболее уместным?

Передняя тампонада носа

Ожоги (34)

Запоминание

1. Какой объем поражения при ожоге второй степени?

ответ: • II степень — средней тяжести ••• При ожогах 20–40% поверхности тела (глубокие ожоги составляют не более 20%) •••

Гибель и отслойка эпидермиса до базального слоя, с образованием пузырей, наполненных прозрачным содержимым

2.Что появляется при химическом ожоге щелочами ?

ответ: Мягкий струп белого цвета

3.С чем связано повышение температуры тела при острой ожоговой токсемии?

Ответ: Интоксикация связана с накоплением продуктов распада белков, токсических веществ, поступающих в кровь из обожжённых тканей и обладающих антигенными свойствами

4.Что необходимо сделать при поступлении в стационар больного с ожоговым шоком?

ответ: Обеспечение проходимости дыхательных путей, катетеризация центральных вен и начало инфузионной терапии, обезболивание, катетеризация мочевого пузыря

5.Какие ожоги относятся к ожогам особой локализации?

Ответ: Ожоги особой локализации (лицо, голова, шея, кисти, ступни, промежность,. внешние половые органы

6. При каком ожоге показана некротомия?

Ответ: Декомпрессионные операции (некротомия). Показаны при глубоких циркулярных ожогах на конечностях, туловище, способных привести к развитию субфасциального отёка.

7.Чему приблизительно равна площадь поражения тела по правилу «ладони»?

0,8-1,5 % (1%)

8.Что является пусковым механизмом ожогового шока ?

гиповолемия, болевой синдром, психо-эмоциональный стресс

9.Какая из этих локализаций ожога более опасна для жизни?

ожог лица, дых путей

10.Какое наиболее тяжелое осложнение ожогов приводит к летальному исходу

впериод ожоговой токсемии?

ОПП

ПОНИМАНИЕ

1. В отделение комбустиологической реанимации поступил больной 24 лет с диагнозом: «Электротравма. Ожог I-II-III степени обеих верхних конечностей 8 (5) % поверхности тела». Из анамнеза: со слов пациента, травму получил дома, потерял сознание на короткое время. Пришел в себя через 5 минут. Локально: имеются «метки» тока на ладонной поверхности обеих кистей. Данные АД 120/80 мм.рт.ст. ЧСС 110 уд/мин, ясный, ритмичный. ЭКГ: ЧСС 115 уд/мин, ритм синусовый, отклонение ЭОС влево. Что доказывает диагноз «Электротравма».

«метки» тока на ладонной поверхности обеих кистей проверено

2. Ребенок 3 года с термическим ожогом лица, шеи, туловища и обеих верхних конечностей общей площадью 35% поверхности тела. Отмечается клиника острой ожоговой токсемии. Какие клинические признаки указывают на период ожоговой болезни?

Лихорадка, бледность, тахикардия, тошнота, рвота, отек ГМ(дезориентация во времени и пространстве, галлюцинации), миокардит, нарушения функции почек (ОПП) проверено

3. У больного 38 лет с термическим ожогом I-II-III степени. S-40 (III-10)% наблюдается картина токсической пневмонии. Какие симптомы характеризуют клиническую картину

Лихорадка, учащение пульса, тахипноэ, одышка, цианоз, кашель с мокротой, притупление перкуторного звука и влажные хрипы

4. У больного 27 лет с термическим ожогом II-III степени. S-35 (15) %. наблюдается парез кишечника. Какие симптомы характеризуют клиническую картину

высокий тимпанит над всей поверхностью живота, равномерное вздутие живота, тошнота, рвота, отсутствие кишечных шумов, эксикоз, тахикардия, вторичная дыхательная недостаточность.

5. В ожоговое отделение поступил больной 27 лет с термической травмой II-III степени. S-25 (8)% поверхности тела. На 3 сутки нахождения в стационаре локально отмечается уменьшение плазмореи и усиление местных воспалительных изменений.

Определите период ожоговой болезни в данной клинической ситуации

Острая ожоговая токсемия

6. Больному 28 лет. Поступил с термическим ожогом от пламени I-II-III степени на площади 55% (23%) поверхности тела с клиникой ожогового шока. При осмотре: дно раны представлено черным плотным струпом.

Выставите клинический диагноз.

Термический ожог пламенем I-II-III ст. площадью 55% (23%). Ожоговый шок II ст.

7. Больной 5 летнего возраста находится в ожоговом отделении 12 сутки по поводу термического ожога горячей жидкостью I-II-III степени, с площадью 20%. Отмечается повышение температуры, нарушение сна и аппетита, вялость. На ожоговых ранах отмечается нагноение. Больному продолжается проведение комплексной терапии. В анализах: Нв-96 г/л, эрит.-3,2, лейкоцит-15,7, СОЭ-40 мм/час, уровень общего белка сыворотки-70,6 г/л, мочевина-7,5 ммоль, креатинин-0,019 ммоль, уровень СМП-0,682 усл.ед. Количество суточного диуреза-1200мл. Определите период ожоговой болезни.

Септикотоксемия

8. Больной 6 летнего возраста находится в ожоговом отделении 10 часов по поводу термического ожога горячей жидкостью I-II-III степени, с площадью 20% поверхности тела. Отмечается сильная боль, ограниченность подвижности в области ран. На ожоговых ранах отмечается гиперемия, отеки, десквамированный эпидермис. Больному назначено проведение комплексной терапии. В анализах: Нв-115 г/л, эрит.-3,2 х 1012/л, лейкоцит-15,7 х 109/л, СОЭ-40 мм/час, Диуреза сохранен.

Определите период ожоговой болезни.

Ожоговый шок I ст.

9. В ожоговом отделении на стационарном лечении находится больной 19 лет с термическим ожогом пламенем I-II-III степени. S-35 (13)% поверхности тела. Локально имеется отторжение некротических тканей с обнажением легко кровоточащей раневой поверхности. Судя по локальным данным у больного какой период ожоговой болезни.

Септикотоксемия

10. В приемное отделение поступил пациент А, 34 года, с жалобами на раны лица, отеки, также раны на животе. Из анамнеза, травму связывает с походом в горы, был на солнце целый день. Общий статус, кожные покровы вне ран бледно розовые, чувствительность сохранена. Дыхание везикулярное, проводиться хрипов нет. Сердечные тоны ритмичны правильны. Живот обычной формы, мягкий, безболезненный. Стул и диурез состоятельны. Локальный статус. Раны на лице, спине и пояснице, гиперемия ран, отеки лица, век, отслаивающиеся эпидермис, эпителиальные пузыри, чувствительность сохранена. Общая поверхность ран до 10% поверхности тела. Ваш диагноз?

Солнечный ожог I-II ст. лица, спины, поясницы площадью 10%

Ожоговая болезнь (индусы)

Применение

1. У больного 12 лет с термическим ожогом пламенем I-II-III с общей площадью 35% и глубоким ожогом 20% поверхности тела. На 5 сутки отмечается образование сухого некротического струпа. Укажите оптимальный метод лечения данного больного.

Некрэктомия

2. У больного А. 22 лет с термическим ожогом пламенем II-III с общей площадью 5%

все глубокие, отмечается образование влажного некротического струпа. Укажите оптимальный метод хирургического лечения данного больного.

Некрэктомия

3. У больного 83 лет с химическим ожогом концентрированным раствором серной кислоты I-II-III степени общая площадь ожоговой поверхности составляет 8%

поверхности тела. На 12 сутки раневая поверхность покрыта некротическим струпом твердой консистенции.

Ваша тактика лечения.

Открытый способ ведения. Консервативное лечение

4. У больного 50 лет с ожогом и электротравмой, при контакте с оголенной проводкой тела, на правой верхней конечности. В области раны поверхность покрыта некротическим струпом твердой консистенции, струп охватывает конечность по всей окружности, дистальная часть конечности холодная, бледная

Ваша тактика лечения.

Некротомия с параллельными разрезами в продольном направлении

5. В ожоговое отделение поступил ребенок 8 месячного возраста с термическим ожогом горячей жидкостью туловища, обеих верхних и нижних конечностей I-II степени. S- 45%. Локально: ожоговые раны расположены на передней и боковых поверхностях туловища, обеих бедрах и предплечьях. На фоне гиперемии и отека имеются участки с отсутствующим эпидермисом. Раневые поверхности розовой окраски, имеется плазморрея. Чувствительность сохранена.

Какой метод местного лечения является в данном случае оптимальным?

Туалет раны. Закрытый способ ведения (наложение раневых повязок)

6. В ожоговое отделение поступил ребенок 8 летнего возраста с термическим ожогом горячей жидкостью туловища I-II степени. S-10% поверхности тела. Локально: ожоговые раны расположены на передней и боковых поверхностях туловища. На фоне гиперемии и отека имеются участки с отсутствующим эпидермисом. Раневые поверхности розовой окраски, имеется плазморрея. Чувствительность сохранена.

Какой метод местного лечения является в данном клиническом случае оптимальным?

Открытый способ ведения. Обработка антисептическими средствами

7. В приемный покой ожогового отделения поступил больной 12 лет с наличием химического ожога раствором каустической соды I-II-III степени площадью 15% поверхности тела. Время, прошедшее после получения травмы – 2 часа.

Каковы правила оказания первой помощи.

Промывание раны водой, затем обработать 2% раствором уксусной или лимонной кислоты

8. В приемный покой ожогового отделения поступил больной 15 лет с наличием химического ожога от серной кислоты, I-II-III степени площадью 3% поверхности тела. Время, прошедшее после получения травмы – 2 часа, рана была обработана каустичекой содой.

Каковы правила оказания первой помощи.

Удалить остатки соды. Некрэктомия. Инфузионная терапия

9. В ожоговое отделение поступил больной 38 лет с диагнозом «Электротравма. Электротермический ожог I-II-III степени обеих верхних и нижних конечностей. S-30 (III степени 12) % поверхности тела. Индекс Франка – 54 единиц. Шок II степени». В области обеих предплечий отмечаются участки с циркулярными глубокими ожогами.

Какова тактика лечения.

Противошоковая терапия. Некротомия

10. Больной 32 лет поступил в ожоговое отделение с электротравмой. Раны локализуются в теменной области и в обеих верхних конечностях. На 15 сутки во время операции отсроченная некрэктомия у больного выявлен остеонекроз костей свода черепа. Размер дефекта составляет 18 х 30 см. При рентгенографии отмечается изменения в кортикальном слое. При рентгеноденситометрии отмечается снижение минерализации этой области. С каким специалистом комбустиолог продолжит вести больного в дальнейшем?

Нейрохирург

11. Больной 78 лет с общей площадью поражения 50% с площадью глубокого ожога 35%. Сопутствующее заболевание ишемическая болезнь сердца и гипертоническая болезнь. Определите необходимую концентрацию и площадь одномоментного использования мази салициловой кислоты для проведения химической некрэктомии.

30-40%???

12. В ожоговое отделение на скорой помощи привезли пациента с ожогом. Локальный статус, рана на правом бедре, расположена циркулярно, на нижней трети, нога сильно отекла, гиперемирована, чувствительность в области раны снижена, отеки

ниже раны, пульс сохранен, ослаблен, чувствительность кожи снижена, температура снижена, бледная. Ваша дальнейшая тактика

Некротомия

13. В приемное отделение обратился за помощью пациент С, с жалобами на раны. Из анамнеза, со слов пациента, за день до этого пролил кофе на руку. Общий статус: кожные покровы вне ран бледно розовые. Дыхание и гемодинамика стабильны. Живот обычной формы, мягкий безболезненный. Стул и диурез состоятельны. Локальный статус: рана на левой руке, в области средней трети предплечья составляет до 0,4% поверхности тела, гиперемирована, чувствительность сохранена, есть отслаивающийся эпидермис, эпителиальные пузыри, чувствительность сохранена. Ваша дальнейшая тактика лечения

Местное лечение (индусы)

Вскрытие пузырей, наложение асептической повязки с мазью

14. В ожоговом отделении, проходит лечение пациент, с диагнозом: Ожог пламенем 3 степени туловища, обеих верхних конечностей 10 (1) % поверхности тела. Из анамнеза, со слов матери пациентки, ребенок получил ожоги в результате пожара, 16 дней назад. Общий статус: кожные покровы вне ран бледно розовые, чистые, чувствительность сохранена. Дыхание везикулярное, хрипов нет. Тоны сердца ритмичные, ясные. Живот обычной формы, мягкий, безболезненный. Локальный статус: раны на груди, до 0,1% поверхности тела, заживают краевой эпителизацией, чувствительность сохранена, чистые, бледно розовые. Ваша дальнейшая тактика лечения

Открытый способ ведения с обработкой антисептическими растворами

Эластичное бинтование (индусы)

ОЗЗ (163)

ЗАПОМИНАНИЕ

1)Какой из факторов, определяющих здоровье населения, лидирует? A. экологический

B.биологический

C.образ жизни

D.уровень организации медицинской помощи

E.наследственный

2)Какие количественные и качественные закономерности определяют состояние здоровья населения и развитие системы здравоохранения?

A.статистика здравоохранения

B.общественная статистика

C. медицинская статистика

D. демографическая статистика

E. статистика здоровья населения

3)Что составляет второй этап статистического исследования?

A.составление плана и программы исследования

B.составление единицы наблюдения и ее признаков

C.составление макетов статистических таблиц

D.составление способов формирования статистической совокупности

E.сбор материала

4)Какой показатель характеризует частоту и силу распространения явления в среде, в которой оно прроисходит непосредственно и органически связано с ней и порождается этой средой?

A.экстенсивный

B.интенсивный

C.наглядности

D.соотношения

E.совокупности

5)Какой показатель позволяет демонстрировать сдвиги явления во времени или по територии, не раскрывая истинного уровня этого явления?

A.экстенсивный

B.интенсивный

C.наглядности

D.показатель

E.совокупности

6)Сезонность заболевания иллюстрируется графиками:

A.На оси координат

B.Секторной диаграммой

C.Столбиковой диаграммой

D.Картадиаграммой.

E. Радиальной диаграммой

7)Какие существуют виды вариационного ряда? A. сложный и простой

B.комбинационный и сгрупированный

C.простой и сгруппированный

D.сложный и комбинационный

E.простой и комбинированный

8)Что из перечисленного входит в обязанности семейного врача

•ведение амбулаторного приема, посещений больных на дому, оказание неотложной помощи;

•проведение комплекса профилактических, лечебно-диагностических и реабилитационных мероприятий, направленных на раннюю диагностику заболеваний, лечение и динамическое наблюдение больных;

•проведение экспертизы временной нетрудоспособности;

•организация медико-социальной и бытовой помощи совместно с органами социальной защиты и службами милосердия одиноким, престарелым, инвалидам, хроническим больным;

•проведение санитарно-просветительной работы по гигиеническому воспитанию населения, пропаганде здорового образа жизни, планированию семьи;

•ведение утвержденных форм учетной и отчетной документации.

9) Выберите правильный состав ГСВ (группы семейных врачей) из перечисленного

это объединение врачей терапевтов, педиатров,

акушеров-гинекологов

10)Какие значения коэффициента корреляции соответствуют прямой сильной связи?

0,7-1

11)О какой силе связи между явлениями свидетельствует коэффициент корреляции равный 1.0? сильная

12)Какие группы потребностей включает теория Альдерфера?

A.существования, связи, роста

B.самосохранения, самовыживания

C.связи, самовыражении, роста

D.роста, самосохранения, самовыживания

E. связи, роста, самосохранения

13) По теории МакКлелланда потребность властвовать включает в себя:

A.власть ради превосходства

B.власть ради самоутверждения

C.власть ради власти и достижения целей

D.власть ради самовыражения

E.власть ради достижения целей и самовыражения

14) Какой ряд означает совокупность однородных величин, отражающих изменение какого-либо явления за промежуток времени?

A.вариационный

B.интервальный

C.динамический

D.ранжированный

E.дискретный

15) В структуре национального состава населения Кыргызской Республики второе место занимают:

узбеки

16)К какому типу возрастной структуры населения относится Кыргызская Республика

прогрессивная

17)Выберите возрастной диапозон фертильного возраста для женщин.

15-49 лет

18) К какому виду движения населения относится изменение численности и состава населения в связи с рождаемостью и смертностью?

A.механическосму

B.социальному

C.естественному

D.равномерному

E.демографическому

19) Какой показатель относится к общей заболеваемости?

A. первичный;

B.госпитализированный

C.неэпидемический

D.инфекционный

E.профессиональный

20) На основании, какого первичного статистического документа изучают смертность населения?

A.статистического талона регистрации случая смерти

B.протокола (карты) патологоанатомического исследования

C.медицинского свидетельства о смерти

D.сведения о лечебно-профилактическом учреждении

E.статистической карты выбывшего из стационара

21)В структуре инфекционной заболеваемости дыхательных путей населения первое место занимают:

A.корь

B.коклюш

C.ОРВИ и грипп

D.эпидемический паротит

E.ветряная оспа

22) Что из нижеследующего занимает первое место в структуре причин первичной инвалидности взрослых?

A.болезни органов дыхания

B.болезни нервной системы

C.несчасные случаи, травмы и отравления

D.болезни системы кровообращения

E.профессиональное заболевание

23) Врачи каких учреждений устанавливают факт временной утраты трудоспособности

иимеют право выдачи листка нетрудоспособности? A. станций скорой медицинской помощи

B.станций переливания крови

C.поликлинических учреждений

D.санаториев, домов отдыха

E. организаций санэпид надзора

24)Какие три группы методов используют руководители медицинских учреждений системы здравоохранения в процессе управления трудовыми ресурсами?

Организационно-распорядительные,экономические, социально-психологические

25)Какими тремя основными видами навыков должен обладать руководитель учреждения?

Концептуальные навыки,социальные навыки, технические навыки

26) В каких четырех основных областях учреждения сферы здравоохранения руководитель должен создавать культуру, которая повлияет на деятельность организации?

-обслуживание, ориентирование на клиента

-постоянное улучшение качества

-сотрудники

-заинтересованные лица

27)Какой процент людей представляет класс «Новаторов» в организации?

2,5%

28)На каких основных принципах строится организационно-правовая и финансовая основа обязательного медицинского страхования в Кыргызской Республике?

Принципы ОМС:

Обязательность

Государственное регулирование.

Солидарность

Застрахованность

Договорной принцип

Гласность

29)Какой принцип является ошибочным для ОМС (обязательное медицинское страхование)?

Принципы ОМС:

• Обязательность

Государственное регулирование.

Солидарность

Застрахованность

Договорной принцип

Гласность

30)Какая характеристика качества медицинской помощи является ошибочной?

Элементы качества медицинских услуг с различных позиций

1. Система здравоохранения:

доступность

безопасность

оптимальность

удовлетворенность

2. Поставщик медицинских услуг:

компетентность

востребованность

улучшение статистических показателей

3. Пациент (аспекты по удовлетворенности пациента полученной медицинской услугой):

улучшение состояния здоровья;

своевременность, удобство и вежливость;

соответствие цены медицинских услуг возможностям пациентов

31) Какой процент соответствия стандартам должен быть для решения МАК (медицинская аккредитационная комиссия) о присуждении учреждению второй категории?

От 50 до 69%

32)Какое требование к клиническим протоколам является ошибочным?

Возможность разного толкования

33)Задачей первичной профилактики является:

устранение и ослабление существующих факторов риска и информационную работу

34)Комплекс каких мероприятий относится к вторичной профилактике?

Медицинские,социальные, санитарно-гигиенические,психологические

35)Основным принципом укрепления здоровья (Оттавская хартия ВОЗ по укреплению здоровья населения) является:

A. уровень благосостояния

B.структура материальных и духовных потребностей

C.политика, способствующая улучшению здоровья и изменению уровня жизни

D.политика, способствующая улучшению здоровья и изменению уровня жизни

E.санитарная культура

36) В каком году была утверждена Алма-Атинская декларация об улучшении организации медицинской помощи на первичном уровне здравоохранения:

A.1947 г.

B.1978 г.

C.1986 г.

D.1990 г.

E.2012 г.

37) Программы ВОЗ по охране и укреплению здоровья населения, работающих в Кыргызской Республике:

A.«Здоровые лицеи»

B.«Здоровые районы»

C.«Здоровый ВУЗ»

D.«Здоровый детский сад»

E.«Здоровый город»

38) В каком году была утверждена Оттавская Хартия Всемирной организации здравоохранения по укреплению здоровья:

A.1946 г.

B.1947 г.

C.1976 г.

D.1986 г.

E. 1991 г.

39) Отчетная форма для областных КУЗ (кабинеты укрепления здоровья):

A.№49

B.№50

C.№ 50y

D.№50-1

E.№50-2

40) Что представляют сельские комитеты здоровья (СКЗ)

A.организация, являющаяся частью ГСВ

B.международные проекты

C.организация сообщества в селе

D.айылокмоту

E.группа независимых сельских экспертов.

41) Каким приказом Министерства здравоохранения Кыргызской Республики утверждены положения и структура республиканского центра укрепления здоровья.

A.приказ №120 от 05.06.2006 г.

B.приказ №132 от 02.04.2008 г.

C.приказ №281 от 24.08.2001 г.

D.приказ №322 от 26.07.2001г.

E.приказ №531 от 13.01.2009 г.

42) Закон Кыргызской Республики «О защите населения от вредного воздействия табачного дыма» гласит:

A.запрет курения на рабочем месте

B.косвенная реклама табачных изделий

C.реализация штучных сигарет только в супермаркетах.

D.реализация табачных изделий с 21 года после предъявления паспорта.

E.запрет реализации альтернативных видов табачных изделий.

43)Укажите административное подчинение Службы укрепления здоровья (СУЗ) Кыргызской Республики: Министерство здравоохранения КР

44)Назовите основную цель организации Службы укрепления здоровья в Кыргызской Республике: улучшение состояния здоровья населения КР(укрепление и

сохранение здоровья, профилактика заболеваний)

45)Что является основной задачей Службы укрепления здоровья в Кыргызской Республике:Реализация долгосрочной стратегии страны “Здравоохранение КР В 21

веке

46)Наиболее эффективной структурой Службы укрепления здоровья на первичном уровне здравоохранения является:

КУЗ (кабинет укрепления здоровья)

47)Выберите фактор, оказывающий наибольшее влияние на здоровье населения:

A.экологический

B.биологический

C.образ жизни

D.уровень организации медицинской помощи

E.наследственный

48)Что означает экономическая эффективность медицинского учреждения?

соотношение экономического эффекта с затратами на его достижение

49)

Как определяется рыночная цена медицинских товаров и услуг?

цена спроса и предложения

50)Из каких источников складывается финансовые ресурсы здравоохранения?

Существует 3 основных источника финансовых ресурсов:

вновь созданный за определенный период времени чистый доход;

использование в качестве источника ресурсов части национального богатства, ранее накопленных средств;

заимствование, привлечение средств из внешних источников.

51)Укажите показатели финансовых результатов организации здравоохранения. Валовая прибыль

52)Какой показатель характеризует отношение прибыли к среднегодовой стоимости основных фондов?Рентабельность основных производственных фондов

53)Из каких категорий складываются валовые затраты медицинских учреждений.Сумма переменных и постоянных затрат

54)Назовите свойства медицинской услуги как товара.Потребительская стоимость

55)Какой характер носят источники финансирования профилактических мероприятий?Комплексный

56)Укажите факторы, снижающие цены на медицинские услуги.Сокращение спроса

при неизменном предложении или при неизменном спросе увеличится предложение

57)Произведение цены одной услуги определенного вида на количества услуг этого же вида это: Выручка

58)Наиболее эффективной структурой Службы укрепления здоровья на первичном

уровне здравоохранения является:КУЗ (кабинет укрепления здоровья)

59) На что направлены мероприятия по первичной профилактике?устранение и

ослабление существующих факторов риска и информационную работу

ПОНИМАНИЕ

1.В университете обучается 6500 студентов, в том числе на 6 курсе 290 человек, из них курящих 25 студентов. Были обследованы студенты 6 курса на случай заболеваний органов пищеварения в зависимости от курения: из числа курящих у 5 студентов выявлена язвенная болезнь желудка; у 20 – хронические гастриты; из

числа некурящих соответственно – 12 и 36. Определите вид совокупности.

А. генеральная

В. выборочная

С.генерализованная D. центральная

Е. централизованная

2. Результаты измерение массы тела у мальчиков в возрасте 16 лет (вариационный ряд):

Масса тела(кг)

55

56

57

58

59

60 всего

Число лиц

2

2

4

6

7

9 30

Что характеризует V-60 (варианта-60) в данном примере? v-60-характеризует

максимальное значение массы тела у мальчиков в 16 лет в данном вариационном ряду

3.Что нужно понимать под принципом семейной медицины

«координация»?Направление на консультацию узкого специалиста

4.16-летний мужчина на приеме у семейного врача жалуется на болезненное мочеиспускание, периодический зуд в мочеиспускательном канале. Какая из нижеперечисленных обязанностей семейного врача наиболее важна в данном случае?

Разъяснение полового воспитания и проблемы планирования семьи

5.Связь между возрастом и распространенности пневмокониозом среди шахтеров равна (-0,5). Какая связь между двумя признаками?

Обратная (отрицательная) связь

6.В городе N численность населения на 2020 год составила 19066 человек из них 8000 приходится на долю лиц от 0-14 лет, 7366лица трудоспособного возраста и 3700лица старше 50 лет. Какому типу возрастной структуры населения относится данный город?

прогрессивный

7.В области А за 2019 год показатель материнской смертности составило - 8,6 00 в т.ч.

женщин от аборта – 2,4 00, от сепсиса на 35 день после родов -1,0 00, от самоубийства на 12 неделе беременности -2,0 00, от позднего токсикоза на 26 неделе беременности

– 2,6 00

и от сепсиса на 35 день после родов -0,6 00. Найдите ошибку в данном

примере?

от самоубийства на 12 неделе беременности не относится

к материнской смертности

8.Мужчина 30 лет работает бухгалтером в одной из государственных юридических компании города N, поступил в поликлинику №2 города N с симптомами ОРВИ. Какому субьекту медицинского страхования относится юридическая компания? Страхователь

9.Вы ведете частную медицинскую практику в Кыргызстане, имеете лицензию на занятие данным видом деятельности и осуществляете медицинские, профилактические, реабилитационные, оздоровительные услуги. Кого в системе медицинского

страхования Вы будете представлять?

Поставщик

10.К функциям какого субъекта системы управления качеством медицинских услуг в КР относятся следующие мероприятия: формирование политики финансирования ЛПУ; определение перспективных направлений в обеспечении эффективного управления качеством медицинского обслуживания; рационализация сети ЛПУ; подготовка кадров? МЗКР (организации и органы здравоохранения)

11.Медицинская услуга считается качественной для стран постсоциалистического пространства если: если медицинская услуга излечивает

12.Одной из основных задач работы специалиста по укреплению здоровья является

проведение интенсивной информационной работы среди населения. Существует несколько каналов коммуникации. Какой из этих каналов является приемлемым при передаче информации по профилактике короновирусной инфекции:

-Графические и аудиовизуальные

-СМИ

13.В университете обучается 6500 студентов, в том числе на 6 курсе 290 человек, из них курящих 25 студентов. Были обследованы студенты 6 курса на случай заболеваний органов пищеварения в зависимости от курения: из числа курящих у 5 студентов выявлена язвенная болезнь желудка; у 20 – хронические гастриты; из числа некурящих соответственно – 12 и 36. Определите объект наблюдения.

А. все студенты

В. курящие студенты С. некурящие студенты

D. больные с хроническими гастритами Е. больные с язвенной болезнью желудка

14.В университете обучается 6500 студентов, в том числе на 6 курсе 290 человек, из них курящих 25 студентов. Были обследованы студенты 6 курса на случай заболеваний органов пищеварения в зависимости от курения: из числа курящих у 5 студентов выявлена язвенная болезнь желудка; у 20 – хронические гастриты; из числа некурящих соответственно – 12 и 36. Определите вид совокупности.

А. генеральная

В. выборочная

С.генерализованная D. центральная

Е. централизованная

15.На вопрос преподавателя: «Что характеризуют экстенсивные показатели?», студент ответил: «Экстенсивные показатели характеризуют распределение целой совокупности на ее составные части, частоту, удельный вес, долю и структуру среды». Укажите параметр, который отсутствует в определении экстенсивного показателя.

А.частота

В. доля С. часть

D. структура Е. среда

16.Какой из коэффициентов рассчитывается по формуле: в числителе – часть явления, в знаменателе – целое явление, умноженное на 100:

A.интенсивности В. соотношения

С. экстенсивности

D.наглядности

E.вариации

17.Число акушер-гинекологов в Кыргызской Республике на конец соответствующего

года: 1990=990, 1991=1100, 1992=1210,1993=2000. Какой тип динамических рядов показан в данном примере?

А. моментальный

В. Интервальный

С. Моментный

D. инертный

Е. временный

18.К какой характеристике качества медицинской помощи относятся действия медицинского учреждения по достижению соответствия оказанной помощи ожиданиям, пожеланиям и надеждам пациентов и их родственников?

А. приемлемость

В. эффективность

C. оптимальность.

D адекватность

Е. справедливость

19.Главный врач стационара определил проблему избытка коечного фонда, в связи с чем для изучения лечебной деятельности в отделениях составил программу и план исследования. Определите этап статистического исследования.

A.1

B.2

C.3

D.4

E.5

20.На вопрос преподавателя: «Что характеризуют интенсивные показатели?», студент ответил: «Интенсивные показатели характеризуют долю, частоту, распространенность явления в среде». Укажите параметр, который отсутствует в определении интенсивного показателя.

А. частота

В. доля

С. распространенность D. Явление

Е. среда

21.Диаграммой, наиболее наглядно характеризующей показатель динамики коэффициента материннской смертности в городе N за 2015-2020 гг. служит:

А. внутристолбиковая В. картодиаграмма С. секторная

D. линейная

Е. радиальная

22.О каком разнообразии признака вокруг средней величины свидетельствует уровень коэффициента вариации равный 15%?

А. слабой

В. средней

С. большой

D. сильной

Е. малой

23.Какой метод вычисления средней арифметической величины рассчитывает вариационный ряд, в котором каждая величина представлена единичным наблюдением?

А. простая

В. взвешенная

С. сложная

D. сгруппированная

Е. несгруппированная

24.Что нужно понимать под принципом семейной медицины «интеграция»?

А. разбор во всех жалобах и проблемах, связанных со здоровьем В. установление доверительных отношений между врачом и пациентом С. лечение отдельного лица от момента рождения до его смерти

D. направление на консультацию специалистов бузкого>> профиля

Е. разбор во всех лечебных аспектах, включая охрану здоровья

25.Что из перечисленного может служить примером для корреляционной связи?

А. количество потребляемой калории в сутки и продолжительности сна

В. количество потребляемой жидкости и выделяемой мочи в сутки

С. уровень артериального давления и температуры тела человека

D. частота сердечных сокращений и уровень холестерина сыворотки

Е. продолжительность сна и регулярность менструального цикла

26.Во сколько раз коэффициент корреляции должен превышать свою ошибку, чтобы он считался достоверным?

А. в 1,2 раза

B. в 1,5 раза С. в 2,5 раза

D. в 2,8 раза

Е. в 3,2 раза

27.Предки кыргызов — енисейские кыргызы — были одним из древнейших кочевых скотоводческих народов, и в наши дни кыргызские фермеры в летние периоды времени поднимаются в горные и предгорные районы республики (в джайлоо), где занимают летние пастбища. Какая миграция осуществляется в данном примере?

А. сезонная

В. социальная С.естественная D.маятниковая Е. временная

28. В области А за 2019 год показатель материнской смертности составило - 8,6 00 в т.ч. женщин от аборта – 2,4 00, от сепсиса на 35 день после родов -1,0 00, от

самоубийства на 12 неделе беременности -2,0 00, от позднего токсикоза на 26 неделе беременности – 2,6 00 и от сепсиса на 35 день после родов -0,6 00. Найдите ошибочное утверждение в данном примере?

A. aбopт-2,4 %‰000

В. сепсис на 35 день после родов-1,000

С. самоубийства на 12 неделе беременности -2,0 %

D. поздний токсикоз на 26 неделе беременности - 2,6 Е. сепсис на 35 день после родов -0,6

29.У мужчины 49 лет наступила стойкая нетрудоспособность после перелома шейки бедра, полученного при падении с лестницы во время ремонта своей квартиры. Какая инвалидность ему будет установлена?

А. от профессионального заболевания

В. до начала трудовой деятельности

С. вследствие общего заболевания

D. вследствие специальных заболевании

Е. вследствие бытовой травмы

30.К каким элементам культуры организации относятся следующие условия: работники организации поддерживают и словесно отстаивают качества, такие как честность, прямота, надежность и справедливость, в одной организации могут ценить готовность подчиняться, в другой — творчество и независимость, а в третьей — чистоту и порядок? А. атрибуты поведения

В. ценности работников

С. основные убеждения

D. цели и задачи организации

Е. личные мотивы работников

31.К функциям какого субъекта системы управления качеством медицинских услуг в Кыргызской Республике относятся следующие мероприятия: формирование политики финансирования организаций здравоохранения; определение перспективных направлений в обеспечении эффективного управления качеством медицинского обслуживания; рационализация сети организаций здравоохранения; подготовка кадров?

А. министерство здравоохранения и социального развития

В. управление организаций медицинской помощи и лекарственной политики С. отдел лицензирования медицинских и фармацевтических услуг

D. Фонд обязательного медицинского страхования

Е. ассоциация больниц, ассоциация групп семейных врачей

32.Медицинская услуга считается качественной для стран постсоциалистического пространства если:

А.избавляет от боли

В. излечивает от болезни

С. предупреждает болезнь

D.избавляет от страданий

Е. устраняет рецидив болезни

33.В 2018 году среди людей пожилого возраста отмечались высокие показатели заболеваемости по сердечно–сосудистым заболеваниям, а также по причинам смертности. Определите основные профилактические меры на государственном уровне по снижению этих показателей:

А. обучить студентов геронтологии и гериатрии

В. увеличить постепенно пенсионный возраст

С. обеспечить бесплатными лекарственными препаратами

D. запретить трудовую деятельность пожилых людей

Е. провести анализ уровня пенсионного обеспечения

34.Врач ГСВ одного из ЦСМ при проведении санитарно просветительных мероприятий, в период эпидемии гриппа наметил прочесть лекцию среди рабочих одного из предприятия. Назовите ошибочную тактику врача в данной ситуации:

А.выбрать тему лекции: «Профилактика гриппа»

В.объявить заранее, чтобы обеспечить явку всех работающих С.организовать лекцию продолжительностью один час

D.показать после лекции санитарно-просветительный фильм <<Профилактика гриппа>>

Е.вручить каждому из присутствующих памятки по этой тематике

35.Одной из основных задач работы специалиста по укреплению здоровья является проведение интенсивной информационной работы среди населения.Существует несколько каналов коммуникации. Какой из этих каналов является приемлемым при передаче информации по профилактике короновирусной инфекции:

А. телевидение

В. беседы С. тренинги D. собрания

Е. лекции

36.Укажите стратегию по охране здоровья детей первого года жизни, которая наименее эффективна:

А. младенческая иммунопрофилактика В. пропаганда грудного вскармливания С. сокращение высокой смертности

D. обеспечение безопасных родов

Е. рационализация питания матери

37.Укажите стратегию, разработанную по сокращению материнской смертности, которая наименее эффективна:

А. обеспечить безопасные роды в условиях родильных домов В. обеспечить доступность услуг по планированию семьи С. стимулировать общественность к привлечению инвестиций

D. обеспечить доступность медицинских услуг при осложнениях криминальных аборто

Е. регулировать возраст вступления женщин в брак

38.При значении коэффициента вариации (Cv), равном 15%, разнообразие признака:

А. очень слабое В. слабое

С. среднее

D. высокое

Е. очень высокое

39.При значении коэффициента вариации (Cv), меньше 10%, разнообразие признака:

А.очень слабое

В. слабое

С. среднее D. высокое

Е. очень высокое

40.Изменение какого из нижеперечисленных параметров влияет на уровень средней ошибки средней арифметической и относительной величин?

А. число наблюдений

В. абсолютный прирост С. величина темпа прироста D. коэффициент вариации

Е. состав сравниваемых совокупностей

ПРИМЕНЕНИЕ

1.Исследователь проводил исследование на студентов, сначала всех студентов сгруппировал по возрасту, затем из каждой возрастной группы производил случайный отбор. Выберите способы отбора единицы наблюдения выборочной совокупности:

Типологический

2. Число заболеваний населения в г. Н. некоторыми заболеваниями за 2019

Нозологическая форма

Эхинококкоз

Паротит

Гепатит

Всего

Число заболеваний

180

3700

9900

13780

Население города – 130 000 человек.

Вычислить частоту всего заболеваний на 1000 населения.

106,0%

3.Вычислите коэффициент вариации, если M=34 а ∂=2,3.

6,76

4.Исследовано 120 больных с абсцессом головного мозга, из них у 52 обнаружены дистрофические изменения пародонта. Основываясь на вышеперечисленных данных, вычислите среднюю ошибку относительной величины (mP). +/-4,52

5.Какую диаграмму целесообразнее применять для отражения гнойничковых заболеваний пальцев правой кисти в некоторых отраслях промышленности?

Пальцы

1 палец

2 палец

3 палец

4 палец

5 палец

Всего

В % к итогу

35,2

28,9

24,0

8,9

3,0

100,0

Секторные (столбиковые, круговые)

6.В 2019 году в городе N численность населения составило 135000 человек рождаемоть - 17‰; смертность - 6‰. Чему равен естественный прирост населения в городе N?

17-6=11%

7.Численность населения в городе S в 2020 году – 340 000 человек; родившихся в данном году – 36 000 детей. Вычислите показатель рождаемости?

36000/34000*1000=105,8 %

8.Численность населения в городе N в 2020 году – 453 000 человек; родившихся в данном году – 23 000 детей, так же в этом году число женшин фертильного возраста составило 231 000. Вычислите показатель плодовитости?

23000/231000*1000=99,5

9.ЦСМ № 6 обслуживают 13200 жителей. Число вновь зарегистрированных заболеваний серца, в частности гипертоническая болезнь – в 2020 году составило 540 больных. Чему равен показатель болезненности гипертонической болезни?

540/13200*1000=40,9

10.ЦСМ № 2 обслуживают 6545 жителей. Число вновь зарегистрированных заболеваний косно-мышечной системой – в 2020 году составило 700 больных, из них 250 выявлены впервые. Чему равен показатель собственной заболеваемости косно-мышечной системой?

250/6545*1000=38,1

11.Какую теорию лидерства необходимо применить руководителю, если труд в коллективе процесс естественный, люди принимают на себя ответственность и стремятся ей следовать, люди приобщены к организационным целям и используют самоуправление и самоконтроль, приобщение для них является функцией вознаграждения, связанного с достижением цели?

А. теория <<Y>> Дугласа МакГрегора

B. ситуационная модель лидерства Фидлера

С. теория <<Х>> Дугласа МакГрегора

D.теория великих людей

E.теория Дэвида МакКлеланда

12.Вы работаете в травматологическом пункте врачом-травматологом. За помощью обратился молодой человек с вывихом плеча. Пациенту 20 лет, студент одного из столичных университетов, гражданин Кыргызстана. В рамках какой формы медицинского страхования вы будете оказывать помощь (неотложная помощь, иммобилизация) пациенту?

ОМС, базовая программа

13.Определите валовой доход медицинского учреждения

Цена одной услуги на количество услуг

14.Укажите стратегию, разработанную по сокращению материнской смертности, которая наименее эффективна:

Регулирование возраста вступления в брак

15.В 2018 году среди людей пожилого возраста отмечались высокие показатели заболеваемости по сердечно–сосудистым заболеваниям, а также по причинам смертности. Определите основные профилактические меры на государственном уровне по снижению этих показателей: Обучение студентов геронтологии и гериатрии

16.Молодой врач проводил исследование студентов, сначала всех студентов сгруппировал по возрасту, затем из каждой возрастной группы производил случайный отбор. Выберите способ отбора единицы наблюдения выборочной совокупности

А. случай-контроль В. Механический

С. типологический

D. серийный

Е. парносопряженный

17.Численность населения в г. Н. за 2020 год составила 130 000 человек. Число заболевших за данный год – 13780 человек. Выберите частоту (интенсивный показатель) заболевания.

A.94,060

B.100,0%

C.106,0%

D.112,00 E. 118,0%

18.Численность населения в г. Н. за 2021 год составила 133 500 человек. Число заболевших за данный год – 15000 человек. Выберите частоту (интенсивный показатель) заболевания.

A.108,3%.

B.109,3%

с. 111,3%%

D.112,3%.

E.114,0%

19.Численность населения в г. Н. за 2020 год составила 130 000 человек. Число заболевших за данный год – 13780, в том числе заболевших эхинококкозом -180 человек. Выберите в структуре всех заболеваний в г. Н. за 2019 удельный вес эхинококкоза.

A.0,9%

B.1,0%

C.1,1%

D.1,2%

E.1,3%

20.За 2019 год численность населения в г. Н. составила 1 200 000, численность врачей 4561.Укажите показатель обеспеченности населения врачебными кадрами в г. Н. (на 10 тыс. нас.)

А.37,0

B.38,0%

C.39,0%

D.40,0

E.41,0%

21.Показатели рождаемости за изучаемый год (на тыс. нас.) составила:

2013 г.- 28,6; 2014г. - 26,7; 2015 г.- 25,5; 2016 г. – 27,1. Выберите показатель наглядности за 2016 год.

A.88,4%

B.90,5%

C.92,6%

D.94,7%

Е. 96,8%

22.Показатели смертности за изучаемый год (на тыс. нас.) составила:

2013 г.- 8,6; 2014г. - 6,7; 2015 г.- 5,5; 2016 г. – 7,1. Выберите показатель наглядности за 2015 год.

A.60,0%

B. 61,5%

C.62,4%

D.62,9%

E.63,9%

23.При исследовании цеховой врач обнаружил у рабочих гнойничковые заболевания пальцев правой кисти в некоторых отраслях промышленности: 1 палец (35,2%); 2 палеца (28,9%); 3 палеца (24,0%); 4 палеца (8,9%); 5 палецев (3,0%). Какую диаграмму целесообразнее применять для отражения гнойничковых заболеваний пальцев правой кисти?

А.внутристолбиковую

В. картодиаграмму

С.столбиковую

D.линейную

Е. радиальную

24.В вариационном ряду приведены результаты измерения массы тела у девочек в возрасте

15 лет: 60 кг – у 6 лиц, 61 кг – у 2, 62 кг – у 3, 63 кг – у 6, 64 кг – у 5, 65 кг – у 4, 66 кг – у 1. Укажите моду в данном примере.

A.2

B.3

C.4

D.5

E. 6

25.Средняя арифметическая величина равна 9, среднее квардратическое отклонение – 1.5. Укажите верный коэффициент вариации.

A.14,6%

B.15,6%

C.16,6

D.17,6%

E.18,6%

26.Средняя арифметическая величина равна 18, среднее квардратическое отклонение – 2. Укажите верный коэффициент вариации.

A. 9,1%

В. 10.1%

C.11,1%

D.12,1%

E.13,1%

27.16-летний подросток на приеме у семейного врача жалуется на болезненное мочеиспускание и периодический зуд в мочеиспускательном канале. Какую из нижеперечисленных обязанностей семейного врача необходимо применить к данной ситуации?

А. опрос оценки взаимоотношений между подростком и его родными

В. семейное просвещение на уровне семьи, переговоры с родителями подростка

С. опрос оценки взаимоотношений между подростком и его родителями

D. разъяснение полового воспитания и проблемы планирования семьи

Е. опрос оценки физического и психического здоровья других членов семьи в целом

28.Определите показатели, характеризующие эффективность диспансеризации пациентов с хроническими заболеваниями.

А. снижение частоты обострений, рост процента госпитализированных, увеличение выхода на первичную инвалидность В. снижение хода на первичную инвалидность, увеличение общей летальности

госпитализированных, увеличение частоты обострений С. снижение заболеваемости с временной утратой трудоспособности, рост процента госпитализированных

D. снижение процента госпитализированных, рост заболеваемости с временной утратой трудоспособности

Е. снижение заболеваемости с временной утратой трудоспособности, снижение частоты обострений

29.Что из перечисленного отображает практическое применение принципа семейной медицины «интеграция»?

А. налаживание «проблемных>> взаимоотношений с пациентом В. направление на консультацию специалиста «узкого» профиля

С. установление доверительных взаимоотношений с пациентом

D. лечение отдельного лица от момента рождения до его смерти

Е. разбор во всех лечебных аспектах, включая охрану здоровья

30.Что из перечисленного отображает практическое применение принципа семейной медицины «координация»?

А. направление на консультацию «узкого» специалиста

В. налаживание «проблемных>> взаимоотношений с пациентом

С. установление доверительных взаимоотношений с пациентом

D. лечение отдельного лица на протяжении всей его жизни. от рождения по смерти.

Е. разбор во псех лечебных и реабилитационных аспектах, включая охрану здоровья

31.Определите приемлемый метод стандартизации при известном возрастном составе населения, а также известной повозрастной смертности от злокачественных новообразований.

А. прямой

В. косвенный

С. обратный

D. керриджа

Е. пирсона

32.Определите силу связи между явлениями, если величина коэффициента корреляции равна r = 0,6.

А. слабая

В. средняя

С. сильная

 

 

 

 

 

 

 

 

 

 

 

 

 

D. Функциональная

 

 

 

 

 

 

 

 

 

 

 

 

Е, полная

 

 

 

 

 

 

 

 

 

 

 

 

 

33.Сезонные колебания случай ангины среди населения города N за 2019 год:

 

 

Месяцы

1

2

3

4

5

6

7

8

9

10

11

12

всего

Число забол.

200

210

234

300 140 134 123 134

125

343

454

234

2631

За квартал

644

 

 

574

 

 

382

 

 

1031

 

2631

Какой способ выравнивания динамического ряда применен в данном примере?

А. сглаживание ряда скользящей средней

В. Укрупнение уровня интервала

С. Укрупнение групповой средней

D. укрупнение скользящей средней

Е. сглаживание ряда групповой средней

34.В 2017 г. число заболевших было 2998, а в 2018 г. число заболевших 2337. Выберите абсолютный прирост за 2018 год.

A.532

B.543

C.661

D.672

E.876

35.В 2018 г. число заболевших было 3500, а в 2019 г. число заболевших 2954. Выберите абсолютный прирост за 2019 год.

A.519

B.526

C.536

D.546

E.556

36.В 2017 г. число заболевших было 2998, а в 2018 г. число заболевших 2337. Выберите темп прироста если за 2018 год. Численность населения (тыс)

A.19,0

B.20,0

C.21.0

D.22,0

E.23,0

37.В 2017 г. число заболевших было 2998, а в 2018 г. число заболевших 2337 Выберите темп роста за 2018 год. Численность населения (тыс)

А. 77.9

B.78,9

C.79,9

D.80.9

E.81,9

38.В 2018 г. число заболевших было 3500, а в 2018 г. число заболевших 3000 Выберите темп роста за 2018 год. Численность населения (тыс)

A.81,7

B.82,7

C.83,7

D.84,7

E.85,7

39.Численность населения в городе S в 2018 году – 200 000 человек; родившихся в данном году – 21 000 детей. Укажите показатель рождаемости?

A.101 %

B.102 %

C.103 %

D.104

E.105%

40.Численность населения в городе N в 2019 году – 350 000 человек; родившихся в данном году – 15 000 детей, также в этом году число женщин фертильного возраста составило 150 000. Укажите показатель плодовитости?

A.100

B.101

C.102

D.103

Е. 104

41.В 2019 году в городе N численность населения составило 150000 человек рождаемость - 18‰; смертность - 4‰. Чему равен естественный прирост населения в городе N?

A.10 %

B.11%

C.12 % D. 13 %

E. 14 %

42.В 2020 году в городе N численность населения составило 150000 человек рождаемость - 20‰; смертность - 5‰. Чему равен естественный прирост населения в городе N?

A.10%

B.13%

C.15%

D.17%

Е. 19%

43.Численность населения в городе D в 2019 году – 250 000 человек; число умерших в данном городе – 1800, в т.ч. число больных умерших в стационаре - 6 человек, число выбывших из стационара больных (выписанных и умерших) – 800. Определите показатель летальности в городе D?

A. 0,55%

B.0,65%

C.0,75%

D.0,85%

E.0,95%

44.Численность населения в городе N в 2019 году – 250 000 человек; число умерших в данном году – 1800. Определите показатель смертности в городе N?

А. 7,2 %.

B.8,2%

C.9,2%

D.10,2 %

E.11,2 %

45.Численность населения в городе N в 2019 году – 300 000 человек; родилось в данном году – 2123. У показатель рождаемости в городе N?

A.4,0 %

B.5,0%

С.6,0 60

D.7,0%

E.8,0 %

46.Выберите показатель материнской смертности в области A, если за год родилось живыми 21040 детей, умерло беременных: от аборта – 2, от самоубийства на 12 неделе беременности -2, от сепсиса на 50 день после родов -1.

A.9,5%

B.10,5%

C.11,5% D. 12,5% E. 13,5

47.ЦСМ № 6 обслуживают 10000 жителей. Число вновь зарегистрированных заболеваний серца, в частности гипертоническая болезнь – в 2018 году составило 620 больных. Чему равен показатель болезненности гипертонической болезни?

A.58%

B.59%

C.60%

D.61%

Е. 62%

48.ЦСМ № 2 обслуживают 3245 жителей. Число вновь зарегистрированных заболеваний костно-мышечной системой – в 2018 году составило 500 больных, из них 300 выявлены впервые. Чему равен показатель собственной заболеваемости костно-мышечной системой?

A.92,4%.

B.93,4%

C.94,4%

D.95,4%

E.96.4%

49.ЦСМ № 15 обслуживает 4000 жителей. Число вновь зарегистрированных заболеваний костно-мышечной системой – в 2018 году составило 750 больных. Чему равен показатель распространенности заболевании костно-мышечной системой?

A.186,5%

B.187,5%.

C.188,5% D. 189,5% E. 190,5%

50.Средняя численность в городе B составило 160000 человек. Число госпитализированных в 3200 человек. Выберите частоту госпитализации?

A.1760

B.1860

C.19%

D.20%.

E.21%

51.Средняя численность в городе B составило 155000 человек. Число госпитализированных в 2200 человек, в т.ч. средняя численность детей от 6-9 лет составило 3400 детей, из них госпитализированы в данном году 700 детей по различным формам заболевания. Выберите частоту госпитализации детей в возрасте 6-9 лет?

A.205,8%

B.215,8%

C.225,8%

D.235,8%

E.245,8%

52.В 2010 году в районе А число работающих – 20000 человек, в первые признанные инвалидами – 167 человек. Выберите показатель первичной инвалидности

А. 8,3%.

Б. 9,3%

B. 10,3%

Г.11,360

Д. 12,36

53.В 2018 году в районе А число работающих – 28000 человек, в первые признанные инвалидами – 167 человек. Выберите первичную инвалидность.

A.5,96%

B.6,96%

B. 7,96%

Г.8,96%

Д. 9,96%

54.В 2018 году в районе А число работающих – 28000 человек. Число состоящие на учете

– 600 человек, в том числе по заболеваемости органов дыхания-300. В структуре общей инвалидности, какой удельный вес приходится на инвалидов по заболеваемости органов дыхания?

A. 30%

Б. 40%

B. 50%

Г.60%

Д.70%

55.Какую теорию лидерства необходимо применить руководителю, если труд в коллективе процесс естественный, люди принимают на себя ответственность и стремятся ей следовать, люди приобщены к организационным целям и используют самоуправление и самоконтроль, приобщение для них является функцией вознаграждения, связанного с достижением цели?

А. теория <<Y>> Дугласа МакГрегора

B. ситуационная модель лидерства Филлера

С. теория <<Х>> Дугласа МакГрегора

D. теория великих людей

Е. теория Дэвида МакКлеланда

56.Какую теорию лидерства необходимо применить руководителю если люди в коллективе изначально не любят трудиться и при любой возможности избегают работы; у людей нет честолюбия, и они стараются избавиться от ответственности, предпочитая, чтобы ими руководили; чтобы заставить людей трудиться необходимо использовать принуждение, контроль, угрозу наказания?

А. теория <<Y» Дугласа МаккГрегора

B. ситуационная модель лидерства Фидлера

С. теория <<Х>> Дугласа МаккГрегора

D.теория великих людей

E.теория Дэвида МакКлеланда

57.Какой тип разрешения конфликта в организации следует применить если конфликтующие в процессе социально-трудовых отношений способны снять проблемы самостоятельно, в границах собственных задач и функций?

А. автономный

B.общеорганизационный

C.самостоятельный

D.публичный

E.административный

58.Вы ведете частную медицинскую практику в Кыргызстане, имеете лицензию на занятие данным видом деятельности и осуществляете медицинские, профилактические, реабилитационные, оздоровительные услуги. Кого в системе медицинского страхования Вы будете представлять?

A.страхователь B. страховщик

С. застрахованный поставщик

D.Поставщик

E.страховой агент

59.Вы работаете в травматологическом пункте врачом-травматологом. За помощью обратился молодой человек с вывихом плеча. Пациенту 20 лет, что студент одного из столичных университетов, гражданина Кыргызстана. В рамках какой формы медицинского страхования вы будете оказывать неотложную помощь пациенту?

А. обязательное медицинское страхование, дополнительная программа

B.обязательное медицинское страхование, базовая программа

C.добровольное медицинское страхование, базовая программа

D.обязательное медицинское страхование, базовая и дополнительная программа

E.добровольное медицинское страхование, дополнительная программа

60.Перед Вами, как перед руководителем медицинского учреждения стоит задача наиболее рационально использовать имеющиеся ресурсы, т.е. обеспечить наименьшую стоимость медицинской помощи без снижения ее результативности - «правильно сделать правильные вещи». Какую характеристику качества медицинской помощи Вы хотите реализовать?

А. результативность

B. эффективность

С. оптимальность

D.приемлемость

E.адекватность

61.Как руководитель медицинского учреждения Вы хотите предоставлять качественные медицинские услуги потребителям. Для этого Вам необходимо определить соответствие деятельности Вашего учреждения установленным критериям и стандартам, а также получить официальное признание возможностей и компетентности учреждения. Какую процедуру нужно пройти Вашему учреждению?

А. аудит

B. мониторинг

С. аккредитация

D.лицензирование

E.экспертиза

62.На территории обслуживания ЦСМ находится средняя школа. Перед тем как проводить информационную работу среди школьников, вы изучили данные анонимного анкетного опроса. При этом были получены следующие данные: 55,0% школьников старших классов курят, из них 28% девушек ,15,0%- употребляют спиртные напитки в кругу друзей;1,2%- употребляли 1раз наркотики;16,2% девушек старших классов плохо информированы о методах контрацепции. Выберите ваши действия в данной ситуации. А. оповестить об этом родителей:

В. поставить на учет в инспекцию по делам несовершеннолетних

С. поставить на учет в кабинет по планированию семьи

D. провести медицинский осмотр среди школьников

Е. провести тренинг среди учителей по формированию ЗОЖ

63.Распространение курения в Кыргызской Республике имеет тенденцию к увеличению, особенно среди женщин и девушек. Так, число курящих женщин в возрасте от 18 до 65 лет составляет 11,6%, среди девочек от 9 до 17 лет - 20%, а от 18 до 25 лет-47%. Выбрать рекомендации по уменьшению показателя табакокурения на уровне врачей первичного звена; А. диспансеризация женщин фертильного возраста

В. информационная работа по охране и укреплению здоровья

С. проведение скринингов по обследованию женщин

D. обязательное обследование пульмонологом

Е. исследования содержания углекислого газа в выдыхаемом воздухе

64.Определите валовой доход медицинского учреждения.

А. цена одной услуги, умноженная на количество услуг

В. доход от реализации медицинских услуг

С. выручка, умноженная на количество услуг

D. разница прибыли и налогов медицинской организации

Е. максимальная прибыль медицинского учреждения

65.Определите средние издержки медицинских товаров и услуг А. сумма переменных и постоянных издержек

В. отношение валовых издержек на объем медицинских услуг

С. разница между валовой прибылью и затратами

D. общая сумма всех затрат

Е. себестоимость медицинской продукции

66.Определите совокупную экономическую прибыль медицинской организации.

А. разницы между валовыми доходами валовыми издержками

В. совокупность чистой и балансовой прибыли С. разница между переменными и постоянными затратами

D. общая сумма реализации медицинских товаров и услуг Е. разница между прибылью и выручкой

Офтальмология (110)

Запоминание

1. Назовите, каким нервом осуществляется двигательная иннервация наружной прямой мышцы?

отводящий нерв-n.abducens (VI пара ЧМН)

ПРОВЕРЕНО

2.Назовите наиболее утолщенный участок склеры, где прикрепляются:

наиболее толстая - у заднего полюса глаза вблизи от зрительного нерва (1,0 мм).

В переднем отделе, до линии прикрепления прямых мышц, толщина склеры равна 0,8 мм.

ПРОВЕРЕНО

3.Тип ожога глаза, при котором развивается коагуляционный некроз тканей:

кислотный (химический)

ПРОВЕРЕНО

4.Автором классификации ожогов органа зрения является:

-В.В. Волков (1972),

-Б.Л. Поляк (1957),

Н.А. Пучковская и Непомнящей (1973)

ПРОВЕРЕНО

5.Симптомом поражения чего являются метаморфопсии?

макулы(сетчатка)

ПРОВЕРЕНО

6.Воспаление слезной железы называется?

дакриоаденит

ПРОВЕРЕНО

7.К какой форме воспаления относится халазион?

хроническое пролиферативное воспаление (вялотекущее гранулематозное)

ПРОВЕРЕНО

8.При ортофории угол между оптической и зрительной осями равен:

3-4 градуса

ПРОВЕРЕНО

9.Наибольшей преломляющей способностью обладает:

роговица

ПРОВЕРЕНО

10. Назовите основную функцию ресничного тела?

-продукция внутриглазной жидкости

-подвешивание хрусталика и обеспечение процесса аккомодации

ПРОВЕРЕНО

11. Назовите нерв, иннервирующий внутреннюю прямую мышцу глаза?

глазодвигательный - n.oculomotorius III пара ЧМН

ПРОВЕРЕНО

Понимание

1. Мужчина 30 лет, обратился на консультацию по поводу отклонения левого глаза кнутри и периодически возникающее двоение при попытке свести глаза. Жалобы появились после травмы головы и левого глаза около 8 месяцев назад после травмы. При объективном осмотре левого глаза: отклонение глазного яблока кнутри на 30 ° по Гиршбергу, ограничение движения глазного яблока кнаружи, среды

прозрачны. Глазное дно: без особенностей. Правый глаз без патологий. Какой нерв в данном случае поврежден у пациента? Отводящий нерв

ПРОВЕРЕНО

2. В ургентную службу глазного отделения обратился мужчина 68 лет, с жалобами на резкую болезненность, покраснение, снижение предметного зрения на правом глазу. Из анамнеза был выставлен диагноз незрелая катаракта правого глаза. При осмотре глаз инъецирован, роговица отечная, передняя камера мельче средней, зрачок широкий, хрусталик мутный, набухает. Глазное дно: не офтальмоскопируется. Какой вид вторичной глаукомы у данного пациента?

Факоморфическая глаукома

ПРОВЕРЕНО

3. При повышении внутриглазного давления, последнее давит на все оболочки одинаково, не выдерживает место выхода зрительного нерва из глазного яблока, где образуется яма-экскавация. Объясните, почему это место является наиболее уязвимым в склере?

В области выхода зрительного нерва, т.к. толщина здесь составляет 1/3 от обычной толщины + перфорирована волокнами зрительного нерва.

ПРОВЕРЕНО

4. Женщина 75 лет обратилась к окулисту с жалобами на отсутствие предметного зрения на правом глазу, со слов пациента правый глаз перестал видеть 3 месяца назад, зрение снижалось постепенно. При визометрии функции правого глаза – правильная цветопроекция, внутриглазное давление

в норме, при офтальмоскопии рефлекса нет. О какой патологии идет речь? Стадия зрелой катаракты

ПРОВЕРЕНО

5. У мужчины при объективном осмотре обнаруживается симблефарон. Со слов несколько лет назад перенес травму глаза. Осложнением какой травмы является данная патология?

-Тяжелый ожог глаз (термический или химический)

-реже хир. вмешательства

ПРОВЕРЕНО

6. Больной 25 лет, жалуется на покраснение обоих глаз, обильное гнойное отделяемое из них, чувство инородного тела, склеивание ресниц по утрам. Зрение не изменилось. Болен уже 5-й день. Началось остро, с покраснения правого глаза, через 2 дня также заболел другой глаз. Ваш предварительный диагноз?

Острый бактериальный конъюнктивит

ПРОВЕРЕНО

7. Студент 22 лет, обратился в поликлинику с жалобами на светобоязнь, слезотечение, снижение зрения, чувство инородного тела и боли в правом глазу. Из анамнеза: данное состояние отмечает после перенесенной острой респираторной вирусной инфекции. Объективно в центре роговицы инфильтрация (проба с флюоресцеином-расплывчатое помутнение в виде ветки дерева с шероховатой поверхностью). Чувствительность роговицы снижена. Visus 0D = 0,2 н/к. Visus OS = 1.0 здоровый. Ваш предположительный диагноз?

-Герпетический кератоконъюнктивит (Эпителиальный кератит -древовидный кератит)

-Эпидемический кератоконьюнктивит(аденовирус 8 типа)

ПРОВЕРЕНО

8. К терапевту, в поликлинику по месту жительства, обратился мужчина 78 лет. Жалуется на боли в правой половине головы, тошноту с позывами на рвоту. Правым глазом не различает ни одной строчки в таблице Сивцева. При взгляде на горящую электрическую лампочку видит ореол в виде радуги. При осмотре правого глаза: выраженная застойная инъекция, прозрачность роговицы снижена, за счет выраженного отека ее эпителия, зрачок расширен, на свет не реагирует; передняя камера мелкая, при пальпации глаз безболезнен, но плотный, как камень. Ваш диагноз?

Закрытоугольная глаукома (острый приступ)

ПРОВЕРЕНО

9. Мужчина 53 года обратился к окулисту по месту жительства с жалобами на покраснение, чувство инородного тела в глазах. Из анамнеза: вышеуказанные жалобы беспокоят последние 6 месяцев, по поводу чего никуда не обращался, свои жалобы связывает с условиями труда, работает грузчиком в запыленном помещении. Объективно: острота зрения обоих глаз = 0,8-0,9, глаз слегка инъецирован, отмечается частое моргание и сухость роговицы при биомикроскопии. Офтальмоскопическая картина в норме. Ваш предполагаемый диагноз?

Синдром сухого глаза

ПРОВЕРЕНО

10. Больная 38 лет обратилась к офтальмологу с жалобами на сильное жжение, снижение предметного зрения, отечность век обоих глаз. Со слов больной при кипячении посуды ошпарилась кипятком. Объективно

функции обоих глаз 0,7, веки гиперемированы, отечные, роговица деэпителизирована, внутриглазные среды прозрачные, глазное дно в норме. Термический ожог какой степени в данной ситуации?

І степени

ПРОВЕРЕНО

11. Женщина 52 лет обратилась к офтальмологу с жалобами на резкое снижение остроты зрения в обоих глазах. Из анамнеза выяснено, что пациентка работает в металлургии. Накануне выполняла срочный заказ и осталась работать в ночную смену. При офтальмологическом обследовании выявлены: гиперемия обоих век, отек и кровоизлияние в сетчатку. Что стало причиной данного состояния?

Воздействие высоких температур, проф.заболевание

Увеличении длительности воздействия факторов производственной среды, что привело к увеличению ПДК(+инфракрасное тепловое воздействие). «Офтальмопатия металлургов»

ПРОВЕРЕНО

12. Больная 35 лет обратилась в поликлинику по месту жительства с жалобами на боль, покраснение, снижение предметного зрения на правом глазу, со слов 2 недели назад получила производственную травму в цеху. Объективно: функция правого глаза 0,05 не корригируется, левого глаза 1,0. Правый глаз: глаз инъецирован, роговица слегка отечная, на радужке отмечается отложение пигмента желто-зеленого цвета, зрачок округлый, в центре, хрусталик прозрачный, глазное дно без особенностей. Ваш предположительный диагноз?

Металлоз глаз(сидероз)

ПРОВЕРЕНО

13. Мужчина поступил в клинику после удара по лицу тупым предметом. При осмотре: Острота зрения правого глаза = 1/∞ pr.l.certa, гематома век, в передней камере кровь на 1/3, зрачок широкий, хрусталик прозрачный, рефлекса с глазного дна нет. Из-за чего отсутствует предметное зрение?

Контузия тяжёлой степени из-за сотрясения сетчатки

-Гифема второй стадии (кровоизлияние в переднюю камеру)

-кровоизлияние в стекловидное тело

ПРОВЕРЕНО

14. Женщина 48 лет с миопией обоих глаз 11,0 диоптрий, передвигая в квартире тяжелую мебель, заметила в нижне-наружном участке поля зрения правого глаза несколько слабых световых вспышек (фотопсии), а затем появление полупрозрачной тени (по определению больного – «занавески»), колеблющейся при движениях глаз и ограничивающей части поля зрения правого глаза. Утром после сна тень исчезла, однако в течение дня появилась вновь и несколько увеличилась. Ваш предположительный диагноз?

Отслойка сетчатки ( регматогенная отслойка обычно при высок степ миопии)

ПРОВЕРЕНО

15. После перенесенной черепно-мозговой травмы мужчина 60л, жалуется на постепенное снижение зрения левого глаза. При офтальмоскопии выявлено: побледнение диска зрительного нерва с височной стороны, границы диска четки, сосудистый пучок в центре, артерии узкие. Назовите наиболее вероятный диагноз.

Нисходящая атрофия зрительного нерва

ПРОВЕРЕНО

16. В приемный покой обратилась женщина 48 лет, с жалобами на сильные боли и затуманивание зрения правого глаза, возникшие на фоне повышения артериального давления до 180/90 мм рт.ст. Пациентка отмечала однократную рвоту, не принесшую облегчения. При осмотре получены следующие данные: острота зрения правого глаза=0,09 не корригируется, острота зрения левого глаза = 0,1 с (+) 3,5Д=1.0; при биомикроскопии правого глаза выявлена застойная инъекция глазного яблока, отек роговицы, мелкая передняя камера, бомбаж радужки, зрачок расширен, на свет не реагирует. Уровень ВГД OD 56 мм рт.ст. по Маклакову. Ваш предварительный диагноз?

Острый приступ закрытоугольной глаукомы

ПРОВЕРЕНО

17. При исследовании ширины угла передней камеры боковой подсветкой по Вуртгафту в лимбе исследуемого глаза с носовой стороны появилась световая полоска шириной 0,5 – 1 мм. Каково ваше заключение о ширине угла в данном случае? Щелевидный

Узкий угол передней камеры

ПРОВЕРЕНО

18. Женщина 62 лет, обратилась к окулисту по месту жительства с жалобами на незначительное снижение зрения на обоих глазах, туман перед глазами, объективно: острота зрения правого глаза=0.1, левого глаза=0.2 глаз спокоен, роговица прозрачная, передняя камера средняя, зрачок округлой формы, хрусталике помутнение, глазное дно за флером. Какая стадия катаракты у больной? Стадия зрелой катаракты

ПРОВЕРЕНО

19. В ургентную службу глазного отделения обратилась женщина 64 лет, с жалобами на резкую болезненность, покраснение, отсутствие предметного зрения на правом глазу, низкое зрение на левом глазу. Из анамнеза был выставлен диагноз перезрелая катаракта правого глаза. При осмотре глаз инъецирован, роговица отечная, передняя камера средняя, зрачок широкий шире среднего, хрусталик мутный. Глазное дно: не офтальмоскопируется. Какая стадия катаракты у больной ?

Перезрелая катаракта

ПРОВЕРЕНО

20. К врачу-окулисту обратился юноша 14 лет с жалобами на периодически возникающее покраснение обоих глаз, боли, ухудшение зрения. Мальчик наблюдается у стоматолога по поводу стоматита и у дерматолога по поводу изъязвления кожных покровов половых органов. При объективном исследовании обоих глаз выявлено: перикорнеальная инъекция, гипопион, мощные задние синехии. Имеются выраженные помутнения в стекловидном теле. Какой диагноз вы поставите?

Болезнь Бехчета (увеит обоих глаз)

ПРОВЕРЕНО

21. В отделение микрохирургии глаза обратился мужчина 45 лет, с жалобами на боли, светобоязнь, слезотечение правого глаза. Из анамнеза: вышеперечисленные жалобы появились 3 дня назад, страдает хроническим гайморитом. При осмотре: VOD= 0,1. Перикорнеальная инъекция, на роговице края язвы зияют, передняя камера мелкая, зрачок узкий. Какое осложнение возможно в данном случае?

Кератоувеит

ПРОВЕРЕНО

22. К окулисту обратился мужчина 49 лет, Жалобы: умеренная светобоязнь и слезотечение обоих глаз, также отмечает, что он плохо слышит. При осмотре заметны резко выступающие лобные бугры, седловидный нос, полулунная выемка в середине передних верхних зубов. Объективно: на глазном яблоке умеренная перикорнеальная инъекция, в строме роговице у лимба диффузная инфильтрация серовато-белого цвета. Какой вид кератита у данного пациента?

Эндогенный ,сифилитический (глубокий) кератит

ПРОВЕРЕНО

23. Больной 65 лет жалуется на обильные слизисто-гнойные выделения из правого глаза. Выделяется гной из слезных точек при надавливании на область слезного мешка. Цветная проба положительная. При применении дезинфицирующих капель улучшений не наблюдается. О какой патологии идет речь?

Гнойный(хронический) дакриоцистит

ПРОВЕРЕНО

24. Больной Н., 46 лет жалуется на сильный зуд, жжение и резь в глазах, частые болезненные моргания. При осмотре выявлено выраженное покраснение по углам глазной щели, здесь кожа экзематозно изменена, имея мокнущие трещины. Отделяемое из конъюнктивы скудное, в виде тягучей слизи, застилая зрачковую область роговицы, оно мешает зрению. Какой конъюнктивит у пациента?

Аллергический конъюнктивит

ПРОВЕРЕНО

25. Больная 23 года, обратилась к врачу, по поводу заболевания правого глаза, больна второй день. При осмотре на вернем веке обнаружено локальное покраснение, припухлость, видна гнойная «головка», боль при пальпации. Ваш диагноз? Глазной ячмень (он же гордеолум)

ПРОВЕРЕНО

26. К офтальмологу обратился пациент 20 лет, с жалобами на низкое предметное зрение и косоглазие левого глаза. Объективно: острота зрения правого глаза -1,0. Острота зрения левого глаза 0,1 н/кор. Девиация кнутри левого глаза 250. Глазное дно в норме. Какую мышцу необходимо отсадить в данном случае? Отсадка внутр прямой мышцы

ПРОВЕРЕНО

27. В ургентную службу глазного отделения обратился мужчина 68 лет, с жалобами на резкую болезненность, покраснение, снижение предметного зрения на правом глазу. Из анамнеза был выставлен диагноз незрелая катаракта правого глаза. При осмотре глаз инъецирован, роговица отечная, передняя камера мельче средней, зрачок широкий, хрусталик мутный, набухает. Глазное дно: не офтальмоскопируется. Какой вид вторичной глаукомы у данного пациента? Факоморфическая глаукома ( из-за набухающей катаракты)

ПРОВЕРЕНО

28. Больной Э., 18 лет, обратился с жалобами на покраснение слизистой оболочки обоих глаз, умеренное слизистое отделяемое, склеивание ресниц по утрам, ощущение песка под веками, слезотечение. Заболел два дня назад после купания в пруду. При осмотре конъюнктива век и глазных яблок резко гиперемирована, умеренно отечная, имеются единичные фолликулы на конъюнктиве обоих глаз, умеренное слизистое отделяемое. Какое осложнение может быть при данной патологии? Бактериальный конъюнктивит,частое осложнение эрозии,язвы роговицы

ПРОВЕРЕНО

29. У пациента 27 лет отмечается покраснение правого глаза, фиолетово-голубой оттенок склеры и плач при смыкании глаза. В районной поликлинике поставили диагнозаденовирусный конъюнктивит, прописали лечение в соответствии с диагнозом, но улучшения не наблюдалось. В дальнейшем выявили постановку неправильного диагноза. Какой диагноз должны поставить для назначения правильного лечения? склерит(эписклерит)

ПРОВЕРЕНО

30. Девушка 23 лет обратилась к окулисту по месту жительства с жалобами на снижение зрения, болезненность в суставах. Данное состояние стала замечать последний месяц. Объективно: острота зрения обоих глаз = 0,4. На роговице в пределах открытой глазной щели лентовидное помутнение шириной 4 мм, на задней поверхности роговице пылевидные преципитаты. Деструкция стекловидного тела. Ваш предположительный диагноз?

Иридоциклит (передний увеит)

ПРОВЕРЕНО

31. Женщина 75 лет обратилась к окулисту ЦСМ с жалобами на отсутствие предметного зрения на правом глазу, со слов пациента правый глаз перестал видеть 3 месяца назад, зрение снижалось постепенно. При визометрии функции правого глаза – правильная цветопроекция, внутриглазное давление в норме, при офтальмоскопии рефлекса нет. Какая стадия катаракты у больной ? зрелая

ПРОВЕРЕНО

32. К дежурному врачу экстренно обратилась пожилая женщина с жалобами на боли в левом глазу, иррадиирующие в левую половину головы, слезотечение, светобоязнь, блефароспазм, покраснение глаза. При объективном осмотре: острота зрения правого глаза 1.0, левого глаза – неправильная светопроекция. Глаз инъецирован, роговица отечная, передняя камера глубокая, определяется дрожание радужной оболочки, зрачок неравномерен, во влаге передней камеры выявляют белые частицы лизированных хрусталиковых масс. Глазное дно: определяется слабый рефлекс. Какой вид вторичной глаукомы у данной

больной? Факогенная глаукома(факотоксическая глаукома)

ПРОВЕРЕНО

33. Родители ребенка 13 лет привели его в глазную клинику с жалобами на снижение зрения на левом глазу. Час назад по левому глазу попали снежком. При осмотре: острота зрения левого глаза 0,2 не корригируется. Гематома век, глаз спокоен, роговица прозрачная, в передней камере уровень крови в 1 мм, зрачок неправильной формы, широкий, хрусталик прозрачный. В передних отделах стекловидного тела – плавающее помутнение. Какая степень контузии глазного яблока у ребенка? 2 степени

ПРОВЕРЕНО

34. У пациента обнаружены следующие изменения: граница поля зрения концентрически сужена в одном или более сегментах находится менее чем в 15° от точки фиксации, краевая субтотальная экскавация диска зрительного нерва. Для какой стадии глаукомы характерны объективные данные? 3 стадия, далеко зашедшая

ПРОВЕРЕНО

35. При патронаже новорожденного педиатр заметила эффект максимально расширенного зрачка одного глаза. Из анамнеза мама во время беременности перенесла грипп. При осмотре – роговица прозрачная, виден ободок корня радужки, при боковом освещении видны контуры хрусталика и ресничного пояска. О какой врожденной аномалии вы подумали? отсутствие радужки (врожденная аниридия )

ПРОВЕРЕНО

Применение

1. Мужчина 54 года, программист по специальности обратился к окулисту по месту жительства с жалобами на покраснение, чувство песка, инородного тела, усталость в глазах. Из анамнеза вышеуказанные жалобы беспокоят последние 6 месяцев, связывает это с длительным пребыванием за компьютером. Объективно: острота зрения обоих глаз =1,0, глаз слегка инъецирован, отмечается ксероз роговицы при биомикроскопии. Офтальмоскопическая картина в норме. Был выставлен предварительный диагноз: синдром сухого глаза. Какой тест необходимо применить для уточнения диагноза?

Проба Ширмера /Норна ПРОВЕРЕНО

2. Женщина 33 лет обратилась к окулисту с жалобами на покраснение, чувство песка в глазах, усталость в глазах. Из анамнеза вышеуказанные жалобы беспокоят последние 8 месяцев, по поводу чего никуда не обращалась, работает офисным работником. Объективно: острота зрения обоих глаз =1.0, глаз слегка инъецирован, отмечается ксероз роговицы при биомикроскопии. Офтальмоскопическая картина в норме.

Был поставлен предварительный диагноз синдром сухого глаза. Какой метод необходим для уточнения диагноза?

Проба Ширмера /Норна ПРОВЕРЕНО

3. Банкир 28 лет, жалуется на светобоязнь, блефароспазм, слезотечение. Начало острое. Объективно на правом глазу перикорнеальной инъекцией, на роговице фликтенулезный инфильтрат, мелкая поверхностная васкуляризация. Выставлен диагноз туберкулезный кератит. Какие препараты Вы назначите данному больному?

Местно применять кортикостероиды в каплях (дексаметазон, 1% р-ра преднизолона) и 5-10 % ПАСК и 3% раствор изониазида(мидриатики, магнитофорез с противовоспалительной смесью, облучение гелийнеоновым лазером)

+общее лечение с фтизиатром(противотуберкулезные препараты- 1 линия изониазид рифампицин 2 линия стрептомицин, этамбутол, пиразинамид)

Лечение противотуб препаратами 1 линии после определения чувствительности, местно гкс,нпвс,противоаллерг препараты

ПРОВЕРЕНО

4. Женщина 46 лет обратилась к офтальмологу с жалобами на снижение предметного зрения обоих глазах. Со слов больной 2 года назад получила ожог щелочами. Объективно функции обоих глаз 0,01, придатки без особенностей, роговица мутная, эндотелиально-эпителиальная дистрофия, передняя камера средняя, зрачок округлый в центре, детали заднего сегмента не просматриваются. Был поставлен диагноз постожоговая дистрофия, тотальное бельмо роговицы обоих глаз. Какая хирургическая операция рекомендована в данной ситуации?Кератопротезирование

ПРОВЕРЕНО

5.Девушка обратилась в клинику с целью улучшить остроту зрения левого глаза. При объективном осмотре у пациента выявлено послеожоговое бельмо роговицы. Какая операция показана данному пациенту? Кератопластика ПРОВЕРЕНО

6.В глазной травмпункт обратилась женщина 40 лет. С жалобами на боли в глазу, слезотечение, светобоязнь, блефароспазм, покраснение правого глаза. Со слов в глаз попали несколько капель клея. С момента травмы прошло 30 минут. Какой из этих растворов необходимо назначить пациенту?

Промыть физ.раствором с последующей инстилляцией скополамина и атропина

ПРОВЕРЕНО

7. К офтальмологу о братился мужчина 60 лет с жалобой на резкую боль в левом глазу. Со слов, он вместо глазных капель, содержащих витамины, случайно закапал «какую-то жидкость». С момента закапывания до обращения прошло 15 минут. Выставлен диагноз химический ожог. Какие капли нужно закапать для облегчения состояния пациента?

После промывания в конъюнктивальный мешок необходимо закапать 20%раствор сульфата натрия и заложить АБ мазь

ПРОВЕРЕНО

8. Сварщик 30 лет обратился в глазной травмпункт с жалобами на слезотечение, чувство инородного тела, покраснение правого глаз. Со слов больного работал с металлом без защитной маски, объективно: функции обоих глаз =1,0, правый глаз инъецирован, роговица прозрачная, на роговице инородное тело металлической природы, передняя камера средняя, зрачок округлый, в центре, хрусталик прозрачный, глазное дно: без особенностей. Был поставлен диагноз: инородное тело роговицы. Какие будут ваши дальнейшие рекомендации?

-Удаление инородного тела роговицы под биомикроскопом, после анестезии каплями.Находящиеся на эпителии незначительные по размерам инородные тела могут быть удалены с помощью смоченного в растворе антисептика ватной палочкой.

-Рекомендуется инстилляция местноанестезирующего средства. Рекомендуется инстилляции антибактериальных средств.ПРОВЕРЕНО

9.Больная 38 лет обратилась в глазное отделение с жалобами на сильные ноющие боли, покраснение, отсутствие предметного зрения на правом глазу. Со слов больного 6 лет назад получил тупую травму правого глаза, к окулистам не обращался. Объективно: функции правого глаза=0 (нуль), глаз субатрофичен, роговица прозрачная, передняя камера неравномерная, зрачок неправильной формы, хрусталик мутный, глазное дно не офтальмоскопируется. Был поставлен диагноз посттравматический увеит правого глаза. Какой метод хирургического лечения необходим в данном случае во избежание симпатического воспаления? Энуклеация правого глаза ПРОВЕРЕНО

10.Мужчина получил проникающую травму глаза. Сам пациент утверждает, что в полость глаза попало инородное тело. Какой метод рентгенографии поможет определить наличие внутриглазного инородного тела? КТ ПРОВЕРЕНО

11.Больной 38 лет, обратился к окулисту с жалобами на резкое снижение зрения правого глаза. 2 месяца назад был избит неизвестными, была тупая травма справа. Объективно: острота зрения правого глаза = 0,08 не корригируется, хрусталик серый, детали глазного дна не просматриваются, но слабый рефлекс зрачка сохранен. Выставлен диагноз полная осложненная катаракта. Какая операция требуется данному пациенту?

Операция по замене хрусталика: факоэмульсификация катаракты с фемтосекундным сопровождением/ЭЭК(экстракапсулярная экстракция катаракты) ПРОВЕРЕНО

12. Больная 25 лет обратилась к врачу после контузии глазного яблока с жалобами на снижение зрения. При осмотре в проходящем свете видны темные плавающие помутнения на фоне розового рефлекса. Какой метод диагностики используется для уточнения диагноза?

УЗИ ПРОВЕРЕНО

13. Студент поступил в клинику с контузией глазного яблока. При осмотре: острота зрения правого глаза = 1/∞ pr.l.certa, гематома век, в передней камере кровь на 1/3, зрачок широкий, хрусталик прозрачный, в стекловидном теле кровоизлияние рефлекса с глазного дна нет. Какая терапия проводится в данном случае?

ГКС,АБТ, кровоостанавливающая терапия(дицинон,викасол)+через несколько дней рассасывающая терапия(в/в NaCl гиперт.р-р,лидаза,трипсин,фибринолизин) ПРОВЕРЕНО

14. Медицинская сестра обратилась с жалобами на ухудшение зрения левого глаза за последние 3 месяца. При объективном осмотре передние отрезки глаза без особенностей, оптические среды прозрачны. Зрачок уже среднего. Какие капли можно использовать для полноценного осмотра сетчатки?

Мидриатики:*М-холиноблокаторы(атропин,тропикамид), *симпатомиметики(ирифрин, мидримакс)ПРОВЕРЕНО

15. К окулисту обратился пациент 59 лет, с жалобами на низкое зрение правого глаза. Со слов больного заметил случайно. Ранее острота зрения на оба глаза была высокой. При осмотре: острота зрения правого глаза = 0,2 не корригируется, левого глаза = 1,0. Правый глаз при наружном осмотре, а также при биомикроскопии без изменений. Глазное дно: диск зрительного нерва бледный монотонный, границы его четкие. Артерии сетчатки сужены, вены обычного калибра. Очаговой патологии не выявлено. Какой дополнительный метод позволит дать полноценное заключение данному пациенту?

Компьютерная периметрия, оптическая когерентная томография, ПРОВЕРЕНО

16. Маляр 62-х лет, жалуется на низкое предметное зрение обоих глаз. При осмотре обоих глаз – диск зрительного нерва бледно-розовый, миопический конус с височной стороны, границы четкие, сетчатка растянута, разрежение пигмента, макулярная область без особенностей, по периферии правого глаза несколько мелких клапанных разрывов сетчатки на фоне решетчатой дистрофии, в левом глазу редкие зоны истончения сетчатки на фоне дистрофии "следы улитки". Выставлен диагноз миопия высокой степени, хориоретинальная дистрофия. Какова Ваша тактика ведения пациента?

Лазерная коагуляция сетчатки (лазерный барраж периферической дегенерации сетчатки)ПРОВЕРЕНО

17. В студенческую поликлинику обратился подросток 18 лет. Он предъявляет жалобы на ухудшение зрения в вечернее время. Со слов больного, симптомы появились около года назад без видимых причин. Объективно: острота зрения обоих глаз= 0,6. Передние отрезки глаза без изменений. Глазное дно: диск зрительного нерва бледноват, границы четкие, на периферии глазного дна по ходу ретинальных сосудов множественные пигментные отложения темно-коричневого цвета в виде «костных телец». Поля зрения резко сужены. Назначение какого витамина может облегчить состояние пациента?

Витамин А(ретинол) ПРОВЕРЕНО

18.Мужчина жалуется на прогрессирующее снижение остроты зрения, ухудшение ночного зрения. При осмотре острота зрения 2 на 2, вокруг глаза, диск зрительного нерва бледный, границы четкие, отложения ретинального пигмента в виде костных телец. Какая терапия проводится при данной патологии? Ретинола пальмитат (витамин А) ПРОВЕРЕНО

19.У больной 77 лет, страдающей сахарным диабетом в течение 30 лет, после обширного кровоизлияния в стекловидное тело в области зрительного нерва организовалась шварта, проминирующая в стекловидное тело. В шварте отмечается появление новообразованных сосудов. Больной следует рекомендовать:

Витрэктомия ПРОВЕРЕНО

20. Беременная женщина 30 недель, жалуется на ухудшение цветощущения, снижение остроты зрения, появление темного пятна перед глазом, искажение предметов на правом глазу. При офтальмоскопии врач обнаружил изменения в центральной зоне сетчатки. Какой тест нужно применить для уточнения диагноза в данном случае?

Тест Амслера ПРОВЕРЕНО

21. В частную клинику обратилась молодая девушка 26 лет, с жалобами на вспышки, молнии, мерцание, плавающие перед глазами черные пятна, деформацию и колебание рассматриваемых предметов, снижение остроты зрения, пелену перед глазами. Вышеуказанные жалобы беспокоят в течение последних 3х дней.

Выставлен диагноз отслойка сетчатки. Какое хирургическое вмешательство показано пациентке?

лазеркоагуляция ПРОВЕРЕНО

22. Пенсионер 75-ти лет, обратился с жалобами на резкое ухудшение зрения и сильные боли в левом глазу и левой половине головы, которые появились ночью, тошноту и рвоту. Несколько дней назад у него было тяжелое эмоциональное переживание. Объективный осмотр левого глаза: острота зрения = 0,04 (не корригируется). Внутриглазное давление = 47 мм рт.ст., глазная щель сужена, выраженная застойная инъекция глазного яблока. Роговица отечная. Передняя камера очень мелкая. Зрачок расширен до 5 мм, неправильной овальной формы. Рефлекс с глазного дна тускло-розовый. Диск зрительного нерва виден в тумане. Выставлен диагноз острый приступ глаукомы. Какие из нижеперечисленных капель улучшат самочувствие пациента?

тимолол 0,5% по 1 капле каждые 30 минут 2 дозы,

1-2% пилокарпин(карбохолин) по 1 капле каждые 15 минут, 2 дозы+диакарб 0,5 г;

глицерин ПРОВЕРЕНО

23. Больной 62 лет, обратился к врачу с жалобами на ухудшение зрения и ориентировки в пространстве. Вышеуказанные жалобы появились около полугода назад, но не вызывали особого беспокойства. Отмечал периодическое проходящее затуманивание зрения. Объективно: острота зрения обоих глаз = 0,4 с (–) 3,5 диоптрий = 1.0. Глаза спокойные. Роговицы прозрачные. Передние камеры средней глубины. Радужки субатрофичные. Зрачки слабо расширены. На глазном дне отмечается экскавация дисков зрительных нервов, их побледнение. Внутриглазное давление обоих глаз = 38 мм рт. ст. Какое исследование необходимо провести для полноценного определения стадии глаукомы?

Периметрия, гониоскопия ПРОВЕРЕНО

24. Больной Ц., 67-ми лет, обратился к окулисту с жалобами на понижение зрения левого глаза. Острота зрения обоих глаз = 1,0. Глаза спокойные. Передние отрезки без видимых изменений. Незначительная деструкция пигментной каймы зрачка левого глаза. Оптические среды прозрачные. На глазном дне слева имеется сдвиг сосудистого пучка в носовую сторону и экскавация зрительного нерва. При периметрии слева имеется сужение поля зрения в верхне-носовом квадранте. Какое дополнительное исследование проведете? МРТ/ КТ

Тонометрия,гониоскопия

ПРОВЕРЕНО

25.У ребенка Н., 8-ми лет, месяц назад выявлена врожденная глаукома. Острота зрения обоих глаз = 0,3 со сферическим стеклом – 2,0 дптр = 1,0. У него отмечается повышение внутриглазного давления. Увеличены размеры слепого пятна. Имеются парацентральные скотомы в зоне Бъеррума. На глазном дне изменений нет. Рекомендации для лечения? Гониотомия,трабекулотомия,гониопунктура ПРОВЕРЕНО

26.К офтальмологу обратилась мать годовалого ребенка. Вскоре после его рождения она заметила, что у него "большие глаза и один глаз больше другого". Отмечает беспокойное поведение ребенка, особенно днем, светобоязнь, слезотечение. При осмотре ребенка врач обратил внимание на большую величину глазных яблок и разноразмерность их, выраженную светобоязнь и слезотечение. При осмотре: роговицы обоих глаз умеренно отечные, диаметр роговиц большой, глубжележащие среды глаз осмотреть не удалось из-за сильной светобоязни и беспокойного поведения ребенка. Ваши рекомендации? Проведение трабекулотомии,гониотомии ПРОВЕРЕНО

27.Больная 64-х лет, обратилась с жалобами на затуманивание зрения по утрам, видение "радужных" кругов в это время, снижение зрения обоих глаз. К 12 часам дня, как правило, "туман" перед глазами проходит. Острота зрения правого глаза=0,5, левого глаза=0,6, не корригируют. Глаза спокойные, среды глаз прозрачны. Тонометрически внутриглазное давление повышено. Какой из этих методов исследований поможет определить стадию данной патологии? Офтальмоскопия, периметрия ПРОВЕРЕНО

28.Женщина 60 лет, обратилась с жалобами на боли в правом глазу, иррадиирующие в правую половину головы, слезотечение, светобоязнь, блефароспазм, покраснение глаза. При объективном осмотре: Острота зрения правого глаза - неправильная светопроекция, левого глаза 1.0. Глаз инъецирован, роговица отечная, передняя камера мелкая, зрачок расширен, хрусталик мутный, набухший. Глазное дно не офтальмоскопируется. Выставлен диагноз катаракта. Какое из этих мероприятий направлено на устранение причины данных жалоб пациента? удаление хрусталика с последующей имплантацией интраокулярной линзы ПРОВЕРЕНО

29.Дедушка 78 лет обратился к окулисту по месту жительства с жалобами на снижение предметного зрения, мелькание мушек, на чувство пелены перед глазами. При визометрии функции правого глаза – 0,02

.функции левого глаза 0,8. внутриглазное давление в норме, При исследовании на щелевой лампе диффузные помутнения на правом глазу, начальные помутнения в хрусталиках на левом глазу. Поставлен клинический диагноз зрелая катаракта обоих глаз. Какой метод исследования необходимо провести для осмотра глазного дна?Офтальмоскопию ПРОВЕРЕНО

30.В ургентную службу глазного отделения обратился мужчина 69 лет, с жалобами на резкую болезненность, покраснение, снижение предметного зрения на левом глазу. Из анамнеза был выставлен диагноз незрелая катаракта правого глаза. При осмотре глаз инъецирован, роговица отечная, передняя камера мельче средней, зрачок широкий, хрусталик мутный, набухает. Глазное дно: не офтальмоскопируется. Был выставлен диагноз: Набухающая катаракта, факоморфическая глаукома правого глаза. Какой метод радикального лечения целесообразно назначить при данном диагнозе? Экстракция катаракты (ФЭК с имплантацией ИОЛ или ЭЭК тоже с имплант ИОЛ) ПРОВЕРЕНО

31.Женщина 65 лет обратилась к окулисту ЦСМ с жалобами на снижение предметного зрения, мелькание мушек, на чувство пелены перед глазами. При визометрии функции обоих глаз – 0,7 внутриглазное давление в норме, При биомикроскопии начальные помутнения в хрусталиках. Поставлен клинический диагноз начальная катаракта обоих глаз. Какие капли целесообразно назначить при данном диагнозе?. В начальной

стадии целесообразна консервативная терапия. Используют ЛС, улучшающие процессы метаболизма в хрусталике: цитохром С + натрия сукцинат + аденозин + никотинамид (офтан катахром), азапентацен (квинакс), витафакол, витайодурол в каплях 2-3 р в день.

Факовит/вицеин/квинекс/тауфон/катаракс/витайодофакол(комплекс витаминов,минералов) ПРОВЕРЕНО

32. Пациент 47 лет, обратился в глазной стационар с жалобами на ломящие боли, светобоязнь, слезотечение, снижение зрения правого глаза. Вышеперечисленные жалобы появились 5 дней, но за последние 2 дня боли усилились. Объективно: острота зрения правого глаза =0,1. Веки отечны, застойная инъекция, роговица отечна, на задней поверхности преципитаты. Зрачок узкий, фибрин в зрачковой зоне, радужка бомбирована, грязно-зеленого цвета. Какое хирургическое вмешательство возможно в данном случае?

Иридэктомия формирование сообщения между задней и передней камерой для оттока ВГД ПРОВЕРЕНО

33. Мужчина 70 лет, обратился в ЦСМ с жалобами на резкое снижение зрения, мелькание мушек перед левым глазом. Из анамнеза отмечено, что накануне он перенес тонзиллит. При осмотре острота зрения левого глаза = 0,02. Левый глаз спокоен, передний отрезок не изменен. При осмотре глазного дна – очаг в центральной зоне 0,3 пд с нечеткими границами, желтовато-серого цвета, сетчатка вокруг инфильтрирована, отечна. Был выставлен диагноз хориоидит. С чего более целесообразно начать лечение? АБ терапия с

учетом чувсвтвительности (либо эмпирически широкого спектра действия , противовоспалительная терапия гкс парабульбарно. Для рассасывания экссудата и кровоизлияний испол-т ферменты(трипсин,лидазу итп)

Мидриатики (1%р-р атропина сульфата,ирифрин,тропикамид-мидримакс

,циклопентолат-цикломед,фенилэфрин)

ПРОВЕРЕНО

34. Пациент 47 лет, обратился в глазной стационар с жалобами на ломящие боли, светобоязнь, слезотечение, снижение зрения правого глаза. Вышеперечисленные жалобы появились 5 дней, но за последние 2 дня боли значительно усилились. Объективно: острота зрения правого глаза =0,1. Веки отечны, застойная инъекция, роговица отечна, на задней поверхности преципитаты. Зрачок узкий, фибрин в зрачковой зоне, радужка бомбирована, грязно-зеленого цвета. Какое дополнительное обследование необходимо провести для облегчения постановки диагноза? Тонометрия, УЗИ

ПРОВЕРЕНО

35.М. 40 лет, поступил в стационар с сильными болями, светобоязнью, слезотечением левого глаза. Анамнез: выше перечисленные жалобы начались накануне вечером, ранее переболел гриппом. Объективно: левый глаз – перикорнеальная иньекция, преципитаты на задней поверхности роговицы, зрачок сужен, радужка отечна, изменена в цвете, задние синехии. Выставлен диагноз: Иридоциклит левого глаза. Какие капли необходимо назначить в качестве первой помощи? 1% раствор атропина и анальгетики ПРОВЕРЕНО

36.Женщина 45 лет, обратилась в ЦСМ к окулисту с жалобами на боли, светобоязнь, слезотечение правого глаза. Из анамнеза: вышеперечисленные жалобы появились 3 дня назад, страдает хроническим гайморитом. При осмотре: острота зрения правого глаза = 0,1. Перикорнеальная инъекция, на роговице язва с черным пузырьком в центре, гной в передней камере, зрачок узкий. Какие капли вы

назначите?Антибактериальные :моксифлоксацин 0,5% или гатифлоксацин 0,3–0,5%, тобрамицин ,15 мг/мл и цефазолин 50 мг/м. ПРОВЕРЕНО

37.Женщина 42 лет, жалуется на упорное слезотечение в правом глазу, гнойное отделяемое из глаза. Вышеперечисленные жалобы появились 2 недели назад. При обследовании больной обращают внимание на избыток слезы по краю нижнего века, фасолевидное мягкое выпячивание кожи под внутренней связкой век. При надавливании на область слезного мешка из слезных точек обильно вытекает слизисто-гнойное содержимое. Канальцевая проба положительная, слезно-носовая проба отрицательная. Какое лечение вы проведете? Бужирование/дакриоцисториностомия + противовоспалительная и антибактериальная терапия. ПРОВЕРЕНО

38.В отделение микрохирургии глаза поступил больной с предварительным диагнозом «Проникающее ранения роговицы». При осмотре острота зрения правого глаза = 0 (ноль), хемоз конъюнктивы, гипопион 3 мм, в просвете зрачка серовато – желтый экссудат. Выставлен диагноз эндофтальмит правого глаза . Какой метод оперативного лечения уместен в данном случае? Эвисцерация ПРОВЕРЕНО

39.К Вам обратилась женщина 33 лет, с жалобами на сильные боли в левом глазу с иррадиацией в висок, брови, челюсть которые не купируются анальгетиками. Из анамнеза: у женщины сахарный диабет 2 типа. Внутриглазное давление в норме. Какой метод диагностики вы назначите для точной постановки диагноза в данном случае? Офтальмоскопия ПРОВЕРЕНО

40.В стационар поступил пациент 23 лет с жалобами на отек, болезненность век. Процесс наблюдался сначала на одном глазу, затем на втором глазу. Клинические проявления появились после переохлаждения. При осмотре были обнаружены точечные геморрагии на конъюнктиве склеры. На слизистой оболочке век образуются белесовато-серые пленки, которые легко снимаются влажной ватой. Какое основное исследование вы должны назначить данному пациенту? Бак.исследование мазков с конъюнктивы ПРОВЕРЕНО

41.Сущность 3х компонентной теории цветового зрения в том, что в макуле имеются 3 типа колбочек с различным пиком спектральной чувствительности, при отсутствии одного типа колбочек, то изменяется восприятие 2х других основных цветов. Какая таблица применяется для исследования цветоощущения? Таблица Рабкина ПРОВЕРЕНО

42.Больной 48лет обратился с жалобами на двоение в глазах. Со слов больного вышеуказанные жалобы беспокоят последние 6 дней. Объективно: отмечается ограничение движения глазного яблока кнаружи левого глаза. Выставлен клинический диагноз: Паретическое косоглазие, парез VI пары ЧМН. Какую пробу необходимо применить для постановки этого диагноза?Проба со спицами,проба с чтением карандашом,способ Гиршпрунга Изучение девиации, исследование подвижности, исследовании тортиколлиса, определение поля взора, электромиографии-1 поток ПРОВЕРЕНО

43.Ученик 8 класса средней школы обратилась к окулисту с жалобами на ухудшение зрения вдаль, из анамнеза близорукостью страдает с 6лет, последние два года отмечает резкое ухудшение зрения, объективно: острота зрения обоих глаз 0,1 с sph -6.5 =1.0. Был поставлен диагноз прогрессирующая миопия

высокой степени обоих глаз. Какой метод хирургической операции показан для предотвращения прогресса миопии? Склеропластика ПРОВЕРЕНО

44.Мужчина 72 лет, обратился в поликлинику с жалобами на низкое зрение вдаль на обоих глазах, со слов пациента оба глаза были оперированы по поводу катаракты 2 года назад без имплантации интраокулярной линзы. Острота зрения обоих глаз 0,01 с коррекцией 0,6. Выставлен диагноз постоперационная афакия обоих глаз. Какие линзы необходимо назначить для коррекции зрения в данной ситуации? КОРРЕКЦИЯ С ПОМОЩЬЮ ОЧКОВ(СЛАБОЙ ЛИНЗОЙ)(в применении собирательных линз силой 11-

13дптр.) И Интраокулярная коррекция ПРОВЕРЕНО

45.Работник полиграфии 42 лет обратился к окулисту с жалобами на нечеткость изображений в даль, в частности жалуется что плохо видит в ночное время, объективно острота зрения обоих глаз: 0,6 с коррекцией =1,0. При скиаскопии выходит: вертикальный меридиан – гиперметропия в 3.0 диоптрию, горизонтальный –

эмметропия. Внутриглазные среды прозрачные, глазное дно без особенностей. Какие линзы необходимо использовать для коррекции аметропии в данной ситуации? Цилиндрические +3,0 ПРОВЕРЕНО

46.Мужчина 52 года обратился к окулисту с жалобами на низкое предметное зрение в даль, особенно в сумерки и ночное время, объективно острота зрения обоих глаз: 0,6 с коррекцией =1,0 При скиаскопии выходит на обеих меридианах миопия в 1.0 диоптрию. Внутриглазные среды прозрачные, глазное дно без

особенностей. Какие линзы необходимо использовать для коррекции аметропии в данной ситуации? Сферические -1,0 ПРОВЕРЕНО

47.Пенсионер 60 лет жалуется на то, что стал плохо различать детали вблизи, ощущает тяжесть в глазах и головные боли после работы. При объективном обследовании врач выявил гиперметропию в (+)1.5 диоптрий. Какие очки для работы нужно выписать данному пациенту для работы? Для работы вблизи у него у самого +1,5 плюс к этому пресбиопия +3,0 =4,5 получается (собирательные линзы+) ПРОВЕРЕНО

48.Молодой человек обратился с жалобами на ухудшение зрения обоих глаз за последний год. Ранее нигде не проверялся. При субъективном исследовании острота зрения обоих глаз = 0,5 без коррекции. Какой объективный метод исследования необходим для дальнейшей коррекции пациента? Рефрактометрия ПРОВЕРЕНО

49.Ювелир 47 лет, предъявляет жалобы на ухудшение зрения при работе на близком расстоянии. Вдаль видит хорошо. Острота зрения обоих глаз = 1,0. Глаза здоровы. Какие линзы необходимо назначить для коррекции зрения в данной ситуации? +1,00+1,50(собирательные линзы+) ПРОВЕРЕНО

50.Менеджер банка 32 лет, пришел на прием к врачу с жалобами на усталость в глазах, быстрое утомление, частое моргание. Данные объективного исследования. Острота зрения обоих глаз = 1,0. Глаза совершенно спокойные. Двигательный аппарат не изменен. Передний отрезок глаз без видимых изменений. Оптические среды прозрачные. Глазное дно в норме. Рефрактометрически – эмметропия. Был выставлен диагноз спазм аккомодации. Какие капли можно назначить данному пациенту?

Мидриатики и циклоплегики:атропин,ирифрин,тропикамид циклопентолат, цикломед,фенилэфрин

ПРОВЕРЕНО

51.Больной 42 года, обратился к окулисту с жалобами на низкое предметное зрение вблизи. Со слов вышеуказанные жалобы отмечаются последние 2 года. Выставлен диагноз “Пресбиопия”. Какую таблицу должен применить офтальмолог для очковой коррекции? ТАБЛИЦА ГОЛОВИНА-СИВЦЕВА ПРОВЕРЕНО

52.Пациент по таблице Головина-Сивцева видит первый ряд с расстояния 2 м. Рассчитайте его остроту зрения с помощью формулы Снеллена. 2/50=0,04 ПРОВЕРЕНО

53.Пациент страдает опухолью гипофиза, при росте опухоли сдавливаются аксоны мультиполярных

клеток центра хиазмы, врач выявил битемпоральную гемианопсию, применив метод: периметрии

(оценивается периферическое зрение пациента и устанавливается объем утраченных полей зрения);

компьютерная кампиметрия (оценивается светоощущение и цветовосприятие);

МРТ (магнитно-резонансная томография) выявляет причину заболевания – опухоль, инфаркт, кровоизлияние

54.Родители привели ребенка к окулисту с жалобами на низкое предметное зрения в сумерках. Со слов родителей заметили при игре в прятки ребенок дезориентируется. Какой метод должен применить окулист для определения нарушения сумеречного зрения? ТЕМНОВАЯ АДАПТОМЕТРИЯ проводят с помощью

таблицы Кравкова-Пуркинье. ПРОВЕРЕНО

55.В стационар поступил мужчина 47 лет, с жалобами на сильные боли в глазу и голове, слабость, повышение общей температуры. Из анамнеза работал на стройке и что-то попало в глаз. Обратился в амбулаторию. Где ему назначили антибиотики в каплях и внутримышечно, но состояние ухудшалось. Обно:VOD=0(ноль). Экзофтальм, выраженный отек век, хемоз конъюнктивы, роговица мутная, передняя камера заполнена гноем. Выставлен диагноз: Панофтальмит правого глаза. Ваша дальнейшая тактика лечения?

АНТИБАКТЕРИАЛЬНАЯ ТЕРАПИЯ (СУЛЬФАНИЛАМИДЫ И А/Б) ПРОВЕРИЛ Эвисцерация, так как Операцию проводят при заболеваниях и патологиях, не поддающихся консервативной терапии или требующих экстренного вмешательства.

56.В ЦСМ обратился пациент 55 лет с жалобами на снижение зрения обоих глаз. Анамнез: зрение снижалось постепенно, к врачам не обращался. Об-но: глаза спокойны, расширение передних ресничных артерий у входа в эмиссарий, роговица прозрачная, зрачок в центре, нарушение целостности пигментной каймы по зрачковому краю. Глазное дно: расширение физиологической экскавации на диске зрительного нерва. ТОД=24мм.рт.ст., ТОS=29мм.рт.ст. Какое обследование необходимо провести для постановки диагноза? ФЛЮОРЕСЦЕНТНАЯ АНГИОГРАФИЯ

57.В глазной стационар обратилась женщина 30 лет с жалобами на боли в левом глазу, светобоязнь, слезотечение, снижение зрения. Из анамнеза: данные жалобы беспокоят 3й день, ни с чем не связывает. Обно: VOD=1,0 VOS=0,1 н/к. OSперикорнеальная инъекция, на задней поверхности роговицы преципитаты, зрачок узкий, слабая реакция на свет. Выставлен диагноз: Иридоциклит левого глаза. Ваша тактика лечения?

ПРЕДОТВРАЩЕНИЕ СРАЩЕНИЙ РАДУЖКИ С ХРУСТАЛИКОМ И СНЯТИЕ БОЛЕВОГО СИНДРОМА После постановки диагноза в первую очередь, как правило, назначают анестетики, мидриатические препараты,

расширяющие зрачок (атропин), антибиотики (цефаоспорины,фторхинолоны),нестероидные противовоспалительные средства (диклофенак, бромфенак, непафенак)ПРОВЕРЕНО

58.В ЦСМ обратился юноша 17 лет, с жалобами на низкое зрение левого глаза, заметил случайно. Об-но. VOS=0,08н/к, глаз спокоен. Роговица конической формы, вершина опущена книзу. При биомикроскопии видны трещины десцеметовой оболочки. Передняя камера в центре глубже чем на парном глазу. Выставлен диагноз кератоконус левого глаза. Что является радикальным методом лечения кератоконуса?

КЕРАТОПЛАСТИКА(ПЕРЕСАДКА РОГОВИЦЫ) ПРОВЕРЕНО

59.Мужчина диспечер 52 лет обратился в ЦСМ с жалобами на резь, жжение, сухость в глазах, усиливающихся к вечеру. Об-но: VOU=0,9 – поверхностная инъекция конъюнктивы, поверхность роговицы тусклая. Проба Ширмера – 5мм. Проба по Норну – 7 сек. Поставлен диагноз: Синдром сухого глаза. Какие

препараты необходимо назначить пациенту? Капли на основе искусственной слезы Слезозаменяющие

препараты (оксиал,лакрисин,офтагель)

ПРОВЕРЕНО

60. Пациент 28 лет, обратился с жалобами на низкое предметное зрение и косоглазие правого глаза. При объективном осмотре: острота зрения правого глаза 0,2 (не коррегируется), левого глаза 1.0. Отмечается отклонение правого глаза кнутри на 300. Какой метод диагностики был применен для определение угла косоглазия?

МЕТОД ГИРШБЕРГА ПРОВЕРЕНО

61.Обратилась больная 65 лет, с жалобами на выворот нижнего века, сухость в глазу, покраснение, чувство ощущения песка в обоих глазах. От предложенной операции по устранению выворота отказывается. Какой из этих местных препаратов может существенно облегчить состояние больной? Использование

заменителей слезы днем(Офтагель, Офтолик, Оптив, Артелак и пр.) и препаратов для увлажнения роговицы (глазных лубрикантов) ночью ПРОВЕРЕНО

62.Больная 40 лет, обратилась с жалобами на гнойное отделяемое из слезных точек при надавливании на проекцию слезного мешка, постоянное слезотечение, слезостояние с правой стороны. Со слов больной, вышеуказанные жалобы беспокоят в течении 1 года. Какое оперативное вмешательство необходимо провести для профилактики осложнения данного заболевания? ДАКРИОЦИСТОРИНОСТОМИЯ ПРОВЕРЕНО

63.Пожилой мужчина 70 лет обратился с жалобами на ухудшение зрения правого глаза. При осмотре врача-офтальмолога: диск зрительного нерва гиперемирован, слегка выступает в стекловидное тело, границы четкие вены расширены, извиты. В сетчатке над диском зрительного нерва и в центральной зоне глазного дна видны множественные кровоизлияния в виде штрихов. Выставлен диагноз тромбоз центральной вены сетчатки. Какой из этих методов диагностики был применен врачом для выставления диагноза?

ОФТАЛЬМОСКОПИЯ ПРОВЕРЕНО

64.Женщина 62 лет, обратилась к окулисту по месту жительства с жалобами на отсутствие зрения левого глаза. С ее слов, зрение снижалось постепенно, за последние 5 лет, При объективном осмотре левого глаза: глаз спокоен, роговица прозрачная, передняя камера средняя, зрачок округлой формы, хрусталик мутный.

Какой метод диагностики был применен в данном случае?

БИОМИКРОСКОПИЯ ПРОВЕРЕНО

Профболезни (58)

Запоминание

1. Какой документ необходим для решения вопроса о связи заболевания с профессией:

санитарно-гигиеническая характеристика условий труда

2.Документ, регламентирующий организацию и проведение предварительных и периодических медосмотров работающих:

трудовой кодекс КР. Постановление правительства КР

3.Кто расследует случаи хронических профессиональных заболеваний? комиссия, организованная работодателем

4.У рабочих какой профессиональной группы развивается силикоз? проходчик

5.Основной критерий диагностики силикоза:

затемнения на рентгенограмме

6.В какой профессиональной группе возможно развитие пылевого бронхита: проходчик

7.Силикоз с размером узелков до 1,5 мм обозначается символом:

P

проверено

8. Силикоз с размером узелков от 1,5 до 3 мм обозначается символом:

q

проверено

9. Силикоз с размером узелков от 3 до 10 мм, обозначается символом:

r

проверено

10.Пневмокониоз с диаметром узлов от 1 до 5 см обозначается символом:

A

проверено

11.Пневмокониоз с диаметром узлов от 5 до 10 см обозначается символом:

B

проверено

12.Интерстициальная форма силикоза обозначается символом:

S, t, u

проверено

13.Крупноузловой пневмокониоз характеризуется затемнениями размером:

до 10 см в диаметре

проверено

14.Перечислите

ртутьорганические

ядохимикаты:

Этилмеркурхлорид,Гранозан,Меркуран,Меркургексан,Меркурбензол,Фенилмеркурацетат

проверено

Для диагностики пылевого бронхита имеет значение?: стаж работы 10лет

15.Каковы особенности клинического течения пылевого бронхита? Постепенное

начала заболевания, хроническое течение.Отставание рентгенологических изменений от клинических,Скудные физикальные данные.Мокроты или нет, или ее мало и она носит слизистый характер,Кашель не носит упорного мучительного характера.Отсутствуют признаки интоксикации .Нет выраженной температурной реакции.В крови отсутствуют

отчетливые признаки воспаления.

+Пылевой бронхит всегда имеет прогрессирующее течение., клиника зависит от стадии и формы(варианта)

16.Аускультативная картина при умеренно выраженном пылевом бронхите: жёсткое

дыхание, сухие хрипы, Крепитация на поздних стадиях

жесткое/ослабленное дыхание, крепитация, сухие рассеянные свистящие хрипы 17.Пылевой бронхит бронхоскопически характеризуется

эндобронхитом:Частичный,Частично-Диффузный,Двусторонний

Диффузный

двусторонний диффузный эндобронхит

18.Работники какой профессии подвергаются действию локальной вибрации?

Работники металлообрабатывающей, металлургической, строительной, авиа- и судостроительной, горнодобывающей промышленностей

обрубщики, формовщики ручной формовки, слесари механосборочных работ,полировщики, кленальщики, проходчики,забойщики

19.Метод определения виброчувствительности: Паллестезиометрия

правильно

20.Состояние периферического кровообращения исследуют методом: Капилляроскопии

правильно

21.Работники какой профессии подвергаются действию общей вибрации?

Бульдозеристы

+водители большегрузных машин, механизаторы сельского хозяйства, машинисты экскаваторов и буровых станков

22.К начальным проявлениям вибрационной болезни от воздействия общей вибрации относится:

Парестезии, неинтенсивные боли в н/ к, судороги в стопах ,акроцианоз , гипергидроз , гипотермия кистей и стоп , Синдром белых пальцев .

ангиодистонический синдром,вегетативно-вестибулярный синдром, синдром вегетативно-сенсорной полинейропатии нижних конечностей

23.К умеренно-выраженным проявлениям вибрационной болезни от воздействия общей вибрации относится:

церебральнопериферический ангиодистонический синдром . Сенсорная полиневропатия конечностей , полирадикулоневропатия

+вторичный пояснично–крестцовый корешковый синдром, синдром неврастении

24.Работники каких прощщфессий контактируют со свинцом: Плавильщики свинца, аккумуляторщики, мельники свинцового порошка, намазчики свинцовых пластин, составители шихты в производстве хрусталя, аппаратчики в производстве свинцовых красителей, изготовители свинцовой дроби, чеканщики свинцовых изделий, пайщики

+ декораторы, расписывающие посуду свинецсодержащими красками 25.Токсическое действие свинца обусловлено:

Свинец относится к ядам кумулятивного действия. Он депонируется в виде нерастворимого трехосновного фосфата свинца. Большая часть свинца откладывается в трабекулах костей, где он вытесняет соли кальция. активно влияет на синтез белка, энергетический баланс клетки, подавляет ферментативные процессы превращения порфиринов и инкорпорацию железа в порфирин с образованием гема

тиоловый яд, блокирующий SH-группы, а также карбоксильные и аминные группы ферментов. нарушается биосинтез порфиринов и гема.

26.Характерным изменением порфиринового обмена для интоксикации свинцом является? (Все 3 ответа верны) 1.Повышенная экскреция дельта-

аминолевуленовой кислоты с мочой; 2.Повышенная экскреция копропорфирина с мочой; 3. Увеличение эритроцитарного протопорфирина

проверно

27.Какой из указанных препаратов наиболее эффективен для лечения больных со свинцовой интоксикацией: Тетацин кальция

проверено

28.Назовите производство, где используется ртуть:

Измерительных приборов

+электронной аппаратуры, ртутных выпрямителей, насосов, целлюлозно-бумажная промышленность

29.30. Назовите неврологический синдром ртутной интоксикации: Эритизм

проверено

30.Дайте объяснение выражению «ртутный эретизм»?

Эретизм при меркуриализме, сочетающийся с боязливостью, внушаемостью, бессонницей или кошмарными сновидениями, часто сопровождающийся тремором пальцев рук и век.

значительные нарушения в эмоциональной сфере

31.Какой препарат используется для элиминационной терапии ртутной интоксикации?

Унитол,натрия-тиосульфат, сукцимер,д-пеницилламин

проверено

32.Назовите производство, где используются бензол и его гомологи: Растворителем

лакокрасочная промышленность, химическая промышленность,электротехническая промышленность, резинотехническая промышленность, автомобилестроение

33.При острой интоксикации бензолом поражается преимущественно? центральная нервная система

проверено

34.Какая форма анемии наблюдается при хронической интоксикации бензолом? Апластическая анемия

35.Лечение тяжёлой формы хронической интоксикации бензолом? Кортикостероид (преднизолон)

36.Для выраженной формы интоксикации свинцом характерны? Анемия

Проверено

37.Какой неврологический синдром может встречаться при свинцовой интоксикации?

Периферическая полинейропатия

Проверено

38.Что относится к выраженным проявлениям вибрационной болезни от локальной вибрации? Сенсерно-моторная полинейропатия,энцефало-полинейропатия, полинейропатия с генерализованными микроангиоспазмами Проверено

39.Перечислите хлорорганические ядохимикаты: Хлорпикрин

+дихлордифенилтрихлорэтан (ДДТ), гексахлорциклогексан (ГХЦГ), Гексахлоран, альдрин ,ТХАН?

40.При каких пневмокониозах гистологически выявляются "конхоидальные тельца": Бериллиоз

Проверено

41.Рентгенографические признаки асбестоза согласно международной классификации пневмокониозов:

Линейно-септическая деформация лёгочного рисунка, прикорневой фиброз, плевральные изменения

линейные затемнение, "сотовое лёгкое"

42.Пневмокониоз с диаметром узлов от 5 до 10 см обозначается символом:

В

Проверено

43. Вибрационная болезнь от воздействия локальной вибрации клинически проявляется в виде синдрома:

Ангиодистонический синдром

Проверено

44.Токсическое действие свинца обусловлено: Угнетение образования порфирина и гема

Проверено

45.Перечислите «свинцовоопасное» производство: Изготовление свинцово-кислотных аккумуляторов

Проверено

46.При свинцовой интоксикации назначают: Тетацин Са

+пентацин, Д-пеницилламин

47.Микромеркуриализм характеризуется:

Хронической интоксикацией ртутью, функциональные изменения ЦНС (астеновегетативный, ртутный эретизм) Проверено

ПОНИМАНИЕ

1.Формовщик литейного цеха, стаж 16 лет, жалуется на одышку, сухой кашель, боли в области лопаток. Физикальные данные не изменены. Рентгенограмма: усиление и деформация легочного рисунка, в средних и нижних отделах с обеих сторон единичные округлые тени размером 2-3 мм с четкими контурами. Корни расширены и уплотнены. Выставьте данному пациенту предварительный диагноз? Силикоз

2.У электросварщика со стажем работы 16 лет, на рентгенограмме выявлены диффузные мелкие округлые затемнения. Предположительный диагноз: пневмокониоз электросварщиков

3.Шлифовщица на наждачных кругах, стаж 10 лет, жалуется на боли в кистях и побеление пальцев рук при общем охлаждении. Кончики пальцев рук отечны, цианотичны, белое пятно выражено. Трофических нарушений кожи нет. Ваш диагноз? Вибрационная болезнь 1 стадия

4.Больной А. 30 лет, с 6 летним стажем работы клепальщика, жалуется на резкие боли в кистях рук, особенно в ночное время, пальцы рук немеют и белеют при охлаждении. Кисти отечны, холодные на ощупь, цианотичные. Резко снижены все виды чувствительности по типу «длинных перчаток». Положительная холодовая проба, капилляроскопически – спастико-атоническое состояние капилляров. На R –грамме со стороны костей кисти –очаги уплотнения костной ткани. Ваш диагноз? Вибрационная болезнь

5.У наборщика типографии выявлен астено - вегетативный синдром. В крови Нв-100 г/л, ретикулоциты- 3,2%, эритроциты с базофильной зернистостью - 52:10000. Цвет мочи розовый. Выставьте предварительный диагноз данному пациенту? Отравление свинцом

6.Лаборант химической лаборатории ртутного комбината, стаж 20 лет, жалуется на раздражительность, дрожание рук. Сухожильные рефлексы оживлены, асимметричный

тремор век и пальцев вытянутых рук, красный дермографизм. ЧСС90 уд. мин. АД150/100 мм рт. ст. Выставьте данному пациенту предварительный диагноз? интоксикация металличесой ртутью

7. Мужчина 40 лет доставлен в приёмное отделение с жалобами на резкую слабость, рвоту и боли в животе, повышенное отделение слюны и судороги. Из анамнеза – за 2 часа до появления симптомов работал на поле, где недавно была проведена обработка ядохимикатами. Объективно: зрачки сужены, гиперсаливация, в легких сухие хрипы, клонико-тонические судороги. Предварительный диагноз? Отравление пестицидами

9.Больной Т., работает в течение 7 лет на прицеферме. Последние 2 года отмечает коньюнктивит, ринорею на работе. Несколько дней назад во время работы у больного возник приступ удушья. Предположительный диагноз? Аллергический коньюктивит , ринит

10. Больной С., с отравлением фосфорорганическими пестицидами. Наблюдается никотиноподобный и мускариноподобный синдромы. Какой препарат следует назначить?

Атропин для устранения бронхоспазма

Пралидоксим вводится после атропина для устранения нервно-мышечных симптомов. Бензодиазепины используются при судорогах

Психиатрия (155)

Запоминание

1.Как называется комплекс мероприятий, направленных на постепенное смягчение представлений общества о больном, страдающим психическим расстройством? дестигматизация

2.Укажите основной метод диагностики психических расстройств из нижеперечисленных: метод расспроса (клинический)

3.Какое из нижеперечисленных утверждений соответствует критериям бреда?

Ложное умозаключение

Нет критики

Не поддается разубеждению

Возникает на патологической почве

Не соответствует этно-культуральным особннностям

Полностью овладевает сознанием

4.Укажите из нижеприведённых утверждений правильное определение деменции. Деменция – это: Синдром, обусловленный заболеванием мозга, обычно хронического и прогрессирующего характера, при котором нарушаются высшие корковые функции

5.Укажите из нижеприведенных утверждений особенность депрессии на современном этапе: соматизированная

6.Укажите какой симптом из нижеперечисленных относится к основным симптомам депрессии:

Анергия (отсутствие двигательной активности)

Ангедония (отсутствие получения удовольствия от того, что ранее интересовало)

Гипотимия (снижение настроения)

аффект тоски, интеллектуальная и двигательная

заторможенность

7. Определите группу препаратов, которые могут вызывать привыкание, злоупотребление и формирование зависимости:

Анксиолитики (транквилизаторы) бензодиазепинового ряда

8.Укажите группу психотропных препаратов, у которых наиболее выражены экстрапирамидные побочные эффекты:

Типичные антипсихотики(нейролептики)

9.Определите какой из нижеперечисленных препаратов наиболее часто используют для купирования психомоторного возбуждения:

Галоперидол либо хлорпромазин (аминазин)